test bank for medical surgical nursing ignatavicius 7th edition

964
TEST BANK FOR MEDICAL SURGICAL NURSING IGNATAVICIUS 7TH EDITION

Transcript of test bank for medical surgical nursing ignatavicius 7th edition

TEST BANK FOR

MEDICAL SURGICAL NURSING

IGNATAVICIUS 7TH EDITION

Chapter 1: Introduction to Medical-Surgical Nursing

Test Bank

TEST BANK OF MEDICAL

SURGICAL NURSING

IGNATAVICIUS 7TH EDITION

MULTIPLE CHOICE

Which action demonstrates that the nurse understands the purpose of the Rapid Response Team?

a. Monitoring the client for changes in postoperative status such as wound infection

b. Documenting all changes observed in the client and maintaining a postoperative flow sheet

c. Notifying the physician of the client’s change in blood pressure from 140 to 88 mm Hg systolic

d. Notifying the physician of the client’s increase in restlessness after medication change

The Joint Commission focuses on safety in health care. Which action by the nurse reflects The Joint Commission’s main objective?

a. Performing range-of-motion exercises on the client three times each day

b. Ensuring that the client is eating 100% of the meals served to him or her

c. Assessing the client’s respirations when administering opioids d. Delegating to the nursing assistant to give the client a complete bath

daily

Which action by the nurse shows an understanding of the principle of self-determination?

a. Allowing a postoperative client to decide to take medication with fruit juice rather than water

b. Allowing a teenager to decide not to go to a clinic when there is evidence that she is having profuse vaginal bleeding

c. Allowing a parent to decide not to proceed with a lifesaving operation for a 12-year-old client

d. Allowing an older client with dementia to decide not to take cardiac medication throughout the shift

The nurse is initiating a series of teaching sessions with an older client. What is the nurse’s highest-priority, client-centered action before

beginning the session? a. Ensure that the client’s family is present and will participate.

b. Make certain that the client is wearing his glasses. c. Have printed handouts ready to use during the session.

d. Schedule the session for early evening after the client’s meal.

Which action best demonstrates the nurse using client-centered care

when planning a menu for a Vietnamese client who is newly diagnosed with diabetes?

a. Asking the client what food he or she would eat on a standard diabetic menu

b. Asking family members to make selections for the client from a diabetic menu

c. Ordering a typical diabetic meal for the client and planning diet teaching

d. Researching the Vietnamese culture before discussing diabetic meal planning

The Institute for Healthcare Improvement (IHI) identified interventions to save client lives. Which actions are within the scope of nursing

practice to improve quality of care? a. Insert a central line to give intravenous fluid to a dehydrated client.

b. Use sterile technique when changing dressings on a new surgical site.

c. Intubate a client whose oxygen saturation is 92%. d. Prescribe aspirin for a client who presents with an acute myocardial

infarction

Which action by the nurse demonstrates the best practice for nursing

documentation on a computerized record? a. Deleting all documentation errors on the computerized record

b. Using red font to denote all significant events that have occurred c. Waiting until the end of the shift to record a summary of

information d. Documenting assessment data at the point of care

A client is scheduled for a mastectomy. As she is about to receive the preoperative medication, she tells the nurse that she does not want to have her breast removed but wants a lumpectomy. Which response

indicates that the nurse is acting as a client advocate? a. Telling the client her surgeon is excellent and knows what is best for

her condition b. Calling the surgeon to come and explain all treatment options to the

client c. Holding the client’s hand and offering to pray with her for a good

outcome d. Arranging for a postoperative visit from a cancer survivor

What priority assessment data should be shared with the interdisciplinary team from a client admitted to the emergency

department with a lacerated artery? a. Information regarding next of kin to notify in case the client dies

b. History about what medications the client is currently taking

c. Measurement of blood pressure and pulse d. Assessment of rate and depth of respirations

Which intervention can the client expect to receive from a medical- surgical nurse in an ambulatory care setting?

a. Drawing blood for routine or preoperative testing b. Teaching the client how to change a dressing on an incision c. Obtaining the client’s signature on a surgical consent form

d. Performing a comprehensive physical examination

An emergency department (ED) nurse gives report on a client who is being transferred to the medical-surgical floor. Because of an identified

risk for suicide, the ED nurse suggests that the floor nurse contact a sitter and behavioral health. This statement represents which part of the

SBAR hand-off? a. Situation

b. Background c. Assessment

d. Recommendation

The nurse is present when the physician discusses the potential effects of a chemotherapy regimen for a client with cancer. Weighing the benefits of the chemotherapy against possible side effects is an example of which

ethical principle? a. Paternalism b. Beneficence

c. Justice d. Autonomy

The physician prescribes warfarin (Coumadin) 15 mg daily. The nurse notes that this is three times the normal dose for this client based on the client’s medication profile and laboratory work. What does the nurse do

first? a. Give the dose and document the concern.

b. Call the pharmacy for a consultation. c. Call the physician to question the order.

d. Hold the medication for that day.

Which statement best describes the process of nursing case management?

a. The coordination of care services to at-risk populations b. A collaborative process to promote quality and cost-effective care

c. The implementation of care to acutely ill, underserved populations

d. A cost-effective care delivery model meeting the needs of specially defined groups

Which client is best served by a case manager? a. An older woman with chronic cystitis

b. A middle-aged man with moderate hypertension c. An older woman with chronic heart failure and diabetes mellitus

d. A young adult with a fractured ankle from a sports injury

The client with a stroke was admitted to a medical-surgical unit. Which tasks does the nurse delegate to the unlicensed assistive personnel?

a. Complete the nursing care plan. b. Assist the client with meals.

c. Evaluate the pulse oximetry reading. d. Assess level of consciousness.

MULTIPLE RESPONSE

The nurse on a medical-surgical unit notices that there has been an increase in the number of client falls. Which methods would be effective in promoting quality improvement on this issue? (Select all that apply.)

a. Identify causes of falls on the unit by looking at specific client cases. b. Look at the research and the literature on prevention of falls.

c. Complain to the manager that team members are neglecting the clients.

d. Use sit and stand alarms because they seem to be working on other units.

e. Try more frequent rounding on clients as suggested by co-workers.

Which settings would require maximum implementation of the nurse supervisor role? (Select all that apply.)

a. Acute care setting b. Home care setting

c. Skilled nursing facility d. Assisted-living facility e. Rehabilitation facility

Which activities are within the role of the case manager? (Select all that apply.)

a. Gathering and organizing data about a client from client records and interviews

b. Planning care for a client with emphasis on client satisfaction c. Coordinating care among a variety of health care professionals and

settings d. Promoting the client’s interests while negotiating necessary health

care e. Advocating for the client and the family throughout the continuum

of care f. Using resources for appropriate client health care services

A client has metastatic lung cancer and is hospitalized for chemotherapy. Which intervention does the nurse delegate to the unlicensed assistive

personnel? (Select all that apply.) a. Assist the client with repositioning.

b. Teach the client to use the incentive spirometer. c. Take vital signs every 4 hours.

d. Record intake and output measurements. e. Promote the expression of grief and loss.

Chapter 2: Introduction to Complementary and Alternative Therapies

Test Bank

MULTIPLE CHOICE The nurse wishes to learn more about the client’s use of natural products

and their effectiveness. The nurse consults the National Center for Complementary and Alternative Medicine because it is known that this

center serves which function? a. Educates health professionals about complementary therapies

b. Educates new mothers on the benefits of massage c. Engages in fundraising to offset client expenses with medical care

d. Provides a scholarship for a student to study naturopathy

A client is anxious about having a dressing change. Which statement indicates that the nurse is promoting appropriate complementary

therapy? a. “I’ll call the doctor and ask for a larger dose of pain medication

before the dressing change.” b. “As we begin the next dressing change, I want you to think of a

beautiful, calm place where you feel happy and peaceful.” c. “I’ll get another nurse to stay in the room with us during the

dressing change so that you have a hand to hold during the procedure.” d. “Are you familiar with acupuncture? It’s a very effective

technique.”

The nurse has designed a treatment plan that includes the use of massage. Which intervention will the nurse implement first?

a. Assess the client to determine the most effective type of massage technique to use.

b. Inspect the skin over the tissue to be massaged to ensure that it is not infected or bruised.

c. Determine whether a licensed therapist will be needed to carry out the massage technique

d. Obtain permission from the client to implement this type of technique.

A client who has been using which therapy requires the most immediate intervention by the nurse?

a. Aromatherapy to treat depression b. Herbal preparations to treat hypertension c. Therapeutic touch to decrease level of pain

d. Tai Chi to improve joint flexibility

A client scheduled for surgery has been taking garlic supplements. Which action is most important for the nurse to take?

a. No action is necessary because the herbal agent is harmless.

b. Notify the charge nurse that the client has been taking garlic. c. Note the information on the client’s record and place in the chart. d. Notify the surgeon that the client has been taking garlic capsules.

For which client does the nurse arrange animal-assisted therapy? a. Middle-aged adult in a psychiatric facility with a history of

schizophrenia b. Older adult client with end-stage lung cancer in hospice care

c. Older adult client in a nursing home who is unresponsive d. Adolescent in a drug treatment facility with a history of violent

outbursts

Which statement indicates that the nurse understands the risks associated with herbal preparations?

a. Herbs are guaranteed to be safe and effective but are not necessarily natural.

b. Herbs require a different type of prescription than is required for standard prescribed medications.

c. Herbs are not classified as drugs and are regulated less strictly by the U.S. Food and Drug Administration (FDA).

d. Herbs are guaranteed to be all natural and of high quality but are not necessarily effective.

A client asks the nurse to pray with him. The nurse is an atheist. Which statement by the nurse is the correct response?

a. “Yes, let’s pray together.” b. “No, I’m sorry, I can’t do that.” c. “No, I don’t believe in prayer.”

d. “I’ll hold your hand while you pray.”

The client has been diagnosed with cancer and is experiencing depression and insomnia as side effects of chemotherapy. The client tells the nurse

that she has been supplementing her antidepressant medication with lavender oil and sandalwood but they aren’t working. Which statement

by the nurse is the best response? a. “Tell me more about exactly what you are taking, how much you

take, and when you take the antidepressants and use the oils.” b. “Perhaps you’re not using enough of the oil or are using it

incorrectly.” c. I’ll speak with your doctor to get you some medication that you can

take while continuing the aromatherapy.” d. “You don’t want your doctor to put you on sleeping pills and

antidepressants. Keep using them.”

The client is undergoing treatment for cancer and is experiencing a high level of anxiety. The client expresses interest in complementary therapies that might decrease the level of anxiety. Which action is the best choice

for the nurse to implement with this client? a. Direct the client to an imaginative peaceful setting using imagery.

b. Provide assistance in finding an acupuncturist. c. Suggest Tai Chi during chemotherapy treatments. d. Encourage the use of acupressure over tumor sites.

Which clients would benefit most from relaxation therapy? a. Middle-age client who is undergoing chemotherapy treatments

b. Young client who is diagnosed with schizophrenia c. Older client who is comatose and unresponsive

d. Young client who is diagnosed with major depression

A nurse is assisting a client in preparing for surgery by using progressive muscle relaxation. Which rationale best supports the use of this therapy

at this time? a. It rebalances or repatterns a person’s energy field.

b. It improves flexibility and assists with positioning during surgery. c. It applies pressure, releasing congestion and promoting energy flow. d. It uses intentional tensing and releasing of successive muscle groups.

A client tells the nurse that he or she is considering using herbal supplements. What is the nurse’s best response?

a. “Herbs are not classified as drugs in the United States, so there is no contraindication to using them.”

b. “Herbs have pharmacologic effects on the body and can interact with some prescription medications.”

c. “It is never permissible to use herbal supplements with prescription medications.”

d. “I will refer you to an herbalist, who can help you decide which medications you can take.”

A client who is status post aortic valve replacement takes sodium warfarin (Coumadin) daily and has started taking Ginkgo biloba. What

is the priority action for the nurse to take? a. Encourage the use of Ginkgo biloba to enhance the client’s systemic

circulation. b. Assess the client for any bruising or petechiae.

c. Explain that replacing Ginkgo biloba with garlic would be much safer.

d. Assess for any forgetfulness or inappropriate speech.

Which statement indicates that the client needs further teaching about complementary therapy?

a. “I’ve decided to use herb therapy for cancer treatment, so I can cancel my radiation treatments.”

b. “I’m hoping that massage therapy will help reduce the amount of pain medication I use for my myalgia.”

c. “I think it helps me get better faster when I picture the drugs punching out the germs in my body.”

d. “I intend to pray about my cancer treatment several times a day. It makes me feel so much better.”

Which teaching strategy is appropriate for a client who wishes to use mind-body complementary therapy to supplement traditional treatment

for cancer? a. Instruct the client to make a follow-up appointment with the health

care provider after using mind-body treatments to assess the client’s response to treatment.

b. Instruct the client never to use alternative or complementary treatments for serious illnesses.

c. Explain to the client that physicians and nurses are not prepared to recommend and monitor alternative treatments.

d. Explain to the client that physicians and nurses do not incorporate such treatments into their practice.

Which complementary or alternative therapy would the nurse recommend to a client with “stiff joints” to improve mobility?

a. Imagery b. Animal-assisted therapy

c. Tai Chi d. Aromatherapy

A client is experiencing nausea and vomiting from chemotherapy. Which alternative or complementary therapy would be best for the nurse to

explore with the client? a. Meditation

b. Imagery c. Yoga

d. Music therapy

The nurse is working in the community and completes home visits with older adult clients. Which statement by a client demonstrates a need for

further instruction about the use of complementary and alternative

therapies? a. “My doctor monitors my kidney function since I started taking

calcium.” b. “I always talk to my doctor first before starting an herbal

preparation.” c. “I heard that St. John’s wort is good for any type of depression.”

d. “I may start a Tai Chi program to help with my mobility and lift my spirits.”

MULTIPLE RESPONSE

During an initial health assessment interview, the nurse learns that the client is taking warfarin (Coumadin) for a history of deep vein

thrombosis. Later, the client admits to taking several herbal preparations as well. Which herbal preparations would the nurse caution the client to

avoid? (Select all that apply.) a. Ginkgo biloba

b. Garlic c. Ginseng

d. Zinc e. St. John’s wort

The nurse wishes to start music therapy with an older adult client who has high anxiety and hypertension. What essential elements should be considered when music is used with this client? (Select all that apply.)

a. Assess the client’s preferences in choice of music. b. Use fast tempo music to energize and motivate the client.

c. Consider rap music to provide diversion. d. Consider live or recorded music such as music performed on a harp.

e. Consider generation-specific music.

Chapter 3: Common Health Problems of Older Adults

Test Bank

MULTIPLE CHOICE

An older client is agitated and develops new-onset confusion on admission to the long-term care unit. What is the best action for the nurse to take to minimize relocation stress syndrome for this client?

a. Provide reorientation during hourly rounding. b. Obtain a certified sitter to remain with the client.

c. Speak to the client as little as possible to avoid overstimulation. d. Provide adequate sedation to lessen fear-provoking situations.

Which intervention would best support a client who relates a feeling of “loss of control” after having a mild stroke?

a. Explain that such feelings are normal, but that expectations for rehabilitation must be realistic.

b. Encourage the client to perform as many tasks as possible and to participate in decision making.

c. Further assess the client’s mental status for other signs of denial or psychopathology.

d. Obtain an order for physical and occupational therapy evaluations.

What will the nurse teach the older client with hypertension who complains that “food does not taste good without salt”?

a. Salt can be used as long as blood pressure remains controlled. b. All salt should be removed from the diet to preserve kidney

function. c. Table salt can be used in small amounts in conjunction with

diuretics. d. Herbs and spices can be substituted to season food.

What is a priority nursing intervention to prevent falls for an older adult client with multiple chronic diseases?

a. Providing assistance to the client in getting out of the bed or chair b. Placing the client in restraints to prevent movement without

assistance c. Keeping all four siderails up while the client is in bed

d. Requesting that a family member remain with the client to assist in ambulation

An older adult client is in physical restraints. Which intervention by the nurse is the priority?

a. Assess the client hourly while keeping the restraints in place. b. Assess the client every 30 to 60 minutes, releasing restraints every 2

hours. c. Assess the client once each shift, releasing the restraints for feeding.

d. Assess the client twice each shift while keeping the restraints in place.

An older adult client has become agitated and combative toward health care personnel on the unit. What is the first action that the nurse will

take? a. Obtain an order for a sedative-hypnotic medication to reduce

combative behavior. b. Attempt to soothe the client’s fears and reorient the client to

surroundings. c. Obtain an order to place the client’s arms in restraints to protect

personnel. d. Arrange for the client to be transferred to a mental health facility.

An older adult client presents with signs and symptoms related to digoxin toxicity. Which age-related change may have contributed to this

problem? a. Increased total body water b. Decreased renal blood flow

c. Increased gastrointestinal motility d. Decreased ratio of adipose tissue to lean body mass

A nurse is assessing a client’s understanding of medication therapy. Which statement indicates that the client needs further instruction? a. “My husband is on the same medication, so we always take our

medications together in the morning.” b. “I prepare all my medication for the week and place the pills in a

container labeled for each day.” c. “When I don’t sleep well at night, I take two thyroid pills the next

day instead of just one.” d. “I take my Coumadin every day when the noon news comes on the

television.”

An older adult client is being discharged from the hospital on several medications. Which intervention best reinforces medication teaching for

this client? a. Have the client actively participate in drug administration during

hospitalization. b. Include the client’s children in discussions regarding medication

administration. c. Give the client a pamphlet with the actions, side effects, and doses of

all drugs. d. Make a chart showing which drugs should be taken at specified

times during the day.

An older adult client’s spouse has died, and the family expresses concern that the client has lost weight recently and now refuses to attend the

annual family reunion. The nurse should assess this client further for what clinical condition?

a. Psychosis b. Depression c. Dementia d. Delirium

Which behavior exhibited by an older adult client alerts the nurse to the possibility that the client is experiencing delirium?

a. Becoming confused within 24 hours after hospital admission b. Displaying a cheerful attitude despite a poor prognosis

c. Becoming withdrawn and sleeping most of the day d. Beginning to use slurred speech and losing coordination

A client with Alzheimer’s disease has been hospitalized for dehydration. In making an assessment, the nurse notes the presence of a cluster of bruises on the client’s buttocks. What is the nurse’s priority action?

a. Call the local police to report a crime. b. Notify the client’s physician and social worker.

c. Confront the client’s caregiver with the suspicions. d. Alert security to prevent visits by the client’s caregiver.

An older adult client is suspected of being neglected by the caregiver. What assessment provides the nurse with the best information about this

possibility? a. Inspect skin in the “bathing suit zone” for bruises.

b. Assess the client for orientation to person, place, and time. c. Compare the client’s current weight with prior recorded weights.

d. Perform orthostatic pulse and blood pressure readings.

A nurse is caring for an older adult client who lives alone. Which economic situation presents the most serious problem for this client?

a. Stock market fluctuations b. Increased provider benefits

c. Social Security as the basis of income d. Costs of creating a living will

An older adult client is in the hospital. To what government resource would the nurse refer the client to help meet the cost of health care?

a. Preferred provider organizations b. Health maintenance organizations

c. Medicare Part A d. Medicare Part B

A nurse is assessing a client at risk for dehydration. Which statement by the client indicates that more education by the nurse is required?

a. “I try to limit coffee to one cup in the morning and one cup in the early evening.”

b. “During the day I drink at least six to seven glasses of water.” c. “Alcohol causes me to frequently urinate so I cut it out of my diet.” d. “I stop drinking fluids in the afternoon to avoid bathroom trips at

night.”

An older adult recently had a hysterectomy and has requested some medication for pain. The physician leaves an order for meperidine

(Demerol). Which action by the nurse is most appropriate? a. Assess the client’s pain 1 hour after giving the medication. b. Call the physician and request a different pain medication.

c. Assess the client’s respiratory rate often after administering the Demerol.

d. Ensure that the client does not receive iron supplements at the same time.

A nurse manager is planning a comprehensive care plan for older clients admitted to the medical-surgical unit. To decrease hospital stays and

lessen the pain that older clients experience, which standard intervention should the manager include in the care bundle for this population?

a. Assess all clients for depression. b. Obtain a dietary consult for nutrition assessment. c. Perform medication reconciliation on admission.

d. Screen all clients for alcohol and drug use.

MULTIPLE RESPONSE

What conditions predispose an older adult client to acute confusion or delirium? (Select all that apply.)

a. Alcoholism b. Chronic pain

c. Acute infection d. Electrolyte imbalances

e. Multi-infarct cerebrovascular disease f. Change in drug regimen

An older adult client has been admitted to a skilled nursing facility following surgery. What interventions should the nurse add to this

client’s care plan to assist with adjusting to this situation? ((Select all that apply.)

a. Make sure the client has hearing aids and glasses. b. Offer the anxiolytic that the physician has prescribed.

c. Encourage the family to bring in favorite pictures. d. Ask where the client wants the room furnishings placed.

e. Encourage the client to eat meals alone in his or her room. f. Set a daily schedule for the client that includes group activities.

The nurse is assessing several clients. Which clients does the nurse identify as being at high risk for falls? (Select all that apply.) The client:

a. With visual impairment such as presbyopia b. Who is reluctant to use a cane while walking

c. Who performs Tai Chi exercise daily d. Who wears a hearing aid and glasses

e. Who has difficulty arising from a sitting position f. Who is male and over 55 years of age

What interventions can the nurse apply to help an older adult client who is having trouble sleeping while in the hospital? (Select all that apply.)

a. Changing the client’s sheets each night before sleep b. Decreasing the level of light surrounding the client’s bed c. Attempting to keep the client awake during the daytime

d. Keeping staff conversations as quiet as possible e. Administering sleeping pills at night

f. Administering pain medication before bedtime g. Asking the client if he or she would like to pray

An 89-year-old is admitted to the medical-surgical floor. The nurse is formulating the client’s plan of care. In assessing the client, which

findings would be considered part of the clinical syndrome of frailty? (Select all that apply.) a. Increased appetite

b. Weight loss c. Weakness

d. Decreased sleep e. Slowed gait

Chapter 4: Cultural Aspects of Health and Illness

Test Bank

MULTIPLE CHOICE

The nurse is caring for a client from another culture. Which action would demonstrate the first step of developing cultural competence?

a. Avoiding assuming that members of the same culture all share the same beliefs and values

b. Developing an understanding of his or her own cultural heritage, feelings, and experiences

c. Becoming bilingual to communicate effectively with the population

of clients served d. Developing an understanding of the religious beliefs of clients served

by the nurse

The nurse is developing a teaching plan on cultural sensitivity for colleagues who are caring for clients from other cultures. What action by

the nurse might a client of another culture interpret as culturally insensitive?

a. Making sure that an Arab female client is covered during assessment

b. Avoiding making direct eye contact with a male American Indian client

c. Not encouraging a Japanese-American client to express feelings openly

d. Hugging an Egyptian-American female client who has received bad news

A Mexican-American client is insistent that her family members stay with her in her hospital room while she is recovering from surgery. What

will be the priority action of the nurse? a. Explain the policy of the hospital regarding visitors.

b. Ask the Spanish-speaking chaplain to help explain why this is unacceptable.

c. Arrange for additional beds to accommodate family members’

overnight stay. d. Notify the physician of the client’s anxiety and suggest a

prescription.

A nurse is caring for a lesbian client. What is a priority assessment question for this client?

a. “When was your last complete physical examination?” b. “How much alcohol do you consume?”

c. “Do you smoke?” d. “Do you use recreational drugs?”

The nurse is providing discharge instructions on medications to an older adult client. The client has expressed concern about paying the hospital

bills. What will the nurse emphasize on the basis of best cultural practice?

a. The client should be prepared to demonstrate filling the daily pill planner at the next appointment.

b. The client should keep a daily diary of side effects experienced from the medications.

c. The client must adhere to the medication regimen, unless the cost becomes prohibitive.

d. The client should ask the pharmacist for the generic version of medications to keep costs to a minimum.

What can health care agencies do to assist in meeting the cultural goals of Healthy People 2020?

a. Ensure accurate recording of a client’s race and/or ethnicity in the medical record.

b. Hire physicians and nurses from culturally diverse backgrounds. c. Provide magazines written in more than one language in waiting

areas of clinics. d. Improve access to health care by establishing clinics closer to low-

income housing.

The nurse is assessing a new home health client. What observation would be relevant for a cultural assessment?

a. The client asks the nurse how to obtain assistance with his utility bills.

b. The client’s wife reports that the children are having difficulty in school.

c. The client tells the nurse that he would like to move into public housing.

d. The client’s wife corrects the husband’s response to questions about his parents.

The client states, “I cannot receive blood transfusions, because it is against my beliefs. I am concerned about having this surgery.” What is

the nurse’s best response? a. “You should allow your health care team to do whatever is needed.”

b. “The blood supply in this country is the safest in the world.” c. “There really is nothing unacceptable about blood transfusion.”

d. “There are good alternatives to transfusions that we can discuss.”

A nurse is working with a new group of immigrants and wants to learn more about their culture. Which method of cultural assessment should the nurse perform to gain a long-term understanding of this culture?

a. Interview a client. b. Observe a group.

c. Participate in the community. d. Visit a group of clients.

The client expresses distress over missing religious services while in the hospital. What is the nurse’s best action?

a. Tell the client that he or she shouldn’t worry about it right now. b. Ask the client’s spouse or family member to pray with the client.

c. Ask the hospital’s professional chaplain to talk to the client. d. Encourage the client to reschedule procedures for another day.

The nurse is caring for a homeless client after surgery. Which statement by the nurse indicates the best understanding of this special population?

a. “When you receive your prescription, fill the medication at the nearest pharmacy.”

b. “To prevent the risk of infection, you need to bathe daily and keep the incision clean.”

c. “Tell me about your home situation and access to food and medications.”

d. “To help with healing, be sure to eat meals high in protein and low in fat and cholesterol.”

MULTIPLE RESPONSE

Which factors are included as part of Purcell’s domains for assessing cultural groups or persons? (Select all that apply.)

a. Nutrition and communication b. Family roles and organization

c. Elementary education d. Biofeedback

e. Death rituals and spirituality f. Alternative and complementary therapies

Which statement by the nurse would potentially offend a client in a predominantly Hispanic clinic? (Select all that apply.)

a. “Did you know that your child has the most beautiful eyes that I have ever seen?”

b. “So I can best treat you, could you share your beliefs on the hot/cold theory of healing?”

c. “Whom do you consider your primary source of healing and treatment?”

d. “How can you believe that eating ice cream will stop the bleeding after having a baby?”

e. “Why do you say that arthritis in older age is caused by childbirth as a young woman?”

Chapter 5: Pain: The Fifth Vital Sign

Test Bank

MULTIPLE CHOICE

The nurse is caring for a client who was medicated for pain 1 hour ago. The client states that the medication is not working and the pain is still

present. What is the first action that the nurse will take? a. Assess the client to determine a pain score.

b. Believe the client’s report of pain. c. Wait until it is time for the next pain medication dose.

d. Teach the client how to use guided imagery.

When is the nurse correct in decreasing the dose of pain medication in a client with end-stage cancer?

a. The spouse is worried that the client may become addicted. b. The client wants to remain alert during the visit of a long-time

friend. c. The client has lost considerable weight and does not want to eat.

d. The client is becoming combative at night.

A client with chronic pain is being discharged from the hospital. When planning the client’s pain relief regimen for home, it is most important for the nurse to communicate with which member of the health care

team? a. Advanced practice nurse b. Home health care nurse

c. Primary physician d. Psychologist

A client with arthritic pain is considering taking an herbal supplement to relieve arthritic pain. What teaching is most important for the nurse to

carry out with this client? a. Inform any health care providers about the use of this supplement.

b. Practice imagery along with taking the herbal supplement. c. Take only herbal supplements that are prescribed.

d. Take herbal supplement at the onset of pain.

What instruction should the nurse include in the discharge teaching plan of a client who has a transcutaneous electrical nerve stimulation (TENS)

unit? a. “Pain relief is sustained when stimulation is stopped.”

b. “The current is adjusted by the physician.” c. “The electrodes are placed away from the painful site.”

d. “You can perceive a pins and needles sensation.”

Why does the nurse always ask the client his or her pain level after taking routine vital signs?

a. To determine whether pain is influencing blood pressure and heart rate

b. To determine the need for more frequent vital sign measurement c. To ensure that pain assessment occurs on a regular basis d. To follow McCaffery’s guidelines on pain management

A client with cholecystitis has pain in the right shoulder area and asks, “What is happening to me? What did I do to my shoulder?” What is the

nurse’s best response? a. “You are weak from staying in bed.”

b. “Does your other arm hurt too?” c. “Sometimes pain from a certain organ is referred elsewhere in the

body.” d. “I am going to hold your medication until we can determine what is

happening.”

The nurse is assigned to care for the following four clients who have the potential for having pain. Which client is most likely not to be treated

adequately for this problem? a. Middle-aged woman with a fractured arm

b. Client with expressive aphasia c. Younger adult with metastatic cancer

d. Client who has undergone an appendectomy

The physician orders a dose of medication that does not resolve the client’s chronic pain. When the nurse questions the order, the physician

explains that he or she fears the client will develop an addiction with higher drug dosages. What is the nurse’s best response?

a. Administer the medication as ordered. b. Assist the client to use guided imagery. c. Consult with the pain control specialist.

d. Explain to the client that lower doses are better.

A client who has been taking oxycodone (OxyContin) for an extended period of time comes to the clinic reporting that the drug is no longer

relieving his pain. Which category would be given to the client’s complaint?

a. Addiction b. Physical dependence

c. Pseudoaddiction d. Tolerance

A home care client who is taking morphine for pain management abruptly stops taking the medication. Which symptom would indicate

physical dependence? a. Abdominal cramping b. Craving for morphine c. Decreased heart rate d. Elevated temperature

A home care client who is currently on hydromorphone (Dilaudid) for pain management presents to the hospital reporting abdominal

cramping, nausea, and sweating. When taking the client’s history, the nurse asks which question first?

a. “Are you currently in severe pain?” b. “Did you take more Dilaudid than prescribed?” c. “When did you take your last dose of Dilaudid?”

d. “When was your last bowel movement?”

The nurse is assessing a client with a long-term history of arthritic pain. Assessment reveals a heart rate of 115 beats/min and blood pressure of

170/80 mm Hg. Which intervention will the nurse carry out first? a. Administer blood pressure medication.

b. Administer a drug to lower the heart rate. c. Assess whether the client needs anti-arthritis medication.

d. Continue to assess for possible causes of elevated vital signs.

The nurse is caring for four clients who are reporting pain. Based on the following assessments and histories, which client’s pain is most likely

chronic in nature? a. Foley catheter inserted 30 minutes ago with a heart rate of 100

beats/min b. History of heart disease with a heart rate of 120 beats/min

c. History of fibromyalgia with a blood pressure of 110/70 mm Hg d. Hip replacement surgery with a blood pressure of 170/90 mm Hg

When a client is assessed, which behavior best indicates that he or she is experiencing changes associated with acute pain?

a. Anger and hostility b. Expressed hopelessness c. Inability to concentrate

d. Psychosocial withdrawal

The nurse anticipates that the client who rates pain as 10 on a scale of 1 to 10 has undergone which surgical procedure?

a. Cranial surgery b. Leg surgery

c. Neck surgery d. Upper abdominal surgery

Which assessment finding is cause for concern in a client who has taken 4 grams of acetaminophen (Tylenol) to relieve back pain?

a. Difficulty with urination b. Decreased respiratory rate c. Gastrointestinal bleeding

d. Increased liver function tests

During preoperative assessment, the client tells the nurse about taking NSAIDs for years. What question is most important for the nurse to ask?

a. “Did you ever have a problem with bleeding?” b. “Do you bruise easily?”

c. “How many tablets do you take every day?” d. “When was the last time you took your NSAID?

The client is taking an oxycodone-acetaminophen combination (Tylox) at home daily for chronic pain management. What instruction does the

nurse give this client? a. “Avoid taking aspirin while you are on this medication.”

b. “Drink plenty of water and eat foods high in fiber.” c. “Stop this medication after 3 days if the pain persists.”

d. “Weigh yourself daily to determine whether you are retaining sodium or water.”

The client was given 15 mg of morphine IM for postsurgical pain. When the nurse checks the client for pain relief 1 hour later, the client is

sleeping and has a respiratory rate of 10 breaths/min. What is the nurse’s first action?

a. Administering naloxone (Narcan) IV push b. Administering oxygen by nasal cannula

c. Arousing the client by calling his or her name d. Documenting the findings and continuing to monitor

The nurse accidentally administers 10 mg of morphine intravenously to a

client who had been given another dose of morphine, 5 mg IV, about 30 minutes earlier. What action must the nurse be prepared to take?

a. Administer naloxone (Narcan). b. Administer oxygen.

c. Assist with intubation. d. Monitor pain level.

A client is admitted to the hospital with a history of oxycodone (Percodan) abuse. For which clinical manifestations does the nurse

observe the client? a. Anorexia and weight loss

b. Decreased heart rate and respirations c. Muscle twitching and profuse perspiration

d. Sedation and constipation

Which client would the nurse suggest should try subcutaneous opioid analgesia for pain management?

a. Client who has had a surgical procedure b. Client with back pain who likes to walk

c. Client with cancer who is nauseous d. Client experiencing acute chest pain

A client with colon cancer is discharged to home with morphine for pain management. He is having episodes of nausea and vomiting. Which route

of morphine administration would be most advantageous to use? a. Oral

b. Rectal c. Intravenous

d. Intramuscular

The nurse is caring for four clients. Which client assessment is the most indicative of having pain?

a. Blood pressure 150/70 mm Hg and sleeping b. Client stating that he is “anxious”

c. Heart rate of 105 beats/min and restlessness d. Postoperative client with a neck incision

A client has a history of alcohol abuse. Which pain relief regimen does the nurse anticipate if morphine (MS Contin) is given for pain?

a. A higher dose of opioids will be needed to provide effective pain relief.

b. A lower dose of opioids will be needed to provide effective pain relief.

c. The appropriate drug selection is an opioid agonist-antagonist combination.

d. The client will receive no pain relief from the morphine.

Which instruction is the most accurate for the nurse to give a client who has a patient-controlled analgesia device (PCA) after abdominal

surgery? a. “Instruct your visitors to press the button for you when you are

sleeping.” b. “Push the button every 15 minutes whether you feel pain at that

time or not.” c. “Push the button when you first feel pain instead of waiting until

pain is severe.” d. “Try to go as long as you possibly can before you press the button.”

The nurse assesses several postoperative clients receiving patient- controlled epidural analgesia (PCEA). Which client does the nurse

prioritize to assess first? a. Client receiving bupivacaine (Marcaine) describing “inability to

move legs” b. Client receiving fentanyl (Sublimaze) describing “itchy arms”

c. Client receiving hydromorphone (Dilaudid) describing “full feeling”

d. Client receiving morphine describing “difficulty staying awake”

A client has epidural analgesia with bupivacaine (Marcaine) for pain relief. For which condition should the nurse assess this client?

a. Extremity itching b. Inability to raise legs off the bed

c. Nausea and vomiting d. Respiratory rate of 8 breaths/min

When assessing a client who is taking long-term ibuprofen (Motrin) for pain, the nurse finds numerous areas of bruising. What is the nurse’s

first action? a. Assess for gastric discomfort.

b. Assess for the presence of pain. c. Continue to monitor bruising.

d. Place client on falls precaution.

Which statement made by a nurse represents the need for further education regarding pain management in older adult clients?

a. “Older adults are at greatest risk for undertreated pain.” b. “Older adults tend to report pain less often than younger adults.”

c. “Older clients usually have more experience with pain than younger clients.”

d. “Older clients have a different pain mechanism and do not feel it as much.”

Before surgery, the nurse observes the client listening to music on the radio. Based on this observation, the nurse may try which

nonpharmacologic intervention for pain relief in the postoperative setting?

a. Cutaneous skin stimulation b. Hypnosis c. Imagery

d. Radiofrequency ablation

A client who is at the end of life is being given morphine for pain management. The family expresses concern that the morphine may cause

the client to stop breathing and die. What is the nurse’s best response? a. “He needs the morphine to prevent pain.”

b. “His respirations are not affected by the morphine.” c. “We will decrease the dose if his breathing slows.”

d. “We will give him oxygen to help with his breathing.”

A client is stating that he has the sensation of burning, aching, and dullness. Which afferent nerve fibers should be transmitting the pain?

a. A delta fibers b. C fibers

c. A alpha fibers d. A beta fibers

A client is postoperative day one and has a patient-controlled analgesia (PCA) pump with a continuous basal dose for pain control. Currently,

the client is stating that the operative pain is escalating. What is the first action of the nurse?

a. Try diversion to take the client’s mind off the pain. b. Ask the client to ambulate around the unit.

c. Assess the client’s pain according to PQRST. d. Call the physician to request an order to increase the basal dose.

Which client does the nurse assess first for pain control?

a. Older client with chronic rheumatoid arthritis b. Client postoperative day three walking in the hallway

c. Sleeping client with an epidural pump d. Quiet client with pancreatic cancer curled up in bed

MULTIPLE RESPONSE

Which is most indicative of pain in an older client who is confused? (Select all that apply.)

a. Decreased blood pressure b. Screaming

c. Facial grimace d. Restlessness

e. Crying f. Decreased respirations

An older client just returned from surgery and is rating pain as “8” on a 0 to 10 scale. Which medications are unsafe choices for treatment of

severe pain in this older adult? a. Meperidine (Demerol)

b. Methadone (Dolophine) c. Propoxyphene (Darvocet)

d. Morphine (Durmorph) e. Codeine

Chapter 6: Genetic Concepts for Medical-Surgical Nursing

Test Bank

MULTIPLE CHOICE

Which statement by the nurse indicates correct understanding of the purpose of a pedigree?

a. “It is used for genetic counseling of the client by the geneticist.” b. “It is used to show family history of a trait over at least three

generations.” c. “It is used to show a specific pattern of inheritance of a trait.”

d. “It is used to identify penetrance of a gene in a family.”

The client exhibits a trait that has appeared in every generation of his or her family. This is an example of which type of inheritance?

a. Autosomal recessive b. Sex-linked recessive c. Autosomal dominant d. Sex-linked dominant

The client has a family history of breast cancer. The physician has recommended that she undergo genetic testing. What action is most important for the nurse to take before scheduling the client for the

procedure? a. Making certain the client is prepared for the risk of psychological

side effects b. Obtaining informed consent from the client and placing it on the

chart c. Simultaneously scheduling genetic counseling with an advanced

practice nurse d. Carefully explaining the procedure to the client and assuring her

confidentiality

The client undergoes genetic testing but chooses not to be told the results of the testing once it is completed. Which action is most important for the

nurse to implement? a. Encourage the client to ask for the results of the genetic testing. b. Share the results of the testing with the client’s family members.

c. Explain to the client how this choice may affect other family members.

d. Respect the client’s right to not know the results of the testing.

The nurse has been working with a client who has asked to receive the results of genetic testing. What will the nurse do before discussing the

results with the client? a. Obtain a signed and witnessed informed consent form and place it

on the client’s chart. b. Assess the client’s ability to communicate clearly with the nurse and

other personnel. c. Reassure the client that it is not necessary to inform other family

members of the test results. d. Encourage the client to agree to undergo several sessions of further

counseling.

Which client needs to undergo carrier genetic testing? a. A middle-aged man whose father died at age 48 of colorectal cancer b. A young woman who has all the symptoms of rheumatoid arthritis

c. A middle-aged woman whose mother died at age 52 of breast cancer d. A young woman of Eastern European Jewish ancestry

The client has just completed the first session of genetic counseling. Which intervention will the nurse perform next?

a. Assessing the phase of the grieving process most applicable to the client at this time

b. Asking the client to explain the various terms used in the discussion c. Determining whether the client has adequate coping methods to deal

with the counseling process d. Asking the client to explain expectations and how they may have

changed after the session

The client has just completed genetic testing and received a negative result. The client tells the nurse that he feels guilty because so many of his family members are carriers of a genetic disease but he is not. What

is the nurse’s best response?

a. Make certain that the client recognizes that although he is not a carrier of the disease, he could still be symptomatic.

b. Encourage the client to undergo a second round of testing to verify that the result was accurate.

c. Arrange for the client to undergo counseling and offer support to him during this time.

d. Emphasize to the client the importance of revealing his test results to other family members.

A client is not certain whether she and her family should participate in a genetic screening plan. She asks the nurse why the X-linked recessive

disorder that has been noted in some of her family members is expressed in males more frequently than in females. What is the nurse’s best

response? a. “The disease tends to show up in males because they don’t have a

second X chromosome to balance expression of the gene.” b. “One X chromosome of a pair is always inactive in females. This

inactivity effectively negates the effects of the gene.” c. “Females are known to have more effective DNA repair mechanisms

than males, thus negating the damage caused by the recessive gene.” d. “Expression of genes from the male’s Y chromosome does not occur in

females, so they are essentially immune to the effects of the gene.”

The client has been found to have a genetic mutation that increases the risk for colon cancer. The client does not want any family to know about

this result. What is the nurse’s best response? a. “It is required by law that you inform your siblings and your

children about this result so that they also can be tested and monitored for colon cancer.”

b. “It is not necessary to tell your siblings because they are adults, but you should tell your children so that they can be tested before they decide

to have children of their own.” c. “It is not required that you tell anyone about this result. However, because your siblings and children may also be at risk for colon cancer,

you should think about how this information might help them.” d. “It is your decision to determine with whom, if anyone, you discuss this test result. However, if you do not tell any of your family members

and they get colon cancer, you could be held liable.”

Which response is accurate regarding a client who has type O blood? The client has:

a. A genotype of AO b. A genotype of OO

c. Heterozygous alleles d. A different genotype and phenotype

Which statement regarding genotype and phenotype is accurate? a. For autosomal recessive traits, the phenotype is the same as the

genotype. b. The only trait in which phenotype always follows genotype is

physiologic gender. c. When a phenotype is fully penetrant, the trait is expressed in the

heterozygous person. d. Genotype changes as a person ages, whereas phenotype is not

affected by the aging process.

Which statement best describes the concept of multifactorial inheritance?

a. A mutation in a single gene results in the expression of problems in a variety of tissues and organs.

b. Susceptibility to a problem is inherited as a single gene trait, but

development of the problem is enhanced by environmental conditions. c. A mutated gene is inherited, but the results of expression of that

gene are not evident until middle or late adulthood. d. Several genes are responsible for the mechanism of hearing, and a

mutation in any one of them results in hearing impairment.

A client’s father has hemochromatosis. Which type of genetic testing would be the primary type that is most appropriate for the client to

have? a. Presymptomatic b. Predisposition

c. Diagnostic d. Carrier

Which disorder qualifies a client for presymptomatic diagnostic genetic testing?

a. Colorectal cancer b. Huntington disease

c. Hemophilia d. Tay Sachs disease

Which disorder presents a need for predisposition genetic testing? a. Huntington disease b. Sickle cell disease

c. Hemophilia d. Breast cancer

The client, whose mother has Huntington disease, is considering genetic testing but is not sure whether she really wants to know the results. She asks what the nurse would do in her situation. What is the nurse’s best

response? a. “I would have the test so I could decide whether to have children or

to adopt children.” b. “I can only tell you the benefits and the risks of testing. You must

make this decision yourself.” c. “Because there is no cure for this disease and testing would not be

beneficial, I would not have the test.” d. “You need to check with your brothers and sisters to determine

whether testing for this disease would be appropriate for you.”

The client, who has been found to have a mutation in the BRCA1 gene

allele and to be at increased risk for breast and ovarian cancer, has asked the nurse to be present when she discloses this information to her adult

daughter. What is the nurse’s role in this situation? a. To act as the primary health care provider

b. To function as a genetic counselor c. To serve as a client advocate

d. To provide client support

After genetic testing, a client is found to have a specific mutation in the a1AT gene (alpha1-antitrypsin). What is the best action for the nurse to

take to guide the client? a. Advise scheduling an annual mammogram and ovarian ultrasound. b. Assess whether close family members have other identified genetic

problems. c. Suggest limiting exposure to secondhand smoke and other

respiratory irritants. d. Advise that cancer may be a risk but not a certainty for this

mutation.

Which client response best indicates that the client has concerns about a genetic link to a specific condition?

a. “Heart disease in women seems to be a growing concern in the

United States.” b. “Obesity is prevalent in the elementary school age population.”

c. “My grandmother died of both breast and bone cancer at age 50.” d. “Both my aunt and my second cousin have osteoarthritis.”

MULTIPLE RESPONSE

What client diagnosis indicates a need for carrier genetic testing? (Select all that apply.)

a. Colorectal cancer b. Huntington disease c. Sickle cell disease

d. Hemophilia e. Breast cancer f. Cystic fibrosis

g. Tay Sachs disease

The client has just been typed and crossmatched for a unit of blood. Which statements by the nurse indicate a need for further genetic

education? (Select all that apply.) a. “The client can receive any unit of blood because all blood types are

basically the same.” b. “Blood type is formed from three possible gene alleles: A, B, and O.” c. “Each blood type allele is inherited from the mother or the father.” d. “If the client’s blood type is AB, then the client is homozygous for

that trait.” e. “If the client has a dominant and a recessive blood type allele, only

the dominant will be expressed.”

In the hospital, the nurse is caring for a client of Asian descent who was just started on warfarin (Coumadin). What would be the best plan of

care for the client? (Select all that apply.) a. Start warfarin at a high dose to decrease the chance for further

clotting problems. b. Monitor international normalized ratio (INR) once a day in the

hospital. c. Teach the client to frequently check for any bruising. d. Initiate fall precautions and strict activity limitations.

e. Start warfarin at a lower-than-normal dose owing to slower metabolism of the drug.

A client recently underwent genetic testing that revealed that she has a BRCA1 gene mutation for breast cancer. What are the best actions of the

nurse? (Select all that apply.) a. Encourage genetic counseling for self and family.

b. Disclose the information to the medical insurance company. c. Recommend self–breast examination every week.

d. Assess the client’s response to the test results. e. Aid in making a plan for prevention and risk reduction.

Chapter 7: Evidence-Based Practice in Medical-Surgical Nursing

Test Bank

MULTIPLE CHOICE

The cardiac nurse wants to know about the best practices to prevent pneumonia after open-heart surgery. What does the nurse do first?

a. Critically appraise relevant evidence. b. Implement acceptable recommendations.

c. Ask clinical experts for their opinions. d. Search for evidence to answer the question.

The nurse is identifying clinical practice problems on a cardiac unit. What question is a foreground question?

a. “What is the pathophysiology of congestive heart failure?” b. “How does smoking affect the internal lumens of arteries? c. “What is the best treatment for a myocardial infarction?

d. “How are a client’s vital signs affected by anxiety?”

The nursing student asks, “What is the difference between qualitative and quantitative clinical questions?” What is the nurse’s best response?

a. “Qualitative questions utilize a strict statistical analysis of information.”

b. “Quantitative questions identify relationships between measurable concepts.”

c. “Qualitative questions ask about associations among defined phenomena.”

d. “Quantitative questions analyze the content of what a person says or does.”

The nurse is assessing the following PICO(T) question: In a 60-year-old woman with osteoarthritis, can a COX-2 inhibitor decrease the risk of gastrointestinal bleeding compared with other NSAIDs? What is the

comparison component in this question? a. Osteoarthritis

b. COX-2 inhibitor c. NSAIDs

d. Gastrointestinal bleeding

The nurse is looking for the best interventions for postoperative pain control. When are the facility’s policies and procedures an appropriate

source of evidence? a. When policies are based on high-quality clinical practice guidelines

b. When evidence is derived from a valid and reliable quantitative research study

c. When procedures originated from opinions of the facility’s chief surgeon

d. When evidence is founded on recommendations from experienced nurses

The nurse researcher is evaluating clinical questions. Which is a qualitative question?

a. What factors affect clients’ responses to postoperative pain? b. Do wound vacuum systems improve surgical wound healing time?

c. What are the effects of hourly rounding on client fall rates? d. Do chlorhexidine swabs decrease central line site infections?

The nurse is searching for evidence related to a PICOT question. What source provides the best evidence?

a. Medline database b. Cochrane library

c. CINAHL database d. Library of Congress

A hospice nurse routinely uses Therapeutic Touch to promote comfort. A current client demonstrates anxiety when this intervention is offered.

What is the nurse’s best response? a. Provide pain medication to manage the client’s comfort and pain. b. Continue with the intervention because it has worked with other

clients. c. Search for alternative interventions to better meet the client’s needs.

d. Share research that supports Therapeutic Touch with the client.

The health care facility is implementing a new evidence-based nursing protocol. What is necessary to ensure successful implementation?

a. Tools to evaluate the protocol are valid and reliable. b. Support from the nurses implementing the protocol is present. c. Recommendations for the protocol are obtained from senior

administrators.

d. The evidence-based protocol is cost-effective for the facility.

MULTIPLE RESPONSE

The nurse is developing a clinical question in a PICOT format. What components are included in the question? (Select all that apply.)

a. Population b. Comparison c. Observation d. Intervention e. Technique

The intervention component of an evidence-based question pertains to the therapeutic effectiveness of a treatment. Which are possible types of

interventions? (Select all that apply.) a. Exposure to disease b. A high-risk behavior

c. Age, gender, or ethnicity d. A prognostic factor

e. A client response

A nurse who wants to incorporate evidence-based practices into client care on a medical unit is meeting resistance. What barriers prevent

nurses from engaging in evidence-based practices? (Select all that apply.) a. Difficulty accessing research materials b. Difficulty understanding client needs c. Lack of value for research in practice d. Lack of value for client preferences

e. Inadequate available time f. Inadequate nurse-client ratios

Chapter 8: Rehabilitation Concepts for Chronic and Disabling Health Problems

Test Bank

MULTIPLE CHOICE A paraplegic client is being evaluated for transfer to a rehabilitation unit.

The nurse refers the client to which interdisciplinary team member for

evaluation of activities of daily living? a. Physical therapist

b. Occupational therapist c. Recreational therapist d. Vocational therapist

The nurse is teaching a client who is a paraplegic about prescribed rehabilitation. The client verbalizes that he doesn’t know why he should

go. What is the nurse’s best response? a. “Your doctor ordered rehabilitation, and he does know what is best

for you.” b. “When new discoveries are made, people in rehabilitation programs

benefit first.” c. “Rehabilitation will teach you how to maintain the functional ability

you have.” d. “You are right. It will not benefit you. I will cancel the orders for

rehabilitation.”

The nurse is caring for a client who has long-standing chronic obstructive pulmonary disease (COPD) and is recovering from a stroke.

Which intervention is a priority when activity tolerance is assessed during rehabilitation?

a. Assess vital signs before, during, and after activity. b. Perform a daily cognitive assessment.

c. Consult physical therapy to ambulate the client. d. Monitor the client’s progress in self-care ability.

A client with a past history of angina had a total knee replacement. What will the nurse teach the client before rehabilitation activities are begun?

a. “Use analgesics even if you are not in pain.” b. “Take nitroglycerin prophylactically before beginning activity.”

c. “Take anti-inflammatory medications before exercising.” d. “Do not exercise if you have knee pain.”

The rehabilitation nurse is caring for an obese client with new bilateral leg amputations. The nurse is planning to move the client from the bed to

the chair. What is the best approach? a. Use the bear-hug method to transfer the client safely. b. Ask several members of the health care team to assist. c. Utilize the facility’s mechanical lift to move the client.

d. Consult physical therapy before performing all transfers.

The nurse is caring for a client in a rehabilitation center. Which test will best assist the nurse in determining the severity of a client’s disability?

a. Instrumental activities of daily living (IADL) b. Minimum data set (MDS)

c. Functional independence measure (FIM) d. Independent living skills test (ILST)

The nurse is planning care for a client who is newly wheelchair bound owing to a spinal cord injury. What priority intervention should the

nurse include in the plan of care to assist the client in transferring from the bed to the wheelchair?

a. A diet high in protein and low in calories b. An occupational therapy consult

c. Bowel and bladder retraining d. Upper arm strengthening exercises

The nurse is performing passive range-of-motion exercises on a semiconscious client and meets resistance while attempting to extend the

right elbow more than 45 degrees. What action by the nurse is best? a. Splint the joint and continue passive range of motion to the shoulder

only. b. Progressively increase joint motion 5 degrees beyond resistance each

day. c. Apply weights to the right distal extremity before initiating any joint

exercise. d. Continue to move the joint only to the point at which resistance is

met.

The nurse is caring for a client with decreased mobility. What intervention should the nurse include in the care plan to best help this

client decrease the risk of fracture? a. Apply shoes to improve foot support.

b. Perform weight-bearing activities. c. Increase calcium-rich foods in the diet.

d. Use pressure-relieving devices.

The nurse assesses a client admitted for rehabilitation. The client has

generalized weakness and needs assistance with activities of daily living. Which exercise should the nurse implement?

a. Passive range of motion b. Active range of motion

c. Resistive range of motion d. Aerobic exercise

The nurse is caring for a bedridden client. Which intervention will the nurse implement to prevent pressure ulcer formation?

a. Adjust nutritional intake based on serum albumin levels. b. Measure the ulcer diameter and depth every shift.

c. Change the gauze dressing whenever drainage is observed. d. Apply antibiotic ointment to all excoriated skin areas.

The nurse is caring for a client who is undergoing rehabilitation. Which nursing intervention would be best to prevent venous stasis and

thrombus formation? a. Range-of-motion exercises b. Foot support while in bed

c. Increased dietary calcium intake d. Avoidance of sudden position changes

The nurse is caring for a rehabilitation client. Which activity plan should the nurse implement to best conserve a client’s energy without

compromising physical or mental health? a. Reduce hygiene activities and restrict visitors.

b. Ensure that the client toilets before and after planned activities. c. Schedule energy-intensive activities when energy levels are high. d. Schedule as many activities as possible in a small block of time.

A nurse catheterizes a client immediately after voiding. The residual volume is 50 mL. What will the nurse do next?

a. Notify the physician. b. Insert an indwelling catheter.

c. Document the finding in the chart. d. Modify the bladder training program.

The client who is performing intermittent self-catheterization at home is concerned about the cost of the catheters. What is the nurse’s best

response? a. “I will try to find out whether you qualify for money to purchase

these necessary supplies.” b. “Even though it is expensive, the cost of taking care of urinary tract

infections would be even higher.” c. “Instead of purchasing new catheters, you can boil the catheters and

reuse them up to 10 times each.” d. “You can reuse the catheters at home. Clean technique, rather than

sterile technique, is acceptable.”

The nurse is providing education for a client at risk for urinary tract infection. Which beverage should the nurse encourage the client to

drink? a. Carbonated beverages

b. Citric juices c. Milk

d. Tomato juice

The nurse delegates the unlicensed nursing assistant (UAP) to ambulate an older adult client. What information must the nurse communicate to

the UAP when delegating this task? a. “The client has skid-proof socks, so there is no need to use your gait

belt.” b. “Teach the client how to use her walker while you are ambulating up

the hall.” c. “Sit the client on the edge of the bed with legs dangling before

ambulating.” d. “Ask the client if she needs pain medication before you walk her in

the hall.”

The nurse is obtaining an admission history of a client with hip problems. The client asks, “Why are you asking about my bowels and

bladder?” What is the nurse’s best response? a. “To plan your care based on your normal elimination routine.”

b. “So we can help prevent side effects of your medications.” c. “We need to evaluate your ability to function independently.” d. “To schedule your activities around your elimination pattern.”

The nurse is planning care for a client who is beginning a structured cardiac rehabilitation program. Before starting the program, what

should the nurse do first? a. Administer nitroglycerin to increase blood flow to the heart.

b. Assess the client for orthostatic hypotension. c. Start oxygen at 2 L/min via nasal cannula.

d. Determine the level of activity before shortness of breath occurs.

The nurse is caring for a client with a spinal cord injury at level T3. How will the nurse assist the client with bladder dysfunction?

a. Insert an indwelling urinary catheter. b. Stroke the medial aspect of the thigh.

c. Use the Credé maneuver every 3 hours. d. Apply a Texas catheter with leg bag.

A client with a flaccid bladder is undergoing bladder training. The nurse begins the client’s bladder training using which technique?

a. Stroking the medial aspect of the thigh b. Valsalva maneuver c. Self-catheterization d. Frequent toileting

The nurse is caring for a client after a stroke. The client has a right facial droop, drools continuously, and chokes on her own saliva. What

rehabilitation team member should the nurse consult to ensure client safety?

a. Speech-language pathologist b. Nutritionist

c. Rehabilitation case manager d. Cognitive therapist

MULTIPLE RESPONSE

The nurse collaborates with a physical therapist when providing care for a rehabilitation client. The role of the physical therapist is to help the

client with which activities? (Select all that apply.) a. Achieve mobility.

b. Attain independence with dressing. c. Use a walker in public.

d. Learn techniques for transferring. e. Perform activities of daily living.

f. Complete job training.

An older adult client tells the nurse, “I tire easily.” Which activities best assist the client to conserve energy? (Select all that apply.)

a. Perform all tasks in the morning. b. Take frequent rest periods.

c. Gather all supplies needed for a chore. d. Use a cart, bag, or tray to carry items. e. Push objects rather than lifting them.

f. Break large activities into smaller parts. g. Hire someone to assist with chores.

The nurse is caring for a client who is wheelchair bound. Which interventions will the nurse implement to prevent skin breakdown?

(Select all that apply.) a. Change the client’s position every 1 to 2 hours.

b. Place pillows under the client’s heels. c. Have the client do wheelchair pushups.

d. Remove the client’s shoes to check for pressure areas. e. Assess the client’s lower legs for pressure from the wheelchair.

f. Massage the client’s calves and feet with lotion.

The nurse is caring for a client with a disabling condition. Which abnormal findings would alert the nurse to an increased risk for skin

breakdown? (Select all that apply.) a. Low serum albumin level

b. High serum transferrin level c. Low serum carboxyhemoglobin

d. High serum hematocrit e. Increased weight gain

f. Incontinence g. Poor fluid intake

The nurse is caring for a client with left-sided weakness. Which gait- training techniques will the physical therapist and the nurse use when

assisting the client to walk with a cane? (Select all that apply.) a. Place the cane in the client’s left hand.

b. Hold the cane with the client’s stronger hand. c. Move the cane forward, followed by legs stepping forward.

d. Take one step forward, followed by the cane moving forward. e. Step forward with the stronger leg, then the weaker leg.

f. Move the weaker leg one step forward, followed by the stronger leg.

The nurse is implementing nutritional changes to reduce the risk for skin breakdown in a client with impaired physical mobility. Which dietary

modifications will the nurse reinforce? (Select all that apply.) a. High-protein b. Low-protein

c. High-carbohydrate d. Low-carbohydrate

e. High-fat f. Low-fat

Chapter 9: End-of-Life Care

Test Bank

MULTIPLE CHOICE

The client tells the nurse that even though it has been 4 months since her sister’s death, she frequently finds herself crying uncontrollably. The client is afraid that she is “losing her mind.” What is the nurse’s best

response? a. “Most people move on within a few months. You should see a grief

counselor.” b. “Whenever you start to cry, distract yourself from thoughts of your

sister.” c. “You should try not to cry. I’m sure your sister is in a better place

now.” d. “Your feelings are completely normal and may continue for a long

time.”

The nurse is discussing advance directives with a client. Which statement by the client indicates good understanding of the purpose of an advance

directive? a. “An advance directive will keep my children from selling my home

when I’m old.” b. “An advance directive will be completed as soon as I’m

incapacitated and can’t think for myself.” c. “An advance directive will specify what I want done when I can no

longer make decisions about health care.” d. “An advance directive will allow me to keep my money out of the

reach of my family.”

The nurse is caring for a client who is considering being admitted to hospice. What is the nurse’s best response?

a. “Hospice admission has specific criteria. You may not be a viable candidate, so we will look at alternative plans for your discharge.”

b. “Hospice care focuses on a holistic approach to health care. It is designed not to hasten death, but rather to relieve symptoms.”

c. “Hospice care will not help with your symptoms of depression. I will refer you to the facility’s counseling services instead.”

d. “You seem to be experiencing some difficulty with this stage of the grieving process. Let’s talk about your feelings.”

A hospitalized American Indian client is approaching death. Family members who are standing vigil in the client’s room begin to divide up

his possessions among themselves as his symptoms progress. What is the nurse’s most important intervention?

a. Ask the family members to step outside the room so the client cannot hear them.

b. Tell the family that they are being insensitive and their behavior is inappropriate.

c. Recognize that this is a culturally appropriate activity and document it in the chart.

d. Report these activities to the client’s physician and the nursing supervisor.

The spouse of a dying client states that she is concerned that her husband is choking to death. What is the nurse’s best response?

a. “Do not worry. The choking sound is normal during the dying process.”

b. “I will administer more morphine to keep your husband comfortable.”

c. “I can ask the respiratory therapist to suction secretions out through his nose.”

d. “I will have another nurse assist me to turn your husband on his side.”

The terminally ill client is prescribed morphine to help cope with increasing discomfort. A family member expresses concern that the client

is on “too much morphine.” What is the nurse’s best response? a. “What has the physician told you about your family member’s

illness?” b. “Don’t worry about that. We’re following the physician’s plan of

care.”

c. “Tell me more about what you mean by too much morphine.” d. “You should talk with your physician about this when he makes

rounds.”

The nurse is teaching a family member about various types of complementary therapies that might be effective for relieving the dying client’s anxiety and restlessness. Which statement by the family member

indicates understanding of the nurse’s teaching? a. “Maybe we should just hire a round-the-clock sitter to stay with

Grandmother.” b. “I have some of her favorite hymns on a CD that I could bring for

music therapy.” c. “I don’t think that she’ll need pain medication along with her herbal

treatments.” d. “I will burn therapeutic incense in the room so we can stop the

anxiety pills.”

A terminally ill client has just died in a hospital setting with family members at the bedside. The health care provider is also present. What should be the nurse’s priority intervention as postmortem care begins? a. Call for emergency assistance so that resuscitation procedures can

begin. b. Ask the family members if they would like to spend time alone with

the client.

c. Ensure that a death certificate has been completed by the physician. d. Request family members to prepare the client’s body for the funeral

home.

The nurse is providing care for a hospice client who is in the last stages of the dying process. The client develops a pressure ulcer on her sacrum,

and family members tell the nurse that they would like a specialist consulted to treat the ulcer. When the nurse discusses this with the client, the client states that the ulcer does not bother her, that it is not causing

her pain, and that she’d rather not have additional caregivers at this time. What should the hospice nurse do next?

a. Tell the family the wound care specialist will be consulted and treatment will begin.

b. Ask the social worker and the chaplain to talk with family members about the dying process.

c. Explain the client’s desires to the family, emphasizing that the client will be made as comfortable as possible.

d. Ask the agency mental health nurse to speak with the client about refusing treatment.

The nurse is being trained in hospice care. Which intervention by the nurse is most compatible with the goals of end-of-life care for the client?

a. Administer influenza and pneumococcal vaccinations. b. Prevent the client with chronic obstructive pulmonary disease from

smoking. c. Perform passive range-of-motion exercises to prevent contractures.

d. Permit the client with diabetes mellitus to have a serving of ice cream.

The nurse is assessing the dying client. Which manifestations of a dying client should the nurse assess to determine whether the client is near

death? a. Level of consciousness

b. Respiratory rate c. Bowel sounds

d. Pain level on a 0 to 10 scale

The wife is concerned because her terminally ill husband does not want to eat. What is the nurse’s best response?

a. “Let him know that food is available if he wants it, but do not insist that he eat.”

b. “A feeding tube can be placed in the nose to provide important nutrients.”

c. “Force him to eat even if he does not feel hungry, or he will die sooner.”

d. “He is getting all the nutrients he needs through his intravenous catheter.”

The family members of a client with a terminal illness tell a nurse that the client keeps asking if she is dying. What is the nurse’s best response?

a. “Whenever she asks about dying, change the subject.” b. “Tell her the truth in as gentle a way as possible.”

c. “Tell her that she will get better eventually.” d. “Ask her if she is afraid to die.”

The client’s family members are concerned that the client should have a urinary catheter placed because of her decreasing urinary output. What

is the hospice nurse’s best response? a. “A Foley catheter is inserted only if she is taking medications that

affect output.” b. “I will insert a Foley catheter if her urinary output drops below 500

mL/day.” c. “A Foley catheter will be inserted if her bladder becomes

distended.” d. “I will insert a Foley catheter if she becomes incontinent of urine.”

The health care provider suggests inpatient hospice for a client. The

family members are concerned that their loved one will receive only custodial care. What is the nurse’s best response?

a. “The goal of palliative care is to provide the greatest degree of comfort possible and help the dying person enjoy whatever time is left.” b. “Palliative care will release you from the burden of having to care for someone in the home. It does not mean that curative treatment will

stop.” c. “A palliative care facility is like a nursing home and costs less than a

hospital because only pain medications are given.” d. “Your relative is unaware of her surroundings and will not notice

the difference between her home and a palliative care facility.”

A dying client’s family members are spending time with the client. What instruction is best to give to family members regarding noise in the

client’s room? a. “Remember that she cannot hear you.”

b. “Try to get her to talk or respond to you.” c. “Avoid making any noise when you are with her.”

d. “Talk in your normal speaking voice.”

A client who is near death appears to be having difficulty breathing. What is the nurse’s highest-priority intervention?

a. Teach the family how to perform nasotracheal suctioning. b. Request that the physician order morphine sulfate.

c. Document the finding in the client’s chart. d. Call a respiratory therapist to intubate the client.

The nurse is caring for a dying client who becomes very agitated. What is the nurse’s best response?

a. Use music therapy to promote relaxation. b. Increase the dose of intravenous opioids.

c. Provide a second antipsychotic medication. d. Assess the client for urinary retention.

An experienced hospice nurse is training a new nurse in the practices of palliative care. What statement by the new nurse indicates

understanding about drug therapy for end-of-life care? a. “I can administer as much pain medication as I want because the

client is dying.” b. “The administration of these medications will hasten the client’s

death.” c. “I can administer medication per the protocol to relieve the client’s

symptoms.” d. “The purpose of palliative sedation is to relieve family members’

distress.”

An older client was admitted to hospice owing to impending death in approximately 6 weeks. After 2 months, the family remains at the

bedside but is becoming increasingly impatient and irritable. What is the best nursing intervention?

a. Ask the family to leave and not return until they are calmer. b. Sit with the family and listen to their concerns and fears.

c. Tell the family members not to worry, the client will die soon. d. Consult the chaplain to come and pray with the client’s family.

An intensive care nurse is discussing withdrawal of care with a client’s family. The family expresses concerns related to discontinuation of

therapy. What is the nurse’s best response? a. “I understand your concerns, but in this state, discontinuation of

care is not a form of active euthanasia.” b. “You will need to talk to the provider because I am not legally

allowed to participate in the withdrawal of life support.” c. “I realize this is a difficult decision. Discontinuation of therapy will

allow the client to die a natural death.” d. “There is no need to worry. Most religious organizations support the

client’s decision to stop medical treatment.”

MULTIPLE RESPONSE

The hospice nurse is caring for a dying client and her family members. What nursing interventions are appropriate to use? (Select all that

apply.) a. Teach family members about physical signs of impending death.

b. Encourage the management of adverse symptoms. c. Assist family members by offering an explanation for their loss.

d. Encourage reminiscence by both client and family members. e. Avoid spirituality because the client’s and the nurse’s beliefs may

not be congruent. f. Do not encourage hope for the terminally ill client.

The nurse is providing care for a dying client. The nurse would place highest priority on treating which symptoms? (Select all that apply.)

a. Anorexia b. Weight loss

c. Pain d. Agitation e. Nausea

f. Hair loss g. Dyspnea

The nurse is admitting a new client to the hospital and needs to determine the plan of care. What criterion is required for the client to

make his own medical decisions? (Select all that apply.) a. Can communicate his treatment preferences b. Is able to read and write at an 8th grade level

c. Is oriented enough to received information d. Can evaluate and deliberate information

e. Has completed an advance directive

Chapter 10: Concepts of Emergency and Trauma Nursing

Test Bank

MULTIPLE CHOICE

While assessing a client in the emergency department, the nurse identifies that the client has been raped. Which health care team member

should the nurse collaborate with when planning this client’s care?

a. Emergency medicine physician b. Case manager

c. Forensic nurse examiner d. Psychiatric crisis nurse

On admission to the emergency department, a client states that he feels like killing himself. When planning this client’s care, it is most important for the nurse to coordinate with which member of the health care team?

a. Case manager b. Forensic nurse examiner

c. Physician d. Psychiatric crisis nurse

The emergency department team is performing cardiopulmonary resuscitation on a client when the client’s spouse arrives at the

emergency department. What should the nurse do next? a. Request that the client’s spouse sit in the waiting room.

b. Ask the spouse if he wishes to be present during the resuscitation. c. Suggest that the spouse begin to pray for the client.

d. Refer the client’s spouse to the hospital’s crisis team.

The emergency department nurse is assigned an older adult client who is confused and agitated. Which intervention should the nurse include in

the client’s plan of care? a. Administer a sedative medication.

b. Ask a family member to stay with the client. c. Use restraints to prevent the client from falling.

d. Place the client in a wheelchair at the nurses’ station.

An emergency department nurse is transferring a client to the medical- surgical unit. What is the most important nursing intervention in this

situation? a. Triage the client to determine the urgency of care. b. Clearly communicate client data to the unit nurse. c. Evaluate the need for ongoing medical treatment.

d. Perform a thorough assessment of the client.

The nurse manager is assessing current demographics of the facility’s emergency department (ED) clients. Which population would most likely

present to the ED for treatment of a temperature and a sore throat? a. Older adults

b. Immunocompromised people c. Pediatric clients

d. Underinsured people

The emergency department (ED) nurse is caring for the following clients. Which client does the nurse prioritize to see first?

a. 22-year-old with a painful and swollen right wrist b. 45-year-old reporting chest pain and diaphoresis

c. 60-year-old reporting difficulty swallowing and nausea d. 81-year-old with a respiratory rate of 28 breaths/min and a

temperature of 101° F

A nurse is triaging clients in the emergency department. Which client complaint would the triage nurse classify as nonurgent?

a. Chest pain and diaphoresis b. Decreased breath sounds due to chest trauma c. Left arm fracture with palpable radial pulses

d. Sore throat and a temperature of 104° F

A client has been injured in a stabbing incident. Assessment reveals the following:

Blood pressure: 80/60 mm Hg

Heart rate: 140 beats/min

Respiratory rate: 35 breaths/min

Bleeding from stabbing wound site

Client is lethargic

Based on these assessment data, to which trauma center should the nurse

ensure transport of the client?

a. Level I b. Level II c. Level III d. Level IV

The emergency medical technicians (EMTs) arrive at the emergency department with an unresponsive client with an oxygen mask in place.

What will the nurse do first? a. Assess that the client is breathing adequately

b. Insert a large-bore intravenous line c. Place the client on a cardiac monitor d. Assess for best neurologic response

A client arrives at the emergency department following a motor vehicle collision. The client is not awake and is being bagged with a bag-valve- mask by paramedics. The client has sustained obvious injuries to the

head and face, as well as an open right femur fracture that is bleeding profusely. What will the nurse do first?

a. Splint the right lower extremity. b. Apply direct pressure to the leg.

c. Assess for a patent airway. d. Start two large-bore IVs.

The nurse is providing care for a client admitted for suicidal precautions. What priority intervention should the nurse implement first?

a. Administer prescribed anti-anxiety drugs. b. Decrease the noise level and the harsh lighting.

c. Remove oxygen tubing from the room. d. Set firm behavioral limits.

A trauma client with multiple open wounds is brought to the emergency department in cardiac arrest. What should the nurse do before providing

advanced cardiac life support?

a. Contact the on-call orthopedic surgeon. b. Don personal protective equipment.

c. Notify the Rapid Response Team. d. Obtain a complete history from the paramedic.

The nurse is triaging clients in the emergency department. Which client should be considered urgent?

a. 20-year-old female with a chest stab wound and tachycardia b. 45 year-old homeless man with a skin rash and sore throat

c. 75-year-old female with a cough and of temperature of 102° F d. 50-year-old male with new-onset confusion and slurred speech

A client in the emergency department has died from a suspected homicide. What is the nurse’s priority intervention?

a. Remove all tubes and wires in preparation for the medical examiner. b. Limit the number of visitors to minimize the family’s trauma.

c. Consult the bereavement committee to follow up with the grieving family.

d. Communicate the client’s death to the family in a simple and concrete manner.

A new nurse is orienting to the emergency department (ED). Which statement made by the nurse would indicate the need for further

education by the preceptor? a. “The emergency medicine physician coordinates care with all levels

of the emergency health care team.” b. “Emergency departments have specialized teams that deal with

high-risk populations of patients.” c. “Many older adults seek emergency services when they are ill

because they do not want to bother their primary health care provider.” d. “Emergency departments are responsible for public health

surveillance and emergency disaster preparedness.”

An unresponsive client with poor ventilator effort and a pulse rate of 120 beats/min arrives at the emergency department. What should the nurse

do first? a. Place the client on a non-rebreather mask.

b. Begin bag-valve-mask ventilation. c. Initiate cardiopulmonary resuscitation.

d. Prepare for chest tube insertion.

The nurse is triaging clients in the emergency department (ED). Which is true about the presentation of client symptoms?

a. Older adults frequently have symptoms that are vague or less specific.

b. Young adults present with nonspecific symptoms for serious illnesses.

c. Diagnosing children’s symptoms often keeps them in the ED longer. d. Symptoms of confusion always represent neurologic disorders.

The emergency department (ED) nurse is assigned to triage clients. What is the purpose of triage?

a. Treat clients on a first-come, first-serve basis. b. Identify and treat clients with low acuity first.

c. Prioritize clients based on illness severity. d. Determine health needs from a complete assessment.

The nurse is caring for a homeless client and consults the emergency department (ED) case manager. What can the ED case manager do for

this client? a. Communicate client needs and restrictions to support staff.

b. Prescribe low-cost antibiotics to treat community-acquired infection. c. Provide referrals to subsidized community-based health clinics.

d. Offer counseling for substance abuse and mental health disorders.

MULTIPLE RESPONSE

The emergency department (ED) nurse is preparing to transfer a client to the critical care unit. What information should the nurse include in

the nurse-to-nurse hand-off report? (Select all that apply.) a. Allergies

b. Vital signs c. Immunizations d. Marital status

e. Isolation precautions

The nurse is discharging an older adult client home from the emergency department (ED) after an acute episode of angina. What should the

nurse do to ensure client safety upon discharge? (Select all that apply.) a. Reconcile the client’s prescription and over-the-counter medications b. Screen the client for functional and cognitive abilities, as well as risk

for falls c. Consult physical therapy to organize for home health services

d. Arrange for the client’s car keys to be taken to prevent an accident e. Review discharge instructions with the client and a family member

Which interventions will be performed during the primary survey for a trauma client? (Select all that apply.)

a. Removing wet clothing b. Splinting open fractures

c. Initiating IV fluids d. Endotracheal intubation

e. Foley catheterization f. Needle decompression

g. Laceration repair

OTHER

The nurse is assessing clients on site at a multi-vehicle accident. Triage clients in the order they should receive care. (Place in order of priority.)

A 50-year-old with chest trauma and difficulty breathing

A mother frantically looking for her 6-year-old son An 8-year-old with a broken leg in his father’s arms A 60-year-old with facial lacerations and confusion

A pulseless male with a penetrating head wound

In what sequence would a client move through the process of admission to disposition in emergency care? (Place in order of priority.)

Client is transported to the medical-surgical floor. Emergency department (ED) nurse gives a report on the client.

Paramedics arrive and start IV access. Nurse and other health care provider(s) perform assessment.

Emergency medical technicians (EMTs) provide oxygen and vital sign monitoring.

Laboratory technician obtains blood specimens.

Chapter 11: Care of Patients with Common Environmental Emergencies

Test Bank

MULTIPLE CHOICE

While the nurse is visiting the community pool, an adult swimmer is pulled out of the pool, unconscious and cyanotic. What is the priority

action of the nurse?

a. Begin chest compressions. b. Move from the pool area. c. Give two rescue breaths.

d. Check for a carotid pulse.

The nurse working at a first aid booth during a summer marathon sees several runners. Which runner should be seen first? A runner who:

a. Has fallen several times b. Is fatigued

c. Thinks he has the flu d. Has tachypnea

The nurse is working at a first aid booth for a spring training game on a hot day. A spectator comes in, reporting that he is not feeling well. Vital

signs are temperature 104.1° F (40.1° C), pulse 132 beats/min, respirations 26 breaths/min, and blood pressure 106/66 mm Hg. He trips over his feet as the nurse leads him to a cot. What is the priority action of

the nurse? a. Encourage drinking of cool water or sports drinks.

b. Sponge the victim with cool water and remove his shirt. c. Administer Tylenol (acetaminophen), 650 mg orally.

d. Encourage rest, and reassess in 15 minutes.

The emergency department nurse assesses a client in extreme pain with an apparent snakebite of the leg. Vital signs are stable. What is the

priority action of the nurse? a. Call the regional poison control center.

b. Administer IV pain medication. c. Place a tourniquet around the leg. d. Apply an immobilization splint.

The nurse is teaching a wilderness survival class. Which statement by a participant indicates that additional teaching is needed?

a. “If I get too cold, I can have some brandy to help me get warmed up.”

b. “My climbing partner should let me know right away if my nose turns white.”

c. “If my partner can’t think straight, we should descend to a lower altitude.”

d. “It is okay to feel a little short of breath when I am climbing, but not at rest.”

A community nurse assesses a client, who has an allergy to bees, after a bee sting. The client’s lips are swollen, and wheezes are audible. What is

the priority action of the nurse? a. Elevate the site and notify the client’s next of kin.

b. Remove the stinger with tweezers and encourage rest. c. Administer diphenhydramine (Benadryl) and apply ice. d. Administer an EpiPen from the first aid kit and call 911.

While on a camping trip, the nurse provides care for a camper who was bitten by a black widow spider. What is the priority action of the nurse?

a. Apply ice to the site of the bite. b. Apply a loose tourniquet to the limb.

c. Give acetaminophen (Tylenol) for pain. d. Cover the camper with a warm blanket.

A client presents to the emergency department after prolonged exposure to the cold. The client is shivering, has slurred speech, and is slow to

respond to questions. Which treatment will the nurse prepare for this client?

a. Dry clothing and warm blankets b. Administration of warmed IV fluids

c. Peritoneal lavage with warmed normal saline d. Continuous arteriovenous rewarming

The emergency department nurse assesses a middle-aged mountain climber who reports headache, nausea, vomiting, and “feeling winded.”

What is the nurse’s priority intervention? a. Administer acetazolamide (Diamox).

b. Administer prochlorperazine (Compazine). c. Perform a neurologic assessment.

d. Assess for bowel sounds.

A nurse is at the scene of a lightning strike during a thunderstorm. Which is the priority action of the nurse?

a. Make sure that victims are not electrically charged. b. Assess victims for second- and third-degree burns.

c. Start emergency resuscitation on anyone not breathing. d. Move victims and first aid responders to a sheltered area.

On a hot, humid day, several clients present to the emergency department with symptoms of heat exposure. Which client will be treated

first? A client who: a. Has normal mental status and flu-like symptoms

b. Is diaphoretic with nausea and vomiting c. Is hypotensive and tachycardic

d. Is anxious and confused

The nurse is teaching a community health class about water safety. Which statement by a participant indicates that additional teaching is

needed? a. “I can go swimming all by myself because I am a certified

lifeguard.” b. “I cannot leave my toddler alone in the bathtub for even a minute.” c. “I will appoint one adult to supervise the pool at all times during a

party.” d. “I will make sure that there is a phone near my pool in case of an

emergency.”

The nurse is assessing a client recently bitten by a coral snake. Which assessment is the priority?

a. Peripheral edema and swelling b. Evaluation of clotting times c. Respiratory rate and depth d. Electrocardiogram rhythm

The nurse is teaching a client with severe allergies how to prevent bug bites. Which statement by the client indicates that additional teaching is

needed? a. “I will avoid wearing perfume when I go outside.”

b. “I will put the picnic food out when we are ready to eat.” c. “I will keep my car windows up at all times.” d. “I will wear sandals whenever I go outside.”

The nurse is planning care for a client admitted with a snakebite to the right leg. With whom should the nurse collaborate?

a. The facility’s neurologist

b. The regional poison control center c. The physical therapy department d. A herpetologist (snake specialist)

The nurse is caring for a client who had a near-drowning incident in a lake. Which action will the nurse take to monitor for possible

complications? a. Assess the client’s temperature every 4 hours.

b. Check the client’s blood glucose level before meals. c. Assess the client’s bowel sounds three times daily.

d. Check the client’s skin for petechiae daily.

The nurse is working in the emergency department on a hot, humid day, when a hiker is brought in after collapsing. The hiker is confused and

tachycardic with a temperature of 105.6° F (40.9° C). Which IV solution and medication will the nurse have ready for the client?

a. Normal saline and methylprednisolone (Solu-Medrol) b. Lactated Ringer’s solution and morphine sulfate

c. Normal saline and lorazepam (Ativan) d. Dextrose 5% and diphenhydramine (Benadryl)

The nurse is assessing a client admitted with a brown recluse spider bite. What priority assessment should the nurse perform?

a. Ask the client about pruritus at the bite site. b. Inspect for a bluish purple vesicle.

c. Assess for redness and swelling. d. Obtain the client’s temperature.

The nurse is providing emergency care to a client with frostbite. Which intervention is performed first?

a. Wrap the affected area in a warm, dry blanket to rewarm. b. Rewarm the affected area in a 104° F water bath.

c. Elevate the affected area above the heart to decrease tissue edema. d. Use a splint to immobilize the affected area.

The nurse is caring for a drowning victim after resuscitation. What focused assessment will the nurse perform to identify complications from

drowning? a. Palpation of abdominal cavity

b. Inspection of skin color c. Auscultation of lungs

d. Palpation of pulse strength

MULTIPLE RESPONSE The nurse is assessing a group of clients. Which clients are at greater risk

for hypothermia or frostbite? (Select all that apply.) a. A young man who has just consumed six martinis b. A young man with a body mass index (BMI) of 42

c. An older man who smokes a pack of cigarettes a day d. A young woman who is anorexic

e. An older woman with hypertension f. A young woman who is diabetic

The nurse is providing health education at a community center. Which instructions should the nurse include in teaching about prevention of

lightning injuries during a storm? (Select all that apply.) a. Seek shelter inside a building or vehicle.

b. Seek shelter under a tall tree.

c. Do not take a bath or shower. d. Turn off the television.

e. Remove body piercings. f. Put down golf clubs or gardening tools.

An emergency department nurse moves to a new city, where heat-related illnesses are common. Which clients should the nurse anticipate as at

higher risk for heat-related illness? (Select all that apply.) a. Homeless individuals

b. Illicit drug users c. Whites

d. Hockey players e. Older adults

Chapter 12: Concepts of Emergency and Disaster Preparedness

Test Bank

MULTIPLE CHOICE

The nurse has been assigned the role of triage nurse after a weather- related disaster. What is the priority action of the nurse?

a. Call in additional staff to assist with care of the victims. b. Splint fractures and clean and dress lacerations.

c. Perform a rapid assessment of clients to determine priority of care. d. Provide psychological support to staff and family members.

A client who is hospitalized with burns after losing the family home in a fire becomes angry and screams at the nurse when dinner is served late.

What is the nurse’s best response? a. “Do you need something for pain right now?”

b. “Please stop yelling. I brought dinner as soon as I could.” c. “I suggest that you get control of yourself.”

d. “You seem upset. I have time to talk if you like.”

A client is receiving follow-up care after surviving a tornado. The client

reports insomnia and the nurse notes that the client jumped as the nurse entered the room. Which action by the nurse is most appropriate?

a. Document findings on the client’s chart and inform the physician. b. Perform additional assessments for post-traumatic stress disorder.

c. Educate the client on nonpharmaceutical methods to promote sleep. d. Plan to initiate a referral to a psychologist experienced in survivor

issues.

An industrial accident has occurred near the hospital, and many victims are brought to the emergency department (ED) for treatment of their

injuries. The nurse triages the victim with which injury with a red tag? a. Dislocated right hip and an open fracture of the right lower leg

b. Large contusion to the forehead and a bloody nose c. Closed fracture of the right clavicle and arm numbness

d. Multiple fractured ribs and shortness of breath

The nurse is working with a paramedic who just finished assisting at the scene of a school shooting where several students were killed. Which

statement by the nurse is most therapeutic? a. “Would you like to talk about what happened?”

b. “Surely the department will give you the day off tomorrow.” c. “At least the gunman was taken into custody.”

d. “Let’s just sit here for a while quietly.”

A young man comes into the foyer of the hospital and says that he has a container of anthrax, which he opens and pours on the floor. Which is

the priority action for the nurse who first comes upon the scene? a. Don a protective gown, mask, and goggles.

b. Escort the man to the decontamination room. c. Begin to evacuate the immediate area.

d. Notify the local health department of a biohazard situation.

Which is the priority action for the emergency department charge nurse in the event of a mass casualty situation?

a. Directing medical-surgical and case management nurses to assist emergency department (ED) staff with critically injured victims

b. Calling additional medical-surgical and critical care nursing staff to come to the hospital to assist when victims are brought in

c. Informing the incident commander at the mass casualty scene about how many victims may be handled by the ED

d. Directing medical-surgical and critical care nurses to assist with clients who are already in the ED while the ED staff prepares to receive

the mass casualty victims

An accident has occurred near the hospital, and a victim is brought to the emergency department with severe chest pain, a pulse of 120

beats/min, blood pressure of 100/60 mm Hg, and a respiratory rate of 28 breaths/min. The nurse assesses shortness of breath and diaphoresis.

Which color tag does the nurse use when triaging this client? a. Red

b. Yellow c. Green d. Black

A nurse is working at the scene of a catastrophic natural event. Which person does the nurse attend to first?

a. Distraught mother looking for her children b. Person walking about with a bleeding head wound

c. Supine person with pale, cool, clammy skin d. Child with a deformed lower leg crying in pain

The hospital is overwhelmed when caring for victims after an earthquake that occurred 48 hours ago. Which responsibility of the

nursing supervisor is most important at this time? a. Assuming leadership for implementation of the hospital emergency

plan b. Releasing updates of client conditions to the media

c. Converting the physical therapy clinic into a treatment area for the injured

d. Arranging relief and coordinating breaks so nursing staff can rest and eat

The nurse is teaching nursing students about personal emergency preparedness. Which statement by a student indicates that further

teaching is indicated? a. “I will get a prescription for antibiotics just in case I have to work in

an area that has been infected with anthrax.” b. “I should keep an extra uniform in my locker in case I get stuck at

work.” c. “I may be torn between caring for my young daughter and caring

for victims at work.” d. “I should make plans for my family to evacuate our house in case of

tornado or earthquake.”

The hospital administration has arranged for critical incident stress debriefing for the staff after a mass casualty incident. Which statement

by the debriefing team leader is most appropriate for this situation? a. “You are free to express your feelings; whatever is said here stays

here.” b. “Let’s determine what we can do better the next time we have this

situation.” c. “This session is only for nursing and medical staff, not for ancillary

personnel.” d. “Let’s pass around the written policy compliance form for

everyone.”

The nurse is caring for a client whose wife just died in an accident. The client says to the nurse, “I can’t believe that my wife is gone and I am left to raise my children all by myself.” Which response by the nurse is most

appropriate? a. “Please accept my sympathy for your loss.”

b. “I can call the hospital chaplain if you wish.” c. “You sound anxious about being a single parent.”

d. “At least your children still have you in their lives.”

The emergency department nurse manager is explaining concepts of emergency and disaster preparedness to a group of students. Which

statement by the nurse manager is most accurate? a. “An internal disaster is something that occurs inside the health care

facility.” b. “An external disaster occurs when someone not employed here

disrupts our operations.” c. “A multi-casualty event involves disasters at several different

locations.” d. “The Joint Commission requires that we participate in a disaster

drill once a year.”

The emergency department (ED) is expecting a large number of casualties after a bridge collapse. Which is a priority consideration for

the ED leadership when activating the disaster plan? a. Responding paramedics and rescue personnel will notify the ED

about exactly how many victims to expect. b. Responding paramedics and rescue personnel will triage all victims

at the bridge collapse site before bringing them to the ED. c. The ED may receive many unexpected victims with minor injuries

from the bridge collapse. d. Victims who have been contaminated with gasoline will be decontaminated by rescue personnel before arriving at the ED.

A nursing administrator is evaluating the hospital’s response to a recent internal disaster. The administrator assesses that goals for disaster

planning have been met when which outcome is assessed? a. The hospital was able to maintain client, staff, and visitor safety

during the disaster. b. Supplies were readily available and were transported rapidly where

needed. c. The hospital incident command officer successfully utilized ancillary

areas for client care. d. All employees followed the chain of command and established

policies and procedures.

A nursing administrator is reviewing a hospital’s disaster planning. The administrator evaluates the plan that addresses which component as

being the best? a. Internal disasters such as fires or power outages

b. All possible catastrophes in the community c. The Joint Commission’s assessment of possible disasters

d. Responses to all types of weather-related emergencies

A nursing instructor is debriefing students who participated in a community-wide disaster drill. Several students are upset with the black-

tagged triage category. Which statement by the nursing instructor is best?

a. “To do the greatest good for the greatest number of people, it is necessary to sacrifice some.”

b. “Not everyone will survive a disaster, so it is best to identify those people early and move on.”

c. “In a disaster, extensive resources are not used for one person at the expense of many others.”

d. “With black tags, volunteers can identify those who are dying and can give them comfort care.”

A nurse wants to become involved in community disaster preparedness and is interested in helping set up and staff first aid stations or community acute care centers in the event of a disaster. Which

organization is the best fit for this nurse’s interests?

a. The Medical Reserve Corps b. The National Guard

c. The Health Department d. A Disaster Medical Assistance Team

A nurse wants to become part of a Disaster Medical Assistance Team

(DMAT) but is concerned about maintaining licensure in several different states. What statement by the nursing supervisor best addresses

these concerns? a. “Deployed DMAT providers are federal employees, so their licenses

are good in all 50 states.” b. “The government has a program for quick licensure activation

wherever you are deployed.” c. “During a time of crisis, licensure issues would not be the

government’s priority concern.” d. “If you are deployed, you will be issued a temporary license in the

state in which you are working.”

A community disaster has occurred and the hospital’s emergency department (ED) has efficiently triaged, treated, and transferred most

clients to appropriate units. The hospital incident command officer wants to “stand down” from the emergency plan. Which question by the

nursing supervisor is most beneficial at this time? a. “Are you sure no more victims are coming into the ED?”

b. “Do all other areas of the hospital have the supplies and personnel they need now?”

c. “Have all ED staff had the chance to eat and rest recently?” d. “Are all other incident command officers and house supervisors in

agreement with you?”

A hospital has “stood down” from a mass casualty disaster. The staff have rested and eaten. Which action by the nursing supervisor takes

priority? a. Restocking the emergency department (ED)

b. Making rounds on each unit to check staffing c. Determining which staff can go home

d. Planning a critical incident stress debriefing

A family in the emergency department is overwhelmed at the loss of several family members due to a shooting incident in the community.

Which intervention by the nurse is most beneficial? a. Offer the family choices as appropriate and possible.

b. Call the hospital chaplain to stay with the family. c. Do not allow visiting of the victims until the bodies are prepared.

d. Provide privacy for law enforcement to interview the family.

An emergency department (ED) supervisor has noted an increase in sick calls and bickering among the ED staff after a week with multiple trauma incidents. What action by the supervisor is most helpful?

a. Organize a pizza party for each shift. b. Remind staff of facility sick-leave policy.

c. Arrange critical incident stress debriefing. d. Talk individually with staff members.

A client has been treated in the emergency department after a tornado and is awaiting discharge instructions. This client is close to losing

control, although other family members are attempting to calm him down. Which response by the nurse is most helpful?

a. Call security and have them standing by in case they are needed. b. Instruct the person to leave the area until he can calm down.

c. Offer the client the choice of waiting in the treatment room or the waiting room.

d. Ask the family to help move the client out of the treatment area.

MULTIPLE RESPONSE

A large number of victims arrive at the emergency department after a bus is hit by a train. Which interventions are performed immediately for

red-tagged victims? (Select all that apply.) a. Splinting a closed tibial fracture

b. Intubating a cyanotic client in respiratory distress c. Initiating IV fluids for a client with a blood pressure of 96/60 mm

Hg and a pulse of 144 beats/min d. Attaching an external pacemaker for a client with a heart rate of 44

beats/min e. Performing postmortem care for a client who has just died

f. Removing glass that is embedded in a client’s arm

The triage nurse is assessing a client who has been brought to the emergency department (ED) by emergency medical services (EMS)

following a mass casualty incident. Which assessment questions are used to determine the appropriate triage category for the client? (Select all

that apply.) a. “Can you wiggle your toes?”

b. “Are you having any difficulty breathing?” c. “Are you allergic to any medications?”

d. “Does your family know that you are here?” e. “Can you tell me what day it is?”

f. “Do you have any abdominal or back pain?”

Emergency medical services (EMS) brings a large number of clients to the emergency department following a mass casualty incident. The nurse

identifies clients with which injuries with yellow tags? (Select all that apply.)

a. Partial-thickness burns covering both legs b. Open fractures of both legs with absent pedal pulses

c. Neck injury and numbness of both legs d. Small pieces of shrapnel embedded in both eyes

e. Head injury and difficult to arouse f. Bruising and pain in the right lower abdomen

A hospital is receiving large numbers of casualties from a disaster. Which clients does the supervisor identify as appropriate for discharge or

transfer to another facility? (Select all that apply.) a. Client who had open reduction and internal fixation of a femur

fracture 3 days ago b. Client who had a colostomy 4 days ago and whose daughter is a

registered nurse c. Client admitted last night with community-acquired pneumonia

d. Infant admitted 2 days ago for fever of unknown origin e. Client in the medical decision unit for evaluation of chest pain

The nurse working with survivors of a disaster wants to assess them for post-traumatic stress disorder. For which clients does the nurse perform

further assessment before administering the Impact of Event Scale– Revised? (Select all that apply.)

a. Older adult survivor with minor injuries b. Woman who lost both her children

c. Middle-aged victim with multiple medical problems d. Young adult who had serious orthopedic injuries

e. Older adolescent who had a traumatic brain injury

A wing of a hospital is on fire. Which actions by the nurse promote safe evacuation of clients? (Select all that apply.)

a. Direct ambulatory clients on where to go to be safe. b. Use ambulatory clients to help push clients in wheelchairs.

c. Use oxygen tanks for all clients who are on oxygen. d. Manually ventilate clients who are on ventilators. e. Move bedridden clients in their beds if possible.

Chapter 13: Assessment and Care of Patients with Fluid and Electrolyte Imbalances

Test Bank

MULTIPLE CHOICE

The nurse observes skin tenting on the back of the older adult client’s hand. Which action by the nurse is most appropriate?

a. Notify the physician. b. Examine dependent body areas.

c. Assess turgor on the client’s forehead. d. Document the finding and continue to monitor.

The client is taking a medication that inhibits aldosterone secretion and release. The nurse assesses for what potential complication?

a. Fluid retention b. Hyperkalemia c. Hyponatremia d. Hypervolemia

Which assessment does the nurse use to determine the adequacy of circulation in a client whose blood osmolarity is 250 mOsm/L?

a. Measuring urine output b. Measuring abdominal girth

c. Monitoring fluid intake d. Comparing radial versus apical pulses

Which statement made by the older adult client alerts the nurse to assess specifically for fluid and electrolyte imbalances?

a. “My skin is always so dry, especially here in the Southwest.” b. “I often use a glycerin suppository for constipation.”

c. “I don’t drink liquids after 5 PM so I don’t have to get up at night.”

d. “In addition to coffee, I drink at least one glass of water with each meal.”

A client has been taught to restrict dietary sodium. Which food selection by the client indicates to the nurse that teaching has been effective?

a. Chinese take-out, including steamed rice b. A grilled cheese sandwich with tomato soup

c. Slices of ham and cheese on whole grain crackers d. A chicken leg, one slice of bread with butter, and steamed carrots

A client is on a potassium-restricted diet. Which protein choice by the client indicates a good understanding of the dietary regimen?

a. 1% or 2% milk b. Grilled salmon c. Poached eggs

d. Baked chicken

Which assessment finding obtained while taking the history of an older adult client alerts the nurse that the client needs further assessment for

fluid or electrolyte imbalance? a. “I am often cold and need to wear a sweater.” b. “I seem to urinate more when I drink coffee.”

c. “In the summer, I feel thirsty more often.” d. “My rings seem to be tighter this week.”

Which client is at greatest risk for dehydration? a. Younger adult client on bedrest

b. Older adult client receiving hypotonic IV fluid c. Younger adult client receiving hypertonic IV fluid

d. Older adult client with cognitive impairment

Which question does the nurse ask the client who has isotonic dehydration to determine a possible cause?

a. “Do you take diuretics, or ‘water pills’?” b. “What do you normally eat over a day’s time?”

c. “How many bowel movements do you have daily?” d. “Have you been diagnosed with diabetes mellitus?”

Which intervention in a client with dehydration-induced confusion is most likely to relieve the confusion?

a. Measuring intake and output every four hours b. Applying oxygen by mask or nasal cannula

c. Increasing the IV flow rate to 250 mL/hr d. Placing the client in a high Fowler’s position

A client is being treated for dehydration. Which statement made by the client indicates understanding of this condition?

a. “I must drink a quart of water or other liquid each day.” b. “I will weigh myself each morning before I eat or drink.”

c. “I will use a salt substitute when making and eating my meals.” d. “I will not drink liquids after 6 PM so I won’t have to get up at

night.”

What intervention is most important to teach the client about identifying the onset of dehydration?

a. Measuring abdominal girth b. Converting ounces to milliliters

c. Obtaining and charting daily weight d. Selecting food items with high water content

A nurse is caring for several clients with dehydration. The nurse assesses the client with which finding as needing oxygen therapy?

a. Tenting of skin on the back of the hand b. Increased urine osmolarity c. Weight loss of 10 pounds

d. Pulse rate of 115 beats/min

Which action does the nurse teach a client to reduce the risk for dehydration?

a. Restricting sodium intake to no greater than 4 g/day b. Maintaining an oral intake of at least 1500 mL/day

c. Maintaining a daily oral intake approximately equal to daily fluid loss

d. Avoiding the use of glycerin suppositories to manage constipation

Which item of assessment data obtained by the home care nurse suggests that an older adult client may be dehydrated?

a. The client has dry, scaly skin on bilateral upper and lower extremities.

b. The client states that he gets up three or more times during the night to urinate.

c. The client states that he feels lightheaded when he gets out of bed or stands up.

d. The nurse observes tenting on the back of the hand when testing skin turgor.

A client is being discharged with mild dehydration. Which statement by the client indicates an understanding of measures to prevent mild

dehydration from becoming more severe? a. “I will weigh myself at the same time daily wearing the same

clothes.” b. “When I feel lightheaded, I will drink a full glass of water.”

c. “I will decrease my fluid intake if my urine output increases.” d. “If I forget to take my diuretic, I will take twice the dose next time.”

During assessment of hydration status, the client tells the nurse that she usually drinks 3 quarts of liquids each day. Which question by the nurse

is best? a. “Do you usually drink liquids that are hot or cold?”

b. “How much salt do you add to your food?” c. “What kinds of liquids do you usually drink?”

d. “Do you drink fluids with meals or between meals?”

A nurse is caring for several clients at risk for overhydration. The nurse assesses the older client with which finding first?

a. Has had diabetes mellitus for 12 years b. Uses sodium-containing antacids frequently

c. Just received 3 units of packed red blood cells d. Had abdominal surgery and has a nasogastric tube

A client has been diagnosed with overhydration and is confused. Which intervention does the nurse include in the client’s plan of care to relieve

the confusion? a. Measuring intake and output every shift

b. Slowing the IV flow rate to 50 mL/hr c. Administering diuretic agents as prescribed d. Placing the client in Trendelenburg position

The nurse assesses distended neck veins in a client sitting in a chair to eat. What intervention is the nurse’s priority? a. Document the observation in the chart.

b. Measure urine specific gravity and volume. c. Assess the pulse and blood pressure.

d. Assess the client’s deep tendon reflexes.

A client in the emergency department has potassium of 2.9 mEq/L. For which disease process or condition does the nurse assess the client?

a. Diabetes mellitus b. Addison’s disease

c. Hyperaldosteronism d. Diabetes insipidus

A client is taking furosemide (Lasix) and becomes confused. Which potassium level does the nurse correlate with this condition?

a. 2.9 mEq/L b. 3.8 mEq/L

c. 5.0 mEq/L d. 6.0 mEq/L

A client has hypokalemia. Which question by the nurse obtains the most information on a possible cause?

a. “Do you use sugar substitutes?” b. “Do you use diuretics or laxatives?” c. “Do you have any kidney disease?”

d. “Have your bowel habits changed recently?”

A client has been treated for hypokalemia. Which clinical manifestation or condition indicates that treatment has been effective?

a. Having a bowel movement daily b. Gaining 2 lb during the past week

c. Electrocardiogram (ECG) showing inverted T-waves d. Fasting blood glucose level of 106 mg/dL

The nurse notes that the handgrip of the client with hypokalemia has diminished since the previous assessment one hour ago. Which

intervention by the nurse is the priority? a. Assess the client’s respiratory rate, rhythm, and depth.

b. Measure the client’s pulse and blood pressure. c. Document findings and monitor the client.

d. Call the health care provider.

The client is receiving an intravenous infusion of 60 mEq of potassium chloride in a 1000 mL solution of dextrose 5% in 0.45% saline. The client states that the area around the IV site burns. What intervention does the

nurse perform first? a. Notify the physician.

b. Assess for a blood return. c. Document the finding. d. Stop the IV infusion.

A client has been taught to increase potassium in the diet. What dietary meal selection indicates to the nurse that teaching has been effective?

a. Toasted English muffin with butter and blueberry jam, and tea with sugar

b. Two scrambled eggs, a slice of white toast, and a cup of strawberries

c. Sausage, one slice of whole wheat toast, cup of raisins, and a glass of milk

d. Bowl of oatmeal with brown sugar, cup of sliced peaches, and coffee

Which client statement indicates the need for more teaching regarding identification of the early manifestations of hypokalemia?

a. “I have been weighing myself every day.” b. “When I am constipated, I drink more fluids.” c. “When my muscles feel weak, I eat a banana.”

d. “I check my pulse each morning and each night.”

A nurse is caring for several clients. Which client does the nurse assess most carefully for hyperkalemia?

a. Client with heart failure using a salt substitute b. Client taking a thiazide diuretic for hypertension

c. Client taking nonsteroidal anti-inflammatory drugs daily d. Client with type 2 diabetes taking an oral antidiabetic agent

A client at risk for continuing hyperkalemia states that she is upset because she cannot eat fruit every day. Which response by the nurse is

best? a. “You are correct. Fruit is usually very high in potassium.”

b. “If you cook the fruit first, that lowers the potassium.” c. “Berries, cherries, apples, and peaches are low in potassium.”

d. “Fresh fruit is higher in potassium than dried fruit.”

A client is being discharged and needs to self-monitor for the development of hyperkalemia. Which intervention is most important for

the nurse to teach the client? a. Weighing self daily at the same time of day

b. Assessing radial pulse for a full minute twice a day c. Ensuring an oral intake of a least 3 L of fluids per day

d. Restricting sodium as well as potassium intake

A client is admitted with hyponatremia. Four hours after the initial assessment, the nurse notes that the client has new hyperactive bowel

sounds in all four quadrants. What analysis about the client’s condition is correct?

a. The hyponatremia is worse. b. The hyponatremia is the same.

c. The hyponatremia is better. d. The client now has hypernatremia.

A nurse is assessing clients for fluid and electrolyte imbalances. Which client is at greatest risk for developing hyponatremia?

a. Client who is NPO receiving intravenous D5W b. Client taking a sulfonamide antibiotic

c. Client taking ibuprofen (Motrin) d. Client taking digoxin (Lanoxin)

The nurse is providing discharge teaching for a client who is at risk for mild hypernatremia. What action is most important for the nurse to

teach the client? a. “Weigh yourself every morning and every night.”

b. “Check your radial pulse twice a day.” c. “Read food labels to determine sodium content.”

d. “Bake or grill the meat rather than frying it.”

A client has a history of hypothyroidism. Which laboratory value is the

nurse most concerned about? a. Na+ 146 mEq/L b. K+ 3.6 mEq/L

c. Ca2+ 8.2 mg/dL d. Mg2+ 1.1 mEq/L

When taking the blood pressure of a very ill client, the nurse observes that the client’s hand undergoes flexion contractions. Which intervention

is most appropriate? a. Administer isotonic intravenous fluids.

b. Remove the blood pressure cuff and give oxygen. c. Ensure the client has a patent intravenous line.

d. Document the finding in the client’s chart.

A client has the following laboratory values: Ca2+ 8.7 mg/dL; K+ 4.2 mEq/L; Na+ 142 mEq/L. Which intervention by the nurse is most

appropriate? a. Prepare to administer IV potassium chloride.

b. Ask the lab to redraw and rerun the tests. c. Document findings and continue to assess.

d. Prepare to administer aluminum hydroxide.

A client has a history of hypocalcemia. What intervention is most important for the nurse to add to this client’s care plan?

a. Push fluids to 2 L/day. b. Strain all urine output.

c. Use nonslip footwear to get out of bed. d. Position the client supine twice a day.

Which client is at greatest risk for developing hypercalcemia? a. Client taking furosemide (Lasix) for heart failure

b. Client with long-standing osteoarthritis c. Woman who is pregnant with twins d. Client with hyperparathyroidism

A client has a calcium level of 14 mg/dL. Which intervention is the priority?

a. Forcing fluids to 2 L/day b. Placing the client on a cardiac monitor

c. Assessing for Chvostek’s sign every 2 hours d. Administering IV calcium chloride

A client is admitted with multiple fractures from a motor vehicle crash (MVC). Which of the client’s previous or concurrent health problems is

most likely to increase the client’s risk for hypophosphatemia? a. Chronic alcoholic pancreatitis b. 50–pack-year smoking history

c. Prostate cancer history d. Heart surgery 8 years ago

A client with hypophosphatemia is being discharged. Which activity demonstrated by the client indicates that discharge teaching has been

effective? a. Assessing radial pulse rate and rhythm

b. Interspersing daily activities with periods of rest c. Selecting foods high in phosphorus and low in calcium

d. Weighing himself or herself correctly at the same time each day

The nurse observes that the handgrip of the client with hypophosphatemia has diminished in strength since the last assessment 2

hours ago. What is the nurse’s primary intervention? a. Document the finding and continue to assess.

b. Assess respiratory status immediately. c. Request an order for a serum calcium level.

d. Administer a rapid bolus of intravenous phosphorus.

MULTIPLE RESPONSE

Which ethnic groups should the nurse screen specifically for hypocalcemia? (Select all that apply.)

a. Whites b. Blacks c. Asians

d. Hispanics e. American Indians

Chapter 14: Assessment and Care of Patients with Acid-Base Imbalances

Test Bank

MULTIPLE CHOICE

In a client with less than the normal amount of bicarbonate in the blood and other extracellular fluids, what response does the nurse anticipate?

a. Increased risk for acidosis b. Decreased risk for acidosis c. Increased risk for alkalosis d. Decreased risk for alkalosis

Which response is an example of compensation for an acid-base imbalance?

a. Increase in the rate and depth of respirations when exercising b. Increased urinary output when blood pressure increases during

exercise c. Increased thirst when spending time in an excessively dry

environment d. Increased release of acids from kidneys during exacerbation of

chronic obstructive pulmonary disease (COPD)

When a client has an arterial blood pH of 7.48, which buffer action will bring the pH back to normal?

a. Absorption of bicarbonate ions from the blood b. Release of bicarbonate ions into the blood

c. Absorption of hydrogen ions from the blood d. Release of hydrogen ions into the blood

A client has moderate acidosis. Which assessment does the nurse perform first?

a. Take the client’s pulse and blood pressure, and analyze the electrocardiogram (ECG) strip.

b. Assess respiratory rate and depth and work of breathing. c. Perform assessments of musculoskeletal strength.

d. Determine whether the client is awake, alert, and oriented.

In the client with hypoventilation, which change in arterial blood gases does the nurse evaluate to determine whether treatment measures are

being effective? a. Decreased arterial blood pH

b. Decreased arterial blood carbon dioxide c. Increased arterial blood bicarbonate

d. Increased arterial blood oxygen

In a client 4 minutes post cardiac arrest, the nurse correlates the largest source of excess hydrogen ions with which cause?

a. Excess renal retention of carbon dioxide due to hypoxia b. Release of intracellular acids due to widespread tissue destruction

c. Anaerobic metabolism, leading to the buildup of lactic acid d. Using fat as a fuel source, resulting in increased fat degradation

A client has mild acidosis but after a day has not compensated for it. Which action by the nurse is best?

a. Review the client’s daily hemoglobin and hematocrit. b. Ask the laboratory to rerun today’s arterial blood gases.

c. Document the finding and notify the physician. d. Apply 2 L of oxygen via nasal cannula.

A client has an arterial blood gas pH of 7.48. How does the nurse interpret this client’s acid-base status?

a. An unknown acid-base balance status b. A normal blood hydrogen ion concentration

c. A deficit in blood hydrogen ion concentration d. An excess in blood hydrogen ion concentration

The nurse reads in the medical record that a client has Kussmaul respirations. Which assessment finding is consistent with this condition?

a. Deep, rapid respirations b. Respirations with an irregular pattern

c. Shallow, grunting respirations d. Use of accessory muscles when breathing

The nurse monitors for which acid-base imbalance in a client who has hypoxemia?

a. Reduced carbon dioxide production leading to alkalosis b. Reduced carbon dioxide retention leading to alkalosis c. Excess carbon dioxide production leading to acidosis d. Excess carbon dioxide retention leading to acidosis

A client has been placed on a ventilator. The physician has ordered that the ventilator be set to deliver a respiratory rate set of 28 breaths/min. The nurse questions the order, citing concerns about which acid-base

problem? a. Acid deficit: alkalosis b. Base excess: alkalosis c. Acid excess: acidosis d. Base deficit: acidosis

The nurse monitors for which acid-base problem in a client who is taking furosemide (Lasix) for hypertension?

a. Acid excess secondary to respiratory acidosis b. Acid deficit secondary to respiratory alkalosis

c. Acid excess secondary to metabolic acidosis d. Acid deficit secondary to metabolic alkalosis

The nurse expects to find renal compensation for an acid-base imbalance in which situation?

a. Mild to moderate dehydration in a middle-aged client who jogged for 2 hours

b. Acute asthma attack with wheezing of 6 hours’ duration in an older man

c. Food poisoning with vomiting for 12 hours in a middle-aged woman d. Hypoxemia for 4 days from pneumonia in an adult woman

A client has moderate metabolic alkalosis. What is the priority intervention for the nurse?

a. Monitor daily laboratory values. b. Assess the client’s muscle strength.

c. Determine the cause of the problem.

d. Teach the client preventive measures.

A client has acute pancreatitis and a risk for acid-base imbalance. The nurse plans to assess for which manifestation consistent with this

condition? a. Agitation

b. Kussmaul respirations c. Seizures

d. Positive Chvostek’s sign

The nurse assesses for acidosis in the client with which assessment data? a. Serum sodium level of 130 mEq per liter and peripheral edema

b. Serum sodium level of 144 mEq per liter and tachycardia c. Serum potassium level of 6.5 mEq per liter and flaccid paralysis d. Serum potassium level of 4.5 mEq per liter and hyperactive deep

tendon reflexes

The hand grasps of a client with acidosis have diminished since the previous assessment 1 hour ago. What action does the nurse take next?

a. Assess client’s rate, rhythm, and depth of respiration. b. Measure the client’s pulse and blood pressure. c. Document findings and continue to monitor.

d. Notify the physician as soon as possible.

In evaluating the electrocardiogram (ECG) in a client with acidosis, the nurse correlates which ECG change with effectiveness of therapy?

a. Small U-waves present after each complex b. Heart rate decreased to 62 beats/min

c. T-waves present, normal height d. P-wave preceding the QRS complex

A client has the following arterial blood results: pH 7.12, HCO3– 22 mEq/L, PCO2 65 mm Hg, PO2 56 mm Hg. The nurse correlates these

values with which clinical situation? a. Diabetic ketoacidosis in a person with emphysema

b. Tracheal obstruction related to aspiration of a hot dog c. Anxiety-induced hyperventilation in an adolescent

d. Diarrhea for 36 hours in an older, frail woman

Which client does the nurse assess for potential metabolic acidosis? a. Client admitted after collapsing during a marathon run

b. Young adult following a carbohydrate-free diet c. Older adult with asthma who is on long-term steroid therapy d. Older client on antacids for gastroesophageal reflux disease

The nurse interprets which arterial blood gas values as partially compensated metabolic acidosis?

a. pH 7.28, HCO3– 19 mEq/L, PCO2 45 mm Hg, PO2 96 mm Hg b. pH 7.45, HCO3– 22 mEq/L, PCO2 40 mm Hg, PO2 98 mm Hg c. pH 7.32, HCO3– 17 mEq/L, PCO2 25 mm Hg, PO2 98 mm Hg d. pH 7.48, HCO3– 28 mEq/L, PCO2 45 mm Hg, PO2 92 mm Hg

A client has just experienced a 90-second tonic-clonic seizure and has these arterial blood gas values: pH 6.88, HCO3– 22 mEq/L, PCO2 60

mm Hg, PO2 50 mm Hg. Which intervention by the nurse is most appropriate?

a. Apply oxygen by mask or nasal cannula. b. Apply a paper bag over the client's nose and mouth.

c. Administer 50 mL of sodium bicarbonate intravenously. d. Administer 50 mL of 20% glucose and 20 units of regular insulin.

A client who was malnourished is being discharged. The nurse evaluates that teaching to decrease risk for the development of metabolic acidosis

has been effective when the client states, “I will: a. Increase my milk intake to at least three glasses daily.”

b. Be sure to eat three well-balanced meals and a snack daily.” c. Avoid taking pain medication and antihistamines together.”

d. Not add salt to food when cooking or during meals.”

The nurse assesses the client with which condition most carefully for the risk of developing acute respiratory acidosis?

a. Allergic rhinitis and sinusitis on sulfa antibiotics b. Type 1 diabetes and urinary tract infection

c. Emphysema and undergoing nasogastric (NG) tube suctioning d. On patient-controlled analgesia after abdominal surgery

The nurse correlates which condition with the following arterial blood gas values: pH 7.48, HCO3– 22 mEq/L, PCO2 28 mm Hg, PO2 98 mm

Hg? a. Diarrhea and vomiting for 36 hours b. Anxiety-induced hyperventilation

c. Chronic obstructive pulmonary disease d. Diabetic ketoacidosis and emphysema

A postoperative client received six units of packed red blood cells (PRBCs) for intraoperative blood loss. The nurse monitors the client for

which acid-base imbalance? a. Metabolic alkalosis b. Metabolic acidosis

c. Respiratory alkalosis d. Respiratory acidosis

A client has severe metabolic alkalosis. Which nursing diagnosis does the nurse choose as the client’s priority problem?

a. Fluid volume excess related to reduced kidney function

b. Fluid volume deficit related to increased insensitive fluid loss through lungs

c. Risk for impaired skin integrity related to accompanying peripheral edema

d. Risk for injury related to increased neuronal sensitivity from hypocalcemia

A client has respiratory acidosis. The nurse evaluates that treatment is being effective with which arterial blood gas values?

a. pH 7.28, HCO3– 12 mEq/L, PCO2 45 mm Hg, PO2 96 mm Hg b. pH 7.32, HCO3– 17 mEq/L, PCO2 25 mm Hg, PO2 98 mm Hg c. pH 7.35, HCO3– 36 mEq/L, PCO2 65 mm Hg, PO2 78 mm Hg d. pH 7.48, HCO3– 12 mEq/L, PCO2 35 mm Hg, PO2 85 mm Hg

The nurse monitors the client with which condition most carefully for metabolic alkalosis?

a. A critical illness receiving total parenteral nutrition b. Type 1 diabetes on once-daily insulin therapy

c. Metastatic breast cancer on continuous IV morphine d. Asthma using an adrenergic agonist inhaler

A client is in the emergency department after an overdose of an unknown substance. Which assessment findings does the nurse correlate with

possible salicylate poisoning? a. Increased deep tendon reflexes

b. Increased rate and depth of respiration c. Decreased capillary refill

d. Decreased intestinal motility and paralytic ileus

A client has a prolonged fever. For which acid-base imbalance does the nurse assess the client further?

a. Metabolic acidosis from excess bicarbonate production b. Metabolic alkalosis from dehydration and hyperkalemia

c. Metabolic acidosis from increased production of hydrogen ions d. Respiratory alkalosis from impaired gas exchange

The nurse is providing discharge teaching. Which statement by the client indicates the need for further teaching regarding increased risk for

metabolic alkalosis? a. “I don’t drink milk because it gives me gas and diarrhea.”

b. “I have been taking digoxin every day for the last 15 years.” c. “I take sodium bicarbonate after every meal to prevent heartburn.”

d. “In hot weather, I sweat so much that I drink six glasses of water each day.”

A client with chronic respiratory acidosis is receiving oxygen by nasal cannula at 6 L/min. The client’s respiratory rate is 8 breaths/min. Which

action by the nurse is the priority? a. Notify the Rapid Response Team and prepare for intubation.

b. Change the nasal cannula to a mask and reassess in 10 minutes. c. Place the client in Fowler’s position if he or she is able to tolerate it.

d. Decrease the flow rate of oxygen to 2 to 4 L/min, and reassess.

A client is being discharged from the emergency department with several broken ribs. For which acid-base imbalance does the nurse provide

discharge teaching? a. Respiratory alkalosis from anxiety and hyperventilation

b. Respiratory acidosis from inadequate ventilation c. Metabolic acidosis from calcium loss from broken bones

d. Metabolic alkalosis from taking base-containing analgesics

The nurse prepares to administer bicarbonate intravenously to the client with which clinical manifestations?

a. pH 7.28, HCO3– 22 mEq/L, PCO2 52 mm Hg, PO2 82 mm Hg secondary to an acute asthma attack

b. pH 7.28, HCO3– 16 mEq/L, PCO2 45 mm Hg, PO2 98 mm Hg secondary to excessive diarrhea

c. Client with chronic emphysema and bronchitis who has the following arterial blood gases: pH 7.30, HCO3– 30 mEq/L, PCO2 60 mm

Hg, PO2 72 mm Hg secondary to chronic bronchitis and emphysema d. pH 7.31, HCO3– 20 mEq/L, PCO2 34 mm Hg, PO2 96 mm Hg

secondary to a urinary tract infection and type 2 diabetes

A client has metabolic alkalosis. Which laboratory results is the nurse most likely to assess as consistent with this condition?

a. Na+ 134 mg/dL b. Mg2+ 1.5 mg/dL c. K+ 3.1 mEq/L

d. Ca2+ 11.5 mg/dL

A client is admitted with mixed respiratory and metabolic acidosis secondary to bronchitis and diabetic ketoacidosis. The nurse evaluates that teaching about the client’s confusion was effective when a family

member makes which statement? a. “It is too early to tell if the ketoacidosis will cause permanent

changes.” b. “Her memory will improve, but loss of some brain cells has

occurred.” c. “The confusion should clear when oxygen and electrolyte levels are

normal.” d. “The confusion should clear when blood glucose levels and other

laboratory tests are normal.”

A client is being discharged and continues to be at risk for developing metabolic alkalosis. Which statement by the client indicates to the nurse

that teaching has been effective? a. “I will avoid excess use of antacids.”

b. “I’ll drink at least three glasses of milk daily.” c. “I’ll avoid medications containing aspirin.”

d. “I will not add salt to my food during meals.”

In clients with any type of acid-base imbalance, the nurse places the priority on monitoring which electrolyte?

a. Sodium b. Calcium

c. Potassium d. Magnesium

A client has acidosis. Which laboratory finding is of greatest concern to the nurse?

a. Sodium 154 mEq/L b. Potassium 5.9 mEq/L c. Calcium 8.9 mg/dL

d. Magnesium 2.1 mg/dL

A client has the following arterial blood gases: pH 7.30, HCO3– 17 mEq/L, PCO2 25 mm Hg, PO2 98 mm Hg. Which intervention by the

nurse is most appropriate? a. Prepare to give intravenous sodium bicarbonate.

b. Document the findings and continue to assess. c. Assist the physician in determining the cause. d. Administer oxygen at 2 L per nasal cannula.

A client has the following arterial blood gases (ABGs): pH 7.30, HCO3– 22 mEq/L, PCO2 55 mm Hg, PO2 86 mm Hg. Which intervention by the

nurse takes priority? a. Assessing the airway

b. Administering bronchodilators c. Administering mucolytics

d. Providing oxygen

A client has been NPO after a colectomy with nasogastric (NG) suction in place. On assessment, the nurse finds the client reporting cramps in the

calves. Which action by the nurse is most appropriate? a. Document findings and notify the physician.

b. Stop suction and request that the laboratory draw arterial blood gases.

c. Prepare to administer lorazepam (Ativan). d. Raise the siderails and notify the physician.

MULTIPLE RESPONSE

In the client with alkalosis, the nurse assesses for which clinical manifestations? (Select all that apply.)

a. Positive Chvostek’s sign b. Positive Trousseau’s sign

c. Hyporeflexia d. Bradycardia

e. Elevated blood pressure f. Elevated urinary output

Chapter 15: Infusion Therapy

Test Bank

MULTIPLE CHOICE

Before the administration of intravenous fluid, it is most important for the nurse to obtain which information from the health care provider’s

orders? a. Intravenous catheter size b. Osmolarity of the solution c. Vein to be used for therapy

d. Specific type of IV fluid

Which IV order does the nurse question? a. Flush Groshong catheter with 10 mL normal saline every 8 hours. b. Infuse 20 mEq potassium chloride in 1000 mL D5W at 50 mL/hr.

c. Infuse 500 mL normal saline over 1 hour. d. Infuse 0.9% normal saline at keep vein open (KVO) rate.

Which infusion device does the nurse select for the older adult client with a medical diagnosis of “dehydration”?

a. Cassette pump b. Elastomeric balloons c. Volumetric controller

d. Syringe pump

A nursing administrator is concerned about the incidence of complications related to IV therapy, including bloodstream infection. Which action by the administrator would have the biggest impact on

decreasing complications? a. Investigate initiating a dedicated IV team.

b. Require inservice education for all RNs. c. Limit IV starts to the most experienced nurses.

d. Perform quality control testing on skin preparation products.

The nurse wants to find written standards for IV therapy. The nursing manager suggests that the nurse investigate publications from which

resource? a. IV Therapy Nursing Society

b. Infusion Nurses Society c. Nurse’s State Board of Nursing d. Hospital’s IV solutions vendor

The RN assigned a new nurse to a client who was receiving chemotherapy through an intravenous extension set attached to a Huber needle. Which information about disconnecting the Huber needle is most

important for the RN to provide to the new nurse? a. “Apply topical anesthetic cream to the area after discontinuing the

system.” b. “Be aware of a rebound effect when discontinuing the system.”

c. “Be sure to flush the system with saline after removing the Huber needle.”

d. “Place pressure over the site to prevent bleeding.”

After discontinuing a nontunneled, percutaneous central catheter, it is most important for the nurse to record which information?

a. Application of a sterile dressing b. Length of the catheter

c. Occurrence of venospasms d. Type of ointment used to seal the tract

When assessing the client’s peripheral IV site, the nurse observes a streak of red along the vein path and palpates a 4-cm venous cord. What

is the most accurate documentation of this finding? a. Grade 3 phlebitis at IV site

b. Infection at IV site c. Thrombosed area at IV site

d. Infiltration at IV site

What information is most important to teach the client going home with a peripherally inserted central catheter (PICC) line?

a. “Avoid carrying your grandchild with the arm that has the IV.” b. “Be sure to place the arm with the IV in a sling during the day.”

c. “Flush the IV line with normal saline daily.”

d. “You can use the arm with the IV for most of the activities of daily living.”

A client is to receive 10 days of antibiotic therapy for urosepsis. The nurse plans to insert which type of intravenous catheter?

a. Hickman b. Midline

c. Nontunneled central d. Short peripheral

A client is admitted to the hospital for excessive nausea and vomiting, and a blood pressure of 90/50 mm Hg. A catheter of which gauge is most

appropriate for the nurse to choose for this client’s peripheral IV? a. 24 b. 22 c. 20 d. 18

To prevent infection when infusing an intermittent “piggyback” line, which intervention does the nurse implement?

a. Backpriming the secondary container from the primary line b. Detaching and capping the secondary line after use

c. Using a new secondary container with each drug infused d. Using sterile gloves when administering medication

The nurse finishes administering an intermittent medication through a Groshong catheter. What is the nurse’s next action?

a. Clamping the catheter b. Flushing the line with saline

c. Flushing with heparin d. Removing the access needle

The nurse is assessing several clients receiving intravenous therapy. Which client situation requires immediate intervention?

a. Completion of an intermittent medication into a Groshong catheter b. Physician’s order to discontinue a peripheral intravenous catheter c. Nonaccessed implanted port placed 1 month ago without problem

d. Peripheral IV catheter dated 5 days ago used for once-daily antibiotics

In examining a peripheral IV site, the nurse observes a red streak along the length of the vein, and the vein feels hard and cordlike. What action

by the nurse takes priority? a. Applying continuous heat b. Continuing to monitor site

c. Elevating the extremity d. Removing the catheter

When an IV pump alarms because of pressure, what action does the nurse take first?

a. Check for kinking of the catheter. b. Flush the catheter with a thrombolytic enzyme.

c. Get a new infusion pump. d. Remove the IV catheter.

The nurse is caring for four clients receiving IV therapy. Which client does the nurse assess first?

a. Client with a newly inserted peripherally inserted central catheter (PICC) line waiting for x-ray

b. Client with a peripheral catheter for intermittent infusions

c. Older adult client with a nonaccessed implanted port d. Older adult client with normal saline infusion

A client who is having a tunneled central venous catheter inserted begins to report chest pain and difficulty breathing. What action does the nurse

take first? a. Administer the PRN pain medication.

b. Prepare to assist with chest tube insertion. c. Place a sterile dressing over the IV site.

d. Place the client in the Trendelenburg position.

A client who has just had an IV started in the right cephalic vein tells the nurse that the wrist and the hand below the IV site feel like “pins and

needles.” Which action by the nurse is best? a. Document the finding and continue to monitor the IV site.

b. Check for the presence of a strong blood return. c. Discontinue the IV and restart it at another site.

d. Elevate the extremity above the level of the heart.

The home care nurse is about to administer intravenous medication to the client and reads in the chart that the peripherally inserted central

catheter (PICC) line in the client’s left arm has been in place for 4 weeks. The IV is patent, with a good blood return. The site is clean and free from manifestations of infiltration, irritation, and infection. Which

action by the nurse is most appropriate? a. Notify the physician.

b. Administer the prescribed medication. c. Discontinue the PICC line.

d. Switch the medication to the oral route.

Which assessment finding for a client with a peripherally inserted central catheter (PICC) line requires immediate attention?

a. Initial dressing over site is 3 days old. b. Line has been in for 4 weeks.

c. A securement device is absent. d. Upper extremity swelling is noted.

A nurse is changing the administration set on a client’s central venous catheter. Which intervention is most important for the nurse to

complete? a. Have the client hold his breath during the set change.

b. Keep the slide clamp on the catheter extension open. c. Position the client in a high Fowler’s position.

d. Position in the client in a semi-Fowler’s position.

When assessing a client’s peripheral IV site, the nurse notices edema and tenderness above the site. What action does the nurse take first?

a. Apply cold compresses to the IV site. b. Elevate the extremity on a pillow.

c. Flush the catheter. d. Stop the infusion of IV fluids.

What action does the nurse take to prevent infection in the older adult receiving IV therapy?

a. Applying skin protectant before applying the dressing b. Avoiding the use of alcohol pads when removing tape c. Shaving the skin before attempting the venipuncture

d. Using maximum friction to cleanse the skin

The nurse is caring for a client who is receiving an epidural infusion for

pain management. Which action has the highest priority? a. Assessing the respiratory rate

b. Changing the dressing over the site c. Using various pain management therapies

d. Weaning the pain medication

The nurse is caring for a client with a radial arterial catheter. Which assessment takes priority?

a. Amount of pressure in fluid container b. Date of catheter tubing change

c. Checking for heparin in infusion container d. Presence of an ulnar pulse

Five days after the start of intraperitoneal therapy, the client reports abdominal pain and “feeling warm.” The nurse prepares to assess the

client further for evidence of which condition? a. Allergic reaction

b. Bowel obstruction c. Catheter lumen occlusion

d. Infection

Which client is the best candidate to receive hypodermoclysis for IV therapy?

a. Client requiring 4000 mL normal saline in 24 hours b. Client with an extensive burn injury c. Client with allergy to hyaluronidase d. Client receiving pain management

The nurse is caring for a client with an intraosseous catheter placed in the leg 20 hours ago. Which assessment is of greatest concern?

a. Length of time catheter is in place b. Poor vascular access in upper extremities

c. Affected leg cool to touch d. Site of intraosseous catheter placement

A client is receiving an infusion of amiodarone (Cordarone), and the nurse notes that the client’s arm has begun to blister around the IV site.

This manifestation is consistent with which condition? a. Extravasation

b. Infiltration c. Infection d. Phlebitis

A client is to receive a blood transfusion. Before the transfusion, what action by the nurse takes priority? a. Verifying the client’s identity

b. Ensuring that the blood bank has enough blood c. Establishing a peripheral IV site

d. Feeding the client before starting the blood

The nurse has just performed an IV start on a client. After the catheter has been threaded its full length in the client’s vein, which action does

the nurse perform next? a. Secure the IV with a securement device or tape. b. Dispose of the IV needle in the sharps container. c. Engage the safety mechanism of the IV catheter

d. Note the date and time of the dressing application over the insertion site.

A new nurse is preparing to start an IV on a client who is dehydrated and needs significant fluid volume. The new nurse selects a butterfly needle for the infusion. What action by the supervising nurse is best?

a. Help the new nurse with the procedure as needed. b. Make sure the new nurse has the correct dressing.

c. Stop the new nurse and review the procedure in private. d. Get the ultrasonic vein finder to help illuminate veins.

A nursing student asks why midline catheters need strict sterile dressing changes when short peripheral IVs do not. Which answer by the

experienced nurse is most accurate? a. “Because of the length of time they stay inserted.” b. “They really don’t need strict sterile technique.”

c. “Because the tip is in the right atrium of the heart.” d. “The tonicity of the fluids used promotes infection.”

A nurse is preparing to administer two drugs at the same time to a client via a double-lumen midline catheter. Which action by the nurse is most

important? a. Check the two drugs for compatibility.

b. Compare the recommended infusion times. c. Schedule any post-infusion lab draws.

d. Flush both lumens with saline before starting the infusion.

A client has just had a central venous access line inserted. What is the nurse’s next action?

a. Beginning the prescribed infusion as soon as possible b. Confirming placement of the catheter by x-ray c. Having the infusion team start the IV therapy

d. Confirming that solutions are appropriate for the central line

A new nurse is securing the connections on a new IV administration set connected to a peripherally inserted central catheter (PICC) line with

tape. Which action by the precepting nurse is most appropriate? a. Make sure the tape being used is from a sterile IV start kit.

b. Stop the nurse and confirm that the Luer-Lok connections are tight. c. Help the new nurse document the set change appropriately.

d. Show the new nurse how to turn back the corner of the tape for easy removal.

The nurse is preparing to administer an infusion of dopamine (Intropin) using a smart pump. After programming the pump and attaching the IV

to the client, what action by the nurse is most important? a. Start the infusion as ordered.

b. Hand-calculate the infusion rate. c. Ensure that the pump is plugged in. d. Place a “time tape” on the IV bag.

A student nurse is preparing to take a blood pressure (BP) on a client who has a peripheral IV line in the left arm. What instruction by the

faculty member is most important? a. “Use the arm that doesn’t have the IV site in it.”

b. “Don’t inflate the cuff too high if you use the left arm.” c. “Make sure the IV line is secure before taking the BP.”

d. “While the BP is taken, a little backflow of the IV is okay.”

The nurse preparing to insert an IV on an older adult client notices that the client’s skin is extremely fragile. Which action by the nurse is best?

a. Use a blood pressure cuff to cause the vein to distend. b. Slap the skin vigorously to cause the vein to rise.

c. Place a gauze pad under the tourniquet before tightening. d. Avoid the use of a tourniquet if the vein is already hard.

The nurse is caring for a client admitted yesterday with an intraosseous (IO) infusion after a car crash. Which action by the nurse takes priority?

a. Ensure that the IV flow rate has been recalculated for an IO infusion.

b. Plan to insert another kind of IV line during the shift. c. Determine which IV medications can be given safely via the IO.

d. Monitor the site and dressings routinely for hemorrhage.

MULTIPLE RESPONSE

The RN is working with an experienced LPN (licensed practical nurse) who has been assigned several clients receiving IV therapy. What actions guide the RN in delegating aspects of IV therapy to the LPN? (Select all

that apply.) a. Look up and read the State Nurse Practice Act.

b. Check facility policy regarding LPNs and IV therapy. c. Ask the LPN what he or she is comfortable performing.

d. Supervise the LPN when performing IV therapy. e. Divide the clients up between the two of them.

The nurse is preparing to administer a medication IV push. What information does the nurse need to know before beginning the infusion?

(Select all that apply.) a. Any dilution required b. Rate of administration

c. Compatibility with infusions d. Other routes of administration

e. Specific monitoring needed

A client has a peripherally inserted central catheter (PICC) line and the primary nurse is updating the care plan. For which common complications does the nurse assess? (Select all that apply.)

a. Phlebitis b. Pneumothorax

c. Thrombophlebitis d. Excessive bleeding

e. Extravasation

The nurse is preparing to give a client an IV push medication through an intermittent IV set (saline lock) using a needleless system. Which actions

by the nurse are most appropriate? (Select all that apply.) a. Cleanse the access port vigorously for at least 30 seconds.

b. Use an antimicrobial agent when cleansing the port. c. Clean the ridges in the Luer-Lok connection well.

d. Rinse the antimicrobial agent off with saline. e. Allow the antimicrobial agent to dry before using IV.

SHORT ANSWER A client is scheduled to receive 1000 mL of normal saline in 24 hours. The

nurse should set the infusion pump to deliver how many milliliters per hour? mL/hr

If a client is to receive an entire 250-mL bag of saline over the next 4 hours and the drop rate of the IV tubing chamber is 15 drops/mL, at what drop rate per minute will the nurse set this IV?

drops/min

OTHER

The nurse is caring for an older adult client who has been admitted for

dehydration and needs IV fluids. Which location does the nurse choose to place a peripheral IV on this client?

The nurse is preparing to flush a PICC line. The protocol specifies using 50 units of heparin. Available is a multidose vial containing heparin, 10 units/mL. Which syringe does the nurse use to draw up and administer

the heparin?

Chapter 16: Care of Preoperative Patients

Test Bank

MULTIPLE CHOICE

A client voluntarily signed the operative consent form. What is the nurse’s next action?

a. Teach the client about the surgery. b. Have family members witness the signature.

c. Sign under the client’s name as a witness. d. Call for the physician to sign the form.

The nurse is caring for an older adult client with a history of chronic lung disease who will be undergoing surgery the following day. When postoperative care is planned, which potential problem is the highest

priority for this client? a. Maintaining oxygenation

b. Tolerating activity c. Anxiety and fear

d. Hypovolemia

The nurse is completing preoperative teaching for a client, and it becomes apparent that the client does not understand the surgery that

will be performed. What is the priority action for the nurse? a. Obtain informed consent from the client.

b. Continue teaching the client about the surgery. c. Revise the teaching plan for the client.

d. Notify the surgeon and document the finding.

During the preoperative assessment, the client tells the nurse that he smokes three packs of cigarettes daily. Which action by the nurse is best?

a. Call the surgeon to cancel the surgery. b. Have baseline laboratory studies drawn.

c. Perform a respiratory assessment.

d. Give a nebulizer treatment.

When the nurse brings a client’s preoperative medications, the client responds, “I don’t need that. I had a good night’s sleep last night.” What

is the nurse’s best response? a. “The doctor ordered this medication so you should take it.” b. “I will make a note that you refused to take the medication.” c. “I will ask your surgeon if you have to take the medication.”

d. “Let me teach you about your medications for surgery.”

A client receiving preoperative medication tells the nurse that she took all the following vitamins and herbs last night before going to bed. Which

one does the nurse report to the surgical team as a priority? a. Valerian root

b. St. John’s wort c. Garlic

d. Chamomile

The nurse reviews a client’s laboratory results before surgery and notes a fasting blood glucose of 120 mg/dL, a prothrombin time (PT) of 25

seconds, and potassium (K+) of 3.8 mEq/L. Which action by the nurse is best?

a. Ask the surgeon for additional laboratory studies. b. Administer a potassium supplement of 20 mEq. c. Increase the IV infusion of D5W to 100 mL/hr.

d. Record laboratory results on the preoperative assessment.

A client is brought to the emergency department (ED) after a motorcycle accident. The client has suffered a ruptured spleen. What is the

immediate priority? a. Emergent surgery to control bleeding

b. Aggressive pain control c. Calling the family members

d. Assessment of neurologic status

The nurse has just completed preoperative teaching with a client who will be having surgery the following day. Which statement by the client

indicates that additional teaching is needed? a. “When I brush my teeth before surgery, I will be sure to spit out the

water.” b. “I will go to the bathroom as soon as I receive all my preoperative

medications.” c. “I will remember to wear my glasses tomorrow instead of my

contact lenses.” d. “I won’t have to worry about putting my makeup on tomorrow

morning.”

The nurse is performing preoperative teaching with an older adult client who will be having colon resection surgery the following day. The

surgeon has ordered bowel preparation the night before. Which action is a priority?

a. Administer antibiotics with a sip of water. b. Encourage the client to drink plenty of juice.

c. Teach the client to eat only low-fat foods the night before surgery. d. Tell the client not to get up and go to the bathroom alone.

When examining an adult client’s preoperative laboratory results, the nurse notes that the potassium level is 2.9 mEq/mL. What is the nurse’s

priority action? a. Document the finding.

b. Alter the client’s diet to include fruit.

c. Increase the IV flow rate. d. Notify the surgeon.

What recently learned information about a client who is scheduled to have surgery within the next 2 hours is the nurse certain to communicate

to the surgical team? a. An allergy to cats b. Hearing problem

c. Consumption of a glass of wine 12 hours ago d. Taking 2000 mg of vitamin C each day

A client will be undergoing palliative surgery. The client’s daughter asks what this means. What is the nurse’s best response?

a. “The surgery will relieve the symptoms but will not cure your father.”

b. “There are fewer risks with this type of surgery.” c. “There is no guarantee of the outcome of the surgery.”

d. “The surgery must be performed immediately to save your father’s life.”

Twenty minutes after a client has received a preoperative injection of atropine and midazolam (Versed), the client tells the nurse that he must

be allergic to the medication because his mouth is dry and his heart seems to be beating faster than normal. What is the nurse’s priority

action? a. Document the findings.

b. Assess the client’s pulse and blood pressure. c. Administer diphenhydramine (Benadryl).

d. Explain to the client that these symptoms are expected.

A client undergoing preoperative assessment informs the nurse that he takes medication for high blood pressure and for asthma. What is the

nurse’s best action? a. Tell the client not to take the medication on the day of surgery.

b. Notify the surgeon and the anesthesiologist. c. Document the information in the client’s record.

d. Tell the client to take medications preoperatively with a sip of water.

Which action is most appropriate during a preoperative chart review? a. Ensure that the consent form is signed, dated, and witnessed.

b. Call the surgeon if the client has any food allergies. c. Make sure all marks are washed off the surgical site.

d. Make sure the client understands the procedure.

The nurse is caring for a client who will be undergoing emergency surgery as soon as possible. Which information is most important for the

nurse to teach the client at this time? a. How the surgery will be performed

b. Importance of early ambulation after surgery c. What to expect in the operating and recovery rooms

d. Complications that may occur after surgery

A client tells the nurse that he has an advance directive with durable power of attorney for health care. The client asks how the advance directive will affect the surgery. What is the nurse’s best response? a. “You will not be intubated during general anesthesia for the

surgery.” b. “There will be no effect on your surgery.”

c. “The surgical staff will resuscitate only if your heart stops during the operation.”

d. “If you are unable to make a decision, your designee will be asked.”

A client is brought to the hospital unconscious and needs emergency surgery. The client’s only family member cannot come to the hospital before the surgery. Which is the best option for obtaining informed

consent for the client’s emergent surgery? a. Proceed with surgery and have the family member sign the consent

as soon as possible. b. Contact the family member by phone and obtain verbal consent

with two witnesses. c. Obtain written consultation with two surgeons that the surgery is

needed. d. Have the hospital administrator appoint a temporary legal

guardian.

Four clients are scheduled for surgery. Which client does the nurse determine is at highest risk for postsurgical complications?

a. 89-year-old scheduled for a knee replacement b. 40-year-old requiring gallbladder surgery

c. 19-year-old requiring a laparoscopy d. 10-year-old admitted for a tonsillectomy

The nurse is conducting preoperative assessments. Which client does the nurse teach about the possibility of developing a venous

thromboembolism (VTE)? a. Client with a latex allergy

b. Client with body mass index (BMI) of 19 c. Client with an international normalized ratio (INR) of 2.2

d. Client undergoing hip replacement surgery

The nurse applies antiembolism stockings to a client preoperatively. When the client says that they are uncomfortably tight, what is the

nurse’s best action? a. Remove the stockings for an hour to relieve the pressure. b. Pull the stockings down so that they are not constricting.

c. Measure the client’s calf to ensure that they are the correct size. d. Teach the client the purpose of wearing the stockings.

MULTIPLE RESPONSE

The nurse is assessing a client before surgery. Which assessments contraindicate the client having surgery as scheduled? (Select all that

apply.) a. Potassium level of 2.8 mEq/L

b. International normalized ratio (INR) of 4 c. Prothrombin time (PTT) of 30 seconds

d. Calcium level of 8.8 mEq/dL e. Positive pregnancy test

f. Platelet count of 150,000

Which medications does the nurse correctly administer preoperatively? (Select all that apply.)

a. Hydroxyzine (Atarax, Vistaril) for sedation b. Lorazepam (Ativan) for anxiety

c. Hydromorphone (Dilaudid) to decrease postoperative secretions d. Metoclopramide (Reglan) to increase stomach emptying

e. Aspirin to decrease blood clotting postoperatively f. Cimetidine (Tagamet) to prevent infection

The nurse is conducting preoperative teaching with a client who will be undergoing pelvic surgery. What teaching is essential for this client?

(Select all that apply.) a. “Wearing elastic stockings and using pneumatic compression

devices are essential after surgery.” b. “Extended bedrest will help you heal after this type of surgery.”

c. “Coughing and deep breathing will help to decrease postoperative complications.”

d. “Turning and moving your legs after surgery will help prevent clots from forming.”

e. “You will need to have your abdomen shaved before surgery.” f. “You cannot wear your hearing aid into the surgical suite.”

What data are essential for the nurse to assess on a client who is scheduled for surgery? (Select all that apply.)

a. Use of tobacco b. Current medications

c. Use of herbal or over-the-counter therapy d. Mental status examination

e. Power of attorney f. Allergies

g. Date of last tetanus shot

OTHER

The nurse is preparing to transfer a client to the operating room for surgery. Put the interventions in order for the nurse to perform. (List in

order of priority.) Take a full set of vital signs.

Have the client go to the bathroom to void. Ask the client to state his or her name and check the ID band.

Administer ordered preoperative sedation.

Chapter 17: Care of Intraoperative Patients

Test Bank

MULTIPLE CHOICE

Which observed action indicates that the nurse is performing the surgical scrub correctly?

a. A small brush is used to scrub under nails and wedding ring. b. The surgical mask is put on before starting the surgical scrub.

c. The soap is rinsed off so that the water runs down to the hands. d. A paper towel is used to turn off the faucet handle.

A client is having epidural anesthesia for knee replacement surgery. Which action by the nurse is the priority during this surgery?

a. Provide emotional support for the client. b. Position the client comfortably and safely.

c. Stay with the client until sedation is effective. d. Teach the client cough and deep-breathing exercises.

Which action indicates to the operating room supervisor that the scrub nurse requires additional teaching about sterile technique?

a. A small amount of sterile saline is poured out before it is poured into the basin.

b. The nurse disposes of any equipment packages that are in poor condition.

c. Sterile surgical supplies are placed in the center of the sterile field. d. The sterile saline bottle cap is placed in the center of the sterile field.

What is the priority action for the scrub person at the conclusion of a surgical procedure?

a. Assist with transferring the client to the postanesthesia care unit. b. Document the procedure in the client’s medical record.

c. Set up the sterile field and drape the client appropriately. d. Document how many sponges and sharps have been utilized.

Before a client’s surgery begins, the circulating nurse notes that the nurse anesthetist did not perform a surgical scrub before coming into the

operating room. Which action by the circulating nurse is most appropriate?

a. Direct the nurse anesthetist to perform the surgical scrub immediately.

b. Proceed with positioning the client on the operating bed. c. Notify the nursing supervisor that sterile technique has been

violated. d. Proceed with setting up the instruments to be used during surgery.

A client is having spinal anesthesia for knee surgery. Which statement by

the client indicates a good understanding of this type of anesthesia? a. “I won’t have to worry about having an allergic reaction.”

b. “I will be able to walk sooner after your surgery.” c. “I will have less risk of developing pneumonia after surgery.”

d. “I will have less risk of bleeding with epidural anesthesia.”

The client is to have a surgical procedure under (moderate) conscious sedation. The client is anxious and asks the nurse what to expect. What is

the nurse’s best response? a. “You will be awake and alert during the procedure but you will feel

no pain.” b. “You will not be able to move your feet or toes during the

procedure.” c. “You will not be able to swallow or talk during the procedure.”

d. “You will be very sleepy and we will monitor you closely.”

The nurse is caring for a client who has had conscious sedation. What is the primary advantage of this type of anesthesia? a. The client can talk through the procedure.

b. The client is able to follow directions. c. No respiratory support is needed.

d. No defensive reflexes are lost.

Before the nurse brings the client to the operating room (OR) for knee surgery, the client reports to the nurse that he did not mark the operative

knee with the surgeon. What is the priority action of the nurse? a. Proceed with transferring the client to the OR as planned.

b. Call a “time out” so the site can be marked before surgery begins. c. Call the surgeon to mark the site with the client before transfer to

the OR. d. Have the client mark the site before transfer to the OR.

The nurse is preparing to bring a young female client to the operating room for a total abdominal hysterectomy (TAH). The client says to the nurse, “I am so glad that I will still be able to have children after this

surgery.” What is the nurse’s best response? a. “That is very good news. How many children do you want?”

b. “Weren’t you taught about your surgery earlier?” c. “You must have misunderstood your surgeon.”

d. “I will call the surgeon to speak with you before surgery.”

An older adult client is being positioned on the operating bed for surgery. Which action is the highest priority for the nurse?

a. Placing gel pads under the client’s shoulders and head b. Placing a soft pillow between the client’s knees

c. Ensuring that the head is elevated to working height d. Assessing skin condition for the need for additional padding

An anesthetized client must be repositioned from the supine to the prone position midway through a surgical procedure. What is the priority

action of the nurse? a. Log roll the client to prevent dislocation of the shoulder.

b. Keep the client covered to maintain dignity and minimize heat loss. c. Ensure that the client’s endotracheal tube does not become

dislodged or kinked. d. Make sure that the client’s indwelling catheter is kept lower than the

bladder.

Which action by the surgical nursing staff indicates that additional teaching is required about nurses’ roles and responsibilities in the

operating room?

a. The circulating nurse and the anesthesiologist accompany the client to the postanesthesia care unit.

b. The circulating nurse goes to the blood bank to pick up 2 units of fresh-frozen plasma for the client.

c. The scrub nurse monitors the amount of irrigation fluid that is used during surgery.

d. The circulating nurse prepares the surgical site before the client is covered with sterile drapes.

The nurse is helping to position a client on the operating bed when the client states, “I am really nervous about having the breathing tube put

down my throat.” What is the nurse’s best response? a. “I will give you some medication so that it won’t hurt.”

b. “The tube is very small and you will hardly know it is there.” c. “The anesthesiologists are experts at this procedure.”

d. “The anesthetist will put the tube in your throat after you are asleep.”

Which statement indicates accountability by the scrub nurse during a surgical procedure?

a. “The client should have epidural anesthesia rather than general anesthesia.”

b. “The client’s endotracheal tube is secured and all monitors are in place.”

c. “I will have retention sutures ready for the surgeon.” d. “A surgical sponge is missing so I will do a re-count.”

The nurse is assisting a client to the operating bed and notes that the client is hyperventilating and cannot keep still. The client states, “I am

really very anxious right now.” What is the nurse’s best action? a. Call the chaplain to calm the client down.

b. Tell the client you will stay with him or her. c. Inform the surgeon so the procedure can be cancelled.

d. Inform the anesthesiologist and suggest antianxiety medication.

Surgery is almost completed for an obese client with diabetes, and the surgeon prepares to close a large abdominal incision. What is the scrub

nurse’s best action? a. Count the number of sponges used.

b. Ask the circulating nurse to count sponges. c. Assist the surgeon with retention sutures.

d. Administer an antibiotic.

The nurse is caring for a client who will be having surgery with spinal anesthesia. The client says to the nurse, “I changed my mind—I don’t want to be awake during surgery!” What is the nurse’s best response?

a. “Spinal anesthesia is safer than being put to sleep with general anesthesia.”

b. “The anesthesiologist has already determined this is best for your surgery.”

c. “It’s too late to change now because all the equipment is in place.” d. “I will call the anesthesiologist to come and talk to you.”

A client is recovering from abdominal surgery and reports unusual muscle pain. The nurse reviews the operative record and notes that the

client received propofol (Diprivan) and ketamine (Ketalar). Which action by the nurse is most appropriate?

a. Request a physical therapy consult. b. Encourage the client to ambulate.

c. Administer the ordered pain medication. d. Call the surgeon and request a potassium level.

MULTIPLE RESPONSE

Before undergoing general anesthesia, the client states, “My brother and

my father had bad reactions to anesthesia. I hope that doesn’t happen to me!” What are the nurse’s best actions? (Select all that apply.)

a. Hook up leads to a temporary pacemaker. b. Have a nasogastric tube ready for insertion.

c. Assess the client’s chest x-ray before surgery. d. Have a cooling blanket ready.

e. Obtain a chest tube insertion kit. f. Have a Foley catheter kit ready.

g. Provide an emergency tracheostomy kit at the bedside. h. Inform the anesthesiologist and the surgeon of the client’s statement.

A client is undergoing an operation under general anesthesia. What are the nurse anesthetist’s best actions to prevent hypoventilation during the

operation? (Select all that apply.) a. Monitor breathing and circulation continuously.

b. Monitor blood pressure and heart rate every 5 minutes. c. Make sure the anesthesia provider remains in the room.

d. Elevate the head of the client’s bed or stretcher. e. Monitor the cardiac rhythm every 30 minutes.

Which clients does the nurse determine have the highest risk for wound infection? (Select all that apply.)

a. Client who has psoriasis b. Middle-aged woman with a body mass index (BMI) of 30

c. Older adult client with a creatinine level of 4.0 d. Client with a family history of malignant hyperthermia

e. Client with peripheral vascular disease f. Teenager with diabetes mellitus type 1

A client is having an operation. Which assessment findings concern the nurse the most? (Select all that apply.)

a. Potassium level of 4.2 mEq/dL b. Calcium level of 12 mg/dL c. Heart rate of 110 beats/min d. Oxygen saturation of 95%

e. pH of 7.37 f. Blood pressure of 90/40 mm Hg

Chapter 18: Care of Postoperative Patients

Test Bank

MULTIPLE CHOICE

The nurse is caring for a client who had surgery 24 hours ago. He is alert and oriented when awakened and reports pain, but goes back to sleep

when not being stimulated. He is on patient-controlled analgesia (PCA). What is the nurse’s next action?

a. Push the PCA control for the client. b. Discontinue the PCA immediately.

c. Assess the client’s respiratory status. d. Keep the client awake as much as possible.

Postoperatively, a client has a heart rate of 120 beats/min, with dysrhythmias noted on the ECG monitor and a respiratory rate of 34

breaths/min, and is very difficult to arouse. Which action by the nurse is most appropriate?

a. Accompany the client to the postanesthesia care unit (PACU). b. Keep the client in the surgical suite.

c. Call a code or the Rapid Response Team. d. Transfer the client to the intensive care unit (ICU).

A client has been transferred to the postanesthesia care unit (PACU). Which action does the receiving nurse perform first?

a. Complete a nursing assessment sheet. b. Change the client’s arm band.

c. Enter client data into the computer. d. Participate in a hand-off report.

The nurse is performing a hand-off report in the PACU. What is the best action for the nurse to perform during the hand-off report?

a. Write all information on a chart and hand it to the nurse who will assume care of the client.

b. Follow the nurse assigned to the new client and give a verbal report that does not interrupt care.

c. Focus on the report and sit with the nurse receiving the client to give a detailed report.

d. Finish the report quickly so the nurse can assume care of the client.

The nurse reviews the initial postanesthesia care unit (PACU) flow record and notes that the client is alert and oriented ´3 when stimulated, pulse is 88 per minute and regular, respirations are 12 per minute and unlabored, and oxygen saturation is 95% on 2 LPM of nasal oxygen.

What is the nurse’s priority action at this time? a. Examine the surgical site; obtain blood pressure and temperature. b. Suction the client and assess anterior and posterior lung sounds.

c. Assess urinary output, the IV site, and the client’s pain. d. Turn the client and perform chest physiotherapy.

A client who has just been transferred to the postanesthesia care unit (PACU) from surgery is very restless and confused. What is the nurse’s

first action? a. Orient the client and remain with him or her. b. Call the surgeon for an intraoperative report.

c. Notify the physician on call. d. Assess the client’s level of pain.

A client had surgical repair of a fractured ankle under local anesthesia and is being transferred from the postanesthesia care unit (PACU) to the

surgical floor. Once admitted, what is the nurse’s priority action? a. Assess pressure points for breakdown.

b. Assess the client’s pain. c. Insert an IV for antibiotic therapy.

d. Assess a full set of vital signs.

The nurse is assessing a client admitted to the postanesthesia care unit (PACU) after abdominal surgery. The client’s respiratory rate is 8

breaths/min and breath sounds are decreased in the bases. What is the nurse’s priority action?

a. Prepare to administer naloxone (Narcan). b. Assess oxygen saturation and level of consciousness.

c. Call a code or the Rapid Response Team. d. Turn the client and perform chest physiotherapy.

The nurse assesses a client who has just been brought to the postanesthesia care unit (PACU). In the operating room, the client’s

blood pressure was 136/80 mm Hg; it is now 110/80 mm Hg. Urine output was 40 mL/hr and is now 10 mL/hr. Which action by the nurse is best?

a. Awaken the client and encourage oral fluids. b. Increase the IV of 0.9 NS as ordered to 100 mL/hr.

c. Put the client in Trendelenburg position. d. Assess the client’s levels of consciousness and pain.

The nurse is caring for a client who has just been brought to the postanesthesia care unit (PACU) after surgery. The client’s oxygen

saturation is 92% and his hemoglobin is 14 g/dL. What is the nurse’s first action?

a. Assess the client’s pain response. b. Determine whether the client is alert and oriented.

c. Increase oxygen and auscultate lung sounds. d. Assess vital signs and temperature.

A client is brought to the postanesthesia care unit (PACU) after surgery that took place with the client in the lithotomy position. Which action

does the nurse take after assessing vital signs? a. Assess for sacral decubiti.

b. Assess dorsalis pedis pulses. c. Turn the client on the left side.

d. Put the client in the Trendelenburg position.

A client is being transferred to the postanesthesia care unit (PACU) after surgery. The client has an endotracheal tube (ET) in place. On

assessment, the client has oxygen saturation of 95%, respiratory rate of 14 breaths/min, and asymmetric chest wall expansion. What is the

nurse’s best action? a. Attempt to awaken the client.

b. “Bag” the client with a resuscitation bag. c. Increase the client’s fraction of inspired oxygen (FIO2).

d. Auscultate lung sounds bilaterally.

A client who is being admitted to the postanesthesia care unit (PACU) has weak hand grasps on assessment and is unable to lift his head off the bed. During hand-off report, the nurse notes that the client has received

a neuromuscular-blocking agent. What is the nurse’s best action?

a. Document the finding. b. Check the client’s pulses.

c. Place the client in Fowler’s position. d. Auscultate the lungs.

The nurse is caring for clients in the postanesthesia care unit (PACU). Which client does the nurse intervene for first?

a. Client with a pulse deficit of 15 b. Client who is reporting leg pain

c. Client with dementia who is confused d. Client who is reporting a headache

The nurse is caring for several clients on the postoperative unit. Which client does the nurse determine has the highest risk of respiratory

complications after general anesthesia? a. Older woman taking a calcium channel blocker for hypertension

b. Middle-aged man with a deviated nasal septum c. Middle-aged woman taking St. John’s wort daily for depression

d. Young adult with a body mass index of 40

One hour after admission to the postanesthesia care unit (PACU), the postoperative client has become very restless. What is the nurse’s first

action? a. Assess for bladder distention.

b. Assess the oxygen saturation level. c. Call the surgeon to assess the client.

d. Administer pain medication as ordered.

The nurse is caring for a client in the postanesthesia care unit (PACU) 2 hours after abdominal surgery. The nurse auscultates the client’s

abdomen and notes that there are no bowel sounds. What action does the nurse take?

a. Position the client on the left side with the bed flat. b. Insert a nasogastric tube to low intermittent suction. c. Palpate the bladder and measure abdominal girth.

d. Document the finding and continue to monitor.

The nurse is changing the client’s dressing on the second postoperative day and notes a small amount of serosanguineous drainage. What is the

nurse’s best action? a. Cleanse the suture line and apply a sterile dressing.

b. Culture the drainage and leave the incision open to air. c. Cover the incision with a transparent dressing.

d. Notify the surgeon to assess the client.

The nurse is caring for a client who had abdominal surgery 3 days ago. He tells the nurse, “I felt something ‘give way’ when I coughed.” What is

the nurse’s best response? a. “It is good that you are coughing and deep-breathing to prevent

pneumonia.” b. “That is a normal feeling in the incision whenever you are moving.” c. “Be sure to splint the incision with a pillow or your hands when you

cough.” d. “Lie down flat on the bed with your knees up and let me examine

your incision.”

A client reports pain 8 hours after surgery. The client has already

received an opioid within the past 2 hours. What is the nurse’s best action?

a. Assess the pain further. b. Administer naloxone (Narcan).

c. Call the surgeon. d. Document the finding.

A client with diabetes mellitus type 1 underwent surgery 24 hours ago. Which precaution does the nurse take to help prevent postoperative

complications for this client? a. Order a high-protein diet.

b. Observe the incision frequently. c. Have suction available at the bedside.

d. Instruct the client to use an electric razor.

The nurse is providing discharge teaching for a client who will be going home with a Jackson-Pratt (JP) drain. Which statement indicates that

the client understands how to care for the drain correctly? a. “I will flush the tubing to make sure that it stays open.”

b. “I will measure the drainage before I discard it.” c. “I will close the drain valve and then compress the bulb to create

suction.” d. “I will pull it out once the surgeon says I don’t need it anymore.”

The nurse is caring for a client who has had surgery the previous day. The client tells the nurse, “Breathing in using this thing (incentive spirometer) is a ridiculous waste of time.” What is the nurse’s best

response? a. “The spirometer will help you cough effectively.”

b. “The spirometer will help your lungs expand.” c. “The spirometer will help prevent blood clots.”

d. “The spirometer will improve blood flow in your lungs.”

After discharge from the postanesthesia care unit (PACU), the client returned to the surgical nursing unit at 10 AM. It is now 6 PM, and the client is not experiencing any complications. How often does the nurse

assess the client’s vital signs? a. Every 15 minutes b. Every 30 minutes

c. Every hour d. Every 4 hours

The nurse is caring for clients in the postanesthesia care unit (PACU).

Which client is ready to be extubated? a. Client with an oxygen saturation of 90%

b. Client with a respiratory rate of 14 breaths/min c. Client who is alert and oriented

d. Client who is coughing and gagging

The nurse is working in the postanesthesia care unit (PACU) and receives a client from the operating room (OR). What does the nurse

assess first? a. Client’s endotracheal tube b. Client’s nasogastric tube c. Client’s Foley catheter

d. Hemovac drain at the incision site

The nurse is caring for a client whose wound dehisces after vomiting. What is the nurse’s first action?

a. Prepare the client for emergency surgery. b. Cover the wound with sterile moist dressings.

c. Give the client medication for nausea. d. Call the surgeon and the operating room.

The nurse is changing the dressing on a postoperative client’s abdominal incision. A Jackson-Pratt (JP) drain is present, along with a moderate amount of serosanguineous drainage. What is the best product for the

nurse to use in performing wound care? a. Half hydrogen peroxide and half sterile saline

b. Sterile water and antibacterial ointment c. Betadine swabs or alcohol wipes

d. Sterile normal saline

The nurse is preparing a client for discharge. The client has a large draining wound. What is the nurse’s best action?

a. Arrange a nurse to come to the house to change the dressing after discharge.

b. Have the client come back to the clinic daily to have the dressing changed.

c. Teach the client and family how to change the dressing. d. Apply a hydrocolloid dressing and change once a week.

A postsurgical client’s urinary output via the Foley catheter is 30 mL in 3 hours. What is the nurse’s first action?

a. Increase the IV infusion rate. b. Assess the client’s skin turgor.

c. Weigh the client. d. Check the patency of the catheter.

The nurse is assisting a client to ambulate several hours after his surgery. The client coughs and says to the nurse, “I feel like something ripped in

my incision.” A large amount of blood is suddenly apparent on the client’s gown near the incision. What action does the nurse take first?

a. Ease the client to the floor and call for assistance. b. Put immediate pressure over the incision with the hands.

c. Call the Rapid Response Team to assess the client. d. Lift up the gown and take off the dressing.

A client is scheduled for an operation. What does the nurse teach the client about postoperative pain control?

a. “You should not ask for IV pain medication more than once every 4 or 5 hours.”

b. “You should not take the pain medication if you are nauseated.” c. “You will not get pain medication until you are transferred to the

floor.” d. “You should ask for pain medication before the pain becomes

severe.”

The nurse is caring for several postoperative clients on the unit. Which client does the nurse assess first?

a. Client with 200 mL dark drainage from the nasogastric tube in an hour

b. Client who received oral pain medication 20 minutes ago c. Client who has not yet ambulated after surgery 4 hours ago

d. Client requiring discharge teaching and whose family is present

The nurse is reviewing postoperative medication orders. Which order can the nurse implement?

a. Acetaminophen orally PRN pain b. Meperidine (Demerol) 75-100 mg every 3-4 hours PRN pain

c. MS .5 mg subcutaneously every 1-3 hours PRN pain d. Hydromorphone hydrochloride (Dilaudid) 1 mg orally every 4 hours

PRN pain

The nurse is caring for a client who is reporting severe postoperative pain. The physician’s order states that the client is to receive

“hydromorphone hydrochloride (Dilaudid) 10-15 mg every 1-2 hours

PRN pain.” What is the nurse’s priority action? a. Call the physician to clarify the order.

b. Give the medication as ordered. c. Refuse to give the medication. d. Call the hospital pharmacist.

Which instruction does the nurse provide to a client to prevent postoperative venous thromboembolism?

a. “Cough and deep-breathe six times every hour after surgery.” b. “Use your incentive spirometer hourly.” c. “Get up and walk as much as possible.”

d. “Keep the sterile dressing on your incision.”

The nurse is assessing clients in the postanesthesia care unit (PACU). A client is shivering and has a temperature of 95.4° F (35.2° C). What is the

nurse’s best action? a. Get the client warm blankets. b. Elevate the head of the bed. c. Auscultate the client’s lungs.

d. Assess the client’s oxygen saturation.

The nurse assesses clients in the postanesthesia care unit (PACU). Which client does the nurse intervene for first?

a. Client with a respiratory rate of 12 breaths/min b. Client with an oxygen saturation of 92% c. Client who is reporting pain (5 out of 10)

d. Client with audible stridor

A client is being discharged after abdominal surgery. What information about the diet does the nurse teach the client? a. “Be sure to monitor your fluid intake.”

b. “Eat foods high in protein and vitamin C.” c. “Call the physician if you develop gas.”

d. “You will need to limit your carbohydrates.”

A client has received an overdose of a benzodiazepine. What medication does the nurse anticipate an order for?

a. Flumazenil (Romazicon) b. Naloxone (Narcan)

c. Acetylcysteine (Mucomyst) d. Digoxin immune fab (Digibind)

MULTIPLE RESPONSE

Which clients are at increased risk for postoperative nausea and vomiting? (Select all that apply.)

a. Older adult with a history of hypertension b. Client who was in the lateral position during surgery

c. Middle aged client with a body mass index (BMI) of 46 d. Woman who has undergone a cholecystectomy

e. Young adult who received 3 L of IV fluid during surgery f. Man who has a history of seasickness

g. Man who has a nasogastric tube to suction

A postoperative client is receiving morphine for pain. For which side

effects does the nurse monitor this client? (Select all that apply.) a. Hypotension

b. Respiratory depression c. Constipation

d. Increased intracranial pressure e. Altered bleeding times

OTHER

A surgical procedure has just been concluded for a client who received a general anesthetic. Place the interventions in order of implementation.

(Select in order of priority.) Determining pain response

Assessing the IV Taking the client’s vital signs Applying warmed blankets

SHORT ANSWER

The nurse is caring for a postoperative client with a nasogastric (NG) tube to suction. The collection container was marked at 125 mL at 7 AM. At 3 PM, 675 mL was in the container. During the shift, the nurse used

45 mL of saline to irrigate the tube three times as prescribed by the physician. What is the total amount of drainage from the NG tube that is

entered into the client’s record? mL

The nurse is to administer 1 mg of butorphanol tartrate (Stadol) IV to a postoperative client. Stadol is available as 2 mg/mL. How much Stadol

does the nurse administer to the client? mL

A client is receiving morphine via patient-controlled analgesia (PCA) pump. Morphine is available in a 5-mg/mL solution. The basal rate is 0.8

mg/hr. What is the total volume the client will receive in 24 hours? mL

Chapter 19: Inflammation and the Immune Response

Test Bank

MULTIPLE CHOICE

A client has a reduction in immune function. What is the nurse’s priority action for this client?

a. Determine whether it is temporary or permanent. b. Take the client’s vital signs every 4 hours.

c. Teach family members to receive the flu shot yearly. d. Wash hands before entering the room.

Which client is at highest risk of compromised immunity? a. Client who has just had surgery

b. Client with extreme anxiety c. Client who is awaiting surgery

d. Client who just delivered a baby

A client who has an extensive burn injury develops inflammation that covers the entire body. What is the nurse’s best action?

a. Notify the health care provider immediately. b. Document the assessment.

c. Take the client’s temperature. d. Ask for an order for antibiotic therapy.

A client’s absolute neutrophil count (ANC) is 550/mm3. What is the nurse’s best action?

a. Use Standard Precautions. b. Place the client on antibiotic therapy.

c. Place client on a low-sodium diet. d. Administer chemotherapy.

The nurse receives a report that a client’s laboratory results show a left shift or an increase in circulating band neutrophils. What is the nurse’s

best action? a. Keep the client on bedrest.

b. Prepare the client for surgery. c. Increase the client’s oxygen flow rate.

d. Assess the client’s vital signs.

A client has an injury to the right ankle. On assessment, the nurse notes that it is red and inflamed. The nurse adds interventions to the care plan

that address which factor? a. An injury that is infected

b. Inflammation without infection c. A secondary infection

d. Dermatitis around the ankle

The nurse is teaching a client with a leg injury and warmth around the injured area. Which statement by the client indicates a good

understanding? a. “The warmth indicates an infection.”

b. “The warmth indicates increased blood flow.” c. “Warmth indicates that the tissues are rebuilding.” d. “Warmth results from localized vasoconstriction.”

A client enters the emergency department (ED) with an injury to the wrist. In assessment, the nurse notes that the area is red, warm, and

edematous. What is the nurse’s best action? a. Apply a heating pad to the area.

b. Inject pain medication directly at the site. c. Start an IV infusion of a vasoconstrictive drug.

d. Assess circulation and elevate the extremity.

The nurse is assessing a client who has a wound on the left calf. Drainage is coming from the wound. What does the nurse tell the client about this

finding? a. “Exudate or drainage is a natural occurrence with inflammation.”

b. “Exudate or drainage means the wound is infected.”

c. “Drainage from a wound is never a good sign.” d. “All wounds result in bleeding and pus formation.”

The nurse is caring for a client who has undergone a kidney transplant. The client asks the nurse what will happen when his body realizes that

the kidney is not “his.” What is the nurse’s best response? a. “The immune system will try to destroy the kidney if we don’t

suppress it.” b. “As long as the kidney is a ‘match’ to your blood type, there will be

no problem.” c. “You will develop a fever or other complications from the

transplant.” d. “Within a week, your body will ‘adjust’ to the new organ.”

The nurse assesses a cut that is 24 hours old and finds that the site is swollen, red, and tender to the touch. Which cell types are responsible for

these assessment findings? a. Erythrocytes and platelets b. Basophils and eosinophils

c. Plasma cells and B-lymphocytes d. Natural killer cells

A client had a splenectomy. Which fact is most important to teach the client regarding immune function?

a. “You won’t get a fever with infection, so you need to learn to identify other symptoms.”

b. “It will no longer be necessary for you to worry about developing allergies.”

c. “Avoid people who are ill because it will be harder for you to develop antibodies.”

d. “You will need to be assessed yearly for the risk of developing cancer.”

A client’s white blood cell (WBC) count value is 10,000 cells/mm3. The nurse reviews the differential. Which counts or percentages is the nurse

sure to report to the provider? a. Eosinophils, 200/mm3

b. A left shift in the white count

c. Segmented neutrophils, 6000/mm3 d. Basophils 100/mm3

The nurse is assessing a client who cannot synthesize suppressor T-cells. For what other condition does the nurse assess the client?

a. Increased seasonal allergies b. Multiple sclerosis

c. Leukemia d. Graft-versus-host disease

An older adult client tells the nurse that her granddaughter has chickenpox. The client is afraid to visit because she is afraid of getting shingles from her granddaughter. What is the nurse’s best response?

a. “If you already had chickenpox, you cannot get shingles.” b. “If you already had shingles, you cannot get them again.” c. “If you already had chickenpox, you can safely visit your

granddaughter.” d. “Shingles is caused by a different virus than the chickenpox virus.”

A client recovering from hepatitis A asks whether he should take the vaccine to avoid contracting the disease again. What does the nurse say?

a. “Yes, because now you are more susceptible to this infection.” b. “Yes, because the hepatitis A virus changes from year to year.”

c. “No, your liver and immune system are too impaired at this time.” d. “No, having the infection has done the same thing a vaccination

would.”

A client reports severe hay fever and allergic rhinitis. Which finding does the nurse expect to see in this client’s laboratory results? a. Band neutrophils outnumber segmented neutrophils.

b. The basophil count is 50/mm3. c. The eosinophil count is 20%. d. The white count is 7500/mm3.

A client has been diagnosed with a deficiency of complement proteins. Which assessment is the item of highest priority for the nurse to

perform? a. Joint stiffness and range of motion

b. Enlarged lymph nodes and night sweats c. Rhinorrhea and conjunctivitis

d. Lung sounds, cough, and oxygen saturation

The nurse prepares to administer a tetanus toxoid vaccination to a client who has suffered a puncture wound. The client reports that he had a

tetanus shot just 1 year ago. What is the nurse’s best action? a. Give the vaccination because strains of tetanus change yearly.

b. Refrain from giving the vaccination if the client is reliable. c. Give a smaller dose because antibody production slows down with

aging. d. Give the shot because it won’t hurt to receive an extra dose of the

toxoid.

The nurse is caring for an older postoperative client. Which assessment finding causes the nurse to assess further for a wound infection? a. Moderate serosanguineous drainage is seen on the dressing. b. The client is now confused but was not confused previously.

c. The white blood cell differential indicates a right shift. d. The white blood cell count is 8000/mm3.

The nurse is providing discharge teaching for a client following a liver transplant. Which statement by the client indicates that additional

teaching is needed?

a. “If I develop an infection, I should stop taking the steroid preparation.”

b. “If I have tenderness in my abdomen, I will call the physician.” c. “I should avoid people who are ill or who have an infection.”

d. “Cyclosporine (Sandimmune) won’t work as well if I change the routine.”

The nurse is caring for a client who is depressed because of acute rejection following a kidney transplant. What is the nurse’s best

response? a. “This is what happens when you don’t take your transplant

medications.” b. “At least you can still have dialysis, unlike people who receive liver

transplants.” c. “One acute rejection episode does not mean that you will lose the

new kidney.” d. “You can always find another donor and get another kidney

transplant.”

When an antibody titer to varicella zoster virus is performed on a nurse, the titer is negative. Which result and action by the nurse’s supervisor

are most appropriate? a. The nurse has chickenpox and is contagious. The supervisor sends

the nurse home.

b. The nurse has never been infected with varicella zoster virus. The supervisor assigns another nurse to care for clients with chickenpox.

c. The nurse had a subclinical case of herpes at least 5 years ago and is now immune to the disease. The supervisor assigns the nurse to a client

with chickenpox. d. The nurse has never been infected with varicella zoster virus and is susceptible to herpes. The supervisor assigns another nurse to a client

with herpes.

Which type of immunity does the hepatitis B immune globulin provide for the nurse? a. Passive

b. Artificial active c. Natural active d. Cell-mediated

The nurse is teaching a client who has recently given birth about immunity that has been passed to the newborn. Which statement by the

client indicates that additional teaching is needed? a. “My baby received some antibodies from me before birth, and I will

give him more when I breast-feed.” b. “I had the measles, so my baby will be protected against it until he is

old enough to receive the MMR vaccine.” c. “I had chickenpox and am immune to it, so my baby will not need to

have the chickenpox vaccine.” d. “Only certain antibodies were able to cross the placenta to protect

my baby.”

The nurse reviews the laboratory results of a client and finds that the white blood cell (WBC) count is 1500/mm3. What is the priority action of

the nurse? a. Have the client wear a mask at all times.

b. Obtain a urine sample for culture and sensitivity. c. Administer two units of fresh-frozen plasma.

d. Institute reverse isolation precautions.

The nurse has sustained a needle stick injury and received a dose of hepatitis B immune globulin. Which statement indicates that the nurse

understands this intervention? a. “I don’t need to receive the hepatitis B vaccine because I already

had the immune globulin.” b. “I will need to receive only two doses of the hepatitis B vaccine

because I had one dose of the immune globulin.” c. “I need to start the hepatitis B vaccination series as soon as

possible.” d. “I will make an appointment to start the hepatitis B vaccination

series in 6 weeks.”

MULTIPLE RESPONSE

For which clients does the nurse assess for inflammation? (Select all that apply.)

a. Client who reports pain b. Client diagnosed with an ear infection

c. Client who has sunburn d. Client taking vitamin C 500 mg daily

e. Client with nausea f. Client reporting reflux

g. Client with frostbite

The nurse is teaching a class about the immune system. The nurse asks the class to list various functions of antibodies. Which class responses

does the nurse evaluate as indicating a good understanding? (Select all that apply.)

a. “They release chemicals to cause blood vessels to dilate.” b. “They deactivate toxins that are released in an infection.”

c. “They tag bacteria so the macrophages know to eat them up.” d. “They cause the person to have a fever.”

e. “They tell cells to make collagen for scar tissue.” f. “They turn on the complement system.”

The nurse suspects acute graft rejection after a client has undergone a kidney transplant. What assessment findings confirm this diagnosis?

(Select all that apply.) a. Temperature of 100.6° F

b. Blood urea nitrogen (BUN) 56 mg/dL c. Creatinine 3.2 mg/dL

d. Urine output 20 mL/hr e. Extreme pain in the lower back

f. Edematous ankles

Chapter 20: Care of Patients with Arthritis and Other Connective Tissue Diseases

Test Bank

MULTIPLE CHOICE

The nurse is teaching a community health class about health promotion techniques. Which statement by a student indicates a strategy to help

prevent the development of osteoarthritis? a. “I will keep my BMI under 24.”

b. “I will switch to low-tar cigarettes.” c. “I will start jogging twice a week.”

d. “I will have a family tree done.”

The nurse is teaching a client who has osteoarthritis ways to slow progression of the disease. Which statement indicates that the client

understands the nurse’s instruction? a. “I will eat more vegetables and less meat.”

b. “I will avoid exercising to minimize wear on my joints.” c. “I will take calcium with vitamin D every day.”

d. “I will start swimming twice a week.”

The nurse is working with a client who will be taking 20 mg of prednisone daily for rheumatoid arthritis. Which precautions does the

nurse give the client about taking this medication? a. “Take this medication at bedtime because it will make you sleepy.”

b. “Take calcium and vitamin D supplements daily.” c. “Eat a high-fiber diet with lots of lean meats.”

d. “Wash your face twice a day with an antibacterial soap.”

An older adult client is scheduled for knee replacement surgery. Which statement by the client indicates a need for further preoperative

instruction? a. “I need to keep my leg positioned away from my body.”

b. “I may have a continuous passive motion machine for a few days.” c. “I may need more pain medicine than I did with my hip

replacement.” d. “I probably can get back to work within 2 to 3 weeks.”

A client returns to the medical-surgical unit after a total hip replacement with a large wedge-shaped pillow between his legs. The client’s daughter

asks the nurse why the pillow is in place. What is the nurse’s best response?

a. “It will help prevent bedsores from developing.”

b. “It will help prevent nerve damage and foot drop.” c. “It will keep the new hip from becoming dislocated.”

d. “It will prevent climbing out of bed if he becomes confused.”

The nurse is caring for a postoperative client on the medical-surgical unit following a total left hip replacement the previous day. During the assessment, the nurse notes that the client’s left leg is cool, with weak

pedal pulses. What is the nurse’s first action? a. Assess circulatory status of the right leg.

b. Notify the surgeon immediately. c. Measure leg circumference at the calf.

d. Check for bilateral Homans’ signs.

A client is admitted for a total hip replacement. Past medical history includes diabetes mellitus type 2, a heart attack 5 years ago, and allergies

to sulfa drugs. The client currently takes insulin on a sliding scale and celecoxib (Celebrex). Before administering the client’s medications,

which action by the nurse is most appropriate? a. Take the client’s blood pressure in both arms.

b. Call the physician to clarify the orders. c. Schedule a preoperative electrocardiogram.

d. Review the client’s laboratory values.

A client is receiving warfarin (Coumadin) daily following total hip replacement surgery. Which laboratory value requires intervention by

the nurse? a. Potassium (K+), 4.2 mEq/L

b. International normalized ratio (INR), 5.1 c. Prothrombin time (PT), 13.4 seconds

d. Hemoglobin (Hg), 16 g/dL

The nurse is caring for a client who is 1 day post total hip replacement. The nurse is instructing the client about how to perform quadriceps-

setting exercises correctly. Which direction does the nurse provide to the client?

a. “Straighten your legs and push the back of your knees into the mattress.”

b. “Straighten your legs and bring each leg separately off the mattress 6 inches.”

c. “Raise each leg 10 inches off the bed, keep it straight, and make ankle circles.”

d. “Bend each knee, and rapidly point your toes downward and then upward.”

The home care nurse is making a follow-up visit to a client who had total hip replacement surgery 2 weeks ago. Which client statement indicates a

need for clarification regarding postoperative routine? a. “My daughter helps me put on my elastic TED (thromboembolic

deterrent) hose every day.” b. “I take 200 mg of Motrin (ibuprofen) at bedtime so that I can sleep.”

c. “Now that my hip doesn’t hurt, I can cross my legs like a lady again.”

d. “Each day, I try to increase my walking time by at least 10 minutes.”

A client who has had bilateral total knee replacements is prescribed enoxaparin sodium (Lovenox) injections twice daily for the next 3 weeks. The client asks the nurse why she has to have the medication. What is the

nurse’s best response? a. “To prevent swelling within your new knee joints.”

b. “To prevent the formation of blood clots in your legs.” c. “To prevent arthritis from developing in your new knee joints.”

d. “To prevent an infection from developing in your new knee joints.”

The nurse is caring for a client who had right total knee replacement surgery 3 days ago. During the assessment, the nurse notes that the

client’s right lower leg is twice the size of the left. What is the nurse’s priority intervention?

a. Elevate the client’s right leg. b. Apply antiembolism stockings.

c. Assess the client’s respiratory status. d. Check the client’s pedal pulses.

A client had a total knee replacement earlier in the day and has a continuous femoral nerve blockade (CFNB). When entering the room to assess the client, the nurse notes that the television volume is quite loud.

The client explains that it is hard to hear with “all the ringing in my ears.” What action by the nurse takes priority?

a. Perform a neurovascular assessment on the operative extremity. b. Call another nurse to notify the anesthesiologist immediately.

c. Take a full set of vital signs and discontinue the CFNB. d. Pad the siderails and instituting other seizure precautions.

A client who has had total hip replacement surgery asks the nurse when she will be able to use a regular-height toilet seat again. What is the

nurse’s best response? a. “As soon as you are able to walk without a limp.”

b. “As soon as the staples are removed from the incision.” c. “When you are off pain medication and warfarin (Coumadin).”

d. “When you can hold your leg 6 inches off the bed for 5 full minutes.”

The nurse is caring for a client who has had hip replacement surgery 2 days before. The client reports severe pain at the surgical site despite

having received 2 Vicodin (acetaminophen and hydrocodone) tablets 2 hours previously. The client is requesting IV pain medication. What is

the nurse’s primary intervention? a. Assess the surgical site for signs of infection.

b. Administer 2 more Vicodin tablets. c. Apply a large ice bag to the operative site.

d. Reassure the client that the Vicodin will work soon.

A client is suspected to have rheumatoid arthritis. Which manifestations does the nurse assess this client carefully for?

a. Crepitus when the client moves the shoulders b. Numbness and tingling in the client’s fingers c. Client has cool feet, with weak pedal pulses

d. Low-grade fever, fatigue, anorexia with weight loss

The nurse is teaching a client how to reduce the pain that she often experiences with fibromyalgia. Which statement does the nurse include

in the teaching? a. “Wear gloves outdoors in cooler temperatures.” b. “Avoid exercising when your muscles are sore.”

c. “Make sure that you get enough sleep every night.” d. “Stay out of the sun as much as possible.”

A client with a history of rheumatoid arthritis will be starting drug therapy with etanercept (Enbrel). What is most important for the nurse

to teach the client? a. The correct technique for subcutaneous injections

b. How to self-monitor blood glucose levels c. How to set up and prime the IV tubing

d. How to calculate the dosage based on symptoms

The nurse provides discharge teaching for a client to prevent a new attack of gout. Which statement by the client indicates that additional

teaching is required? a. “I will keep a food and symptom diary for a few weeks.”

b. “If I get a headache, I will take Tylenol instead of aspirin.” c. “I hate to start limiting my fluid intake so much!”

d. “Citrus juices and milk may keep me from having kidney stones.”

The school nurse is working with a group of high school students who will be going on a field trip to a nature center. Which student is at highest

risk for a tick bite? a. Male student with a beard and a baseball cap

b. Female student with long hair pulled back in a ponytail c. Male student wearing a long-sleeved shirt and shorts d. Female student who is wearing scented hand lotion

The school nurse removes a tick embedded in a student’s scalp by the hairline. Which follow-up instruction is the nurse sure to provide to the

mother? a. “Call your pediatrician right away if a fever or a red rash develops

at the bite.” b. “If your child does not have symptoms within 2 weeks, you can

relax.” c. “Call your pediatrician tomorrow to get antibiotics to prevent Lyme

disease.” d. “Keep the site clean, but you don’t have to worry about further

problems.”

The nurse has taught a client with lupus about skin protection in the clinic. Later, the nurse sees the client at an outdoor music festival. Which

observation by the nurse indicates that the client requires further instruction?

a. Client is wearing a thin, long-sleeved shirt. b. Client is wearing a hat with a full brim.

c. Client is discussing her new perm. d. Client is seen applying sunscreen twice.

A client with diagnosed osteoarthritis comes to the clinic reporting a low- grade fever, fatigue, and bilateral joint pain. What action by the nurse is

most appropriate? a. Assess the client for a systemic infection.

b. Discuss increasing the dose of anti-arthritis drugs. c. Prepare the client for a laboratory draw for rheumatoid factor.

d. Teach the client joint protection activities.

The nurse is working in a primary care clinic and sees a young male client. The client is athletic and is well over 6 feet tall, with size 14 shoes.

What diagnostic test does the nurse facilitate for the client? a. Coagulation studies b. Echocardiography

c. Electromyelography d. Genetic testing

The nurse is working in a clinic when a young male client presents with reports of pain with urination. The client wants testing for sexually

transmitted diseases (STDs). The nurse notes that the client’s eyes are red and inflamed. What question by the nurse is most important?

a. “Do you have more than one sexual partner?” b. “Do you have any new joint pain?”

c. “What eyedrops have you used for your red eyes?” d. “Are you allergic to any antibiotics?”

A client presents with painful, inflamed fingers with small, hard, yellow nodules that have a sandy yellow drainage. Which medication does the

nurse prepare to administer to the client? a. Colchicine (Colasalide) b. Allopurinol (Zyloprim)

c. Methotrexate (Rheumatrex) d. Aspirin

A client had a total knee replacement this morning and has a continuous passive motion (CPM) machine. What activity related to the CPM does

the RN delegate to the unlicensed assistive personnel? a. Placing controls out of the reach of confused clients

b. Assessing the client’s response to the CPM c. Teaching the client’s family the rationale for the CPM d. Assessing neurovascular status of the leg in the CPM

A client with chronic gout takes probenecid (Benemid) and comes to the clinic reporting frequent severe headaches and a new gout flare. The

client is frustrated because the gout had been under good control. Which

question by the nurse is most helpful? a. “What do you take for your headaches?” b. “Do you know what triggers your gout?”

c. “Have you been following your diet?” d. “Did you switch from wine to beer lately?”

The nurse is caring for a pregnant client who is taking Humira (adalimumab) to control symptoms of rheumatoid arthritis. The client

mentions the pain and inconvenience of the subcutaneous injections and asks, “While I’m pregnant, can I take this drug by mouth instead?”

What is the nurse’s best response? a. “I will ask the physician to write a prescription for you today.” b. “Humira takes much longer to work when it is given orally.”

c. “Humira can be given only by subcutaneous injection.” d. “You can switch from Humira to oral leflunomide (Arava).”

A female client with rheumatoid arthritis has taken Rheumatrex (methotrexate) for the past year to control her symptoms. The client comes to the clinic and tells the nurse that a home pregnancy test was

positive. What is the nurse’s best response? a. “You need to schedule a prenatal appointment with your

obstetrician right away.” b. “Stop taking Rheumatrex immediately. I’ll tell the physician you are

pregnant.”

c. “Continue taking the Rheumatrex, and increase the dose if you have a flare.”

d. “See a genetic counselor to determine whether your baby will have rheumatoid arthritis.”

The nurse is instructing a client about management of discoid lupus erythematosus (DLE). Which statement indicates that the client requires

additional teaching? a. “I will be sure to apply sunscreen whenever I am outside.”

b. “I will apply small amounts of the steroid cream to my face twice a day.”

c. “I will take Plaquenil (hydroxychloroquine sulfate) with breakfast each morning.”

d. “Steroids weaken the immune system, so I will wash my hands frequently.”

Which statement by a client indicates that additional teaching is needed in the management of fibromyalgia?

a. “I will switch to decaffeinated coffee in the mornings.” b. “Water aerobics classes will be a good form of exercise.” c. “Limiting my physical activity will reduce my fatigue.”

d. “I will take my sertraline (Zoloft) right before I go to bed.”

The nurse is caring for a client who has dysphagia caused by systemic sclerosis. What is the best intervention for the nurse to implement for

this client? a. Encourage frequent, high-protein, easy to swallow foods. b. Teach the client to lie flat after meals to prevent reflux.

c. Thicken liquids to a nectar or honey consistency. d. Have the client hyperextend his or her neck while swallowing.

The nurse is instructing a client about the management of systemic sclerosis. Which statement indicates that the client requires additional

teaching? a. “I will let my doctor know right away if I develop a fever.”

b. “Ice packs will help relieve the aching pain in my hips and knees.” c. “I will wear mittens when I am in the freezer section of the grocery

store.” d. “I will apply a rich moisturizer to my skin every morning after my

shower.”

The nurse is working at a clinic, where several clients are waiting to be seen. Which client does the nurse assess first?

a. Client with temporal arteritis with new onset of blurry double vision b. Client with polymyalgia rheumatica with low-grade fever and

fatigue c. Client with polymyositis reporting generalized rash and joint pain

d. Client with ankylosing spondylitis who presents with back pain and weight loss

The nurse is caring for a female client who has a history of chronic fatigue syndrome. Which finding is the nurse surprised to see in the

client’s record? a. Hemoglobin, 7.2 g/dL

b. Serum creatinine, 0.9 mg/dL c. Multiple tender lymph nodes

d. Newly red, swollen, warm knee

A client with rheumatoid arthritis had abdominal surgery and has returned to the postoperative nursing unit. The client is unable to use the incentive spirometer correctly, demonstrating limited lung volume and

fatiguing easily. What action by the nurse takes priority? a. Notify the physician immediately.

b. Have respiratory therapy re-instruct the client. c. Assess for pain and medicate if necessary.

d. Let the client rest for a few hours.

A nurse is caring for a client who has had rheumatoid arthritis (RA) for 5 years. Which laboratory value requires the most immediate

intervention by the nurse? a. White blood cell count (WBC), 3800/mm3

b. Hemoglobin (Hg), 10.6 g/dL c. Blood urea nitrogen (BUN), 16 mg/dL

d. Creatinine, 3.2 mg/dL

The nurse is caring for an older adult client who will be discharged after being hospitalized for a total hip replacement. Which statement indicates that arrangements may have to be made to have the client’s medications

supervised at home? a. “I will take my Coumadin pill every day just before the evening

news.” b. “My wife takes iron too, so we will take our pills together every

morning.” c. “I prepare all my pills for the week and will place them in a labeled

medi-set.” d. “If my legs get swollen, I will take an extra Coumadin pill that day.”

The nurse is working with a client who has severe rheumatoid arthritis in her hands. The client states that she is frustrated at mealtime because it is difficult for her to manage cups and silverware. What is the nurse’s

best response? a. “I’ll have the nursing assistants set up your meal trays while you are

in the hospital.” b. “Let’s see if the occupational therapist can provide you with some

utensils that are easier for you to use.” c. “I’ll arrange for a home nursing assistant to help you with your

meals after you are discharged from the hospital.” d. “Let’s see if the physical therapist can suggest some muscle

strengthening exercises for you.”

The nurse is caring for an older adult client who will be discharged home to live with an adult daughter. The client will be given prescriptions for

four new medications for rheumatoid arthritis. How does the nurse ensure that the client will be able to take the medications correctly at

home? a. Monitor the client self-administering medications while in the

hospital. b. Include the client’s daughter when teaching the client about the

medications. c. Provide the client with pamphlets and information about all the

medications. d. Make a chart showing which medications the client should take at

different times.

The nurse is caring for an older adult client who has fallen and fractured her hip. The client will have hip replacement surgery followed by

extensive rehabilitation. The client confides in the nurse, “I feel like I don’t have any control over anything anymore now that I am old.” What

is the nurse’s best response? a. “I’ll make sure that the physical and occupational therapists see you

after surgery to help get your strength back.” b. “It’s normal to feel this way, but hopefully you will be back on your

feet after a stay in rehab.” c. “It’s important to control what you can right now, like making out

your menu every day and working with the therapists.” d. “I sense that you are feeling depressed about the situation. I will ask

the doctor to prescribe an antidepressant for you.”

The nurse is caring for an older adult client who has had a hip replacement 2 days previously. Which assessment finding is the best indicator that the client does not need pain medication at this time?

a. The client received 2 pain pills 2 hours ago. b. The client states that she has no pain.

c. The client is sleeping quietly. d. The client’s vital signs are stable.

The nurse is caring for a client who has a history of severe rheumatoid arthritis. The client becomes combative and abusive to the staff when she

is unable to perform personal care independently. What is the best statement the nurse can make to the client at this time?

a. “I will have to restrain your hands if you cannot keep them to yourself.”

b. “I will ask your doctor for a psychiatrist to talk to you about anger management.”

c. “You seem frustrated. Would you like to try to dress again in a few minutes?”

d. “Would you like me to get an order for medication to help you settle down?”

MULTIPLE RESPONSE

What interventions does the nurse recommend for a client who is to be discharged home following total hip replacement surgery? (Select all that

apply.) a. Continuous passive motion machine

b. Elevated toilet seat c. Walker

d. Crutches e. TED hose

f. Heating pad

After hip replacement surgery, a client receives two doses of enoxaparin (Lovenox) during the day shift. What orders does the nurse anticipate

for the client? (Select all that apply.) a. Laboratory draw for platelet count

b. Laboratory draw for prothrombin time (PTT) c. Laboratory draw for international normalized ratio (INR)

d. Order for protamine sulfate e. Order for vitamin K

The nurse is teaching a client with rheumatoid arthritis (RA) about joint protection principles. What information does the nurse include? (Select

all that apply.) a. Use smaller joints to rest the larger ones.

b. Hold objects with two hands, not one. c. Sit most often in a reclining chair.

d. Use assistive-adaptive devices. e. Bend at your knees to lift objects.

Chapter 21: Care of Clients with HIV Disease and Other Immune Deficiencies

Test Bank

MULTIPLE CHOICE

Which action by the nurse is most effective to prevent becoming exposed to the human immune deficiency virus (HIV)?

a. Always use Standard Precautions with all clients in the workplace. b. Place clients who are HIV positive in Contact Precautions.

c. Wash hands before and after contact with clients who are HIV positive.

d. Convert parenteral medications to an oral form for clients who are HIV positive.

The nurse is caring for a young client who has acquired immune deficiency syndrome (AIDS) and a very low CD4+ cell count. The nurse is teaching the client how to avoid infection at home. Which statement by

the client indicates that additional teaching is needed? a. “I will let my sister clean my pet iguana’s cage from now on.”

b. “My brother will change the kitty litter box from now on.” c. “It will seem funny but I’ll run my toothbrush through the

dishwasher.” d. “I will not drink juice that has been sitting out for longer than an

hour.”

The nurse is working with a client at a public health clinic. The client says to the nurse, “The doctor said that my CD4+ count is 450. Is that

good?” What is the nurse’s best response? a. “Your count is high so you can cut back on your medication.”

b. “Your count is normal because your medications are working well.” c. “Your count is a bit low and you are susceptible to infection.”

d. “Your count is very low and you actually now have AIDS.”

The nurse is caring for a young woman at the primary health care clinic. Which assessment finding leads the nurse to question the client about

risk factors for HIV? a. Six vaginal yeast infections in the last 12 months b. Unable to become pregnant for the last 2 years

c. Severe cramping and irregular periods d. Very heavy periods and breakthrough bleeding

A client who is positive for HIV presents with confusion, fever, headache, blurred vision, nausea, and vomiting. What does the nurse do first?

a. Assess the client’s deep tendon reflexes. b. Ask the client to place his chin on his chest.

c. Start an IV line with normal saline. d. Assess the client’s pupil reaction.

The nurse is caring for a client with AIDS who has just been diagnosed with cryptococcal meningitis. Which is the best nursing intervention for

this client? a. Initiate respiratory isolation for the next 72 hours.

b. Initiate seizure precautions with padded siderails. c. Thicken the client’s liquids to honey consistency. d. Administer IV pentamidine isethionate (Pentam).

A client with AIDS has been admitted with fever, night sweats, and weight loss of 6 pounds in 2 weeks. The client’s purified protein

derivative (PPD) test, placed 3 days ago in the clinic, is negative. Which action by the nurse is most appropriate?

a. Place the client in Airborne Precautions. b. Facilitate the client’s chest x-ray.

c. Initiate a 3-day calorie count. d. Start an IV of normal saline.

The nurse is caring for a newly diagnosed HIV-positive client who will be taking enfuvirtide (Fuzeon). Which precaution is important for the nurse

to communicate to this client? a. “Stop taking the medication if you develop a fever.”

b. “Rotate the sites where you will be giving the injections.” c. “Take this medication with a snack or a small meal.”

d. “Do not drive or operate machinery while taking this drug.”

A client who is receiving highly active antiretroviral therapy (HAART) tells the nurse, “The doctor said that my viral load is reduced. What does

this mean?” What is the nurse’s best response? a. “The HAART medications are working well right now.”

b. “You are not as contagious as you were anymore.” c. “Your HIV infection is becoming resistant to your medications.”

d. “You are developing an opportunistic infection.”

The nurse is seeing clients at a drop-in primary health clinic. Which client does the nurse teach about the risks of acquiring HIV?

a. Middle-aged woman with a new sexual partner b. Young male who has male sexual partners

c. All clients who come to the clinic d. Young woman having her first gynecologic examination

An HIV-positive client is taking lopinavir/ritonavir (Kaletra) and reports nausea, abdominal pain, and diarrhea. What orders does the nurse

anticipate? a. Renal function studies

b. Liver enzymes c. Blood glucose monitoring d. Albumin and prealbumin

The nurse has been exposed to HIV through splashing of urine from a client who is HIV positive with a low viral load. The urine came into

contact with the nurse’s face. Which drug regimen does the nurse prepare to initiate?

a. Retrovir (zidovudine) for 14 days b. Retrovir (zidovudine) for 28 days

c. Retrovir (zidovudine) and Epivir (lamivudine) for14 days d. Retrovir (zidovudine) and Epivir (lamivudine) for 28 days

The nurse is teaching a client how to prevent transmitting HIV to his sexual partner. Which statement by the client indicates that additional

teaching is needed? a. “I can throw the condoms in the trash after I have used them.”

b. “I will store my condoms in my wallet so they are always handy.” c. “Water-based lubricants are best to prevent condom breakage.”

d. “The condom needs to stay on until I withdraw my penis.”

The nurse is teaching a seminar about preventing the spread of HIV. Which statement by a student indicates that additional teaching is

required? a. “A woman can still get pregnant if she is HIV positive.”

b. “I won’t get HIV if I only have oral sex with my partner.” c. “Showering after intercourse will not prevent HIV transmission.”

d. “People with HIV are still contagious even if they take HAART drugs.”

The nurse is teaching a client who has AIDS how to avoid infection at home. Which statement indicates that additional teaching is needed?

a. “I will wash my hands whenever I get home from work.” b. “I will make sure to have my own tube of toothpaste at home.”

c. “I will run my toothbrush through the dishwasher every evening.” d. “I will be sure to eat lots of fresh fruits and vegetables every day.”

The nurse is teaching a postmenopausal client about the risk of acquiring HIV infection. The client states, “I’m an old woman! I cannot possibly

get HIV.” What is the nurse’s best response?

a. “Your vaginal walls become thicker after menopause, which increases your risk.”

b. “Women in your age-group are the fastest growing population of AIDS clients today.”

c. “Hormonal fluctuations after menopause make it harder to fight off infection.”

d. “You might be right. How often do you engage in sexual activities?”

A client has selective immune globulin A (IgA) deficiency. The provider orders an infusion of immune globulin (IVIG). Which action by the

nurse is best? a. Start a second IV line for the client’s antibiotics.

b. Call the physician to clarify the order. c. Review the client’s renal panel before administration.

d. Obtain baseline vital signs and another set after 15 minutes.

The nurse is working with a client who has AIDS-related dementia and will soon be discharged to the care of family members. What teaching

topic is best for the nurse to include in the discharge plan? a. Feed the client when he will not do it by himself.

b. Make sure that a clock and a calendar are easily visible. c. Remove locks from bathroom and bedroom doors.

d. Do not allow the client to smoke when he is alone.

A client with HIV who is taking highly active antiretroviral therapy (HAART) medications is in radiology waiting for a chest x-ray when

medications are due. What action by the nurse is best? a. Call radiology to see when the client will be brought back to the

nursing unit. b. Send the nursing assistant to radiology to bring the client back to

the nursing unit. c. Take the client’s medications to radiology and administer them there

if possible. d. Stagger the next dose of the medication if the current dose is given

late.

An HIV-positive client verbalizes concerns about the high cost of antiretroviral medications. What is the nurse’s best response?

a. “The medications are actually less expensive than they used to be.” b. “These medications are the best course of treatment for you.”

c. “You should be glad the medications will help prolong your life.” d. “Let’s talk to the social worker about getting financial assistance for

you.”

The nurse is caring for a client who is HIV positive. The client has become confused over the course of the shift, and the client’s pupils are no longer reacting to light equally. The nurse anticipates an order for

which medication? a. Prednisone (Deltazone)

b. Trimethoprim/sulfamethoxazole (Bactrim) c. Pentamidine isethionate (Pentam)

d. Ketoconazole (Nizoral)

A client verbalizes a fear of contracting HIV because she has a history of intravenous substance abuse. What instructions does the nurse provide

to the client to help minimize this risk? a. “Boil all needles and syringes for at least 20 minutes before using

them again and be sure not to share them.” b. “Rinse used needles and syringes with water followed by laundry

bleach after using them.” c. “Rinse used needles and syringes with rubbing alcohol before and

after using them.” d. “Run all needles and syringes through the dishwasher with an extra

rinse cycle before using them again.”

The nursing supervisor is working with an HIV-positive nurse who has open weeping blisters on her arms after being exposed to poison ivy.

Which instructions should the nursing supervisor provide to the nurse before she starts her shift?

a. “You should reassure your clients that you are not contagious.” b. “You should work phone triage at the desk today rather than taking

clients.” c. “You should wear a long-sleeved scrub jacket today while working

with clients.” d. “You should not care for clients who are immune compromised or in

isolation.”

The nurse is caring for an HIV-positive client. What assessment finding assists the nurse in confirming progression of the client’s diagnosis to

AIDS? a. Generalized lymphadenopathy b. HIV-positive status for 8 years

c. Low-grade fever for the last 10 days d. Thick white patches on the client’s tongue

A nursing assistant asks the nurse if respiratory isolation is needed for a client with Pneumocystis jiroveci pneumonia. What is the nurse’s best

response? a. “This type of pneumonia is an opportunistic infection, so the staff is

not at risk.” b. “You should wear a mask and a gown to provide care.”

c. “Yes, please institute respiratory isolation because this is very contagious.”

d. “You are not at risk for this infection if you have had a vaccination.”

When obtaining a sexual history from a client in a clinic setting, the nurse notes that the client appears very uncomfortable and pauses for

long periods before answering the nurse’s questions. What is the nurse’s best response?

a. “I am sorry that my questions are making you very uncomfortable.” b. “Don’t worry. We’ll be done with these questions in no time at all.”

c. “Take your time. I realize that this is a very private topic to talk about.”

d. “These questions are making you uncomfortable, so we’ll finish next time.”

The nurse asks a young adult client if she is sexually active. The client asks why the nurse needs to know. What is the nurse’s best response?

a. “I just need to make sure that the information you are providing is

reliable.” b. “I have to fill in answers to all of the questions on the health history

form.” c. “If you are sexually active, we should talk about ways to prevent

getting HIV.” d. “I will have to notify your partner if you have a sexually transmitted

disease.”

The nurse is completing a health history for a client and begins to obtain a sexual history. What is the nurse’s best opening question?

a. “How long have you been sexually active?” b. “Are you in a monogamous relationship with your spouse?”

c. “How do you feel about answering questions about your sexual history?”

d. “Have you noticed any problems with your ability to have or enjoy sex?”

The nurse is caring for a client with HIV who has been prescribed didanosine (Videx EC). Which action by the nurse is most appropriate?

a. Help the client plan specific meal and dosing times. b. Explain that the client will have frequent complete blood counts

(CBCs) drawn.

c. Advise the client to take Videx EC with milk or a small meal. d. Tell the client to take Tylenol (acetaminophen) for any abdominal

pain.

MULTIPLE RESPONSE

The nurse is caring for a hospitalized client who has AIDS and is severely

immune compromised. Which interventions are used to help prevent infection in this client? (Select all that apply.)

a. Use sterile gloves and gowns whenever the nursing staff is in contact with the client.

b. Provide an incentive spirometer to encourage coughing and deep breathing by the client.

c. Keep a blood pressure cuff, thermometer, and stethoscope in the client’s room for his or her use only.

d. Use N95 respirators (all nursing staff) when in the client’s room. e. Request that the family take home the fresh flowers that are at the

client’s bedside. f. Assist the client with meticulous oral care after meals and at

bedtime.

SHORT ANSWER

The nurse is to give a client ganciclovir (Cytovene) for cytomegalovirus (CMV) retinitis. The dosage is 5 mg/kg IV every 12 hours. The client

weighs 185 pounds. How many milligrams of ganciclovir does the client receive per dose? mg/dose

The nurse is to give a client rifampin (Rifadin) for tuberculosis. The dosage is 10 mg/kg/day. The client weighs 198 lb, and the medication is available in 150-mg capsules. How many capsules of rifampin does the

client receive daily? capsules/day

Chapter 22: Care of Patients with Immune Function Excess: Hypersensitivity (Allergy) and Autoimmunity

Test Bank

MULTIPLE CHOICE

Which characteristic is common to all types of hypersensitivity reactions?

a. Decreased inflammatory responses b. Presence of tissue-damaging reactions c. Enhanced natural killer cell activity

d. Inability to recognize extraneous cells

What intervention does the nurse implement to provide for client safety during intradermal allergy testing?

a. Stay with the client and ensure that emergency equipment is in the room.

b. Pretreat the skin area to be tested with a cortisone-based cream. c. Apply oxygen by mask or nasal cannula before injecting the test

agent. d. Cover the examination table and pillow with plastic or an ultrafine

mesh.

What is most important for the nurse to teach the client with allergic rhinitis and glaucoma?

a. “If your heartbeat increases, be sure to contact your health care provider.”

b. “Avoid allergy drugs containing pseudoephedrine or

phenylephrine.” c. “Be sure to drink plenty of water with antihistamines.”

d. “You should use an eye-moistening agent such as Restasis.”

A client has received diphenhydramine (Benadryl) and is currently oriented but drowsy. What is the best action for the nurse to take?

a. Perform a neurologic assessment every 2 hours. b. Document the response and continue to monitor.

c. Prepare to administer epinephrine subcutaneously. d. Have the nursing assistant stimulate the client every hour.

Which condition is a type II hypersensitivity reaction? a. Allergic rhinitis

b. Positive purified protein derivative (PPD) test for tuberculosis c. Transfusion reaction to improper blood type

d. Serum sickness after receiving immune globulin

A client is receiving an IV infusion of an antibiotic. The client calls the nurse about feeling uneasy and uncomfortable owing to congestion.

Which action by the nurse is most appropriate? a. Elevate the head of the client’s bed to 45 degrees.

b. Have another nurse call the Rapid Response Team. c. Prepare to administer diphenhydramine (Benadryl).

d. Slow the rate of the IV infusion.

Which client characteristic places her or him at high risk for latex hypersensitivity?

a. Allergy to shellfish b. History of spina bifida c. Total hip replacement

d. Taking oral contraceptives

What dose of epinephrine does the nurse prepare for a client in anaphylaxis who is 6 feet 3 inches tall and weighs 250 lb?

a. 0.2 mL of a 1:1000 solution b. 0.5 mL of a 1:1000 solution

c. 0.3 mL of a 1:10,000 solution d. 0.5 mL of a 1:10,000 solution

Which intervention is most important for the nurse to teach the client who is recovering from an allergic reaction to a bee sting?

a. How to use an EpiPen b. Wearing a medical alert bracelet

c. Avoiding contact with the allergen d. Keeping diphenhydramine (Benadryl) available

A client has angioedema of the lower face. What will the nurse assess next?

a. Pulse oximetry b. Airway patency c. Breath sounds

d. Chest wall symmetry

A mother brings her child to the clinic requesting “genetic testing” to determine whether her child suffers from the same multiple allergies as

herself. What action by the nurse is most appropriate? a. Provide a referral to an allergist so the child can be tested.

b. Refer the mother to a geneticist for genetic testing on the child. c. Ask the mother about specific symptoms the child may have had.

d. Have the mother list her allergies and the symptoms they cause her.

A client states that he is “allergic” to poison ivy. Which statement by the client indicates a good understanding of this type of sensitivity?

a. “Drinking 3 liters of water a day will prevent kidney damage.” b. “I will always wear a medical alert bracelet for this allergy.” c. “I need to try to avoid coming into contact with poison ivy.”

d. “I should carry diphenhydramine (Benadryl) with me at all times.”

A client is hospitalized with Goodpasture’s syndrome. Which intervention by the nurse takes priority?

a. Monitor urine output and renal function tests. b. Teach the client to manage peritoneal dialysis.

c. Administer antibiotics strictly on time. d. Have separate IV access for immune globulin (IVIG)

administration.

A client is in the clinic having had rhinorrhea and headache for the last 2 weeks. Which laboratory value alerts the nurse to the possibility of a

type I hypersensitivity reaction? a. White blood cell count, 8900/mm3

b. Eosinophils, 10% c. Neutrophils, 65%

d. Hemoglobin, 14 g/dL

How does the type V hypersensitivity reaction differ from other reactions?

a. It is cell mediated rather than antibody mediated. b. It is an immediate response rather than a delayed response.

c. It produces a stimulatory response to normal tissues. d. It results in more severe tissue damage than is caused by other types

of reactions.

A nurse is planning care for a client with Sjögren’s syndrome. At what point does the nurse determine that priority outcomes have been met?

a. The client states that he or she is not as fatigued as previously. b. The client dresses attractively despite gaining a large amount of

weight.

c. The oral mucosa is intact and no systemic signs of infection are present.

d. The client is able to complete activities of daily living with minimal shortness of breath.

An unknown unconscious client with an elevated temperature is ordered intravenous penicillin. What is the best action for the nurse to take?

a. Administer the medication. b. Check the chart for allergies.

c. Look for medical alert identification. d. Notify the nursing supervisor.

A nurse suspects that a client has serum sickness. For which manifestation does the nurse assess the client?

a. Joint pain b. Allergic rhinitis

c. Stridor d. Wheezing

The nurse enters a client’s room and observes the manifestations shown below. What action should the nurse take first?

a. Prepare to administer diphenhydramine (Benadryl). b. Prepare to administer epinephrine.

c. Assess the client’s respiratory status. d. Get a full set of vital signs.

MULTIPLE RESPONSE

A client is admitted for a cardiac catheterization. It is essential for the nurse to ask the client about which allergies? (Select all that apply.)

a. Penicillin b. Latex c. Iodine

d. Shellfish e. Keflex

f. Dilantin

g. Bananas The nurse is preparing to administer a medication when the client states, “I’m allergic to that.” How will the nurse proceed? (Select all that apply.)

a. Check the chart for allergies. b. Notify the health care provider.

c. Ask what reaction the client gets. d. Continue to give the medication.

e. Perform a skin test first. f. Notify the pharmacist.

g. Document the allergy on the chart.

SHORT ANSWER

The nurse is to give a client 80 mg of diphenhydramine (Benadryl) by IV push. The vial contains a solution with a concentration of 25 mg/mL. How many milliliters of diphenhydramine does the nurse administer?

mL

Chapter 23: Cancer Development

Test Bank

MULTIPLE CHOICE

The nurse includes which information about benign tumors when presenting an in-service on cancer?

a. They can wander far throughout the body. b. They are smaller than 2 cm.

c. They retain a small nuclear-to-cytoplasmic ratio. d. They look different from the tissue they arose from.

In reviewing the pathophysiology of a particular type of cancer, the nurse correlates the generation time for cancer development with which

description? a. The rate at which cancer cells are able to migrate and metastasize to

different sites b. How long it takes for a malignant tumor to double in size by mitotic

cell divisions c. The period of time needed for one cell to divide into two cells by

mitosis d. The period of time between cell damage and expression of a

malignancy

Which biologic characteristic is specific to normal differentiated adult cells but not to cancer cells?

a. Anaplasia b. Hypertrophy c. Aneuploidy

d. Loose adherence

A client states that his brain tumor is benign and does not need to be removed. What is the nurse’s best response?

a. “As your tumor grows, it can damage your brain, so it should be

removed.” b. “Benign tumors consist of normal cells, so removal is only for

cosmetic purposes.” c. “Benign tumors turn into cancer, so they should be removed as soon

as possible.” d. “Because benign tumors can migrate, they should be removed

before they spread.”

Which comment made by a client with breast cancer indicates a need for clarification regarding cancer causes and prevention? a. “I will eat a low-fat, high-fiber diet from now on.”

b. “Probably nothing I did or didn’t do caused this cancer.” c. “I hope my daughter doesn’t develop breast cancer.”

d. “Regular mammograms on my other breast will prevent cancer.”

Malignant cell growth is uncontrolled because of which action? a. Cancer cells always divide more rapidly than normal cells.

b. Mitosis of malignant cells usually produces more than two daughter cells.

c. Malignant cells bypass one or more phases of the cell cycle during cell division.

d. Malignant cells enter the cell cycle frequently, making cell division

continuous.

A client has known lung cancer and has been admitted for abdominal pain and jaundice. A computed tomography (CT) scan reveals tumors in the client’s liver. The client is distraught and says, “So now I have liver

cancer too?” Which response by the nurse is most appropriate? a. “Yes, liver cancer is common in people who already have lung

cancer.” b. “Yes, your chemotherapy left you vulnerable to a virus that causes

liver cancer.” c. “No, the tumors are actually from your lung cancer, which has

metastasized.” d. “No, having tumors in two different organs is rare; you probably

have hepatitis.”

An occupational health nurse is working with management in a firm that provides commercial building restoration, including asbestos removal.

Which action does the nurse recommend to management? a. Provide annual screening chest x-rays for those exposed to asbestos.

b. Purchase protective gear and develop policies mandating its use. c. Offer “stop smoking” programs on site several times a year.

d. Routinely distribute testing kits for occult fecal blood.

The nurse correlates “initiation” in cancer development with which action?

a. Inflicting mutations that lead to excessive cell division b. Increasing the capacity of the transformed cell for error-free DNA

repair c. Stimulating contact inhibition in cells damaged by a carcinogen

d. Making cancer cells appear more normal to escape immune surveillance

The middle-aged client with lung cancer asks whether his adult children are at increased risk for this cancer. What is the nurse’s best response? a. “This disease is a random event and there is no way to prevent it.”

b. “This disease is inherited, so your children have a 50% risk for developing it.”

c. “Smoking is the main cause. Helping your children not smoke decreases their risk.”

d. “They can avoid cancer by decreasing the fat they eat and by exercising more.”

An adult client who has a suspicious mammogram says that her mother died of bone cancer when she was around the same age. Which is the

most important question for the nurse to ask this client? a. “Have any other members of your family had bone cancer?”

b. “Did your mother ever have any other type of cancer?” c. “How old were you when you started your periods?”

d. “Did your mother have regular mammograms?”

A client with prostate cancer says that he is now having a lot of pain in his lower back and legs. The nurse educates the client about which

intervention? a. X-rays of the spine and legs

b. Administering ibuprofen (Motrin) for pain c. Referral to the pain control specialist

d. Referral to physical therapy

A middle-aged client is having a physical examination and is worried about cancer risk. Which question is most important for the nurse to

ask? a. “How much time do you spend in the sun?”

b. “How many servings of fruits and vegetables do you eat every day?” c. “How often do you eat processed meats like bologna?”

d. “Do you smoke cigarettes or have you ever used tobacco products?”

The nurse is counseling a client who smokes and drinks heavily about cancer risk. The client is adamant that he or she will never stop smoking.

Which question by the nurse is most appropriate? a. “Would you be willing to stop drinking alcohol?”

b. “Have you ever tried the nicotine patch?” c. “Why are you so determined to continue smoking?”

d. “Do you understand that smoking is the leading cause of cancer?”

A client’s cancer is staged by the TNM classification as T1, N3, M1. What is the nurse’s interpretation of this classification?

a. The client has a large tumor involving the lymph nodes, but no distant metastasis.

b. The client has a tumor, and metastasis cannot be determined by the staging method.

c. The client has a 2-cm tumor, one involved lymph node, and local metastasis.

d. The client has a small tumor, many involved lymph nodes, and distant metastasis.

A client says that she has heard that the origin of most cancers is genetic and wants “genetic testing because of a family history of cancer.” What is

the nurse’s best response? a. “I will ask your physician about a referral for genetic testing.” b. “Let’s look at your family history back to your grandparents’

generation.” c. “Genetic testing is so expensive; let’s talk about reducing your risk

instead.” d. “Inherited cancers are much more common in males than in

females.”

In preparing a community teaching program, which information does the nurse plan to present to address secondary cancer prevention?

a. Receiving cancer treatment with chemotherapy b. Annual measurement of prostate-specific antigen levels

c. Avoiding known cancer-causing substances or conditions d. Having adolescent children receive the Gardasil vaccination

The nurse correlates the role of suppressor genes in cancer development with which action?

a. The presence of suppressor genes increases risks for gene damage by carcinogens.

b. People with a greater number of suppressor genes are at increased

risk for getting cancer. c. Suppressor genes enhance immune function, suppressing cancer

development. d. Suppressor genes limit cell division, reducing risks for developing

cancer.

The nurse most likely would construct a three-generation pedigree for a

client who had a relative treated for which cancer? a. Lung cancer

b. Prostate cancer c. Cervical cancer

d. Bone cancer

The nurse counsels a woman who has a BRCA1 gene that she has what chance for developing breast cancer during her lifetime?

a. None; this gene has a protective effect b. Same as the general population

c. Lower than the general population d. Higher than the general population

The nurse wishes to present a cancer program to a group of people at high risk for cancer. In planning the program, which group does the

nurse consider the priority? a. Older adults

b. People who smoke c. Clients with family histories of cancer

d. People with poor immune function

The nurse is planning a cancer education event in an Asian community center. The nurse plans to present information specifically on which

types of cancer? a. Breast and colorectal b. Skin and lymphoma c. Liver and stomach

d. Uterine and ovarian

In preparing a cancer risk reduction pamphlet for African-American clients, it is most important that the nurse include information on

prevention and early detection for which types of cancer?

a. Lung and prostate b. Bone and leukemia c. Skin and lymphoma

d. Stomach and esophageal

The nurse is seeing clients in a clinic. Which client does the nurse assess further for the development of cancer?

a. Client with a cough that has lasted for 4 months b. Client whose mother died of lung cancer

c. Client with a 10-pound weight gain d. Woman whose last mammogram was 3 years ago

It is most important that the nurse include which activity for the young

adult client with Down syndrome? a. Encouraging more fruit and leafy green vegetables in the diet

b. Teaching him how to perform testicular self-examination c. Assessing the skin for unusual bruises and petechiae

d. Testing the client’s stool for occult blood

The nurse is interested in primary prevention for cancer. Which activity does the nurse most likely participate in?

a. Distributing occult fecal blood test kits to people at the shopping mall

b. Arranging transportation volunteers for clients undergoing radiation therapy

c. Teaching high school students the dangers of using tobacco d. Educating adolescent girls about getting an annual Papanicolaou

(PAP) smear

The nurse assesses which client most carefully for cancer development? a. Young man receiving radiation therapy for a brain tumor

b. Young adult woman who recently had postpartum hemorrhage c. Adolescent male recently diagnosed with acquired immune

deficiency syndrome (AIDS) d. Older woman undergoing chemotherapy for bowel cancer

MULTIPLE RESPONSE

A client has colorectal cancer. Which activities are especially important for the nurse to conduct for this client? (Select all that apply.)

a. Monitor liver function studies. b. Maintain accurate intake and output.

c. Obtain daily weight using the same scale. d. Palpate lymph nodes at each clinic visit.

e. Ask the client about changes in belly size.

Chapter 24: Care of Patients with Cancer

Test Bank

MULTIPLE CHOICE

What statement indicates that the client understands teaching about neutropenia?

a. “I need to use a soft toothbrush.” b. “I have to wear a mask at all times.”

c. “My grandchildren may get an infection from me.” d. “I will call my doctor if I have an increase in temperature.”

A client is undergoing radiation therapy and asks the nurse about skin care for the exposed area. Which statement by the nurse is most

accurate? a. “No products work well to reduce the skin reactions you get from

radiation.” b. “No one product works best, so you can choose what you would like

to use.” c. “The only medication that works well for skin reactions is very

expensive.” d. “No good studies on skin care with radiation have been conducted to

date.”

A client who has just had a mastectomy is crying. When the nurse asks about her crying, the client responds, “I know I shouldn’t cry because this surgery may well save my life.” What is the nurse’s best response?

a. “It is all right to cry. Mourning this loss will help make you stronger.”

b. “I know this is hard, but your chances of survival are better now.” c. “I can arrange for someone who had a mastectomy to come visit if

you like.” d. “How have you coped with difficult situations in the past?”

In evaluating dietary teaching for a client with chemotherapy-induced neutropenia, the nurse becomes concerned when the client makes which

food choice? a. Fruit salad b. Applesauce

c. Steamed broccoli d. Baked potato

What teaching is essential for a client who has received an injection of iodine-131?

a. “Do not share a toilet with anyone else or let anyone clean your toilet.”

b. “You need to save all your urine for the next week.” c. “No special precautions are needed because this type of radiation is

weak.” d. “Avoid all contact with other people until the radiation device is

removed.”

A client has bone cancer. What intervention does the nurse implement as a priority for this client?

a. Using a lift sheet when repositioning the client b. Positioning the client’s heels to keep them from touching the

mattress c. Providing small, frequent meals rich in calcium and phosphorus d. Applying pressure for 5 minutes after intramuscular injections

A client is undergoing radiation therapy and says, “I will be so glad when this is over and I don’t have to worry about my skin.” What response by

the nurse is most appropriate? a. “Unfortunately, your skin will be permanently damaged from the

radiation.” b. “You need to protect your skin from the sun for at least a year

afterward.” c. “You can get a prescription for special lotions that reduce the effects

of radiation.” d. “You’re having skin problems? That is unusual; let me take a look

at your skin.”

A client scheduled to undergo radiation therapy for breast cancer asks why 6 weeks of daily treatment is necessary. What is the nurse’s best

response? a. “Your cancer is widespread and requires more than the usual

amount of radiation treatment.” b. “Giving larger doses of radiation for a shorter period of time does

not produce better effects and has worse side effects.” c. “Research has shown that more cancer cells are killed if radiation is

given in smaller doses over a longer time period.” d. “It is less likely that your hair will fall out or that you will become

anemic if radiation is given in this manner.”

A client’s radiation implant has become dislodged overnight, and the nurse finds it in the client’s bed. What does the nurse do first?

a. Assess the client’s skin for radiation burns. b. Use tongs to put the implant into the radiation container.

c. Notify the safety officer and move the client to a different room. d. Don gloves and attempt to replace the implant.

A client is receiving a chemotherapeutic agent intravenously through a peripheral line. What is the nurse’s first action when the client reports

burning at the site? a. Check for a blood return. b. Slow the rate of infusion. c. Discontinue the infusion. d. Apply a cold compress.

A client receiving intravenous chemotherapy asks the nurse the reason for wearing a mask, gloves, and gown while administering drugs to the

client. What is the nurse’s best response? a. “These coverings protect you from getting an infection from me.”

b. “I am preventing the spread of infection from you to me or any other client here.”

c. “The policy is for any nurse giving these drugs to wear a gown, gloves, and mask.”

d. “The clothing protects me from accidentally absorbing these drugs.”

A client’s spouse reports that the last time the client received lorazepam (Ativan) before receiving chemotherapy, the client was extremely drowsy

and didn’t remember the trip home. Which is the nurse’s best action? a. Hold the dose of lorazepam for this round of chemotherapy.

b. Explain that this is a normal response to the drug. c. Perform a Mini-Mental State Examination. d. Document the response in the client’s chart.

A client is on chemotherapy and has a platelet count of 25,000. Which intervention is most important to teach this client?

a. “Eat a low-bacteria diet.” b. “Take your temperature daily.”

c. “Use a soft-bristled toothbrush.” d. “Avoid alcohol-based mouthwashes.”

A client with chemotherapy-induced bone marrow suppression has received filgrastim (Neupogen). Which laboratory finding indicates that

this therapy is effective for the client? a. Hematocrit is 28%. b. Hematocrit is 38%.

c. Segmented neutrophil count is 2500/mm3. d. Segmented neutrophil count is 3500/mm3.

What is the priority problem for a client experiencing chemotherapy- induced anemia?

a. Risk for injury related to fatigue b. Fatigue related to decreased oxygenation

c. Body image problems related to skin color changes

d. Inadequate nutrition related to anorexia

A client is hospitalized for chemotherapy. The registered nurse intervenes when observing which action by the nursing assistant?

a. Allowing the client to rest instead of making him or her perform oral hygiene

b. Helping the client wash the groin and axillary areas every 12 hours c. Cutting food and opening food packages when the client’s meal tray

arrives d. Reminding the client to use the incentive spirometer every hour

while awake

The student nurse overhears several staff members referring to a client who is receiving chemotherapy as having “chemo brain.” The student

asks the instructor what that means. Which response by the instructor is best?

a. “That is an awful thing to say and the staff should not call a client by that name.”

b. “It refers to the client’s brain as being irreversibly damaged by the chemotherapy.”

c. “The client has reduced cognitive function that may last for several years.”

d. “The client has delirium related to the toxic effects of the chemotherapy.”

A client with prostate cancer is taking estrogen daily to control tumor growth. He reports that his left calf is swollen and painful. Which is the

nurse’s best action? a. Instruct the client to keep the leg elevated. b. Measure and compare calf circumferences.

c. Apply ice to the calf after massaging it. d. Document this expected response.

A client is receiving interleukin-2 (IL-2) for cancer. Which drug is the nurse prepared to administer if needed?

a. Lorazepam (Ativan) b. Meperidine (Demerol)

c. Furosemide (Lasix) d. Epoetin alfa (Epogen)

A nurse manager on an oncology nursing unit notes an increased incidence of infection and serious consequences for clients on the unit.

Which action by the nursing manager is most beneficial in this situation?

a. Review asepsis policies at a mandatory in-service for staff. b. Spot-check all staff for good handwashing practices.

c. Develop standard protocols to identify and treat clients with infection.

d. Institute protective precautions for all clients receiving chemotherapy.

A client has small cell lung cancer. Which laboratory result requires immediate intervention by the nurse? a. Serum potassium of 5.1 mEq/L

b. Serum sodium of 118 mEq/L c. Hematocrit of 45%

d. Blood urea nitrogen (BUN) of 10 mg/dL

A client with advanced cancer is being treated with intravenous mithramycin (Mithracin). Which clinical manifestation indicates that the

treatment is effective? a. Bowel sounds are active in all four quadrants. b. The client’s serum sodium level is 138 mEq/L. c. The pulse rate is 68 beats/min and bounding.

d. Urine output has increased to 30 mL/hr.

A nurse is reviewing the white blood cell count with differential for a client receiving chemotherapy for cancer. Which finding alerts the nurse

to the possibility of sepsis? a. Total white blood cell count is 9000/mm3.

b. Lymphocytes outnumber basophils. c. “Bands” outnumber “segs.”

d. Monocyte count is 1800/mm3.

A client is receiving high-dose chemotherapy for multiple myeloma. Which intervention is most important for the nurse to implement to

prevent complications during chemotherapy? a. Ensure that the client’s fluid intake is 3000 to 5000 mL/day.

b. Monitor telemetry every hour during therapy. c. Apply pressure to all injection sites for 5 minutes.

d. Assist the client in all ambulatory activities.

The nurse teaches a client with superior vena cava syndrome that

improvement is characterized by which clinical manifestation? a. The client’s hands are less swollen. b. Breath sounds are clear bilaterally. c. The client’s back pain is relieved.

d. Pedal edema is present.

A client has late-stage colon cancer with metastasis to the spine and bones. Which nursing intervention does the nurse add to the care plan to

address a priority problem? a. Provide six small meals and snacks daily. b. Offer the client prune juice twice a day. c. Ensure that the client gets adequate rest.

d. Give the client pain medications around the clock.

After receiving change-of-shift report, which client does the nurse assess first?

a. Client with leukemia who needs an antiemetic before chemotherapy b. Client with breast cancer scheduled for external beam radiation

c. Client with xerostomia associated with laryngeal cancer d. Client with neutropenia who has just been admitted with a possible

infection

The nurse questions which activity for the client with thrombocytopenia? a. Application of warm compresses to bruises

b. Cleaning teeth with a soft-bristled brush c. Taking acetaminophen (Tylenol) for pain

d. Using stool softeners daily for constipation

The nurse prioritizes which intervention in a client with xerostomia secondary to radiation therapy to the neck area? a. Applying lotions and oils to affected areas

b. Wearing a hat to decrease heat loss c. Providing oral care after meals and at bedtime

d. Monitoring vital signs every 4 hours

Which statement indicates that the client needs more teaching about mucositis?

a. “I will rinse my mouth with water after every meal.” b. “I will use a soft-bristled toothbrush to prevent trauma.”

c. “I should use an alcohol-based mouth rinse to kill bacteria.” d. “I cannot use floss because it may irritate my gums.”

MULTIPLE RESPONSE

In planning a teaching session for a client undergoing photodynamic therapy for lung cancer, the nurse includes which statements? (Select all

that apply.) a. “This is a palliative treatment that should decrease your pain.”

b. “Avoid exposure to the sun for 1 to 3 months after the treatment.” c. “Do not eat or drink anything before your treatments.”

d. “Do not remove skin markings between treatments.” e. “You need to wear sunglasses to protect your eyes after treatments.”

f. “Make sure you keep your curtains closed at home afterward.”

The nurse is planning care for a client with hypercalcemia secondary to

bone metastasis. Which interventions are included in the plan? (Select all that apply.)

a. Increase oral fluids. b. Place an oral airway at the bedside.

c. Monitor for Chvostek’s sign. d. Implement seizure precautions. e. Assess for hyperactive reflexes. f. Observe for muscle weakness.

The nurse is caring for a client who has a sealed radiation implant for cervical cancer. Which activities by the nurse are appropriate? (Select all

that apply.) a. Inform the supervisor of the nurse’s positive pregnancy test. b. Obtain the dosimeter badge from the nurse going off shift.

c. Keep the client’s door open for frequent observation. d. Dispose of dirty linen in a red “biohazard” bag. e. Wear a lead apron while providing client care.

Chapter 25: Care of Patients with Infection

Test Bank

MULTIPLE CHOICE

After an infection control in-service, which statement by the nurse demonstrates an accurate understanding of the mode of transmission of

influenza? a. “I will not develop the infection unless I have physical contact with

the client.” b. “I should wear an N95 respirator to provide care for the client with

influenza.” c. “I should try to stay at least 3 feet away from the client, if at all

possible.” d. “The infection is spread through droplets suspended in the air and

inhaled.”

The nurse is told that a client with measles is being admitted. Which action by the nurse is best?

a. Implement Contact Precautions. b. Check negative airflow monitors.

c. Ensure that hand sanitizer is readily available. d. Place the client in a room with another measles client.

A client and his family are waiting for the results of clinical tests to determine whether the client has an infection. They are becoming

anxious. What is the most important assessment that the nurse should make of the client and family members?

a. Understanding of insurance reimbursement for testing b. Use of appropriate coping mechanisms for anxiety

c. Understanding of the infectious disease process d. Understanding of the diagnostic procedures

The nurse is preparing to administer a prescribed IV antibiotic to a client admitted with a serious infection. Which action by the nurse is

most important? a. Check the IV for patency.

b. Assess the client for allergies. c. Double check the “five rights.” d. Teach the client about the drug.

A client is being treated with acetaminophen (Tylenol). Which assessment finding is most likely to occur after a dose of the medication?

a. A febrile seizure b. Nausea and vomiting c. Episodes of sweating

d. Syncope

Which client does the nurse consider to be at increased risk for infection? a. Young adult who wears contact lenses

b. Adult with type 1 diabetes mellitus c. Adult with known hypersensitivity to latex

d. Adolescent using analgesics for migraine headaches

The nurse is assigned to work with a new nursing assistant. Which action by the nursing assistant requires intervention by the registered nurse?

a. Using an alcohol-based hand rub after caring for a client with diarrhea

b. Washing hands for 20 seconds using warm water and friction c. Cleaning especially carefully under fingernails and around a

wedding band d. Using chlorhexidine for handwashing when caring for clients on

neutropenic precautions

The nurse is caring for a client with a large leg wound that has been slow to heal. Which action by the nurse is most appropriate?

a. Use Contact Precautions when caring for the client. b. Double-glove when providing wound care.

c. Help the client choose high-protein items at meals. d. Assess the client’s knowledge of causative factors.

A client comes to the emergency department with a fever, diarrhea, and general malaise. Which information obtained during assessment does the

nurse communicate immediately to the health care provider? a. Blood pressure of 110/90 mm Hg

b. Allergy to aspirin c. The client having just returned from a 14-day trip to Asia

d. A blood transfusion 12 years ago

While sponging a client who has a high fever, the nurse observes the client shivering. Which is the nurse’s priority action?

a. Administering oral acetaminophen b. Placing a heated blanket on the client

c. Stopping sponging the client d. Warming up the water and continuing sponging

The new nurse is caring for a client with a high temperature. Which action by the nurse warrants intervention by the new nurse’s preceptor?

a. Sponging the client while monitoring for shivering b. Applying cool packs to the client’s axillae and groin

c. Monitoring the client’s temperature more often than ordered d. Obtaining a fan from central supply for the client’s room

A client has been admitted with suspected Clostridium difficile infection. Which medication does the nurse plan to administer as a priority?

a. Metronidazole (Flagyl) b. Acetaminophen (Tylenol) c. Tetracycline (Sumycin)

d. Doxycycline (Vibramycin)

An older adult client is admitted with an infection. On assessment, the nurse finds the client slightly confused. Vital signs are as follows:

temperature 99.2° F (37.3° C), blood pressure 100/60 mm Hg, pulse 100, and respiratory rate 20. Which action by the nurse is most appropriate?

a. Perform a Mini-Mental Status Examination. b. Assess the client for other signs of infection.

c. Document the findings and continue to monitor. d. Assess the client’s pain level and treat if needed.

A client is being treated at home for vancomycin-resistant Enterococcus (VRE). The client and the family are worried about spreading the

infection. Which action by the nurse is best? a. Instruct the client to use a separate bathroom.

b. Encourage the family to stay 3 feet away from the client. c. Tell the client to cough into tissues and dispose of them immediately. d. Teach the family ways to increase their immune system functioning.

A client has scabies. In addition to Standard Precautions, which information is most important to communicate to visitors and health

care providers? a. Do not allow children to visit.

b. Wear gloves when entering the room. c. Wear a mask when within 3 feet of the client.

d. Keep head covered when providing care.

Before discharge, the nurse confirms that the client understands antibiotic therapy for a wound infection by which statement?

a. “I should take the antibiotic until my temperature is normal.”

b. “If my temperature elevates, I should increase my dose of antibiotic.”

c. “If my drainage is clear, I do not need the antibiotic.” d. “I need to take the medication until the prescription is finished.”

A client has been admitted for the second time to treat tuberculosis (TB). Which referral does the nurse initiate as a priority?

a. Social worker to see if the client can afford the medications b. Visiting nurses to arrange directly observed therapy on dismissal

c. Psychiatric nurse liaison to assess reasons for noncompliance d. Infection control nurse to arrange testing for drug resistance

The nurse is caring for a client with a suspected infection. Which action by the nurse is most appropriate?

a. Give antibiotics as soon as possible to prevent sepsis. b. Obtain all required cultures, then administer the antibiotic.

c. Wait for culture results to give the most appropriate antibiotic. d. Defer cultures unless the client shows signs of drug resistance.

The nurse reviews laboratory results for a client and notes that the erythrocyte sedimentation rate (ESR) is 32 mm/hr. What action by the

nurse is best? a. Document the findings and call the health care provider.

b. Assess the client for any manifestations of infection or inflammation. c. Review the client’s chart to see what medications have been given.

d. Call the physician and request blood cultures and a chest x-ray.

The nurse works in a long-term care facility. Which resident does the nurse assess most carefully for manifestations of infection?

a. Resident who has long-standing dementia b. Resident with incontinence

c. Resident who eats a lot of sweets and little protein d. Resident whose family won’t allow an influenza vaccination

A client is diagnosed with varicella (chickenpox). The nurse places the client on which precautions?

a. Airborne

b. Standard c. Contact d. Droplet

MULTIPLE RESPONSE

A client is admitted with infection and a high fever. Which assessments by the nurse take priority? (Select all that apply.)

a. Blood pressure b. Mental status c. Pulse quality

d. Respiratory effort e. Skin turgor

f. Bowel sounds

The nurse is assessing a client’s skin for local signs of infection. Which signs does the nurse assess for? (Select all that apply.)

a. Fever b. Redness c. Warmth

d. Pain e. Swelling

f. Increased erythrocyte sedimentation rate (ESR)

)

Chapter 26: Assessment of the Skin, Hair, and Nails

Test Bank

MULTIPLE CHOICE

The nurse is planning care for an older client who has very thin skin on the backs of the hands and arms. What is the client’s priority problem?

a. Risk for injury b. Infection

c. Poor self-image d. Discomfort

A client has a suspected superficial fungal infection. The nurse prepares

the client for a culture by explaining the procedure. Which statement by the client indicates a correct understanding of the procedure?

a. “The doctor will shave off a small piece of the lesion.” b. “You will be performing what is called a punch biopsy.”

c. “A sample is obtained by simply scraping the lesion.” d. “You’ll squeeze material from the lesion to send to the laboratory.”

The nurse observes yellow-tinged sclera on a client with dark skin. Based on this observation, what is the nurse’s best action?

a. Evaluate the client further for hepatitis. b. Examine the soles of the client’s feet.

c. Inspect the client’s oral mucosa. d. Place the client in contact isolation.

A client has a bluish tinge to the palms, soles, and conjunctivae. Based on these assessment data, what does the nurse do next?

a. Take a medication history. b. Assess pulse oximetry.

c. Assess the client’s personal hygiene. d. Palpate the soles and palms.

An older client with age spots is fearful of contracting skin cancer but wants to continue his hobby of outdoor gardening. Which statement by

the client indicates a good understanding of the teaching about this issue?

a. “I will avoid staying outside during the day.” b. “I can use only oil-based tanning lotion.” c. “I have to start growing plants indoors.”

d. “I will wear a hat and gloves when gardening.”

An older client expresses concern about developing new “age spots.” Which instruction is most important for the nurse to provide to the

client? a. “Limit the time you spend in the sun.”

b. “Monitor for signs of infection.” c. “Monitor spots for color change.”

d. “Use skin creams to prevent drying.”

A client is seen in the clinic for a persistent hand rash. When taking the client’s history, the nurse places priority on obtaining information

related to which topic? a. Age

b. Gender c. Occupation and hobbies

d. Socioeconomic status

A client is admitted with inflamed soft tissue folds around his nail plates. Which question by the nurse elicits the most useful information about

the possible condition? a. “What do you do for a living?”

b. “Do you keep your nails manicured?” c. “Do you have diabetes?”

d. “Have you had any fungal nail infections?”

A client has multiple bruises on the arms. Which question provides the nurse with the most information?

a. “Are you using lotion on your skin?” b. “Do you have a family history of this?”

c. “Do your arms itch?”

d. “What medication are you taking?”

A client had an excisional biopsy on a neck lesion. Which information does the nurse include in the discharge instructions?

a. “Stay in bed today to prevent excessive bleeding from the incision.” b. “Do not drive until you have recovered from the anesthesia.”

c. “You will need to change the dressing daily for a week.” d. “Keep the dressing on until tomorrow, then you may remove it.”

A client asks the nurse if a Wood’s light examination is painful. Which response by the nurse is accurate?

a. “A local anesthetic will be used to prevent pain.” b. “The pain lasts only a few seconds.”

c. “Some clients feel a pressure-like sensation.” d. “The examination does not cause discomfort.”

A client expresses concern about a rash located beneath her breast. What

statement by the client indicates a good understanding of this condition? a. “This rash is probably due to fluid overload.”

b. “I need to wash this daily with antibacterial soap.” c. “I can use powder to keep this area dry.”

d. “I will schedule a mammogram as soon as I can.”

A client comes to the clinic reporting pain and itching from blisters on both arms. This finding indicates an abnormality in which layer of the

skin? a. Adipose tissue

b. Dermis c. Epidermis

d. Stratum corneum

A client has two skin lesions, each the size of a nickel, on his chest. Both lesions are flat and are a darker color than the rest of the client’s skin.

How does the nurse document this finding? a. Two 2-cm hyperpigmented patches b. Two 1-inch erythematous plaques

c. Two 2-mm pigmented papules d. Two 1-inch moles

The nurse reads in the chart that a client has a fine, macular rash on the lower extremities. The nurse inspects the client’s skin, looking for lesions

that can be described with which term? a. Flat

b. Raised c. Rough

d. Blood-filled

On assessing a client’s lower extremities, the nurse notices that one leg is pale and cooler to the touch. Which assessment does the nurse perform

next? a. Ask about a family history of skin disorders. b. Palpate the client’s pedal pulses bilaterally.

c. Check for the presence of Homans’ sign. d. Assess the client’s skin for adequate skin turgor.

The nurse observes hirsutism in a female client. What does the nurse do next?

a. Assess for deepening of the voice. b. Assess personal hygiene habits.

c. Document the finding.

d. Prepare the client for a biopsy.

MULTIPLE RESPONSE

The nurse is assessing for skin changes in an older woman. Which findings require immediate referral? (Select all that apply.)

a. Excessive moisture under axilla b. Increased hair thinning

c. Increased presence of fungal toenails d. Lesion with various colors

e. Spider veins on legs f. Asymmetric 6-mm dark lesion on forehead

Chapter 27: Care of Patients with Skin Problems

Test Bank

MULTIPLE CHOICE

A client has very dry skin. Which is the best intervention for the nurse to teach the client?

a. “Be sure to use lots of moisturizer several times a day.” b. “Avoid wearing stockings or other constricting clothing.”

c. “Use antimicrobial soap so scratching won’t cause infection.” d. “After you bathe, put lotion on before your skin is totally dry.”

Which intervention best assists a client with pruritus? a. “Keep your fingernails cut short and keep them clean.”

b. “Drinking extra fluids decreases stimulation of itch receptors.” c. “Wear soft, breathable clothing made from material like cotton.”

d. “Avoid immersing the areas in water and dry thoroughly after bathing.”

A client has urticaria and has been prescribed diphenhydramine (Benadryl). Which information is most important for the nurse to teach

the client? a. “Wear sunscreen when you are outside.”

b. “Avoid drinking alcoholic beverages.” c. “Do not take aspirin-containing drugs.” d. “Take this medicine on a full stomach.”

When changing the dressing on a partial-thickness wound, a nurse observes small, pale pink bumps within the wound bed. Which action by

the nurse is best? a. Remove the bumps with a sterile scalpel.

b. Document and continue the current treatment. c. Clean the wound vigorously to remove the bumps. d. Culture the wound and place the client in isolation.

Which nursing intervention is best for the nurse to use to enhance healing of a 1-week-old partial-thickness wound?

a. Ensure that the client is systemically oxygenated. b. Restrict the client’s movement with bedrest. c. Cover the wound with an airtight dressing.

d. Apply hydrocortisone cream as ordered.

A client is going home with a surgical wound on the coccyx that is to heal

by second intention. Which priority problem must the nurse address in the teaching plan?

a. Pain b. Infection

c. Poor body image d. Dehydration

Which nursing intervention best assists a bedridden client to keep skin intact?

a. Apply talcum powder to the perineal area. b. Turn the client every 2 to 4 hours.

c. Use a foam mattress pad. d. Use a lift sheet to move the client in bed.

The nurse determines that a client has a Braden Scale score of 9. Which is the nurse’s best intervention related to this assessment?

a. Document the finding per protocol. b. Reassess the client in 3 days.

c. Increase the client’s fluid intake. d. Consult with the health care provider.

Which client does the nurse assess to be at greatest risk for pressure ulcer development?

a. Client who has pneumonia b. Client who requires assistance with ambulation

c. Client with hypertension on multiple medications d. Incontinent client with limited mobility

Which dressing choice does the nurse make to protect a heavily draining deep pressure ulcer?

a. Wet-to-dry gauze b. Dry cotton gauze

c. Alginate packing, dry gauze cover d. Hydrocolloidal transparent film cover

When getting a client up in a chair, the nurse notices that the pressure- relieving mattress overlay has deep imprints of the client’s buttocks,

heels, and scapulae. Which is the nurse’s best action? a. Turn the mattress overlay to the opposite side.

b. Do nothing because this is an expected occurrence. c. Apply a different pressure-relieving device. d. Reinforce the overlay with extra cushions.

A client has a deep wound covered with a wet-to-damp dressing. Which intervention does the nurse include on this client’s care plan?

a. Change the dressing every 6 hours around the clock. b. Leave the dressing intact until next week.

c. Change the dressing when the current dressing is saturated. d. Apply a new dressing when the seal breaks and the dressing leaks.

A client has a wound that is draining heavily. Which type of dressing does the nurse use on this wound?

a. Hydrophilic b. Synthetic

c. Hydrophobic d. Biologic

Which client is receiving appropriate treatment? a. Client with an ulcer and slight necrosis receiving whirlpool

treatment b. Client with an eschar-covered sacral ulcer receiving whirlpool

therapy

c. Client with sunburn and erythema soaking in warm water for 20 minutes

d. Client with urticaria instructed to take warm showers twice a day

ANS: A The nurse observes a small opening that is draining purulent material on

the skin over the trochanter area of a bedridden client. Which is the nurse’s next best action?

a. Probe for a larger pocket of necrotic tissue. b. Apply a transparent film dressing.

c. Measure the reddened area on the skin surface. d. Apply alginate dressing daily.

A client has a wound on his left trochanter that is 4 inches in diameter, with black tissue at the perimeter, and bone is exposed. Which is the

nurse’s best action? a. Document as a stage I pressure ulcer and apply a transparent

dressing. b. Document as a stage II pressure ulcer and start wet-to-dry gauze

treatments. c. Document as a stage III pressure ulcer and start antibiotic therapy.

d. Document as a stage IV pressure ulcer and prepare the client for débridement.

A client presents with a pressure ulcer on the ankle. Which is the first intervention that the nurse implements?

a. Draw blood for albumin, prealbumin, and total protein. b. Prepare for and assist with obtaining a wound culture.

c. Place the client in bed and instruct him or her to elevate the foot. d. Assess the affected leg for pulses, skin color, and temperature.

Which finding puts a client at greatest risk for wound infection? a. Immune compromised status

b. Presence of a deep wound c. Severely reddened skin

d. Coexisting medical conditions

A client has a chronic wound that is being treated with a vacuum-assisted wound closure (VAC) device. Which intervention by the nurse takes

priority? a. Provide pain medication as needed.

b. Assess the VAC every 2 hours for bleeding.

c. Check the integrity of the dressing seal every 4 hours. d. Document the wound size with each dressing change.

Which statement made by the caregiver of a home care client indicates a need for clarification regarding pressure ulcer prevention and

treatment? a. “I help him shift his position every hour when he sits in the chair.”

b. “I massage his tailbone every morning when he gets up because it is red.”

c. “I apply lotion to his arms and legs every evening because they are so dry.”

d. “He drinks a nutritional supplement between meals to maintain his weight.”

A client has been identified as being at risk for formation of pressure ulcers. Which dietary choices by the client indicate a good understanding

of teaching related to this condition? a. Low-fat diet with whole grains and cereals and vitamin supplements

b. High-protein diet with vitamins and mineral supplements c. Vegetarian diet with nutritional supplements and fish oil capsules

d. Low-fat, low-cholesterol, high-fiber, low-carbohydrate diet

A client with a pressure ulcer has the following laboratory values: white blood count 8000/mm3, prealbumin 15.2 mg/dL, albumin 4.2 mg/dL, and

lymphocyte count 2000/mm3. Which action by the nurse is most appropriate?

a. Document the findings. b. Request a dietary consult.

c. Place the client in isolation. d. Assess the client’s vital signs.

The nurse sees a client with which condition first to evaluate for wound infection?

a. Pending blood cultures b. Thin serous wound drainage

c. White blood cell count of 23,000/mm3 d. Decrease in wound size

A client has been admitted for vacuum-assisted wound closure (VAC) treatment for a chronic leg wound. The client’s past medical history

includes atrial fibrillation and stroke, and medications include warfarin sodium (Coumadin) and sotolol (Betapace). Which action by the nurse is

most appropriate? a. Place the client on continuous telemetry monitoring. b. Call the health care provider with this information.

c. Let the wound care nurse know that the client has arrived. d. Order the VAC and gather other needed supplies.

The nurse assesses the client with which condition first? a. Folliculitis b. Furuncles c. Cellulitis

d. Stage II ulcer

Which client should be placed in isolation awaiting possible diagnosis of infection with methicillin-resistant Staphylococcus aureus (MRSA)?

a. Client admitted from a nursing home with furuncles and folliculitis b. Client with a leg cut and other trauma from a motorcycle crash c. Client with a rash noticed after participating in sporting events

d. Client transferred from intensive care with an elevated white count

The occupational health nurse is seeing several nurses with skin problems. The nurse with which condition was most likely infected by a

client? a. A herpes simplex virus 1 (HSV-1) oral lesion

b. Herpetic whitlow of the fingertip c. Herpes zoster on one side of the body

d. Severe postherpetic neuralgia

A client had a skin graft with a pedicle flap. Which is the priority nursing intervention for this client in the early postoperative phase?

a. Monitor the donor site to detect hemorrhage. b. Check the flap edges for adequate perfusion.

c. Turn the client often to prevent pressure ulcers. d. Perform interventions to prevent contractures.

A home care client with a leg wound is unable to climb stairs to the second floor, where the bathtub is located. Which is the nurse’s best

intervention? a. “I’ll show you how to use a syringe to cleanse the wound.”

b. “It is not necessary to clean this wound because it is not infected.” c. “You can use the kitchen sink and clean tap water for this purpose.” d. “You will have to come to the hospital each day for hydrotherapy.”

A client has a furuncle in the axilla. Which statement by the client indicates a good understanding of how to care for this condition?

a. “I’ll apply cortisone cream to reduce the inflammation.” b. “I will squeeze the lesion until all pus is removed.”

c. “I’ll keep my arm down at my side to prevent spread.” d. “I will cleanse the area and apply warm compresses and antibiotic

cream.”

The home care nurse is visiting an older adult client who has diabetes and “skinned his shin” yesterday. An intact scab is seen over the

abrasion, and the skin around it is red, warm, and hard. Which is the nurse’s best action?

a. Teaching the client how to apply cold compresses to the area b. Lifting an area of scab to see whether any exudate can be expressed

c. Measuring the length and width of the red area d. Calling the health care provider for a prescription to treat cellulitis

An older client is observed scratching and rubbing white ridges on the skin between fingers, on the wrists, in the axillae, and around the waist.

Which is the nurse’s priority intervention? a. Placing the client in a single room b. Administering an antihistamine

c. Assessing for allergies d. Applying cold compresses

The home health nurse is visiting a client who is treating a chronic wound. The nurse assesses that the client only performs daily wound care twice a week owing to cost. Which statement by the nurse best

addresses this issue? a. “You can use tap water instead of sterile saline to clean your

wound.” b. “If you don’t clean the wound properly, you could end up in the

hospital.” c. “Sterile procedure is necessary to keep this wound from getting

bigger.” d. “The only thing that really matters is good handwashing with

wound care.”

Which statement by a client with psoriasis indicates a need for further teaching?

a. “At the next family reunion, I’m going to ask my relatives if they have psoriasis.”

b. “I have to make sure I keep my lesions covered, so I do not spread this to others.”

c. “I expect that these patches will get smaller when I lay out in the sun.”

d. “I should continue to use the cortisone ointment as the patches shrink and dry out.”

Which question does the nurse ask to identify a possible trigger for

worsening of a client’s psoriatic lesions? a. “Have you eaten a large amount of chocolate lately?”

b. “Have you been under a lot of stress lately?” c. “Have you used a public shower recently?”

d. “Have you been out of the country recently?”

The nurse notes that a client who has been using psoralens–ultraviolet A (PUVA) therapy for psoriasis for 1 month has darkening of the skin.

Which is the nurse’s best action? a. Document this assessment finding.

b. Instruct the client to reduce the dose. c. Instruct the client to drink more water.

d. Instruct the client to apply cortisone cream.

The nurse is teaching a community group of older adults about skin problems. Which intervention by the nurse is most important?

a. Encourage them to get Zostavax. b. Instruct them to monitor skin dryness.

c. Teach them how to moisturize skin. d. Discuss how skin disorders are spread.

A client has a group of vesicles on top of a red base on the trunk. The nurse prepares the client for which intervention?

a. Venipuncture for blood cultures b. Tzanck smear and viral cultures

c. Cotton swab culture of the vesicles d. Scraping of the lesions for examination

An African-American woman had a breast biopsy 1 year ago. The incision site is elevated, dark, and protruding. Which information does

the nurse provide to the client? a. A keloid has formed over the biopsy scar.

b. The benign tumor has undergone malignant changes. c. A deep infection has probably become symptomatic.

d. Chronic inflammatory changes have occurred in the skin.

A client at a community skin screening has numerous skin lesions. Which one does the nurse evaluate first?

a. Beige freckles on the backs of both hands b. Irregular blue mole with white specks on the lower leg

c. Large cluster of pustules in the right axilla

d. Raised, tubular, white areas on the inner aspects of the wrists

A client has methicillin-resistant Staphylococcus aureus (MRSA) and is receiving vancomycin (Vancocin) 500 mg IV every 6 hours. What is an

important nursing intervention related to this drug? a. Administering it over 30 minutes using an IV pump

b. Giving the client diphenhydramine (Benadryl) before the drug c. Assessing the IV site at least every 2 hours for thrombophlebitis

d. Ensuring that the client has increased oral intake during therapy

The nurse inspects the site where a client’s basal cell carcinoma has been treated with cryosurgery and finds that the area is red, with a blister in

the center. Which action does the nurse take? a. Culture the site.

b. Notify the surgeon. c. Apply hydrocortisone cream.

d. Reassure the client.

A client presents to the clinic with a swollen arm lesion that contains old

blood and a sunken-in center. Which question by the nurse yields the most useful information?

a. “Have you traveled out of the country recently?” b. “What do you do for a living or for hobbies?” c. “Do you hike or engage in outdoor activities?”

d. “Are you exposed to places where spiders might be?”

Which is the most important question for the nurse to ask a young adult woman about to begin taking isotretinoin (Accutane)? a. “Do you spend a great deal of time in the sun?”

b. “Have you or any family members ever had skin cancer?” c. “Which method of contraception are you using?”

d. “Do you drink alcoholic beverages?”

A client has a widespread fungal infection. For which drug does the nurse anticipate an order? a. Clindamycin (Cleocin)

b. Acyclovir (Zovirax) c. Linezolid (Zyvox)

d. Ketoconazole (Nizoral)

Which characteristic regarding leprosy is true? a. A few cases have been found in underdeveloped countries.

b. Affected clients must be confined away from the general population. c. Treatment with multiple antibiotic agents is necessary.

d. Treatment of leprosy involves immunosuppressive drugs.

A client who has had a rhinoplasty is swallowing frequently and belching. Which action does the nurse take?

a. Notify the surgeon. b. Raise the head of the bed.

c. Assist the client with liquids. d. Continue to assess.

A client is at high risk for developing skin cancer but will not perform total skin self-examination (TSSE) consistently. Which nursing

intervention is the most important? a. Reinforce previous teaching on the TSSE technique.

b. Teach the client the dangers of skin cancer. c. Determine whether the client has a partner to help.

d. Carefully document all existing skin lesions.

The nurse assessing a client notices a lesion on the skin as shown in the photograph below. For which diagnostic test does the nurse prepare the

client?

a. Punch skin biopsy b. Viral cultures

c. Wood’s lamp examination d. Diascopy

MULTIPLE RESPONSE The nurse is prioritizing care to prevent pressure sores for a client who is

immobilized. Which interventions are appropriate? (Select all that apply.)

a. Place a small pillow between bony surfaces. b. Elevate the head of the bed to 45 degrees.

c. Limit fluids and proteins in the diet. d. Use a lift sheet to assist with repositioning.

e. Reposition the client who is in a chair every 2 hours. f. Keep the heels off the bed surfaces.

g. Use a rubber ring to decrease sacral pressure when up in the chair.

In preparation for a client being admitted with herpes zoster, what does the nurse do? (Select all that apply.)

a. Prepare a room for reverse isolation. b. Assess staff for a history of or vaccination for chickenpox.

c. Check the admission orders for analgesia. d. Choose a roommate who also is immune suppressed.

e. Ensure that gloves are available in the room.

Which preventive measures does the nurse use to prevent skin lesions in older adults? (Select all that apply.)

a. Use a lift sheet when moving the client in bed. b. Avoid tape when applying dressings. c. Avoid any type of restraining device.

d. Avoid whirlpool therapy. e. Use loose dressing on all wounds.

f. Apply dressings only when wounds are draining.

In assessing a client’s wound, which finding assists the nurse in determining that the wound is infected? (Select all that apply.)

a. It is open. b. It has granulation tissue.

c. It is inflamed. d. It has an odor.

e. It has heavy exudates. f. It contains necrotic tissue.

Chapter 28: Care of Patients with Burns

Test Bank

MULTIPLE CHOICE

The RN has assigned a client who has an open burn wound to the LPN. Which instruction is most important for the RN to provide the LPN?

a. Administer the prescribed tetanus toxoid vaccine. b. Assess wounds for signs of infection.

c. Have the client cough and breathe deeply. d. Wash hands on entering the client’s room.

When providing care for a client with an acute burn injury, which nursing intervention is most important to prevent infection by

autocontamination? a. Avoid sharing equipment such as blood pressure cuffs between

clients. b. Change gloves between wound care on different parts of the client’s

body. c. Use the closed method of burn wound management for all wound

care. d. Use proper and consistent handwashing by all members of the staff.

The nurse is teaching burn prevention to a community group. Which information shared by a member of the group causes the nurse the

greatest concern? a. “I get my chimneys swept every other year.”

b. “My hot water heater is set at about 120 degrees.” c. “Sometimes I wake up at night and smoke.”

d. “I use a space heater when it gets below zero.”

The nurse is conducting a home safety class. It is most important for the nurse to include which information in the teaching plan?

a. Have an escape route everyone knows about.

b. Keep a smoke detector in each bedroom. c. Use space heaters instead of gas heaters.

d. Use carbon monoxide detectors in the garage.

A client with facial burns asks the nurse if he will ever look the same. Which response is best for the nurse to provide?

a. “With reconstructive surgery, you can look the same.” b. “We can remove the scars with the use of a pressure dressing.”

c. “You will not look exactly the same but cosmetic surgery will help.” d. “You shouldn’t start worrying about your appearance right now.”

The nurse has provided instruction on the facial pressure garment to a client with facial burns. Which statement indicates that the client

understands these instructions? a. “My scars should be less severe with the use of this mask.” b. “The mask will help protect my skin from sun damage.”

c. “This treatment will help prevent infection.” d. “Using the mask will keep scars from being permanent.”

Which finding indicates to the nurse that a client with a burn injury has a positive perception of his appearance?

a. Allowing family members to change the dressings b. Discussing future surgical reconstruction c. Performing morning care independently

d. Wearing the pressure dressings as ordered

Which statement best exemplifies a client’s understanding of rehabilitation after a full-thickness burn injury?

a. “I am fully recovered when all the wounds are closed.” b. “I will eventually be able to perform all my former activities.”

c. “My goal is to achieve the highest level of functioning that I can.” d. “Full recovery from a major burn injury never occurs.”

Which finding indicates to the nurse that a client understands the psychosocial impact of a severe burn injury? a. “It is normal to feel some depression.” b. “I will go back to work immediately.”

c. “I will not feel anger about my situation.” d. “Once I get home, things will be normal.”

A client is in the emergency department after being rescued from a house fire. After the initial assessment, the client develops a loud, brassy cough.

What intervention by the nurse takes priority? a. Apply oxygen and continuous pulse oximetry.

b. Allow the client to suck on small quantities of ice chips. c. Request an antitussive medication from the physician.

d. Have the respiratory therapist provide humidified room air.

A client has burns on both legs. These areas appear white and leather- like. No blisters or bleeding is present, and the client describes just a “small amount of pain.” How does the nurse categorize this injury?

a. Partial thickness deep b. Partial thickness superficial

c. Full thickness d. Superficial

A client has a large burned area on the right arm. The burned area appears pink, has blisters, and is very painful. How does the nurse

categorize this injury? a. Full thickness

b. Partial thickness superficial c. Partial thickness deep

d. Superficial

A client with a new burn injury asks the nurse why he is receiving intravenous cimetidine (Tagamet). What is the nurse’s best response?

a. “Tagamet will stimulate intestinal movement so you can eat more.” b. “Tagamet can help prevent hypovolemic shock, which can be fatal.”

c. “This will help prevent stomach ulcers, which are common after burns.”

d. “This drug will help prevent kidney damage caused by dehydration.”

A client who is burned is drooling and is having difficulty swallowing. Which action does the nurse take first?

a. Assess level of consciousness and pupillary reactions. b. Ascertain the time food or liquid was last consumed.

c. Auscultate breath sounds over the trachea and mainstem bronchi. d. Measure abdominal girth and auscultate bowel sounds.

On assessment, the nurse notes that a client has burns inside the mouth

and is wheezing. Several hours later, the wheezing is no longer heard. What is the nurse’s next action?

a. Document the findings and reassess in 1 hour. b. Loosen any constrictive dressings on the chest.

c. Raise the head of the bed to a semi-Fowler’s position. d. Gather appropriate equipment and prepare for intubation.

A client who is receiving fluid resuscitation per the Parkland formula after a serious burn continues to have urine output ranging from 0.2 to 0.25 mL/kg/hour. After the health care provider checks the client, which

order does the nurse question? a. Increase IV fluids by 100 mL/hr.

b. Administer furosemide (Lasix) 40 mg IV push. c. Continue to monitor urine output hourly.

d. Draw blood for serum electrolytes stat.

A client is 24 hours post burn and has the following laboratory results. Which result does the nurse report to the health care provider

immediately? a. Arterial pH, 7.32 b. Hematocrit, 52%

c. Serum potassium,7.5 mEq/L d. Serum sodium, 131 mEq/L

Ten hours after a client with 50% burns is admitted, her blood glucose level is 152 mg/dL. What action by the nurse is most appropriate?

a. Document the finding. b. Obtain a family history for diabetes.

c. Repeat the glucose measurement. d. Stop IV fluids containing dextrose.

A client who was burned has crackles in both lung bases and a respiratory rate of 40 breaths/min and is coughing up blood-tinged

sputum. Which action by the nurse takes priority? a. Administer digoxin.

b. Perform chest physiotherapy. c. Document and reassess in an hour.

d. Place the client in an upright position.

A client who is admitted after a thermal burn injury has the following vital signs: blood pressure, 70/40; heart rate, 140 beats/min; and

respiratory rate, 25 breaths/min. He is pale, and it is difficult to find pedal pulses. Which action does the nurse take first?

a. Begin intravenous fluid resuscitation. b. Check pulses with a Doppler device.

c. Obtain a complete blood count (CBC). d. Obtain an electrocardiogram (ECG).

A client is brought to the emergency department by an emergency medical services (EMS) squad after being burned with unknown

chemicals. The client’s body is covered with a white, powdery substance, and the client cries out, “Get this stuff off me! It’s burning me!” Which

action by the nurse is most appropriate? a. Have the client take a shower, and bag all clothing.

b. Brush the substance off the client and remove clothes. c. Call poison control to try to identify the chemical.

d. Start an IV line and prepare to administer analgesics.

A client suffered a 45% total body surface area (TBSA) burn and was intubated. Twelve hours later, bowel sounds were absent in all four

abdominal quadrants. Which is the nurse’s best action?

a. Administer a laxative. b. Document the finding.

c. Prepare to insert a nasogastric (NG) tube. d. Reposition the client on the right side.

A client has experienced an electrical injury of the lower extremities. Which priority assessment data should be obtained from this client?

a. Range of motion in all extremities b. Heart rate, rhythm, and electrocardiogram (ECG)

c. Respiratory rate and pulse oximetry d. Orientation to time, place, and person

A client is receiving fluid resuscitation after a burn. Which finding indicates that fluid resuscitation is adequate for this client?

a. Hematocrit = 60% b. Heart rate = 130 beats/min c. Increased peripheral edema

d. Urine output = 50 mL/hr

The nurse provides wound care for a client 48 hours after a burn injury. To achieve the desired outcome of the procedure, which action does the

nurse perform first? a. Apply silver sulfadiazine (Silvadene) ointment.

b. Cover the area with an elastic wrap. c. Place a synthetic dressing over the area.

d. Remove loose nonviable tissue.

Which nursing intervention is likely to be most helpful in providing adequate nutrition while a client is recovering from a thermal burn

injury? a. Allowing the client to eat whenever he or she wants

b. Beginning parenteral nutrition high in calories c. Including 3000 kcal/day of calories with meals

d. Providing a low-protein, high-fat diet

The family of a client who has been burned asks when the client will no longer be at greater risk for infection. What is the nurse’s best response?

a. “As soon as the antibiotics have been finished.” b. “As soon as albumin levels returns to normal.”

c. “When fluid remobilization has started.” d. “When the burn wounds are closed.”

A client with open burn wounds begins to have diarrhea. The client is found to have a below-normal temperature, with a white blood cell count

of 4000/mm3. Which action by the nurse is most appropriate? a. Continue to monitor the client.

b. Increase the temperature in the room. c. Increase the rate of intravenous fluids.

d. Prepare to do a workup for sepsis.

The nurse uses topical gentamicin sulfate (Garamycin) on a client’s burn injury. Which laboratory value does the nurse monitor?

a. Creatinine b. Red blood cells

c. Sodium d. Magnesium level

The nurse has been teaching a client about skin grafting procedures. What statement indicates that the client needs further education about

allografts? a. “Because the graft is my own skin, there is no chance it won’t

‘take.’”

b. “For a few days after surgery, the donor sites will be painful.” c. “I will have some scarring in the area where the skin is removed.” d. “I am still at risk for infection after the procedure until the burn

heals.”

Which intervention by the nurse is most appropriate to reduce a client’s pain after a burn injury?

a. Administering morphine sulfate 4 mg intravenously b. Administering morphine sulfate 4 mg intramuscularly

c. Applying ice to the burned area for 20 minutes d. Avoiding tactile stimulation near the burned area

The nurse assesses a client in the burn unit after the client was repositioned by the nursing assistant. The nurse intervenes after finding

the client repositioned in what manner? a. Supine with one pillow behind the head

b. Semi-Fowler’s position with arms elevated c. Wrists extended to 30 degrees in a splint

d. A towel roll placed under the neck or shoulder

A client has severe burns around the right hip. Which position does the nurse instruct the nursing assistant to use to maintain maximum

function of this joint? a. Hip maintained in 30-degree flexion

b. Hip at zero flexion with leg flat c. Knee flexed at 30-degree angle d. Leg abducted with foam wedge

A client who suffered burns in a house fire reports a headache and is not consistently oriented to time. Which intervention by the nurse is most

appropriate? a. Increase the client’s oxygen and obtain blood gases.

b. Draw blood for a carboxyhemoglobin level. c. Increase the client’s intravenous fluid rate.

d. Perform a thorough Mini-Mental Status Examination.

A client who has had a full-thickness burn is being discharged from the hospital. Which information is most important for the nurse to provide

before discharge? a. How to maintain home smoke detectors

b. Joining a community reintegration program c. Learning to perform dressing changes

d. Options available for scar removal

An older adult client with burns has a white blood cell count of 10,000/mm3. The client is afebrile with a heart rate of 110 beats/min, a

respiratory rate of 20 breaths/min, and blood pressure of 112/68 mm Hg. The client’s wound is pale, and edema is noted in the surrounding

tissues. Which intervention by the nurse is most appropriate? a. Assess the client’s skin for signs of adequate perfusion. b. Calculate intake and output ratio for the last 24 hours.

c. Prepare to obtain blood and wound cultures. d. Place the client in an isolation room.

SHORT ANSWER

A client who weighs 90 kg and had a 50% burn injury at 10 AM arrives at the hospital at noon. Using the Parkland formula, calculate the rate

that the nurse should use to deliver fluid when the IV is started at noon.

OTHER

A client is in the emergency department with a burn calculated to be 35% TBSA. The nurse prepares the client for an IV insertion in which

location?

Chapter 29: Assessment of the Respiratory System

Test Bank

MULTIPLE CHOICE

A client has undergone a thoracentesis. Which assessment finding requires immediate action by the nurse?

a. Decreased level of consciousness b. Tachycardia

c. Increased temperature d. Slowed respiratory rate

The nurse assesses a client after an open lung biopsy. Which assessment finding is matched with the correct intervention?

a. Client feels “dizzy;” nurse applies oxygen and pulse oximeter. b. Client’s heart rate is 55 beats/min; nurse withholds pain medication.

c. Client has reduced breath sounds; nurse calls physician immediately.

d. Client’s respiratory rate is 18 breaths/min; nurse decreases oxygen flow rate.

The nurse is caring for four clients who had arterial blood gases (ABGs). Which laboratory value warrants immediate intervention by the nurse?

a. HCO3– of 25 mEq/L b. SpO2 of 96%

c. pH of 7.38 d. PaCO2 of 48 mm Hg

The nurse is calculating a client’s smoking history in pack-years. The client has recently been diagnosed with lung cancer. Which is the nurse’s

priority intervention during the interview? a. Encourage the client to quit smoking to stop further cancer

development. b. Encourage the client to be completely honest about both tobacco

and marijuana use. c. Maintain a nonjudgmental attitude to avoid causing the client to feel

guilty. d. Avoid giving the client false hope regarding cancer treatment and

prognosis.

When assessing a client’s respiratory status, which information is of highest priority for the nurse to obtain?

a. Average daily fluid intake b. Neck circumference c. Height and weight

d. Occupation and hobbies

The nursing assistant reports to the nurse that an African-American client’s pulse oximetry reading is 93%. The client has no complaints.

Which action by the nurse is most appropriate? a. Replace the sensor probe of the oximeter.

b. Place the probe on another finger. c. Assess other signs of respiratory adequacy.

d. Prepare to obtain arterial blood gases.

The nurse is caring for an older adult client with a pulmonary infection. Which nursing action is a priority with this client? a. Encouraging the client to increase fluid intake

b. Assessing the client’s level of consciousness c. Raising the head of the bed to at least 45 degrees

d. Providing the client with humidified oxygen

The nurse is assessing a client’s breath sounds. Which assessment finding has been correctly linked to the nurse’s primary intervention?

a. Hollow sounds heard over trachea; increase oxygen flow rate. b. Crackles heard in bases; have the client cough forcefully.

c. Wheezes heard in central areas; administer inhaled bronchodilator. d. Vesicular sounds heard over the periphery; have the client breathe

deeply.

A client has a long-standing history of chronic obstructive pulmonary disease (COPD). Which laboratory finding does the nurse correlate with

this condition? a. White blood cell count, 7500/mm3

b. Hemoglobin, 22 g/dL c. Neutrophils, 6000/ mm3

d. Monocytes, 600/mm3

The nurse is caring for several clients on a respiratory unit. Which client does the nurse see first?

a. Older adult with an SaO2 of 96% on room air b. Adult client with an SaO2 of 94% on 2 L/min

c. Young adult with an arterial oxygen level of 85% d. Young adult with an arterial oxygen level of 94%

A client with a history of chronic obstructive pulmonary disease (COPD) presents to the clinic with increased cough and low-grade temperature.

Which question by the nurse elicits the most useful information? a. “How long have you been sick?”

b. “Has your sputum changed color?” c. “Is anyone else in your house sick?”

d. “Do you take any medications?”

A client tells the nurse that he usually expectorates about 2 ounces of

thin, clear, colorless sputum each day, mostly in the morning after getting out of bed. What is the nurse’s initial action after gaining this

information? a. Ask the client to provide a morning sputum sample for laboratory

analysis. b. Obtain a specimen of the sputum in a sterile container for culture. c. Monitor for an increase in sputum production or a change in color. d. Notify the health care provider and prepare the client for possible

bronchoscopy.

The nurse observes that a client’s anteroposterior (AP) chest diameter is

the same as his lateral chest diameter. What is the nurse’s most important question for the client in response to this finding? a. No questions are needed regarding this normal finding.

b. “Do you have any chronic breathing problems?” c. “How often do you perform aerobic exercise?”

d. “What is your occupation and what are your hobbies?”

A client with long-standing pulmonary problems is classified as having class III dyspnea. Based on this classification, what type of assistance

does the nurse anticipate providing for ADLs? a. Dyspnea is minimal and the client requires no additional assistance. b. The client may require rest periods during performance of ADLs.

c. The client requires assistance for some but not all tasks. d. Owing to severe dyspnea, this client cannot participate in any self-

care.

A postoperative client has an oxygen saturation of 96% but is pale and dyspneic and says, “I can’t get enough air!” The client’s lung sounds are

clear. Which action by the nurse is most appropriate? a. Call the physician and request a hemoglobin and hematocrit level.

b. Notify respiratory therapy and request a breathing treatment. c. Encourage the client to cough and deep breathe 10 times each hour.

d. Take the client’s temperature and give antipyretics if needed.

A client had a flexible bronchoscopy 2 hours ago and has become mildly cyanotic despite the application of oxygen. When giving change-of-shift

report, which question by the oncoming nurse elicits the most useful information?

a. “How long was the client sedated for the procedure?” b. “Were the oximetry readings during the test normal?”

c. “Are you sure the client was NPO before the bronchoscopy?” d. “What kind of topical anesthetic was used on the client?”

A client is scheduled to undergo a thoracentesis. What is the nurse’s priority intervention?

a. Measure oxygen saturation before and after a 12-minute walk. b. Verify that the client understands all possible complications. c. Explain the procedure in detail to the client and the family. d. Verify that informed consent has been given by the client.

The nurse is caring for a client after a thoracentesis. Which assessment finding by the nurse warrants immediate action?

a. Client rates pain as 5/10 at the site of the procedure.

b. Small amount of drainage is noted from the site. c. Pulse oximetry is 93% on 2 liters of oxygen.

d. Trachea is deviated toward opposite side of the neck.

A client had a bronchoscopy 2 hours ago and is requesting water to drink. Which action by the nurse is most appropriate?

a. Call the physician and request an order for food and water. b. Give the client ice chips instead of a drink of water. c. Assess the client’s gag reflex before giving anything.

d. Let the client have a small sip to see whether he or she can swallow.

A client is scheduled for pulmonary function tests (PFTs) in the morning. The nurse calls the client to teach about the procedure. Which statement

by the client indicates a need for further teaching? a. “I should not smoke for at least 6 hours before the test.”

b. “PFTs can determine whether my lung problem has gotten worse.” c. “I should use my inhaler anytime during the test if I need it.”

d. “If I get really short of breath, I’ll tell the technician.”

Chapter 30: Care of Patients Requiring Oxygen Therapy or Tracheostomy

Test Bank

MULTIPLE CHOICE

The nurse is caring for a client with a new tracheostomy. Which assessment finding requires the nurse’s immediate action?

a. Cuff pressure readings consistently between 14 and 20 mm Hg. b. Need to change Velcro tube holders three times in 1 day.

c. Crackling sensation around the neck when skin is palpated. d. Small amount of bleeding around the incision for the first few days.

A client has a new tracheostomy and is receiving 60% oxygen via tracheostomy collar. Which assessment finding requires immediate

action by the nurse? a. Constant, nonproductive coughing

b. Blood-tinged sputum c. Rhonchi in upper lobes

d. Dry mucous membranes

A client has been placed on 6 L of humidified oxygen via nasal cannula. Which action by the nurse is most appropriate?

a. Drain condensation back into the humidifier, maintaining a closed system.

b. Keep the water sterile by draining it from the water trap back into the humidifier.

c. Turn down the humidity when condensation begins to collect in the tubing.

d. Remove condensation in the tubing by disconnecting and emptying it appropriately.

A client is being discharged with a tracheostomy and voices concern about his appearance. What discharge teaching will assist the client with

maintaining a positive body image? a. “Tell people how sick you were when they ask about the

tracheostomy.” b. “Your clothing can help hide the tracheostomy so it is not as

noticeable.” c. “You can put a bandage around your tracheostomy so no one will

see it.” d. “You have to ignore comments that people make about your

appearance.”

A client is becoming frustrated because of an inability to communicate with a tracheostomy. Which intervention by the nurse most effectively

enhances communication? a. Explain to the client that speech will be clear and distinct with a

fenestrated tube. b. Reassure the client that in time he or she will get used to the speech

difficulties. c. Place a sign above the client’s bed indicating that the client cannot

speak. d. Provide the client with a communication board and call light within

easy reach.

A client is receiving oxygen via Venturi mask at 40%. On assessment the nurse finds the client cyanotic with labored respirations. Which action

does the nurse perform first? a. Remove bedding from around the adaptor opening. b. Listen to lung sounds and obtain a respiratory rate. c. Call respiratory therapy to check oxygen saturation.

d. Notify the provider or Rapid Response Team immediately.

A client requires oxygen received via a face mask but wants to remain as mobile as possible once discharged home. Which intervention by the home health nurse best provides the client with maximal mobility?

a. Arrange a consultation with pulmonary rehabilitation to decrease oxygen needs.

b. Encourage the client to remove the mask occasionally to assess tolerance.

c. Add extra connecting pieces of tubing to the client’s existing oxygen setup.

d. Change the face mask to a nasal cannula occasionally, such as at mealtimes.

A client has been brought in by the rescue squad to the emergency department. The client is having an acute exacerbation of chronic

obstructive pulmonary disease (COPD) and is severely short of breath. On arrival, the client is on 15 L/min of oxygen via rebreather mask.

Which action by the nurse takes priority? a. Immediately reduce the oxygen flow to 2 to 4 L/min via nasal

cannula. b. Perform a thorough respiratory assessment and attach pulse

oximetry. c. Call the laboratory to obtain arterial blood gases as soon as possible. d. Obtain a stat chest x-ray, then slowly wean the client’s oxygen down.

The nurse is caring for a client with orders for oxygen at 5 L/min. Approximately how much FiO2 is the client receiving?

a. 24%

b. 28% c. 36% d. 40%

A client who is receiving continuous oxygen therapy by nasal cannula for an acute respiratory problem is becoming increasingly confused. What

does the nurse do first? a. Notify the health care provider.

b. Assess the client’s pulse oximetry. c. Document the observation. d. Raise the head of the bed.

The nurse assesses a client who is receiving oxygen via a partial rebreather mask. Which assessment finding does the nurse intervene to

correct? a. The bag is two thirds inflated during inhalation.

b. The client’s pulse oximetry reading is 93%. c. The oxygen flow rate is 2 L/min. d. The arterial oxygen level is 90%.

A client is to be discharged home on oxygen therapy. What information does the nurse teach the client?

a. “Carry the H cylinder tank on short trips.” b. “Only use the E tank when stationary.” c. “The D or C cylinder can be carried.”

d. “Roll the tank gently when transporting.”

The nurse assesses a client with a new tracheotomy, and the tracheostomy tube is pulsating in synchrony with the client’s heartbeat.

Which is the nurse’s priority action? a. Notify the health care provider immediately.

b. Stabilize the tube by reapplying the ties. c. Change the inner cannula of the tube.

d. Increase the inflation pressure of the cuff.

A client is 24 hours postoperative after a tracheostomy has been performed. The nurse finds the client cyanotic, with the tracheostomy

tube lying on his chest. Which action by the nurse takes priority? a. Auscultate breath sounds bilaterally.

b. Ventilate with a resuscitation bag and mask. c. Call a code or the Rapid Response Team.

d. Insert a new obturator into the neck.

While suctioning a client who had a tracheostomy placed 4 days ago, the nurse notes particles of food in the tracheal secretions. Which action by

the nurse is most appropriate? a. Increase the inflation pressure in the tracheostomy cuff.

b. Add blue dye to a beverage to assess for aspiration. c. Make the client NPO and notify the health care provider.

d. Perform a more thorough assessment of the client.

The nurse is teaching a client about his fenestrated tracheostomy tube. Which statement by the client indicates an accurate understanding of the

tube? a. “I’m glad I will still be able to talk with this tube in place.”

b. “It is great that this tube does not have to be cleaned regularly.” c. “This tube will not get dislodged because it never needs suctioning.”

d. “Because I can’t swallow, I will need another tube for eating.”

The nurse observes a nursing student suctioning a client. Which

intervention by the student nurse requires the supervising nurse to intervene?

a. Checking oxygen saturation post suctioning b. Hyperoxygenating the client after removal of the catheter

c. Applying intermittent suction during catheter removal d. Applying suction when the catheter is inserted

The nurse assesses a client during suctioning. Which finding indicates that the procedure should be stopped?

a. Heart rate increases from 86 to 102 beats/min. b. Respiratory rate increases from 16 to 20 breaths/min.

c. Blood pressure increases from 110/70 to 120/80 mm Hg. d. Heart rate decreases from 78 to 40 beats/min.

A client is being discharged home with a tracheostomy. Which action does the nurse teach the client to decrease the risk for aspiration while

eating? a. Swallow quickly.

b. Thicken all liquids. c. Rinse all food with water.

d. Chew food completely.

The nursing student is performing tracheostomy care on a client. Which action by the student leads the supervising nurse to intervene?

a. Using folded gauze dressings on both sides of the stoma b. Cutting a slit in a gauze 4 ´ 4 pad to fit around the stoma

c. Applying new tracheostomy ties before removing old ones d. Tying the twill tape in a square knot on the side of the neck

A client receiving high-flow oxygen has new crackles and diminished breath sounds since the last assessment 1 hour ago. Which action by the

nurse is most appropriate? a. Call respiratory therapy and request a bronchodilator treatment.

b. Instruct the client to use the spirometer and to cough and deep breathe.

c. Consult with the health care provider and request an order for diuretics.

d. Ensure that the ordered FiO2 is what is being provided.

Which statement by a client indicates an accurate understanding of home self-care of a tracheostomy?

a. “The stoma should be left uncovered during the day to dry.” b. “I need to put normal saline in my airway twice daily.”

c. “While showering, I need to keep water out of my airway.” d. “I don’t need to use tracheostomy ties on a daily basis.”

A client is being weaned from a tracheostomy tube and has tolerated capping of the tube for 24 hours. Which action by the nurse is most

appropriate? a. Collect all materials needed for suturing the stoma shut. b. Place a dry dressing over the stoma and tape it securely.

c. Assess the client for air leaking around the tube. d. Select a smaller tracheostomy tube to be inserted.

The nurse is preparing to receive a postoperative client who just had a tracheostomy. Which action by the nurse takes priority?

a. Obtain report from the postanesthesia care unit. b. Place a second tracheostomy tube and obturator at the bedside.

c. Review orders for postoperative pain medications. d. Order supplies for tracheostomy care for 24 hours.

A family member has been taught to provide oral care to a client with a tracheostomy. Which statement by the family member indicates an accurate understanding of the correct way to provide mouth care?

a. “I can use glycerin swabs.” b. “I’ll use water and a toothette.” c. “I can use hydrogen peroxide.” d. “It is okay to use mouthwash.”

The nurse is teaching a family member how to suction the client’s tracheostomy at home. Which information does the nurse include in the

teaching plan? a. Always suction using sterile technique.

b. Suction the mouth first and then the airway. c. Be prepared to recannulate the tube frequently. d. Suctioning with clean technique is acceptable.

MULTIPLE RESPONSE

Which interventions help to prevent aspiration during eating for a client with a tracheostomy? (Select all that apply.)

a. Provide close supervision for the client during eating and drinking. b. Add liquids to foods to make them thinner and easier to swallow. c. Inflate the tracheostomy cuff tube to maximum pressure before

starting. d. Let the client indicate readiness for another bite when being fed.

e. Have the client tuck the chin down and forward while swallowing. f. Instruct the client to dry swallow to clear food particles from the

throat. g. Place the client in a semi-Fowler’s position for an hour after eating.

Chapter 31: Care of Patients with Noninfectious Upper Respiratory Problems

Test Bank

MULTIPLE CHOICE

A high school athlete has suffered a nasal fracture. What is the priority action of the nurse caring for the client?

a. Assess for pain. b. Pack the nares to prevent blood loss.

c. Assess for bone displacement. d. Assess for airway patency.

After facial trauma, a client has a nasal fracture and is reporting constant nasal drainage, a headache, and difficulty with vision. What is

the nurse’s first action? a. Collect the nasal drainage on a piece of filter paper.

b. Send the client for a facial x-ray. c. Perform a vision test.

d. Palpate all facial areas for crepitus.

What is the nurse’s most important action after a client’s gag reflex has returned post rhinoplasty?

a. Teach the client to change position every 2 hours. b. Tell the client to put heating pads on the face.

c. Instruct the client to lay flat. d. Have the client drink at least 2500 mL/day.

A client reports waking up feeling very tired, even after 8 hours of good sleep. What is the nurse’s best action?

a. Ask for an order for sleep medication. b. Tell the client not to drink beverages with caffeine.

c. Tell the client not to lie flat at night. d. Ask the client whether he or she has ever been evaluated for sleep

apnea.

A client had a partial laryngectomy and has received instructions on the supraglottic method of swallowing. Which action by the nurse is most

appropriate? a. Place a chart in the client’s room detailing the steps in the process.

b. Order a dynamic swallow study. c. Repeat the instruction each day.

d. Have the client demonstrate swallowing.

A client has open vocal cord paralysis. Which technique does the nurse teach the client to do to prevent aspiration?

a. Tilt the head back as far as possible when swallowing. b. Tuck the chin down when swallowing.

c. Breathe slowly and deeply while swallowing. d. Keep the head very still and straight while swallowing.

Which client does the nurse safely delegate to the LPN/LVN who has been assigned to the unit for the first time?

a. Young adult who is 6 hours post radical neck dissection b. Older adult client with esophageal cancer who is awaiting gastric

tube placement c. Client who is status post laryngectomy and is awaiting discharge

teaching d. Client who is awaiting preoperative teaching for laryngeal cancer

A client has a closed fracture of the nose. Which intervention is best when encouraging self-care for this client?

a. Advise the client not to eat or drink for 24 hours after sustaining the fracture.

b. Teach the client how to apply cold compresses to the area to reduce swelling.

c. Urge the client to sleep without a pillow to hasten resolution of the swelling.

d. Reassure the client that his or her appearance will normalize after the swelling is gone.

Which statement indicates that the client needs more teaching regarding rhinoplasty?

a. “I will take my temperature twice each day and will report any fever to my doctor.”

b. “I will wait a few weeks to have my photograph taken, when the swelling is gone.”

c. “I will take acetaminophen instead of aspirin for pain to avoid excessive bleeding.”

d. “I will drink at least 3 quarts of liquids a day and will use a stool softener.”

What is the highest priority for the nurse to teach the client who is being discharged after a fixed centric occlusion for a mandibular fracture?

a. How to use wire cutters b. Eating six soft or liquid meals each day

c. How to irrigate the mouth every 2 hours d. Sleeping in semi-Fowler’s position postoperatively

Which client is at greatest risk for development of obstructive sleep apnea?

a. Woman who is 8 months pregnant b. Middle-aged man with gastroesophageal reflux disease

c. Middle-aged woman who is 50 pounds overweight d. Older man with type 2 diabetes and a history of sinus infections

Which clinical manifestation in a client with paralysis of one vocal cord alerts the nurse to the possibility of aspiration?

a. Oxygen saturation is decreased. b. Voice is weak and tremulous.

c. The client coughs immediately after swallowing. d. An audible wheeze is present on exhalation.

Which statement made by the client who is prescribed “voice rest” therapy for vocal cord polyps indicates the need for more teaching? a. “I will stay out of rooms and places where people are smoking.”

b. “When I speak at all, I will whisper rather than use a normal tone of voice.”

c. “For the next several weeks, I will not lift more than 10 pounds.” d. “I will drink at least 3 quarts of water each day and will use stool

softeners.”

A client states that he is going to relax on the beach between radiation treatments for laryngeal cancer to help his “mental status.” What is the

nurse’s best response? a. “You deserve to do something for yourself.”

b. “Make sure someone is with you because you shouldn’t be alone right now.”

c. “Your skin can become severely burned, and you should not be out in the sun.”

d. “You should make sure you use sunscreen that is at least SPF 15.”

The nurse is observing a client performing stoma care for a laryngectomy for the first time. Which action does the nurse reinforce?

a. Washing the stoma with soap and water b. Covering the stoma with a gauze pad

c. Irrigating the stoma with half-strength peroxide d. Making sure any scab around the stoma is removed

A client has undergone a nasoseptoplasty 2 hours ago. It is a priority for the nurse to assess for which factor?

a. Nasal drainage b. Bleeding

c. Pain d. Airway patency

A client develops posterior nasal bleeding and has packing inserted. What is the nurse’s priority action?

a. Assess the client’s pain level.

b. Keep the client’s head elevated. c. Teach the client about the causes of nasal bleeding. d. Make sure the string is taped to the client’s cheek.

A client who has sleep apnea is reporting constant daytime sleepiness. The client has multiple other chronic diseases. What is the nurse’s best

action? a. Refer the client for surgery.

b. Perform a health history. c. Request an order for a sleeping pill. d. Move the client to a private room.

MULTIPLE RESPONSE

The nurse is assessing a client who is 6 hours post surgery for a nasal fracture. The client has facial pain (5 out of 10) and nasal packing in place. What actions by the nurse are most appropriate at this time?

(Select all that apply.)

a. Observe for clear drainage. b. Observe for bleeding.

c. Observe the client for frequent swallowing. d. Ask the client to open his or her mouth.

e. Administer a nasal steroid to decrease edema. f. Change the nasal packing.

g. Administer pain medication. h. Place the client in Trendelenburg position.

The client with which conditions requires immediate nursing intervention? (Select all that apply.)

a. Shortness of breath b. Sternal retractions

c. Pulse oximetry reading of 95% d. Occasional expiratory wheeze

e. Respiratory rate of 8 breaths/min f. Arterial blood gas showing a pH of 7.35

g. Stridor

A client develops epistaxis. Which conditions in the client’s history could have contributed to this problem? (Select all that apply.)

a. Diabetes mellitus b. Hypertension

c. Leukemia

d. Cocaine use e. Migraine

f. Elevated platelets g. High cholesterol

The nurse is assessing a client with facial trauma. Which assessment findings require immediate intervention? (Select all that apply.)

a. Stridor b. Nasal stuffiness

c. Edema of the cheek d. Ecchymosis behind the ear

e. Eye pain f. Swollen chin

Chapter 32: Care of Patients with Noninfectious Lower Respiratory Problems

Test Bank

MULTIPLE CHOICE

A client with asthma reports “not being able to take deep breaths.” The nurse auscultates decreased breath sounds in the bases, and no wheezes.

What is the nurse’s best action? a. Encourage the client to stay calm and take deep breaths.

b. Document the findings and continue to monitor. c. Have the client cough forcefully.

d. Assess the client’s oxygen saturation.

A client with asthma has been having frequent asthma attacks. What is the nurse’s best action?

a. Teach the client to stay away from pets. b. Assist the client in using an incentive spirometer.

c. Administer aspirin for its anti-inflammatory properties. d. Administer montelukast (Singulair).

A client diagnosed with asthma has not responded well to medication. The client is concerned and asks the nurse, “What is wrong with me, and

why am I not getting better?” What is the nurse’s best response? a. “You just weren’t used to the medication yet.”

b. “The medication dose has to be increased.” c. “It is possible that genetic testing may help.”

d. “You should try homeopathic medicine.”

The nurse is caring for four clients with asthma. Which client does the nurse assess first?

a. Client with a barrel chest and clubbed fingernails b. Client with an SaO2 level of 92% at rest

c. Client whose expiratory phase is longer than the inspiratory phase d. Client whose heart rate is 120 beats/min

The nurse is caring for an older adult who reports experiencing frequent asthma attacks and severe arthritic pain. What action by the nurse is

most appropriate? a. Review pulmonary function test results. b. Assess use of medication for arthritis.

c. Assess frequency of bronchodilator use. d. Review arterial blood gas results.

The nurse is evaluating a client’s response to medication therapy for asthma. The client has a peak flowmeter reading in the yellow zone.

What does the nurse do next? a. Nothing; this is an acceptable range.

b. Teach the client to take deeper breaths. c. Assist the client to use a rescue inhaler.

d. Assess the client’s lungs.

Which statement indicates that the client understands teaching about the use of long-acting beta2 agonist medications?

a. “I will not have to take this medication every day.” b. “I will take this medication when I have an asthma attack.”

c. “I will take this medication daily to prevent an acute attack.” d. “I will eventually be able to stop using this medication.”

Which statement indicates that a client understands teaching about the correct use of a corticosteroid medication?

a. “This drug can reverse my symptoms during an asthma attack.” b. “This drug is effective in decreasing the frequency of my asthma

attacks.” c. “This drug can be used most effectively as a rescue agent.”

d. “This drug can be used safely on a long-term basis for multiple applications daily.”

A client is using omalizumab (Xolair) for the first time. What is the priority nursing action?

a. Make sure the client takes the medication with water. b. Administer ibuprofen (Motrin) because Xolair often causes

headaches. c. Teach the client how to use a syringe.

d. Remain with the client and assess for anaphylaxis.

A client is demonstrating diaphragmatic breathing for the nurse. Which action by the client shows adequate understanding of this breathing

technique? a. Lying on his or her side with knees bent b. Having his or her hands on the abdomen

c. Having his or her hands over the head d. Lying in the prone position

A client is undergoing lung reduction surgery. What is the nurse’s highest priority preoperatively?

a. Administer medications.

b. Discuss the possibility of ventilator dependency. c. Teach how to cough and deep breathe.

d. Teach about preoperative testing.

The nurse assesses a client receiving chemotherapy for lung cancer and notes red swollen mucous membranes and open sores in the mouth. The client reports mouth pain and difficulty swallowing. Which action does

the nurse perform first? a. Document the size of the sores.

b. Perform mouth hygiene. c. Have the client rinse his or her mouth.

d. Call the health care provider and hold chemotherapy.

A client is undergoing radiation therapy as treatment for lung cancer and has developed esophagitis. Which is the best diet selection for this

client? a. Spaghetti with meat sauce, ice cream

b. Scrambled eggs, bacon, toast

c. Omelet, whole wheat bread d. Pasta salad, custard, orange juice

A client with lung cancer refuses pain medications because he or she is “afraid of addiction.” What is the nurse’s best response?

a. “I can ask the physician to change your medication to a drug that is less potent.”

b. “I can use other measures such as music therapy to distract you.” c. “It is unlikely you will become addicted from taking medicine for

pain.” d. “I can just give you aspirin or acetaminophen (Tylenol) if you like.”

What is the best instruction for a client who has step II (mild persistent) asthma?

a. “Avoid participating in aerobic exercise.” b. “You will need daily inhaled low-dose steroids.”

c. “You need to evaluate your diet for asthma triggers.” d. “Make sure you use a rescue inhaler three times per day.”

The nurse assesses a client with asthma and finds wheezing throughout the lung fields and decreased pulse oxygen saturation. In addition, the nurse notes suprasternal retraction on inhalation. What is the nurse’s

best action? a. Perform peak expiratory flow readings.

b. Assess for a midline trachea. c. Administer oxygen and a rescue inhaler.

d. Call a code.

The nurse is teaching a client with asthma about self-management. Which statement by the nurse is best?

a. “Keep a daily symptom and intervention diary.” b. “Measure your anterior/posterior diameter weekly.”

c. “Note your symptoms when you don’t take your medications.” d. “Exercise before and after taking inhalers and compare tolerance.”

Which statement indicates that a client needs additional teaching about using an inhaler?

a. “I will not exhale into the inhaler.” b. “I will store the inhaler in a drawer in my bedroom.”

c. “I will soak my inhaler in water to clean it.” d. “I will inhale and hold my breath.”

The home care nurse observes white patches on the oral mucosa of a client with severe, chronic airflow limitation. What is the nurse’s best

action? a. Ask the client whether he or she uses a steroid inhaler.

b. Inquire about any recent viral illnesses. c. Have the client rinse the mouth with salt water.

d. Have the client brush the patches with a soft-bristled brush.

What statement indicates that a client needs further teaching regarding therapy with salmeterol (Serevent)?

a. “I will be certain to shake the inhaler well before I use it.” b. “It may take a while before I notice a change in my asthma.”

c. “I will use the drug when I have an asthma attack.” d. “I will be careful not to let the drug escape out of my nose and

mouth.”

The nurse is teaching a client about different medications for asthma. Which medication does the nurse teach the client to administer to control

the prolonged inflammatory response? a. Diphenhydramine (Benadryl)

b. Montelukast (Singulair) c. Aspirin

d. Bitolterol (Tornalate)

A client with chronic obstructive pulmonary disease (COPD) reports social isolation. What does the nurse encourage the client to do?

a. Join a support group for people with COPD. b. Ask the client’s physician for an antianxiety agent.

c. Verbalize his or her thoughts and feelings. d. Participate in community activities.

The nurse is teaching a client with cystic fibrosis. What activity does the nurse teach as the priority? a. Taking daily antibiotics

b. Having genetic screening c. Maintaining good nutrition

d. Exercising daily

The nurse is assessing a client with lung disease. Which symptom does the nurse intervene for first?

a. The client’s anterior-posterior chest diameter is 2:2. b. Clubbing of the finger tips is noted.

c. The client has bilateral dependent leg edema. d. The client is pale.

A client with lung cancer is lying flat in bed and reports shortness of breath. What action does the nurse take first?

a. Notify the health care provider. b. Elevate the head of the bed. c. Assess oxygen saturation.

d. Have the client take deep breaths.

The nurse observes hematuria in a client receiving IV cyclophosphamide (Cytoxan). After notifying the health care provider, what intervention is

the nurse’s priority? a. Obtain a urine specimen. b. Assess laboratory studies.

c. Increase hydration. d. Stop the medication.

A client with pulmonary fibrosis is being discharged home. What is the highest priority teaching need?

a. Dietary modifications b. Determining activity tolerance

c. Avoiding infection d. Medication therapy

The nurse is caring for a client with bronchiolitis obliterans organizing pneumonia (BOOP) and assesses decreased vital capacity during

pulmonary function testing. What is the nurse’s best action? a. Administer intermittent positive-pressure breathing treatments.

b. Administer a short-acting beta-adrenergic medication. c. Prepare to administer IV antibiotics.

d. Document the finding in the client’s chart.

The nurse is teaching a client with bronchiolitis obliterans organizing

pneumonia (BOOP) about corticosteroid therapy. What statement is accurate for the nurse to teach the client?

a. “You will be on this drug the rest of your life.” b. “You will be prone to many long-term side effects of this drug.”

c. “A short course of therapy will help with acute episodes.” d. “This medication cannot be taken with antibiotic therapy.”

A client recently diagnosed with lung cancer is being taught by the nurse. What information does the nurse teach the client?

a. “You will receive 6 weeks of daily radiation therapy.” b. “Lung cancer has a very good prognosis.”

c. “Further testing is not needed because lung cancer rarely metastasizes.”

d. “It is very likely that surgery will be curative.”

Which nursing intervention is an example of primary prevention for lung cancer?

a. Teaching clients with lung cancer how to cough and deep breathe b. Teaching clients with lung cancer to avoid infection

c. Teaching clients about prophylactic antibiotics d. Teaching people about smoking and secondhand smoke

A client’s chest tube is accidentally dislodged. What action by the nurse is best?

a. No action is necessary because the area will reseal itself. b. Cover the insertion site with a sterile gauze and tape three sides.

c. Obtain a suture kit and prepare for the physician to suture the site. d. Cover the area with an occlusive dressing.

Which is the highest priority problem for a client with late-stage lung cancer?

a. Malnutrition b. Constipation

c. Weakness and fatigue d. Pain

The nurse assesses a client’s chest tube and finds continuous bubbling in the water seal chamber. When the nurse clamps the chest tube close to the client’s dressing, the bubbling stops. How does the nurse interpret

this finding? a. An air leak is present at the chest tube insertion site or in the

thoracic cavity. b. An air leak is present in the drainage system.

c. More water needs to be added to the water seal. d. The system is functioning appropriately and no intervention is

needed.

A client was diagnosed with lung cancer and appears distressed. The client states, “I am so afraid.” What is the best action for the nurse to

take? a. Provide comfort by holding the client’s hand.

b. Offer to give the client a back rub for relaxation. c. Offer the client a PRN antianxiety medication.

d. Ask the client what is causing the most fear right now.

The nurse is assessing a client who has a chest tube. Which assessment finding requires intervention by the nurse?

a. Pain at the insertion site b. Bloody drainage in the collection chamber

c. Intermittent bubbling in the water seal chamber d. Tidaling in the water seal chamber

The nurse assesses a client who is on fluticasone (Flovent) and notes oral lesions. What is the nurse’s best action?

a. Teach the client to rinse the mouth after Flovent use. b. Have the client use a mouthwash daily.

c. Start the client on a broad-spectrum antibiotic. d. Document the finding as a known side effect.

A client has recently been placed on prednisone (Deltasone). What is the highest priority instruction the nurse will provide?

a. “Expect to experience weight gain.” b. “Watch your diet while on this medication.”

c. “Take the drug with food or milk.” d. “Report any abdominal pain or dark-colored vomit.”

A client infected with Burkholderia cepacia is admitted to the unit. What is the nurse’s priority action when caring for this client?

a. Instruct the client to wash his or her hands after contact with other people.

b. Place the client on strict isolation. c. Keep the client isolated from other clients with cystic fibrosis.

d. Administer IV vancomycin daily.

The nurse assesses the following lung sounds in a client. What is the nurse’s best action? (Click the media button to hear the audio clip.)

a. Administer a rescue inhaler. b. Administer oxygen. c. Assess vital signs.

d. Elevate the client’s head.

The nurse assesses an older adult after an upper respiratory infection and notes the following lung sound on auscultation. What is the nurse’s

best action? (Click the media button to hear the audio clip.) a. Assess the client for the development of asthma.

b. Ask the client if he or she finished all the medication. c. Administer oxygen immediately.

d. Assess arterial blood gas.

MULTIPLE RESPONSE

A client has a mediastinal chest tube. Which symptoms require the nurse’s immediate intervention? (Select all that apply.)

a. Production of pink sputum b. Tracheal deviation

c. Oxygen saturation greater than 95% d. Sudden onset of shortness of breath

e. Drainage greater than 70 mL/hr f. Pain at insertion site

g. Disconnection at Y site

What information about nutrition does the nurse teach a client with chronic obstructive pulmonary disease (COPD)? (Select all that apply.)

a. “Avoid drinking fluids just before and during meals.” b. “Rest before meals if you have dyspnea.”

c. “Have about six small meals a day.” d. “Practice diaphragmatic breathing against resistance four times

daily.” e. “Eat high-fiber foods to promote gastric emptying.”

f. “Eat dry foods rather than wet foods, which are heavier.” g. “Increase carbohydrate intake for energy.”

The nurse is assessing a client with chronic obstructive pulmonary disease (COPD) to determine activity tolerance. Which questions elicit

the most important information? (Select all that apply.) a. “What color is your sputum?”

b. “Do you have any difficulty sleeping?” c. “How long does it take to perform your morning routine?”

d. “Do you walk upstairs every day?” e. “Have you lost any weight lately?”

A client has a chest tube. What assessment findings require immediate intervention from the nurse? (Select all that apply.)

a. Intermittent bubbling in the water seal chamber in the client with a pneumothorax

b. “Silent chest” in the client with a pneumothorax c. Tidaling in the water seal chamber in a client with a pneumothorax

d. Bloody drainage in the tubing of a client with a hemothorax e. Tracheal deviation in a client after chest trauma

f. No drainage in the chest tube of a client with a pneumothorax g. Constant bubbling in the water seal chamber in a client post chest

surgery

Which symptoms in chronic lung disease require nursing intervention? (Select all that apply.)

a. Clubbed fingers b. Increased residual volume

c. Decreased peak flow d. Increased anterior-posterior diameter

e. Elevated platelets f. Expiratory wheezing

g. Stridor h. Change in sputum color and amount

The nurse is teaching a client with asthma how to avoid attacks. What information does the nurse give the client? (Select all that apply.)

a. “You should not dust your furniture.” b. “Stay inside as much as possible.”

c. “Stay away from people who are sick.” d. “Do not go out in the fall.”

e. “Stay out of the snow.” f. “Do not take aspirin.”

The nurse is assessing a client with asthma. Scattered wheezes are noted, and the client’s oxygen saturation is 88%. What other assessments are

essential for the nurse to perform? (Select all that apply.) a. Assess for accessory muscle use.

b. Assess anterior-posterior diameter. c. Assess inspiration/expiration ratios.

d. Assess the suprasternal notch. e. Perform a stress test. f. Assess a chest x-ray.

g. Assess mucous membranes.

OTHER

The nurse is teaching a client to cough productively. Put the actions in proper sequence.

Have the client flex the head and hold a pillow to the stomach. Assist the client to a sitting position with feet on the floor.

Instruct the client to bend forward and to cough two or three times. Have the client return to an upright position and take a deep breath.

Encourage the client to take several deep breaths.

Place the steps for obtaining a peak expiratory flow rate in the order in which they should occur.

Take as deep a breath as possible. Stand up (unless you have a physical disability).

Place the meter in your mouth, and close your lips around the mouthpiece.

Make sure the device reads zero or is at base level.

Blow out as hard and as fast as possible for 1 to 2 seconds. Write down the value obtained.

Repeat the process two additional times, and record the highest number in your chart.

Chapter 33: Care of Patients with Infectious Respiratory Problems

Test Bank

MULTIPLE CHOICE

A client has acute rhinitis. What is the most important intervention for the nurse to perform?

a. Assess for symptoms of infection. b. Ascertain whether the client has allergies.

c. Question the client on the use of nasal sprays. d. Do blood and urine screenings for drug use.

A client has pharyngitis. Which symptom helps the nurse determine whether the infection is bacterial versus viral?

a. Redness in the back of the throat b. Enlarged lymph glands in the neck

c. Nasal discharge d. Skin rash

It is suspected that a client has bacterial pharyngitis. What is the best intervention?

a. Administer a broad-spectrum antibiotic. b. Have the client produce a sputum specimen. c. Obtain samples for culture and sensitivity.

d. Assess a rapid antigen test (RAT).

The nurse is caring for a client with recurrent bacterial pharyngitis. Which is the nurse’s highest priority intervention?

a. Assess for symptoms of human immune deficiency virus (HIV). b. Ask about exposure to allergens.

c. Perform nasal cultures. d. Teach the client about antibiotic therapy.

A client who has had acute tonsillitis develops drooling and reports severe throat pain. What is the nurse’s priority intervention?

a. Assess the throat for deviation of the uvula. b. Prepare the client for surgery.

c. Teach the client about antibiotic therapy. d. Prepare the client for percutaneous needle aspiration.

The nurse has determined that a client has an acute sore throat. What is the nurse’s best action?

a. Assess whether the client can speak. b. Call an ear-nose-throat specialist.

c. Administer an antibiotic. d. Give the client ice chips.

A client who is immune compromised develops muscle aches and fever. The client is admitted to the hospital for several days and is diagnosed

with influenza. At discharge, the client asks when he can go back to work. What is the nurse’s best response?

a. “You should be able to return to work in 5 days.” b. “You can return to work as soon as you feel ready.”

c. “You cannot return to work for several weeks.” d. “You will need to have cultures performed before returning to

work.”

A client is worried about contracting influenza. What is the nurse’s best response to the client?

a. “Flu is no longer a prevalent problem.” b. “Did you receive a flu vaccine this year?” c. “Current flu strains are generally mild.”

d. “If you develop symptoms, antibiotics will cure you.”

The nurse assesses a client with pneumonia and notes decreased lung sounds on the left side and decreased lung expansion. What is the nurse’s

best action? a. Have the client cough and deep breathe.

b. Check oxygen saturation and notify the health care provider. c. Perform an arterial blood gas analysis.

d. Increase oxygen flow to 10 L/min.

An older adult is admitted to the emergency department with respiratory symptoms. Which client symptom requires the nurse to intervene

immediately? a. Confusion

b. Scattered wheezing c. Crackles

d. Flushed cheeks

Which is the highest priority goal to set for a client with pneumonia? a. Absence of cyanosis

b. Maintenance of SaO2 of 95% c. Walking 20 feet three times daily

d. Absence of confusion

The nurse is teaching a client with pneumonia ways to clear secretions. Which intervention is the most effective?

a. Administering an antitussive medication b. Administering an antiemetic medication c. Increasing fluids to 2 L/day if tolerated

d. Having the client cough and deep breathe hourly

A client who works in a day care facility is admitted to the emergency department. The client is diagnosed with pneumonia, and a sputum

culture is taken. Infection with Streptococcus pneumoniae is confirmed. What is the nurse’s primary action?

a. Have emergency intubation equipment nearby. b. Teach the client about the treatment.

c. Isolate the client. d. Perform chest physiotherapy.

What is the priority nursing intervention when caring for a client with severe acute respiratory syndrome (SARS)?

a. Maintaining Standard Precautions b. Administering antibiotics

c. Assessing oxygenation d. Making sure the client stays hydrated

The newly employed nurse received a bacillus Calmette-Guérin (BCG) vaccine before moving to the United States. The nurse needs to receive a tuberculin (TB) test as part of the pre-employment physical. What does

the nurse do? a. The nurse should not receive the tuberculin test.

b. The nurse will need a two-step TB test. c. The nurse will need a chest x-ray instead.

d. A physician should examine the nurse before the TB test is given.

The nurse is caring for several clients on a respiratory floor. The nurse should place the client with which condition in isolation?

a. Fever and weight loss b. Negative QuantiFERON TB gold test

c. Negative acid-fast bacillus (AFB) stain d. Positive nucleic acid amplification test (NAAT)

A client has multidrug-resistant tuberculosis (TB). What is the most important fact for the nurse to teach the client?

a. “You will need to take medications longer than clients with other strains.”

b. “You will need to remain in the hospital until cultures are negative.” c. “You will need to wear a mask when you go out in public.”

d. “You will need to have drug cultures done weekly.”

The nurse is worried that a client who is not entirely reliable is being discharged home on therapy for multidrug-resistant tuberculosis. What

strategy is the best to use for this client? a. Directly observed therapy

b. IV drug administration c. Remaining in the hospital

d. Isolation

A client is admitted with suspected avian influenza. The family asks the nurse what kind of care the client will get. Which statement by the nurse

is correct? a. “He will be given standard antibiotic agents and will be placed in

contact isolation.” b. “He will be placed on airborne and contact isolation.”

c. “Oseltamivir (Tamiflu) will reduce complications of this infection.” d. “All family members should be tested for evidence of the same

disease.”

Which client does the nurse caution to avoid taking over-the-counter decongestants for manifestations of a cold or flu?

a. Young man with a latex allergy b. Middle-aged woman with hypertension

c. Teenage woman who is taking oral contraceptives d. Older man who has had type 1 diabetes mellitus for 20 years

An older client reports having a cold and a “full bladder.” What does the nurse obtain for or from the client?

a. Order for a Foley catheter b. Order for a one-time catheterization

c. Urine specimen d. History focusing on current medications

A client has a peritonsillar abscess. Which priority instruction does the nurse provide to this client?

a. “If you notice an enlarged node on the side of your neck where the abscess is, call your health care provider.”

b. “Stay home from work or school until your temperature has been

normal for 24 hours.” c. “You may gargle with warm water that has a teaspoon of salt in it as

often as you like.” d. “Take the antibiotic for the entire time it is prescribed, not just until

you feel better.”

An older adult client with heart failure asks if she should get a flu shot. Which is the nurse’s best response?

a. “Yes, because of your heart failure you are at greater risk for complications.”

b. “Yes, if it has been longer than 5 years since your last flu vaccination.”

c. “No, your heart failure makes you too weak to get the live virus vaccine.”

d. “No, the vaccine will interact with your heart medications.”

Which person is at greatest risk for developing a community-acquired pneumonia?

a. Middle-aged teacher who typically eats a diet of Asian foods b. Older adult who smokes and has a substance abuse problem

c. Older adult with exercise-induced wheezing d. Young adult aerobics instructor who is a vegetarian

Which is the nurse’s best response to an older adult client who is hesitant to take the pneumococcal vaccination and influenza vaccine in the same

year? a. “You need both injections. A risk factor for getting pneumonia is

infection with influenza.” b. “Take both injections. They will protect you against respiratory

problems for this year.” c. “The flu shot may protect you against influenza but not against

bacteria that cause pneumonia.” d. “You should get the pneumococcal vaccination so you won’t infect

other people.”

Which is a priority teaching intervention for the client who is using a nicotine patch?

a. “Abruptly discontinuing this patch can cause high blood pressure.” b. “Abruptly discontinuing this patch can cause nausea and vomiting.” c. “Smoking while using this patch increases the risk for pneumonia.”

d. “Smoking while using this patch increases the risk for a heart attack.”

A client is admitted with left lower lung pneumonia. Which assessment finding does the nurse correlate with this condition?

a. Expiratory wheeze on the right side b. Dullness to percussion on the lower left side

c. Crepitus of the skin around the left lung d. Crackles heard on expiration bilaterally

The nurse auscultates the following lung sound in the client with pneumonia. What is the best intervention? (Click the media button to

hear the audio clip.) a. Have the client cough and deep breathe. b. Prepare to administer a bronchodilator.

c. Have the client use an incentive spirometer. d. Administer IV fluids.

A client has a tuberculin skin test as a pre-employment physical requirement. Which statement by the nurse is best made to the client

who has the test result seen in the photograph below?

a. “Your PPD is negative. No further follow-up is necessary.” b. “You will need to have a second PPD.”

c. “You will need to have titers drawn.” d. “You will need further testing.”

MULTIPLE RESPONSE

What teaching is appropriate for a client with acute rhinitis and sinusitis? (Select all that apply.)

a. Using hot packs over the sinuses b. Fluid restriction c. Saline irrigations

d. Staying in a dry climate e. Taking echinacea

f. Antifungal medications

A client enters the clinic with an acute sore throat and a temperature of 101.5° F (38.5° C). What diagnostic testing does the nurse educate the

client about? (Select all that apply.) a. Complete blood count (CBC)

b. Throat culture c. Monospot test

d. Arterial blood gas e. Biopsy

f. HIV testing

What is the best way for the nurse to decrease the risk of ventilator- associated pneumonia (VAP) in a ventilator-dependent client? (Select all

that apply.) a. Provide prophylactic antibiotics.

b. Provide frequent oral care. c. Keep the head of the bed elevated.

d. Maintain good hand hygiene. e. Perform chest percussion frequently.

A client who previously had a bacillus Calmette-Guérin (BCG) vaccine has a positive tuberculosis (TB) test. What symptoms assist in

determining that the client has active disease? (Select all that apply.) a. Nausea

b. Weight loss c. Insomnia

d. Ankle edema e. Night sweats

f. Increased urination

A client started on therapy for tuberculosis infection is reporting nausea. What does the nurse teach this client? (Select all that apply.)

a. Eat a diet rich in protein, iron, and vitamins. b. Do not drink fluids with medications.

c. Take medications at bedtime. d. Space medications 12 hours apart.

e. Take medications with milk. f. Take an antiemetic daily.

The nurse is caring for a client who is suspected of having severe acute respiratory syndrome (SARS). What actions by the nurse are most

appropriate? (Select all that apply.) a. Wash hands when entering the client’s room and use Standard

Precautions. b. Wear a gown and goggles when entering the client’s room.

c. Teach the client to wear a mask at all times when someone is in the room.

d. Use a disposable particulate mask respirator when the client is coughing.

e. Keep the door to the client’s room open to allow close monitoring. f. Place the client in a negative airflow room, if available in the facility.

The nurse is caring for a client who has inhalation anthrax. What nursing actions are of the highest priority? (Select all that apply.)

a. Placing the client in an isolation room b. Teaching the client how to use a mask

c. Teaching the client about long-term antibiotic therapy d. Using handwashing and other Standard Precautions e. Reporting suspected cases to the proper authorities

Chapter 34: Care of Critically Ill Patients with Respiratory Problems

Test Bank

MULTIPLE CHOICE

The nurse is caring for a client receiving heparin and warfarin therapy for a pulmonary embolus. The client’s international normalized ratio

(INR) is 2.0. What is the nurse’s best action? a. Increase the heparin dose. b. Increase the warfarin dose.

c. Continue the current therapy. d. Discontinue the heparin.

The nurse is caring for a postoperative client who suddenly reports difficulty breathing and sharp chest pain. After notifying the Rapid

Response Team, what is the nurse’s priority action? a. Elevate the head of the bed and apply oxygen.

b. Listen to the client’s lung sounds. c. Pull the call bell out of the wall socket.

d. Assess the client’s pulse oximetry.

It is determined that a client has a large pulmonary embolism (PE). Fibrinolytic therapy is initiated. What is the nurse’s priority action?

a. Monitor the client’s oxygenation. b. Teach the client about potential side effects.

c. Monitor the IV insertion site. d. Monitor for bleeding.

A client with a large pulmonary embolism is receiving alteplase (Activase). The nurse notes frank red blood in the Foley catheter

drainage bag. What is the nurse’s first action?

a. Irrigate the Foley. b. Administer an antibiotic.

c. Clamp the Foley. d. Notify the health care provider.

The nurse is caring for a client with a pulmonary embolus who also has right-sided heart failure. Which symptom will the nurse need to

intervene for immediately? a. Respiratory rate of 28 breaths/min

b. Urinary output of 10 mL/hr c. Heart rate of 100 beats/min

d. Dry cough

A client states, “At night, I usually need to sleep propped up on two pillows in the chair, but now it seems I need three pillows.” What is the

nurse’s best response? a. “You should try to rest more during the day.”

b. “You should try to lie flat for short periods of time.”

c. “You need to stay in the hospital for further evaluation.” d. “You can take medication at night so you can sleep.”

A client is admitted owing to difficulty breathing. The nurse assesses the client’s color, lung sounds, and pulse oximetry reading. The pulse

oximetry is 90%. What is the nurse’s next action? a. Give an intermittent positive-pressure breathing treatment.

b. Administer a rescue inhaler. c. Call for a chest x-ray.

d. Assess an arterial blood gas.

A client with dyspnea is becoming very anxious. An arterial blood gas (ABG) shows a PaO2 of 93 mm Hg. How does the nurse best intervene?

a. Increase the oxygen. b. Administer an antianxiety medication.

c. Administer a bronchodilator. d. Assist with relaxation techniques.

The nurse notes that each time the mechanical ventilator delivers a

breath to a client with acute respiratory distress syndrome (ARDS), the peak inspiratory pressure alarm sounds. What is the nurse’s best

intervention? a. Suction the client.

b. Perform chest physiotherapy. c. Administer an inhaler.

d. Assess the airway.

The nurse is caring for several clients on the respiratory floor. Which client does the nurse assess most carefully for the development of acute

respiratory distress syndrome (ARDS)? a. Older adult with COPD

b. Middle-aged client receiving a blood transfusion c. Older adult who has aspirated his tube feeding

d. Young adult with a broken leg from a motorcycle accident

The nurse is caring for a client with acute respiratory distress syndrome (ARDS) who is receiving mechanical ventilation and positive end-

expiratory pressure (PEEP). The alarm sounds, indicating decreased pressure in the system. What is the nurse’s best action?

a. Change the client’s position. b. Suction the client.

c. Assess lung sounds. d. Turn off the pressure alarm.

The nurse is caring for a client who has been intubated and placed on a ventilator for treatment of acute respiratory distress syndrome (ARDS). Aside from assessing oxygenation, what is the nurse’s priority action?

a. Assess hemoglobin. b. Administer ferrous sulfate.

c. Assess muscle strength. d. Consult with the registered dietitian.

The nurse is caring for a client who is intubated with an endotracheal tube and on a mechanical ventilator. The client is able to make sounds.

What is the nurse’s first action? a. Check cuff inflation on the endotracheal tube.

b. Listen carefully to the client. c. Call the health care provider.

d. Auscultate the lungs.

Which assessment finding of a client requires the nurse’s immediate action?

a. Being intubated for 4 days b. Uneven breath sounds

c. Wheezing on auscultation d. Having the endotracheal (ET) tube taped to the lower jaw

The pilot balloon on the endotracheal tube of a client being mechanically ventilated is deflated. What is the nurse’s priority action?

a. Nothing; this is required during ventilation. b. Inflate the cuff using minimal leak technique.

c. Call the Rapid Response Team. d. Increase the tidal volume.

The nurse is caring for a client with a ventilation/perfusion mismatch who is receiving mechanical ventilation. Which intervention is a priority

for this client? a. Administering antibiotics every 6 hours

b. Positioning the client with the “good lung dependent” c. Making sure that the pilot balloon line on the endotracheal tube is

deflated d. Ensuring that the client is able to speak clearly

The nurse is caring for a client who is receiving mechanical ventilation accompanied by positive end-expiratory pressure (PEEP). What

assessment findings require immediate intervention? a. Blood pressure drop from 110/90 mm Hg to 80/50 mm/Hg

b. Pulse oximetry value of 96% c. Arterial blood gas (ABG): pH, 7.40; PaO2, 80 mm Hg; PaCO2, 45

mm Hg; HCO3–, 26 mEq/L d. Urinary output of 30 mL/hr

The client receiving mechanical ventilation has become more restless over the course of the shift. Which is the nurse’s first action?

a. Sedate the client. b. Call the health care provider.

c. Assess the client for pain. d. Assess the client’s oxygenation.

The pressure reading during inspiration on the ventilator of a client receiving mechanical ventilation is fluctuating widely. What is the

nurse’s first action? a. Determine whether an air leak is present in the client’s endotracheal

tube cuff. b. Have the respiratory therapist check the pressure settings.

c. Assess the client’s oxygenation. d. Manually ventilate the client with a resuscitation bag.

A client is admitted to the emergency department several hours after a motor vehicle crash. The car’s driver-side airbag was activated during

the accident. Which assessment requires the nurse’s immediate intervention?

a. Disorientation b. Hemoptysis

c. Pulse oximetry reading of 94% d. Chest pain with movement

The nurse assesses a client admitted for chest trauma who reports dyspnea. The nurse finds tracheal deviation and a pulse oximetry

reading of 86%. What is the nurse’s priority intervention? a. Notify the health care provider and document the symptoms. b. Intubate the client and prepare for mechanical ventilation.

c. Administer oxygen and prepare for chest tube insertion. d. Administer an intermittent positive-pressure breathing treatment.

The nurse assesses a client who has a hemothorax and a chest tube inserted on the right side. What finding requires immediate attention?

a. Pain at the chest tube insertion site b. Fluctuation in the water seal chamber with breathing

c. Puffiness of the skin around the chest tube insertion site and a crackling feeling

d. Dullness to percussion on the affected side

The nurse is caring for a client who is taken off a ventilator and placed on continuous positive airway pressure (CPAP). What intervention is

most appropriate for this client? a. Administering antianxiety medications PRN

b. Administering a medication to help the client sleep c. Telling the client to relax and let the ventilator do the work

d. Making sure the client is breathing spontaneously

The nurse assesses a client who suffered chest trauma and finds that the left chest sucks in during inhalation and out during exhalation. The

client’s oxygen saturation has dropped from 94% to 86%. What is the priority action by the nurse?

a. Encourage the client to take deep, controlled breaths.

b. Document findings and continue to monitor the client. c. Notify the health care provider and prepare for intubation.

d. Stabilize the chest wall with rib binders.

The nurse auscultates the lungs of a client on mechanical ventilation and hears vesicular breath sounds throughout the right side but decreased

sounds on the left side of the chest. What is the nurse’s best action? a. Turn the client to the right side.

b. Elevate the head of the bed. c. Assess placement of the endotracheal (ET) tube.

d. Suction the client.

What is the best way for the nurse to communicate with a client who is intubated and is receiving mechanical ventilation? a. Ask the client to point to words on a board. b. Ask the client to blink for “yes” and “no.”

c. Have the client mouth words slowly. d. Teach the client some simple sign language.

A client admitted with respiratory difficulty and decreased oxygen saturation keeps pulling off the oxygen mask. What action does the nurse

take? a. Stays with the client and replaces the oxygen mask

b. Asks the client’s spouse to hold the oxygen mask in place c. Restrains the client per facility policy

d. Contacts the health care provider and requests sedation

A client with severe respiratory insufficiency becomes short of breath during activities of daily living. Which nursing intervention is best?

a. Call the Rapid Response Team. b. Decrease involvement in care until the episode is past. c. Cluster morning activities to provide long rest periods. d. Space out interventions to provide for periods of rest.

The nurse is assessing arterial blood gases (ABGs). The client with which ABG reading requires the nurse’s immediate attention?

a. pH, 7.32; PaCO2, 55 mm Hg; PaO2, 70 mm Hg b. pH, 7.45; PaCO2, 42 mm Hg; PaO2, 70 mm Hg c. pH, 7.48; PaCO2, 38 mm Hg; PaO2, 60 mm Hg d. pH, 7.55; PaCO2, 32 mm Hg; PaO2, 50 mm Hg

The nurse auscultates the following lung sound in a client with a respiratory disorder. What is the nurse’s best action? (Click the media

button to hear the audio clip.) a. Have the client use an incentive spirometer.

b. Have the client cough and deep breathe. c. Suction the client after auscultating the lower lobes of the lungs.

d. Call for the Rapid Response Team.

MULTIPLE RESPONSE

Which symptoms in a client assist the nurse in confirming the diagnosis of pulmonary embolus (PE)? (Select all that apply.)

a. Wheezes throughout lung fields b. Hemoptysis

c. Sharp chest pain d. Flattened neck veins

e. Hypotension f. Pitting edema

Which clients are at highest risk for pulmonary embolism (PE)? (Select all that apply)

a. Middle-aged client awaiting surgery b. Older adult with a 20–pack-year history of smoking

c. Client who has been on bedrest for 3 weeks d. Obese client who has elevated platelets

e. Middle-aged client with diabetes mellitus type 1 f. Older adult who has just had abdominal surgery

A client admitted for difficulty breathing becomes worse. Which assessment findings indicate that the client has developed acute respiratory distress syndrome (ARDS)? (Select all that apply.)

a. Oxygen administered at 100%, PaO2 60 b. Increased dyspnea

c. Anxiety d. Chest pain

e. Pitting pedal edema f. Clubbing of fingertips

The nurse is caring for a client on a ventilator when the high-pressure alarm sounds. What actions are most appropriate? (Select all that apply.)

a. Assess the tubing for kinks.

b. Assess whether the tubing has become disconnected. c. Determine the need for suctioning.

d. Call the health care provider. e. Call the Rapid Response Team.

f. Auscultate the client’s lungs.

The nurse is prioritizing care for a client on a ventilator. What are essential nursing interventions for this client? (Select all that apply.)

a. Change the settings in accordance with provider orders. b. Modify the settings for weaning the client.

c. Assess the reasons for alarms. d. Compare the ventilator settings with ordered settings.

e. Assess the water level in the humidifier. f. Change the ventilator tubing according to hospital policy.

The nurse is caring for a client with a high risk for pulmonary embolism (PE). Which prevention measures does the nurse add to the client’s care

plan? (Select all that apply.) a. Use antiembolism stockings.

b. Massage calf muscles per client request. c. Maintain supine position with the legs flat.

d. Turn every 2 hours if client is in bed. e. Refrain from active range-of-motion exercises.

SHORT ANSWER

A nurse is making initial rounds on assigned clients at the beginning of the shift. One client is receiving a heparin infusion at 5 mL/hr. The nurse notes that 25,000 units of heparin are mixed in 250 mL of solution. How

many units per hour is the client receiving? units/hr

A client is ordered heparin 5000 units at 7 AM. The heparin is provided in a vial labeled 20,000 units per mL. How much does the nurse

administer? mL

Chapter 35: Assessment of the Cardiovascular System

Test Bank

MULTIPLE CHOICE

The nurse is caring for a client who has had a recent myocardial infarction involving the left ventricle. Which assessment finding is

expected? a. Faint S1 and S2 sounds

b. Decreased cardiac output c. Increased blood pressure d. Absent peripheral pulses

The nurse is caring for a client with coronary artery disease. What assessment finding does the nurse expect if the client’s mean arterial

blood pressure decreases below 60 mm Hg? a. Increased cardiac output

b. Hypertension c. Chest pain

d. Decreased heart rate

The nurse is assessing a client following a myocardial infarction. The client is hypotensive. What additional assessment finding does the nurse

expect? a. Heart rate of 120 beats/min

b. Cool, clammy skin c. Oxygen saturation of 90%

d. Respiratory rate of 8 breaths/min

The nurse administers a beta blocker to a client after a myocardial infarction. What assessment finding does the nurse expect?

a. Blood pressure increase of 10% b. Increasing respiratory rate c. Increased cardiac output

d. Pulse decrease from 100 to 80 beats/min

The nurse is assessing clients at a community health center. Which client does the nurse determine is at high risk for cardiovascular disease?

a. Older adult man with a history of asthma b. Asian-American man with colorectal cancer

c. American Indian woman with diabetes mellitus d. Postmenopausal woman on hormone therapy

The nurse is obtaining a client’s health history. Which illness alerts the nurse to the possibility of abnormal heart valves?

a. Tuberculosis b. Recurrent viral pneumonia

c. Rheumatic fever

d. Asthma

A nurse is performing an admission assessment on an older adult client with multiple chronic diseases. The nurse assesses the heart rate to be 48

beats/min. What does the nurse do first? a. Document the finding in the chart.

b. Evaluate for a pulse deficit. c. Assess the client’s medications. d. Administer 1 mg of atropine.

The nurse is assessing clients at a clinic. Which activity takes priority? a. Teaching smoking cessation to a middle-aged woman who smokes

b. Planning an exercise regimen with a woman with a sedentary lifestyle

c. Teaching an older man who is moderately obese to keep a food diary d. Assessing a man with familial coronary artery disease for specific

risk factors

The nurse is assessing a client in the emergency department. Which

client statement alerts the nurse to the occurrence of heart failure? a. “I get short of breath when I climb stairs.”

b. “I see halos floating around my head.” c. “I have trouble remembering things.”

d. “I have lost weight over the past month.”

The nurse is assessing a client newly admitted to the medical unit. Which statement made by the client alerts the nurse to the presence of edema?

a. “I wake up to go to the bathroom at night.” b. “My shoes fit tighter by the end of the day.” c. “I seem to be feeling more anxious lately.”

d. “I drink at least eight glasses of water a day.”

When obtaining a client’s vital signs, the nurse assesses a blood pressure of 134/88 mm Hg. What is the nurse’s best intervention? a. Call the health care provider and report the finding.

b. Reassess the client’s blood pressure at the next yearly physical. c. Administer an additional antihypertensive medication to the client. d. Teach the client lifestyle modifications to decrease blood pressure.

The nurse is performing a focused cardiac assessment. What assessment finding should be reported to the health care provider?

a. Bruit heard on the side of the neck b. Bounding peripheral pulses c. Pulse rate of 90 beats/min

d. Blood pressure of 140/90 mm Hg

A client consistently reports feeling dizzy and lightheaded when moving

from a supine position to a sitting position. What nursing assessment takes priority at this time?

a. Pulse oximetry b. Blood pressure

c. Respiratory rate d. Neurologic evaluation

The nurse is assessing an older adult client who is experiencing a myocardial infarction. What clinical manifestation does the nurse expect

in this client? a. Excruciating pain on inspiration

b. Left lateral chest wall pain c. Disorientation and confusion

d. Numbness and tingling of the arm

A client is newly diagnosed with a heart murmur and asks the nurse to explain what this means. What is the nurse’s best response?

a. “It is a rushing sound that blood makes moving through narrow places.”

b. “It’s the sound of the heart muscle stretching in an area of weakness.”

c. “It’s a term doctors use to describe the efficiency of blood circulation.”

d. “It is the sound the heart makes when it is has an increased workload.”

A client has returned from a cardiac angiography via the left femoral artery. Two hours after the procedure, the nurse notes that the left pedal

pulse is weak. What is the nurse’s best action? a. Elevate the leg and apply a sandbag to the entrance site.

b. Increase the flow rate of intravenous fluids. c. Assess the color and temperature of the left leg. d. Document the finding as left pedal pulse of +1/4.

The nurse is recovering a client after a left-sided cardiac catheterization. What assessment finding requires immediate intervention?

a. Urinary output less than intake b. Bruising at the insertion site

c. Slurred speech and confusion d. Discomfort in the left leg

The nurse is preparing a client for a cardiac catheterization. What assessment is a priority before the procedure?

a. Client’s level of anxiety b. Ability to turn self in bed

c. Cardiac rhythm and heart rate d. Allergies to iodine and shellfish

The client is scheduled for a resting electrocardiography. Which

statement by the client indicates a good understanding of the

preprocedure teaching? a. “I cannot eat or drink before the electrocardiography.”

b. “I must lie as still as possible during the procedure.” c. “I am likely to feel warmth as the dye enters the heart.”

d. “I will increase my fluid intake on the day of the procedure.”

The nurse is monitoring a client undergoing an exercise electrocardiography (stress test). Which assessment finding necessitates

that the test be stopped? a. Heart rate increases to 140 beats/min

b. Blood pressure of 100/80 mm Hg c. Respiratory rate exceeds 36 breaths/min

d. Significant ST-segment depression

A client who is scheduled for an echocardiography today asks why this test is being performed. What is the nurse’s best response?

a. “This procedure is a noninvasive way to assess the structure of the heart.”

b. “This procedure assesses for abnormal electrical impulses within the heart.”

c. “This procedure will evaluate the oxygen saturation in your blood.” d. “This procedure assesses for blockages within the coronary

arteries.”

The nurse is caring for a client who is scheduled for magnetic resonance imaging (MRI) of the heart. The client’s history includes a previous

myocardial infarction and pacemaker implantation. Which action by the nurse is most appropriate?

a. Schedule an electrocardiogram just before the MRI. b. Notify the health care provider before scheduling the MRI.

c. Call the physician and request a laboratory draw for cardiac enzymes.

d. Instruct the client to increase fluid intake the day before the MRI.

The nurse obtains a pulmonary artery pressure reading of 25/12 mm Hg in a client recovering from a myocardial infarction. What is the nurse’s

first intervention? a. Compare the results with previous readings.

b. Increase the IV fluid rate because these readings are low. c. Immediately notify the physician of the elevated pressures.

d. Document the finding and continue to monitor.

The nurse is preparing to measure a client’s pulmonary artery wedge pressure (PAWP). In what position will the nurse place the client for the

most accurate results? a. Supine, with the head elevated to 45 degrees b. Supine, with the head elevated to 30 degrees

c. Reverse Trendelenburg position at 15 degrees d. Supine, with the head of the bed flat

The nurse is caring for a client with an 80% blockage of the right coronary artery (RCA). While waiting for bypass surgery, what is

essential for the nurse to have available? a. Furosemide (Lasix) b. External pacemaker

c. Lidocaine d. Central venous access

The nurse is caring for a client with pericarditis. What assessment

finding correlates with this disorder? a. Pericardial friction rub

b. Systolic murmur c. Ventricular gallop

d. Paradoxical splitting

The nurse is auscultating heart tones on an older client and hears the following sound. What is the nurse’s best action? (Click the media button

to hear the audio clip.) a. Administer a diuretic. b. Document the finding.

c. Decrease the IV flow rate. d. Evaluate the medications.

The nurse is auscultating cardiac tones. Where should the nurse listen to best hear a cardiac murmur related to aortic regurgitation?

a. Location A b. Location B c. Location C d. Location D

MULTIPLE RESPONSE

A client with a history of renal insufficiency is scheduled for a cardiac catheterization. What does the nurse expect to do for this client before

the catheterization? (Select all that apply.) a. Insert a Foley catheter. b. Administer IV fluids.

c. Assess for allergies to iodine. d. Assess laboratory results. e. Assess and mark pulses.

f. Insert a central venous catheter. g. Have the client sign the consent.

A female client is admitted to the emergency department. Which symptoms cause the nurse to order an electrocardiogram? (Select all that

apply.) a. Hypertension

b. Fatigue despite adequate rest c. Indigestion

d. Abdominal fullness

e. Shortness of breath f. Feeling of choking g. Abdominal pain

A client is recovering after a coronary catheterization. What assessment findings in the first few hours after the procedure require immediate

action by the nurse? (Select all that apply.) a. Blood pressure of 140/88 mm Hg b. Serum potassium of 2.9 mEq/L c. Warmth and redness at the site

d. Expanding groin hematoma e. Rhythm changes on the cardiac monitor

When reviewing a client’s laboratory results, which findings alert the nurse to the possibility of atherosclerosis? (Select all that apply.)

a. Total cholesterol of 280 mg/dL b. High-density cholesterol of 50 mg/dL

c. Triglycerides of 200 mg/dL d. Serum albumin of 4 g/dL

e. Low-density cholesterol of 160 mg/dL

Chapter 36: Care of Patients with Dysrhythmias

Test Bank

MULTIPLE CHOICE

A client’s cardiac status is being observed by telemetry monitoring. The nurse observes a P wave that changes shape in lead II. What conclusion

does the nurse make about the P wave? a. It originates from an ectopic focus.

b. The P wave was replaced by U waves. c. It is from the sinoatrial (SA) node.

d. Multiple P waves are present.

The nurse is assessing the client’s electrocardiography (ECG). What does the P wave on the ECG tracing represent?

a. Contraction of the atria b. Contraction of the ventricles c. Depolarization of the atria

d. Depolarization of the ventricles

A nurse notes that the PR interval on a client’s electrocardiograph (ECG) tracing is 0.14 second. What action does the nurse take?

a. Assess serum cardiac enzymes. b. Administer 1 mg epinephrine IV.

c. Administer oxygen via nasal cannula. d. Document the finding in the client’s chart.

When analyzing a client’s electrocardiograph (ECG) tracing, the nurse observes that not all QRS complexes are preceded by a P wave. What is

the nurse’s interpretation of this observation? a. The client has hyperkalemia causing irregular QRS complexes. b. Ventricular tachycardia is overriding the normal atrial rhythm.

c. The client’s chest leads are not making sufficient contact with the skin.

d. Ventricular and atrial depolarizations are initiated from different sites.

The nurse observes a prominent U wave on the client’s electrocardiograph (ECG) tracing. What is the most appropriate action

for the nurse to take? a. Document the finding as a normal variant. b. Review the client’s daily electrolyte results.

c. Move the crash cart closer to the client’s room. d. Call for an immediate electrocardiogram.

The client’s heart rate increases slightly during inspiration and decreases slightly during expiration. What action does the nurse take?

a. Evaluate for a respirator disorder. b. Assess the client for chest pain.

c. Document the finding in the chart. d. Administer antidysrhythmic drugs.

A client with tachycardia is experiencing clinical manifestations. Which manifestation requires immediate intervention by the nurse?

a. Mid-sternal chest pain b. Increased urine output

c. Mild orthostatic hypotension d. P wave touching the T wave

A client is experiencing sinus bradycardia with hypotension and dizziness. What medication does the nurse administer?

a. Atropine (Atropine) b. Digoxin (Lanoxin)

c. Lidocaine (Xylocaine) d. Metoprolol (Lopressor)

A client experiences occasional premature atrial contractions (PACs) accompanied by palpitations that resolve spontaneously without treatment. What instruction does the nurse include in the client’s

teaching plan? a. “Minimize or abstain from caffeine.”

b. “Lie on your side until the attack subsides.” c. “Use your oxygen when you experience PACs.”

d. “Take quinidine (Cardioquin) daily to prevent PACs.”

The nurse identifies a client’s rhythm to be a sustained supraventricular tachycardia. What medication does the nurse administer?

a. Atropine (Atropine) b. Epinephrine (Adrenalin)

c. Lidocaine (Xylocaine) d. Diltiazem (Cardizem)

A client has a heart rate averaging 56 beats/min with no adverse symptoms. What activity modifications does the nurse suggest to avoid

further slowing of the heart rate? a. “Make certain that your bath water is warm (100° F).”

b. “Avoid bearing down or straining while having a bowel movement.” c. “Avoid strenuous exercise, such as running, during the late

afternoon.” d. “Limit your intake of caffeinated drinks to no more than 2 cups per

day.”

The nurse is assessing clients at a community clinic. Which client does the nurse assess most carefully for atrial fibrillation?

a. Middle-aged client who takes an aspirin daily b. Client who is dismissed after coronary artery bypass surgery

c. Older adult client after a carotid endarterectomy d. Client with chronic obstructive pulmonary disease

The nurse is caring for a client on a cardiac monitor. The monitor shows a rapid rhythm with a “saw tooth” configuration. What physical

assessment findings does the nurse expect? a. Presence of a split S1 and wheezing

b. Anorexia and gastric distress c. Shortness of breath and anxiety

d. Hypertension and mental status changes

The nurse is caring for a client with atrial fibrillation. What manifestation most alerts the nurse to the possibility of a serious

complication from this condition? a. Sinus tachycardia b. Speech alterations

c. Fatigue d. Dyspnea with activity

The nurse is caring for a client with chronic atrial fibrillation. Which drug does the nurse expect to administer to prevent a common

complication of this condition? a. Sotalol (Betapace)

b. Warfarin (Coumadin) c. Atropine (Atropine)

d. Lidocaine (Xylocaine)

The nurse is caring for a client admitted for myocardial infarction. The client’s monitor shows frequent premature ventricular contractions

(PVCs). What dysrhythmia does the nurse remain alert for? a. Sinus tachycardia

b. Rapid atrial flutter c. Ventricular tachycardia

d. Atrioventricular junctional rhythm

A client with myocardial ischemia is having frequent early, wide ventricular complexes seen on the cardiac monitor. Which medication

does the nurse administer? a. Lanoxin (Digoxin)

b. Amiodarone (Cordarone) c. Dobutamine (Dobutamine) d. Atropine sulfate (Atropisol)

The nurse has administered adenosine (Adenocard). What is the expected therapeutic response?

a. Increased intraocular pressure b. A brief tonic-clonic seizure c. A short period of asystole

d. Hypertensive crisis

A client’s electrocardiograph (ECG) tracing shows a run of sustained

ventricular tachycardia. What is the nurse’s first action? a. Assess airway, breathing, and level of consciousness. b. Administer an amiodarone bolus followed by a drip.

c. Cardiovert the client with a biphasic defibrillator. d. Begin cardiopulmonary resuscitation (CPR).

A client with unstable ventricular tachycardia is receiving amiodarone by intravenous infusion. The nurse notes that the client’s heart rate has decreased from 68 to 50 beats/min. The client is asymptomatic. What is

the nurse’s priority intervention? a. Stop the infusion and flush the IV. b. Slow the amiodarone infusion rate.

c. Administer a precordial thump. d. Place the client in a side-lying position.

A client with ischemic heart disease has an electrocardiograph (ECG) tracing that shows a PR interval of 0.24 second. What is the nurse’s best

action? a. Document the finding in the chart.

b. Measure blood pressure. c. Notify the health care provider.

d. Administer oxygen.

The physician is about to perform carotid sinus massage on a client with supraventricular tachycardia. What equipment is most important for the

nurse to have ready? a. Emesis basin

b. Magnesium sulfate c. Resuscitation cart

d. Padded tongue blade

The nurse is caring for a client with a complete heart block (third-degree atrioventricular [AV] block). What is the nurse’s priority intervention?

a. Perform a cardioversion. b. Assist with carotid massage.

c. Begin external pacing.

d. Administer adenosine (Adenocard) IV.

A client with third-degree heart block is admitted to the telemetry unit. The nurse observes wide QRS complexes on the monitor with a heart

rate of 35 beats/min. What priority assessment does the nurse perform? a. Pulmonary auscultation

b. Pulse strength and amplitude c. Level of consciousness

d. Mobility and gait stability

The nurse is caring for a client with a temporary pacemaker. The client’s bedside monitor shows a spike followed by a QRS complex. What is the

nurse’s best action? a. Remove the pacemaker; it is not needed. b. Decrease the threshold of the pacemaker.

c. Document the finding in the client’s chart. d. Set the pacemaker to the synchronous mode.

A client with ventricular tachycardia (VT) is unresponsive and has no pulse. The nurse calls for assistance and a defibrillator. What is the

nurse’s priority intervention while waiting for the defibrillator to arrive? a. Perform a pericardial thump.

b. Initiate cardiopulmonary resuscitation. c. Start an 18-gauge IV in the antecubital.

d. Ask the client’s family about code status.

A client has an epicardial pacemaker. The nurse observes the presence of a pacing spike but no QRS complex on the client’s electrocardiograph

(ECG) tracing. How does the nurse interpret this event? a. Loss of capture

b. Ventricular fibrillation c. Failure to sense

d. A normal tracing

The nurse is assisting with resuscitation of a client. What priority intervention does the nurse perform before defibrillating a client?

a. Make sure the defibrillator is set to the synchronous mode. b. Deliver a precordial thump to the upper portion of the sternum.

c. Test the equipment by delivering a smaller shock at 100 J. d. Ensure that all personnel are clear of contact with the client and the

bed.

The nurse is recovering a client after insertion of an implantable cardioverter-defibrillator (ICD). What complication must the nurse

intervene for immediately? a. 2/4 bilateral peripheral edema

b. Heart rate of 56 beats/min c. Temperature of 96° F (35.5° C)

d. Muffled heart sounds

A client was admitted for a permanent pacemaker insertion. What priority instruction does the nurse include in the client’s discharge

teaching? a. “Do not submerge your pacemaker, take only showers.” b. “Report pulse rates lower than your pacemaker setting.” c. “If you feel weak, apply pressure over your generator.”

d. “Have your pacemaker turned off before having an MRI.”

The nurse is providing discharge instructions for a client with an implantable cardioverter-defibrillator (ICD). What statement by the

client indicates a good understanding of the instructions? a. “I should wear a snug-fitting shirt over the ICD.”

b. “I will avoid sources of strong electromagnetic fields.” c. “I can’t perform activities that increase my heart rate.”

d. “Now I can discontinue my antidysrhythmic medication.”

A nurse assesses the following electrocardiography (ECG) strip from a client’s telemetry monitor. What does the nurse chart as the client’s

ventricular heart rate?

a. 40 beats/min b. 80 beats/min c. 120 beats/min d. 160 beats/min

The nurse is assessing a client’s ECG. What is the nurse’s interpretation of the following ECG strip?

a. Sinus rhythm with premature ventricular contractions (PVCs) b. Ventricular tachycardia c. Ventricular fibrillation

d. Sinus rhythm with premature atrial contractions (PACs)

The nurse notes the following rhythm on a client’s telemetry monitor. How does the nurse interpret these findings?

a. Ventricular tachycardia b. Second-degree heart block

c. Supraventricular tachycardia d. Premature ventricular contractions

A nurse assesses the following ECG strip from a client’s telemetry monitor. What does the nurse do next?

a. Measure hourly urine output. b. Assess the client’s vital signs.

c. Administer 0.5 mg atropine IV. d. Prepare for external pacing.

The nurse is alerted to a client’s telemetry monitor. After assessing the following ECG, what is the nurse’s priority intervention?

a. Start a large-bore IV. b. Administer atropine.

c. Prepare for intubation. d. Perform defibrillation.

MULTIPLE RESPONSE

A client has a consistently regular heart rate of 128 beats/min. Which related physiologic alterations does the nurse assess for? (Select all that

apply.) a. Decrease in cardiac output b. Increase in cardiac output c. Increase in blood pressure d. Decrease in blood pressure

e. Increase in urine output

Chapter 37: Care of Patients with Cardiac Problems

Test Bank

MULTIPLE CHOICE

A client is admitted with early-stage heart failure. Which assessment finding does the nurse expect?

a. A decrease in blood pressure and urine output b. An increase in creatinine and extremity edema

c. An increase in heart rate and respiratory rate d. A decrease in respirations and oxygen saturation

A client with systolic dysfunction has an ejection fraction of 38%. The nurse assesses for which physiologic change?

a. Increase in stroke volume b. Decrease in tissue perfusion

c. Increase in oxygen saturation d. Decrease in arterial vasoconstriction

The nurse is assessing clients on a cardiac unit. Which client does the nurse assess most carefully for developing left-sided heart failure?

a. Middle-aged woman with aortic stenosis b. Middle-aged man with pulmonary hypertension

c. Older woman who smokes cigarettes daily d. Older man who has had a myocardial infarction

The nurse is assessing a client in an outpatient clinic. Which client statement alerts the nurse to possible left-sided heart failure?

a. “I have been drinking more water than usual.” b. “I have been awakened by the need to urinate at night.”

c. “I have to stop halfway up the stairs to catch my breath.” d. “I have experienced blurred vision on several occasions.”

A client with a history of myocardial infarction calls the clinic to report the onset of a cough that is troublesome only at night. What direction

does the nurse give to the client? a. “Please come into the clinic for an evaluation.”

b. “Increase your fluid intake during waking hours.” c. “Use an over-the-counter cough suppressant.”

d. “Sleep on two pillows to facilitate postnasal drainage.”

The nurse is assessing a client admitted to the cardiac unit. What statement made by the client alerts the nurse to the possibility of right-

sided heart failure? a. “I sleep with four pillows at night.”

b. “My shoes fit really tight lately.” c. “I wake up coughing every night.”

d. “I have trouble catching my breath.”

The nurse notes that the client’s apical pulse is displaced to the left. What conclusion can be drawn from this assessment?

a. This is a normal finding. b. The heart is hypertrophied.

c. The left ventricle is contracted. d. The client has pulsus alternans.

The nurse assesses a client and notes the presence of an S3 gallop. What is the nurse’s best intervention?

a. Assess for symptoms of left-sided heart failure. b. Document this as a normal finding.

c. Call the health care provider immediately. d. Transfer the client to the intensive care unit.

A client asks the nurse why it is important to be weighed every day if he has right-sided heart failure. What is the nurse’s best response?

a. “Weight is the best indication that you are gaining or losing fluid.” b. “Daily weights will help us make sure that you’re eating properly.”

c. “The hospital requires that all inpatients be weighed daily.” d. “You need to lose weight to decrease the incidence of heart failure.”

A client has been admitted to the intensive care unit with worsening pulmonary manifestations of heart failure. What is the nurse’s best

action? a. Place the client in a high Fowler’s position.

b. Begin cardiopulmonary resuscitation (CPR). c. Promote rest and minimize activities.

d. Administer loop diuretics as prescribed.

A client with heart failure is experiencing acute shortness of breath. What is the nurse’s priority action?

a. Place the client in a high Fowler’s position. b. Perform nasotracheal suctioning of the client. c. Auscultate the client’s heart and lung sounds. d. Place the client on a 1000 mL fluid restriction.

A client with heart failure is prescribed enalapril (Vasotec). What is the nurse’s priority teaching for this client?

a. “Avoid using salt substitutes.”

b. “Take your medication with food.” c. “Avoid using aspirin-containing products.”

d. “Check your pulse daily.”

The nurse is administering captopril (Capoten) to a client with heart failure. What is the priority intervention for this client?

a. Administer this medication before meals to aid absorption. b. Instruct the client to ask for assistance when arising from bed.

c. Give the medication with milk to prevent stomach upset. d. Monitor the potassium level and check for symptoms of

hypokalemia.

The client who just started taking isosorbide dinitrate (Imdur) reports a headache. What is the nurse’s best action?

a. Titrate oxygen to relieve headache. b. Hold the next dose of Imdur.

c. Instruct the client to drink water. d. Administer PRN acetaminophen.

The client with heart failure has been prescribed intravenous nitroglycerin and furosemide (Lasix) for pulmonary edema. Which is the

priority nursing intervention? a. Insert an indwelling urinary catheter. b. Monitor the client’s blood pressure.

c. Place the nitroglycerin under the client’s tongue. d. Monitor the client’s serum glucose level.

The nurse is starting a client on digoxin (Lanoxin) therapy. What intervention is essential to teach this client?

a. “Avoid taking aspirin or aspirin-containing products.” b. “Increase your intake of foods high in potassium.”

c. “Hold this medication if your pulse rate is below 80 beats/min.” d. “Do not take this medication within 1 hour of taking an antacid.”

A client is taking triamterene-hydrochlorothiazide (Dyazide) and furosemide (Lasix). What assessment finding requires action by the

nurse? a. Cough

b. Headache c. Pulse of 62 beats/min

d. Potassium of 2.9 mEq/L

The rehabilitation nurse is assisting a client with heart failure to increase activity tolerance. During ambulation of the client, identification of what

symptom causes the nurse to stop the client’s activity? a. Decrease in oxygen saturation from 98% to 95%

b. Respiratory rate change from 22 to 28 breaths/min c. Systolic blood pressure change from 136 to 96 mm Hg

d. Increase in heart rate from 86 to 100 beats/min

The nurse is concerned that an older adult client with heart failure is developing pulmonary edema. What manifestation alerts the nurse to

further assess the client for this complication? a. Confusion b. Dysphagia

c. Sacral edema d. Irregular heart rate

A client with a history of heart failure is being discharged. Which priority instruction will assist the client in the prevention of

complications associated with heart failure? a. “Avoid drinking more than 3 quarts of liquids each day.” b. “Eat six small meals daily instead of three larger meals.”

c. “When you feel short of breath, take an additional diuretic.” d. “Weigh yourself daily while wearing the same amount of clothing.”

A client has been admitted to the acute care unit for an exacerbation of heart failure. Which is the nurse’s priority intervention?

a. Assess respiratory status. b. Monitor electrolyte levels.

c. Administer intravenous fluids. d. Insert a Foley catheter.

The nurse is caring for a client with mitral valve stenosis. What clinical manifestation alerts the nurse to the possibility that the client’s stenosis

has progressed? a. Oxygen saturation of 92%

b. Dyspnea on exertion c. Muted systolic murmur

d. Upper extremity weakness

The nurse is caring for a client diagnosed with aortic stenosis. What assessment finding does the nurse expect in this client?

a. Bounding arterial pulse b. Slow, faint arterial pulse c. Narrowed pulse pressure d. Elevated systolic pressure

A client who has had a prosthetic valve replacement asks the nurse why he must take anticoagulants for the rest of his life. What is the nurse’s

best response? a. “The prosthetic valve places you at greater risk for a heart attack.”

b. “Blood clots form more easily in artificial replacement valves.” c. “The vein taken from your leg reduces circulation in the leg.” d. “The surgery left a lot of small clots in your heart and lungs.”

The nurse is discharging a client home following mitral valve replacement. What statement indicates that the client requires further

education?

a. “I will be able to carry heavy loads after 6 months of rest.” b. “I will have my teeth cleaned by the dentist in 2 weeks.”

c. “I will avoid eating foods high in vitamin K, like spinach.” d. “I will use an electric razor instead of a straight razor to shave.”

The nurse is obtaining the admission health history for a young adult who presents with fever, dyspnea, and a murmur. What priority data

does the nurse inquire about? a. Family history of coronary artery disease b. Recent travel to Third World countries

c. Pet ownership, especially cats with litter boxes d. History of a systemic infection within the past month

The nurse is providing care to a client with infective endocarditis. What infection control precautions does the nurse use?

a. Standard Precautions b. Bleeding Precautions

c. Reverse isolation d. Contact isolation

A client with pericarditis is admitted to the cardiac unit. What assessment finding does the nurse expect in this client?

a. Heart rate that speeds up and slows down b. Friction rub at the left lower sternal border

c. Presence of a regularly gallop rhythm d. Coarse crackles in bilateral lung bases

The nurse is providing discharge education to a client with hypertrophic cardiomyopathy (HCM). What priority instruction will the nurse

include? a. “Take your digoxin at the same time every day.” b. “You should begin an aerobic exercise program.”

c. “You should report episodes of dizziness or fainting.” d. “You may have only two alcoholic drinks daily.”

The nurse reminds the client who has received a heart transplant to change positions slowly. Why is this instruction a priority?

a. Rapid position changes can create shear and friction forces, which can tear out internal vascular sutures.

b. The new vascular connections are more sensitive to position changes, leading to increased intravascular pressure.

c. The new heart is denervated and is unable to respond to decreases in blood pressure caused by position changes.

d. The recovering heart diverts blood flow away from the brain when the client stands, increasing the risk for stroke.

A client is being discharged home after a heart transplant with a prescription for cyclosporine (Sandimmune). What priority education

does the nurse provide with the client’s discharge instructions? a. “Use a soft-bristled toothbrush and avoid flossing.”

b. “Avoid large crowds and people who are sick.” c. “Change positions slowly to avoid hypotension.”

d. “Check your heart rate before taking the medication.”

A client with end-stage heart failure is awaiting a transplant. The client appears depressed and states, “I know a transplant is my last chance, but I don’t want to become a vegetable.” What is the nurse’s best response?

a. “Would you like to speak with a priest or chaplain?” b. “I will consult a psychiatrist to speak with you.” c. “Do you want to come off the transplant list?”

d. “Would you like information about advance directives?”

The nurse is assessing a client with a history of heart failure. What priority question assists the nurse to assess the client’s activity level?

a. “Do you have trouble breathing or chest pain?” b. “Are you able to walk upstairs without fatigue?”

c. “Do you awake with breathlessness during the night?” d. “Do you have new-onset heaviness in your legs?”

An older adult client with heart failure states, “I don’t know what to do. I don’t want to be a burden to my daughter, but I can’t do it alone.

Maybe I should die.” What is the nurse’s best response? a. “Would you like to talk about this more?”

b. “You’re lucky to have such a devoted daughter.” c. “You must feel as though you are a burden.”

d. “Would you like an antidepressant medication?”

An older adult client is admitted with fluid volume excess. Which diagnostic study does the nurse facilitate as a priority?

a. Echocardiography

b. Chest x-ray c. T4 and thyroid-stimulating hormone (TSH)

d. Arterial blood gas

The nurse is caring for a client with severe heart failure. What is the best position in which to place this client?

a. High Fowler’s, pillows under arms b. Semi-Fowler’s, with legs elevated c. High Fowler’s, with legs elevated

d. Semi-Fowler’s, on the left side

The nurse is instructing a client with heart failure about energy conservation. Which is the best instruction?

a. “Walk until you become short of breath and then walk back home.” b. “Gather everything you need for a chore before you begin.”

c. “Pull rather than push or carry items heavier than 5 pounds.” d. “Take a walk after dinner every day to build up your strength.”

A client with heart failure is due to receive enalapril (Vasotec) and has a

blood pressure of 98/50 mm Hg. What is the nurse’s best action? a. Administer the Vasotec.

b. Recheck the blood pressure. c. Hold the Vasotec.

d. Notify the health care provider.

A client in severe heart failure has a heparin drip infusing. The health care provider prescribes nesiritide (Natrecor) to be given intravenously. Which intervention is essential before administration of this medication?

a. Insert a separate IV access. b. Prepare a test bolus dose.

c. Prepare the piggyback line. d. Administer furosemide (Lasix) first.

MULTIPLE RESPONSE

The nurse is assessing a client with left-sided heart failure. What conditions does the nurse assess for? (Select all that apply.)

a. Pulmonary crackles b. Confusion, restlessness

c. Pulmonary hypertension d. Dependent edema

e. S3/S4 summation gallop f. Cough worsens at night

The nurse is evaluating the laboratory results for a client with heart failure. What results does the nurse expect? (Select all that apply.)

a. Hematocrit (Hct), 32.8% b. Serum sodium, 130 mEq/L

c. Serum potassium, 4.0 mEq/L d. Serum creatinine, 1.0 mg/dL

e. Proteinuria f. Microalbuminuria

Chapter 38: Care of Patients with Vascular Problems

Test Bank

MULTIPLE CHOICE

A client with atherosclerosis asks a nurse which factors are responsible for this condition. What is the nurse’s best response?

a. “Injury to the arteries causes them to spasm, reducing blood flow to the extremities.”

b. “Excess fats in your diet are stored in the lining of your arteries, causing them to constrict.”

c. “A combination of platelets and fats accumulates, narrowing the artery and reducing blood flow.”

d. “Excess sodium causes injury to the arteries, reducing blood flow and eventually causing obstruction.”

The nurse is working with clients at a health fair. Which teaching takes priority to reduce the risk of atherosclerosis?

a. Instructing a diabetic client not to smoke or use any tobacco b. Teaching diet changes to a client with elevated cholesterol levels c. Suggesting limiting alcohol to an older client with hypolipidemia

d. Encouraging exercise to an obese client who lives a sedentary lifestyle

A client with hyperlipidemia who is being treated with dietary fat restrictions and an exercise program asks the nurse why his serum lipid levels are still elevated. What activity by the nurse is most appropriate?

a. Developing a very low-fat diet that the client will adhere to b. Explaining familial tendencies in hyperlipidemia

c. Referring the client to a registered dietitian for weight loss d. Educating the client on antihyperlipidemic medications

A client with atherosclerosis is attempting to stop cigarette smoking with the use of a nicotine patch. Which statement by the client indicates a

good understanding of smoking cessation education? a. “Abruptly discontinuing this patch can cause high blood pressure.” b. “Abruptly discontinuing this patch can cause nausea and vomiting.”

c. “Smoking while using this patch increases the risk of respiratory infection.”

d. “Smoking while using this patch increases the risk of a heart attack.”

A client with hypercholesterolemia and atherosclerosis is prescribed nicotinic acid (Niaspan). Which instruction does the nurse provide the

client? a. “This medication may make you flush.”

b. “Take this medication on an empty stomach.” c. “You will not need to change your diet with this medication.”

d. “Take this medication when you experience chest pain.”

The nurse incorporates dietary teaching into the plan for a client with a low-density lipoprotein (LDL) level of 158 mg/dL. What dietary

instruction by the nurse is most appropriate? a. “You should keep your saturated fat intake below 10% of your total

calories.” b. “This result is normal, so continue your current dietary practices.”

c. “Your total cholesterol intake should be less than 300 mg/day.” d. “You should restrict protein sources to fish and chicken only.”

The nurse is assisting the hospitalized client with his food selections for breakfast. The client is on a low-cholesterol diet. What recommendations

are most appropriate for this client? a. Cheese omelet, skim milk, whole wheat toast, coffee

b. Skim milk, oatmeal, banana, orange juice, coffee c. Whole wheat French toast, a side of bacon, coffee

d. Blueberry muffin, orange juice, decaffeinated coffee

The nurse is reviewing the menu selections of a client who has ordered a low-cholesterol diet. What meal items does the nurse question?

a. Vegetarian wrap b. Cheesesteak sandwich c. Fruit salad with yogurt d. Grilled fish sandwich

After reviewing the client’s chart upon admission to the unit, the nurse consults the health care provider about a new order for lovastatin

(Mevacor). What triggered the nurse’s action? a. Blood glucose of 182 mg/dL

b. History of peptic ulcers c. History of high cholesterol

d. Elevated liver enzymes

A client with high cholesterol is beginning treatment with simvastatin (Zocor). What priority instruction does the nurse give this client?

a. “Increase your intake of dietary fiber to minimize constipation.” b. “Take this drug on an empty stomach to promote absorption.” c. “Report any muscle tenderness to your health care provider.”

d. “You may experience flushing of the skin with this medication.”

A client has been diagnosed with Cushing’s syndrome. What assessment does the nurse perform to detect vascular complications associated with

this illness? a. Auscultation of heart and lung sounds

b. Assessment of blood pressure c. Daily weight using the same scale

d. Monitoring of urine output every 24 hours

The nurse is providing care for a client with hypertension. What priority physical assessment does the nurse include in examination of this client?

a. Skin examination for telangiectasia b. Otoscopic examination of the inner ear c. Funduscopic examination of the retina

d. Neurologic examination of the cranial nerves

The nurse is caring for a client with newly diagnosed hypertension. What statement by the client indicates adequate understanding of his or her

diet restrictions?

a. “I will give my canned soups to the food pantry.” b. “I’m going to miss my evening glass of wine.”

c. “I will mostly use salt substitutes for flavoring.” d. “I can have regular coffee only in the morning.”

The nurse is assessing a client’s understanding of his hypertension therapy. What client statement indicates a need for further teaching?

a. “If I lose weight, I might be able to reduce my blood pressure medication.”

b. “If my blood pressure stays under control, I will reduce my risk for a heart attack.”

c. “When my blood pressure is normal, I will no longer need to take medication.”

d. “When getting out of bed in the morning, I will sit for a few moments then stand.”

A nurse is about to administer the first dose of captopril (Capoten) to a client with hypertension. Which is the priority nursing intervention?

a. Take the client’s apical pulse for 1 full minute before drug administration.

b. Place the client in Trendelenburg position to facilitate blood flow to the heart.

c. Educate the client to sit on the side of the bed for a few minutes before rising.

d. Instruct the client to drink 3 L of fluid daily when taking this medication.

The nurse is a assessing a client with hypertension. Which client outcome is indicative of effective hypertension management?

a. Pedal edema is not present in the lower legs. b. No complaints of sexual dysfunction occur.

c. No indication of renal impairment is present. d. The blood pressure reading is 148/94 mm Hg.

The nurse is assessing a client who reports claudication after walking a distance of one block. The nurse notes a painful ulcer on the fourth toe of the client’s right foot. What condition do these findings correlate with?

a. Diabetic foot ulceration b. Peripheral arterial disease c. Peripheral venous disease

d. Deep vein thrombosis

The nurse notes a venous ulcer on the client’s left ankle. What additional assessment finding does the nurse expect in this client?

a. Absence of hair on the left lower extremity b. Skin surrounding the ulcer mottled but blanchable

c. Brownish discoloration of the lower extremity d. Cold and gray-blue lower extremity

A client with chronic peripheral arterial disease and claudication tells the nurse that burning pain often awakens him from sleep. What is the

nurse’s interpretation of this change? a. The client has inflow disease.

b. The client has outflow disease. c. The client’s disease is worsening.

d. The client’s disease is stable.

The nurse is educating a client before a right leg atherectomy. What priority education does the nurse provide?

a. “You may use the bathroom after the procedure.” b. “You will be sedated for 6 hours after the procedure.”

c. “You will not need to take a daily aspirin anymore.” d. “You may be on heparin during the procedure.”

The nurse is caring for a client with peripheral arterial disease. What priority nursing intervention does the nurse perform to promote

vasodilation? a. Increase the client’s exercise regimen daily. b. Apply a heating pad to the affected limb. c. Administer an aspirin on a daily basis.

d. Educate the client to abstain from smoking.

The nurse is recovering a client with peripheral arterial disease who has just undergone percutaneous transluminal angioplasty. What

complication does the nurse monitor for in the immediate postprocedure period?

a. Bleeding b. Aspiration

c. Hypertensive crisis d. Chest pain

The nurse is monitoring a client who has returned to the unit after arterial revascularization. The client reports pain in the affected limb

that is similar to the pain experienced before the procedure. What is the nurse’s best action?

a. Assess the peripheral pulses in the limb. b. Elevate the affected extremity on pillows.

c. Administer pain medication as prescribed. d. Place a warm blanket on the operative limb.

A client is recovering after an embolectomy. What clinical manifestations consistent with compartment syndrome does the nurse watch for?

a. Elevated temperature and excessive diaphoresis b. Loss of sensation and pallor near the surgical site

c. Swelling, pain, and tension of the affected limb d. Increased pulse and warmth below the surgical site

The nurse is caring for a client who develops compartment syndrome after an embolectomy for peripheral arterial disease. What is the nurse’s

best action? a. Perform passive range-of-motion exercise to improve distal blood

flow.

b. Prepare the client for return to the operative suite for surgical correction.

c. Medicate the client for pain and place the client in a knee-chest position.

d. Loosen the dressing and elevate the extremity to the level of the heart.

The new graduate nurse is assessing a client with an unrepaired abdominal aortic aneurysm. What assessment technique requires further

education by the supervising nurse? a. Measurement of abdominal girth

b. Observation of abdominal wall movement c. Auscultation of any area of the abdomen d. Palpation of the abdominal midline area

A client with a diagnosed abdominal aortic aneurysm (AAA) develops lower back pain radiating to the groin. What is the nurse’s interpretation

of this information? a. The aneurysm clotted and is obstructing blood flow.

b. The aneurysm is expanding and is preparing to rupture. c. The client feels the inflammation of the aneurysm. d. This is a normal sensation associated with an AAA.

The nurse is preparing a client with an aortic aneurysm for surgery. The nurse notes that the client’s systolic blood pressure has increased by 30 mm Hg compared with the reading 1 hour ago. What is the nurse’s best

action? a. Measure abdominal girth. b. Auscultate the abdomen.

c. Increase the IV infusion rate. d. Reassess the blood pressure.

A nurse is recovering a client who has undergone surgical repair of an abdominal aortic aneurysm (AAA). The client develops coolness of the extremities and reports a bloated feeling in the abdomen. What is the

nurse’s best action? a. Measure the abdominal girth and check pulses.

b. Raise the head of the bed to 90 degrees. c. Assess cardiac output and blood pressure. d. Auscultate and then palpate the abdomen.

The nurse is providing discharge education to a client after repair of an abdominal aortic aneurysm (AAA). What priority instruction does the

nurse include? a. “No restrictions on driving your car are necessary.”

b. “Avoid sleeping on your left side for 6 weeks.” c. “Avoid lifting heavy objects for about 3 months.” d. “You will have a distended abdomen for 2 weeks.”

The nurse is caring for a client with Buerger’s disease. What client education does the nurse provide to minimize disease progression?

a. “Keep environmental temperatures warm.” b. “Avoid highly stressful activities.”

c. “Use a heating pad on your extremities.” d. “Abstain from all forms of tobacco.”

The nurse is assessing the extremities of a client with Buerger’s disease. What clinical manifestation does the nurse correlate with this disease?

a. Reddened, with diminished distal pulses b. Cold and pale, with proximal bounding pulses c. Cyanotic, with decreased deep tendon reflexes

d. Brownish discoloration, with pitting edema

The nurse is providing disease management education to a client with Raynaud’s disease. What intervention does the nurse suggest to prevent

complications of this disease? a. “Take vasoconstrictive agents when you have symptoms.”

b. “Wear warm clothing when exposed to cool temperatures.” c. “Avoid placing alcohol-based lotion on affected extremities.” d. “Check the strength of pulses in your arms and legs daily.”

The nurse is caring for a client who is receiving heparin therapy for a venous thromboembolism (VTE). The client’s activated partial

thromboplastin time (aPTT) before heparin therapy was 30 seconds. Which aPTT result indicates that anticoagulation is adequate at this

time? a. 15 seconds b. 30 seconds c. 60 seconds d. 150 seconds

The health care provider has prescribed a client sodium warfarin (Coumadin) while he is still receiving intravenous heparin. Which is the

nurse’s best action? a. Administer both heparin and warfarin as prescribed.

b. Turn off the heparin before administering the warfarin. c. Clarify the warfarin order with the nursing supervisor.

d. Hold the warfarin dose until the heparin is discontinued.

The nurse is discharging home a client at risk for venous thromboembolism (VTE) on low-molecular-weight heparin. What

instruction does the nurse provide to this client? a. “You must have your aPTT checked every 2 weeks.”

b. “Massage the injection site after the heparin is injected.” c. “Notify your health care provider if your stools appear tarry.”

d. “An IV catheter will be placed to administer your heparin.”

The nurse is providing health education to a client with chronic venous stasis ulcers. What priority instruction does the nurse include?

a. “Apply antiembolism stockings before getting out of bed in the

morning.” b. “Clean venous ulcers with Betadine before applying a dressing.” c. “Take 1 low-dose aspirin (81 mg) daily to prevent inflammation.”

d. “Remove and reapply a new DuoDerm dressing to your ulcers each day.”

The nurse is assessing for skin changes in an African-American client admitted with peripheral arterial disease. What does the nurse monitor

for? a. Excess hair growth

b. Pitting edema in the feet c. Cyanosis of the nail beds

d. Loss of toenails

The nurse assesses a client’s legs. Which assessment finding indicates arterial insufficiency?

a. Ankle discoloration and pitting edema b. Dependent mottling and absence of hair c. Pain with activity but not while resting

d. Full veins present in dependent extremity

MULTIPLE RESPONSE

The nurse is reviewing a client’s laboratory results. The nurse correlates elevations in which values as risk factors for atherosclerosis? (Select all

that apply.) a. Total cholesterol, 280 mg/dL

b. High-density cholesterol, 50 mg/dL c. Triglycerides, 200 mg/dL d. Serum albumin, 4 g/dL

e. Low-density cholesterol, 160 mg/dL

An older adult client is prescribed furosemide (Lasix) for control of hypertension. What client education does the nurse provide? (Select all

that apply.) a. “Confusion can occur when taking this medication.”

b. “Drink at least 3 liters of water every day.” c. “Arise slowly from a chair or from your bed.”

d. “Persistent coughing is a side effect of this drug.” e. “You should eat foods high in potassium.”

The nurse is caring for a client who is experiencing excessive bleeding after receiving unfractionated heparin. What orders does the nurse

anticipate from the health care provider? (Select all that apply.) a. Laboratory draw for activated partial thromboplastin time (aPTT)

b. Administer vitamin K c. Laboratory draw for prothrombin time (PT)/international

normalized ratio (INR) d. Administer protamine sulfate

e. Administer enoxaparin (Lovenox)

Chapter 39: Care of Patients with Shock

Test Bank

MULTIPLE CHOICE

The intensive care nurse is educating the spouse of a client who is being treated for shock. The spouse states, “The doctor said she has shock.

What is that?” What is the nurse’s best response? a. “Shock occurs when oxygen to the body’s tissues and organs is

impaired.” b. “Shock is a serious condition, but it is not a life-threatening

emergency.”

c. “Shock progresses slowly and can be stopped by the body’s normal compensation.”

d. “Shock is a condition that affects only specific body organs like the kidneys.”

The nurse is caring for multiple clients in the emergency department. The client with which condition is at highest risk for distributive shock?

a. Severe head injury from a motor vehicle accident b. Diabetes insipidus from polycystic kidney disease

c. Ischemic cardiomyopathy from severe coronary artery disease d. Vomiting of blood from a gastrointestinal ulcer

The nurse is assessing a client who has hypovolemic shock. Which laboratory value indicates that the client is at risk for acidosis?

a. Decreased serum creatinine b. Increased serum lactic acid

c. Increased urine specific gravity d. Decreased partial pressure of arterial carbon dioxide

A client brought to the emergency department after a motor vehicle accident is suspected of having internal bleeding. Which question does the nurse ask to determine whether the client is in the early stages of

hypovolemic shock? a. “Are you more thirsty than normal?”

b. “When was the last time you urinated?” c. “What is your normal heart rate?”

d. “Is your skin usually cool and pale?”

A client who has acidosis resulting from hypovolemic shock has been prescribed intravenous fluid replacement. Which fluid does the nurse

prepare to administer? a. Normal saline

b. Ringer’s lactate c. 5% dextrose in water

d. 5% dextrose in 0.45% normal saline

The nurse is monitoring a client in hypovolemic shock who has been placed on a dopamine hydrochloride (Intropin) drip. Which

manifestation is a desired response to this medication? a. Decrease in blood pressure

b. Increase in heart rate

c. Increase in cardiac output d. Decrease in mean arterial pressure

The nurse is caring for a client who has hypovolemic shock. After administering oxygen, what is the priority intervention for this client?

a. Administer an aminoglycoside. b. Initiate a dopamine hydrochloride (Intropin) drip.

c. Administer crystalloid fluids. d. Initiate an intravenous heparin drip.

The nurse is administering prescribed sodium nitroprusside (Nipride) intravenously to a client who has shock. Which nursing intervention is a

priority when administering this medication? a. Ask if the client has chest pain every 30 minutes.

b. Assess the client’s blood pressure every 15 minutes. c. Monitor the client’s urinary output every hour. d. Observe the client’s extremities every 4 hours.

The nurse is preparing to administer sodium nitroprusside (Nipride) to a

client. Which important action related to the administration of this drug does the nurse implement?

a. Assess the client’s respiratory rate. b. Administer the medication with gravity tubing.

c. Protect the medication from light with an opaque bag. d. Monitor for hypertensive crisis.

The nurse is caring for a client who has had an anaphylactic event. Which priority question does the nurse ask to determine whether the

client is experiencing distributive shock? a. “Is your blood pressure higher than usual?”

b. “Are you having pain in your throat?” c. “Have you been vomiting?”

d. “Are you usually this swollen?”

A client who has septic shock is admitted to the hospital. What priority intervention does the nurse implement first?

a. Obtain two sets of blood cultures. b. Administer the prescribed IV vancomycin (Vancocin). c. Obtain central venous pressure (CVP) measurements.

d. Administer the prescribed IV norepinephrine (Levophed).

The nurse is assessing a client who was admitted for treatment of shock. Which manifestation indicates that the client’s shock is caused by sepsis?

a. Hypotension b. Pale clammy skin

c. Anxiety and confusion d. Oozing of blood at the IV site

A client was admitted 2 days ago with early stages of septic shock. Today the nurse notes that the client’s systolic blood pressure, pulse pressure, and cardiac output are decreasing rapidly. Which intervention does the

nurse do first? a. Insert a Foley catheter to monitor urine output closely.

b. Ask the client’s family to come to the hospital because death is near. c. Initiate the prescribed dobutamine (Dobutrex) intravenous drip.

d. Obtain blood cultures before administering the next dose of antibiotics.

The nurse is assessing clients in the emergency department. Which client is at highest risk for developing septic shock?

a. 25-year-old man who has irritable bowel syndrome b. 37-year-old woman who is 20% above ideal body weight

c. 68-year-old woman who is being treated with chemotherapy d. 82-year-old man taking beta blockers for hypertension

The nurse is caring for a client in the hyperdynamic phase of septic shock. Which medication does the nurse expect to be prescribed?

a. Heparin sodium b. Vitamin K

c. Corticosteroids d. Hetastarch (Hespan)

The nurse is planning care for a client with late-phase septic shock. All of the following treatments have been prescribed. Which prescription does

the nurse question? a. Enoxaparin (Lovenox) 40 mg subcutaneous twice daily

b. Transfusion of 2 units of fresh frozen plasma c. Regular insulin intravenous drip per protocol

d. Cefazolin (Ancef) 1 g IV every 6 hours

The nurse is assessing a client at risk for shock. The client’s systolic

blood pressure is 20 mm Hg lower than baseline. Which intervention does the nurse perform first?

a. Increase the IV fluid rate. b. Administer oxygen.

c. Notify the health care provider. d. Place the client in high Fowler’s position.

A client recovering from septic shock is preparing for discharge home. What priority information does the nurse include in the teaching plan

for this client? a. “Clean your toothbrush with laundry bleach daily.”

b. “Bathe every other day with antimicrobial soap.” c. “Wash your hands after changing pet litter boxes.” d. “Use an electric razor when you shave your face.”

The nurse is providing community education for clients at risk for dehydration. One client states, “We are not at risk because we live in a

hot and dry climate.” What is the nurse’s best response? a. “You are still at risk but not as high a risk as those who live in hot

and humid climates.” b. “Any type of heat can cause peripheral vasoconstriction, which

causes the body to lose water.” c. “In a hot and dry environment, the body can lose an increased

amount of water without your knowledge.”

d. “Even though you are not at risk, you should drink adequate fluids when you exercise.”

The emergency department nurse is triaging clients. Which client does the nurse assess most carefully for hypovolemic shock?

a. 15-year-old adolescent who plays high school basketball b. 24-year-old computer specialist who has bulimia

c. 48-year-old truck driver who has a 40-pack-year history of smoking d. 62-year-old business executive who travels frequently

The nurse is planning discharge education for a client who had an exploratory laparotomy. Which nursing statement is appropriate when

teaching the client to monitor for early signs of shock? a. “Monitor how much urine you void and report a decrease in the

amount.” b. “Take your temperature daily and report any below-normal body

temperatures.” c. “Assess your radial pulse every day and report an irregular

rhythm.” d. “Monitor your bowel movements and report ongoing constipation or

diarrhea.”

A client who has a local infection of the right forearm is being discharged. The nurse teaches the client to seek immediate medical

attention if which complication occurs? a. Dizziness on changing position

b. Increased urine output c. Warmth and redness at site

d. Low-grade temperature

The intensive care nurse is caring for an intubated client who has severe sepsis that led to acute respiratory distress. Which nursing intervention

is most appropriate during this stage of sepsis? a. Check blood glucose levels every 4 hours.

b. Monitor intake and urinary output twice each shift. c. Decrease ventilator rate and tidal volume.

d. Administer prescribed low-dose corticosteroids.

The nurse is assessing a client who has septic shock. The following assessment data were collected:

Baseline Data Today’s Data Heart rate 75 beats/min 98 beats/min Blood pressure 125/65 mm Hg 128/75 mm Hg

Respiratory rate 12 breaths/min 18 breaths/min Urinary output 40 mL/hr 40 mL/hr

The nurse correlates these findings with which stage of shock?

a. Early b. Compensatory c. Intermediate d. Refractory

MULTIPLE RESPONSE

The nurse is assessing a client who is in early stages of hypovolemic shock. Which manifestations does the nurse expect? (Select all that

apply.) a. Elevated heart rate

b. Elevated diastolic blood pressure c. Decreased body temperature

d. Elevated respiratory rate e. Decreased pulse rate

The nurse is providing health education to a client on immunosuppressant therapy. Which instructions does the nurse include

in this client’s teaching? (Select all that apply.) a. “Wear a facemask at all times.”

b. “Take your temperature once a day.” c. “Drink only bottled water.”

d. “Avoid any contact with pets.” e. “Wash dishes with hot sudsy water.”

f. “Rinse your toothbrush in liquid laundry bleach.”

A client has septic shock. Which hemodynamic parameters does the nurse correlate with this type of shock? (Select all that apply.)

a. Decreased cardiac output b. Increased cardiac output c. Increased blood glucose

d. Decreased blood glucose e. Increased serum lactate f. Decreased serum lactate

Chapter 40: Care of Patients with Acute Coronary Syndromes

Test Bank

MULTIPLE CHOICE The nurse is taking the history of a client with suspected coronary artery

disease (CAD). Which situation correlates with stable angina? a. Chest discomfort at rest and inability to tolerate mowing the lawn b. Chest discomfort when mowing the lawn and subsiding with rest

c. Indigestion and a choking sensation when mowing the lawn d. Jaw pain that radiates to the shoulder after mowing the lawn

The nurse is assessing a client who has a history of stable angina. The

client describes a recent increase in the number of attacks and in the intensity of the pain. Which question does the nurse ask to assess the

client’s change in condition? a. “How many cigarettes do you smoke daily?” b. “Do you have pain when you are resting?” c. “Do you have abdominal pain or nausea?”

d. “How frequently are you having chest pain?”

The community health nurse assesses clients at a health fair. Which statement assists the nurse to identify modifiable risk factors in clients

with coronary artery disease? a. “Would you please state your full name and birth date?”

b. “Have you ever had an exercise tolerance stress test?” c. “In what activities do you participate on a daily basis?”

d. “Does anyone in your family have a history of heart disease?”

The nurse teaches a client who is newly diagnosed with coronary artery disease. Which instruction does the nurse include to minimize

complications of this disease? a. “Rest is the best medicine at this time. Do not start an exercise

program.” b. “You are a man; therefore there is nothing you can do to minimize

your risks.” c. “You should talk to your provider about medications to help you

quit smoking.” d. “Decreasing the carbohydrates in your diet will help you lose

weight.”

The emergency department nurse is assessing an 82-year-old client for a potential myocardial infarction. Which clinical manifestation does the

nurse monitor for? a. Pain on inspiration

b. Posterior wall chest pain c. Disorientation or confusion

d. Numbness and tingling of the arm Eight hours after presentation to the emergency department with reports

of substernal chest pain, a client’s laboratory results demonstrate myoglobin levels of 55 ng/mL. What does the nurse do next?

a. Prepare the client for an emergency coronary bypass graft surgery.

b. Administer nitroglycerin to prevent further myocardial cell death. c. Assess the client to identify another potential cause of the chest pain.

d. Provide client education related to complications of myocardial infarctions.

The nurse evaluates diagnostic results for a client who has chest pain. Which laboratory test is most specific for acute coronary syndromes?

a. Troponin markers b. Serum lactate dehydrogenase (LDH)

c. Serum myoglobin d. Creatine kinase (CK)-MB isoenzyme

While evaluating a client’s electrocardiogram (ECG) before surgery, the preoperative nurse identifies large, wide Q waves. What is the nurse’s

best interpretation of this finding? a. An acute myocardial infarction is occurring.

b. The client had a myocardial infarction in the past. c. The ventricles are enlarged and failing.

d. The ECG is a common variation of normal sinus rhythm.

The nurse is providing care for a client admitted to the hospital with reports of chest pain. After receiving a total of three nitroglycerin

sublingual tablets, the client states, “The pain has not gotten any better.” What does the nurse do next?

a. Place the client in a semi-Fowler’s position. b. Administer intravenous nitroglycerin. c. Begin supplemental oxygen at 2 L/min.

d. Notify the health care provider.

The nurse assesses a client who has received thrombolytic therapy after having a myocardial infarction. Which clinical manifestation indicates to

the nurse that reperfusion has been successful? a. ST-segment depression

b. Cessation of diaphoresis c. Sudden onset of pleuritic chest pain d. Onset of ventricular dysrhythmias

A client who presented with an acute myocardial infarction is prescribed thrombolytic therapy. The client had a stroke 1 month ago. Which action

does the nurse take? a. Administer the medication as prescribed.

b. Perform a CT scan before administering the medication. c. Contact the health care provider to discontinue the prescribed

therapy. d. Administer the therapy with a normal saline bolus.

The nurse is administering thrombolytic therapy to a client who had a myocardial infarction. Which intervention does the nurse implement to

reduce the risk of complications in this client? a. Administer prescribed heparin. b. Apply ice to the injection site.

c. Place the client in Trendelenburg position. d. Instruct the client to take slow deep breaths.

The nurse is assessing a client who has been prescribed a nonselective beta-blocking agent. Which adverse effect does the nurse monitor for in

this client? a. Headache

b. Postural hypotension c. Nonproductive cough

d. Wheezing

The nurse is assisting a client to walk in the hall on the third day after a myocardial infarction. Which clinical manifestation indicates to the

nurse that the client is not ready to advance to the next level of activity? a. Facial flushing

b. Onset of chest pain c. Heart rate increase of 10 beats/min at completion of the activity

d. Systolic blood pressure increase of 10 mm Hg at completion of the activity

The nurse is assessing a client who has left ventricular failure secondary to a myocardial infarction. Which clinical manifestation of poor organ

perfusion does the nurse monitor for in this client? a. Headache

b. Hypertension c. Urine output of less than 30 mL/hr

d. Heart rate of 55 to 60 beats/min

The nurse is caring for a client who had a myocardial infarction. The client develops increased pulmonary congestion; an increase in heart rate from 80 to 102 beats/min; and cold, clammy skin. Which action does the

nurse implement before notifying the health care provider? a. Administer oxygen.

b. Increase the IV flow rate. c. Place the client in supine position.

d. Prepare the client for surgery.

The nurse is teaching a client who is prescribed a calcium channel blocking agent after a percutaneous transluminal coronary angioplasty

(PTCA). Which instruction does the nurse include in this client’s teaching?

a. “Change position slowly.” b. “Avoid crossing your legs.”

c. “Weigh yourself daily.” d. “Decrease salt intake.”

A client who is post percutaneous transluminal coronary angioplasty (PTCA) reports severe chest pain. Which action does the nurse take

first? a. Administer the prescribed IV morphine.

b. Administer the prescribed sublingual nitroglycerin. c. Assess the client’s vital signs and notify the health care provider.

d. Perform an immediate 12-lead ECG.

The nurse is teaching a client prescribed sublingual nitroglycerin for chest pain. Which statement indicates that the client needs further

teaching? a. “I carry my medicine around in a clear plastic bag so that I can get

to it easily if I have chest pain.” b. “Even if I have not used any of the nitroglycerin from one refill, I

get another refill every 3 months.” c. “If I still have chest pain after I have taken 3 nitroglycerin tablets, I

will go to the hospital.” d. “When my nitroglycerin tablet tingles under my tongue, I know that

it is strong enough to work.”

The nurse is assessing a client who had percutaneous transluminal coronary angioplasty (PTCA) 1 hour ago. Which complication does the

nurse monitor for? a. Hypertensive crisis

b. Hyperkalemia c. Infection d. Bleeding

The nurse is assessing a client who has undergone a percutaneous transluminal coronary angioplasty (PTCA) and is ordered to receive an

IV infusion of abciximab (ReoPro). Which clinical manifestation does the nurse monitor for in this client?

a. Bleeding b. Joint pain

c. Pedal edema d. Excessive thirst

The nurse is assessing a client who has a serum potassium level of 4.5 mEq/L after coronary artery bypass graft (CABG) surgery. Which action

does the nurse take? a. Notify the health care provider.

b. Document the finding. c. Administer prescribed diuretics.

d. Administer prescribed potassium replacements.

The nurse is assessing a client who is 6 hours postoperative from coronary artery bypass graft surgery. The client’s mediastinal tubes are

not draining. Which action does the nurse implement at this time? a. Replace the drainage tubing. b. Check for kinks in the tubing.

c. Irrigate the tubing with normal saline. d. Document the finding.

The nurse is caring for an 80-year-old client who has had coronary artery bypass graft surgery. Which assessment does the nurse prioritize

for this client? a. Skin

b. Otoscopic c. Mental status

d. Gastrointestinal

The nurse is planning discharge education for a client after coronary artery bypass graft surgery. Which instruction does the nurse include in

this client’s teaching? a. “Remember to drink at least 3 liters of fluid daily.”

b. “You should abstain from sexual activity for 6 months.” c. “Take your pulse before, midway through, and after exercising.” d. “Stop taking your antihyperlipidemic medication at this time.”

MULTIPLE RESPONSE

The nurse is planning a community health promotion program for cardiovascular disease. Which risk factors of coronary artery disease (CAD) does the nurse include in the education? (Select all that apply.)

a. Cigarette smoking b. Use of alcohol

c. Insomnia d. Hypertension

e. Obesity f. Depression

The nurse is monitoring the electrocardiogram (ECG) of a client who has a myocardial infarction. Which changes does the nurse expect to see in

the ECG tracing? (Select all that apply.) a. ST-segment depression

b. T-wave inversion c. Normal Q waves

d. ST-segment elevation e. T-wave elevation

f. Abnormal Q wave

The nurse administers intravenous dobutamine (Dobutrex) to a client who has heart failure. Which clinical manifestations indicate that the

client’s status is improving? (Select all that apply.) a. Decreased heart rate

b. Increased heart rate c. Increased contractility d. Decreased contractility

e. Increased respiratory rate

A client is hospitalized after a myocardial infarction. Which hemodynamic parameters does the nurse correlate with cardiogenic

shock? (Select all that apply.) a. Decreased cardiac output b. Increased cardiac output

c. Increased mean arterial pressure (MAP) d. Decreased MAP

e. Increased afterload f. Decreased afterload

Chapter 41: Assessment of the Hematologic System

Test Bank

MULTIPLE CHOICE

The nurse helps to ambulate a client who has anemia. Which clinical manifestation indicates that the client is not tolerating the activity?

Blood pressure of 120/90 mm Hg Heart rate of 110 beats/min

Pulse oximetry reading of 95% Respiratory rate of 20 breaths/min

The nurse is assessing a client with liver failure. Which assessment is the highest priority for this client? Auscultation for bowel sounds

Assessing for deep vein thrombosis Monitoring of blood pressure hourly

Assessing for signs of bleeding

)

The nurse observes yellow-tinged sclera in a client with dark skin. Based on this assessment finding, what does the nurse do next?

Assess the client’s pulses. Examine the soles of the client’s feet.

Inspect the client’s hard palate.

Auscultate the client’s lung sounds.

The nurse is assessing a client with numerous areas of bruising. Which question does the nurse ask to determine the cause of this finding?

“Do you take aspirin?” “How often do you exercise?”

“Are you a vegetarian?” “How often do you take Tylenol?”

A client who has a chronic vitamin B12 deficiency is admitted to the hospital. When obtaining the client’s health history, which priority

question does the nurse ask this client? “Are you having any pain?”

“Are you having blood in your stools?” “Do you notice any changes in your memory?”

“Do you bruise easily?”

The nurse is planning discharge teaching for a client who has a splenectomy. Which statement does the nurse include in this client’s

teaching plan? “Avoid crowds and people who are sick.”

“Do not eat raw fruits or vegetables.” “Avoid environmental allergens.”

“Do not play contact sports.”

The nurse is teaching a client who has undergone a bone marrow biopsy. Which instruction does the nurse give the client?

“Wear protective gear when playing contact sports.” “Monitor the biopsy site for bruising.” “Remain in bed for at least 12 hours.”

“Use a heating pad for pain at the biopsy site.”

The nurse is assessing a 75-year-old male client. Which blood value indicates that the client is experiencing normal changes associated with

aging? Hemoglobin, 13.0 g/dL

Platelet count, 100,000/mm3 Prothrombin time (PT), 14 seconds

White blood cell (WBC) count, 5000/mm3

The nurse is planning care for a client who has a platelet count of 30,000/mm3. Which intervention does the nurse include in this client’s

plan of care? Oxygen by nasal cannula

Bleeding Precautions Isolation Precautions

Vital signs every 4 hours

The nurse is obtaining the health history of a client who has iron deficiency anemia. Which factor in this client’s history does the nurse

correlate with this diagnosis? Eating a meat-free diet

Family history of sickle cell disease History of leukemia

History of bleeding ulcer

The nurse is caring for a client who has a decreased serum iron level. Which intervention does the nurse prioritize for this client?

Dietary consult Family assessment

Cardiac assessment Administration of vitamin K

The nurse is caring for a client who has an elevated white blood cell count. Which intervention does the nurse implement for this client?

Administer the prescribed Tylenol. Hold the client’s prescribed steroids. Assess the client’s respiratory rate.

Obtain the client’s temperature.

A female client is admitted with the medical diagnosis of anemia. The nurse assesses for which potential cause?

Diet high in meat and fat Daily intake of aspirin

Heavy menses Smoking history

The nurse is assessing a client’s susceptibility to rejecting a transplanted kidney. Which result does the nurse recognize as increasing the client’s

chances of rejection? Decreased T-lymphocyte helper Decreased white blood cell count

Increased cytotoxic-cytolytic T cell Increased neutrophil count

The nurse is caring for a client who is receiving chemotherapy for cancer. Which intervention does the nurse implement for this client?

Assess the client’s fibrinogen level. Administer the prescribed iron.

Maintain strict Standard Precautions. Monitor the client’s pulse oximetry.

The nurse is performing an admission assessment on a 46-year-old client, who states, “I have been drinking a 12-pack of beer every day for the

past 20 years.” Which laboratory abnormality does the nurse correlate with this history?

Decreased white blood cell (WBC) count Decreased bleeding time

Elevated prothrombin time (PT) Elevated red blood cell (RBC) count

The nurse is assessing a client whose warfarin (Coumadin) therapy was discontinued 3 weeks ago. Which laboratory test result indicates that the

client’s warfarin therapy is no longer therapeutic? International normalized ratio (INR), 0.9

Reticulocyte count, 1% Serum ferritin level, 350 ng/mL

Total white blood cell (WBC) count, 9000/mm3

The nurse is completing the preoperative checklist on a client. The client states, “I take an aspirin every day for my heart.” How does the nurse

respond? “I will call your doctor and request a prescription for pain medication.”

“I need to call the surgeon and reschedule your surgery.” “I’ll give you the prescribed Tylenol to minimize any headache before

surgery.” “I need to administer vitamin K to prevent bleeding during the

procedure.”

The nurse is caring for a client who is receiving heparin therapy. How does the nurse evaluate the therapeutic effect of the therapy?

Evaluate platelets. Monitor the partial thromboplastin time (PTT).

Assess bleeding time. Monitor fibrin degradation products.

The nurse is administering a prescribed fibrinolytic to a client who is having a myocardial infarction (MI). Which adverse effect does the nurse

monitor for? Bleeding

Orthostatic hypotension Deep vein thrombosis

Nausea and vomiting

The nurse is caring for a client who had a bone marrow aspiration. The client begins to bleed from the aspiration site. Which action does the

nurse perform? Apply external pressure to the site.

Elevate the extremities. Cover the site with a dressing.

Immobilize the leg.

The nurse is preparing a client for a bone biopsy and aspiration. The client asks, “Will this be painful?” How does the nurse respond?

“The procedure is always done under general anesthesia.” “The biopsy lasts for only 2 minutes.”

“There is a chance that you may have pain.” “You can relieve pain with guided imagery.”

The nurse is caring for four clients with hematologic-type problems. Which client does the nurse prioritize to see first? 18-year-old female with decreased protein levels

36-year-old male with increased lymphocytes 60-year-old female with decreased erythropoietin

82-year-old male with an increased thromboxane level

The nurse is assessing the following laboratory results of a client before discharge. Which instruction does the nurse include in this client’s

discharge teaching plan? Test Result

Hemoglobin 15 g/dL

Hematocrit 45%

White blood cell (WBC) count 2000/mm3

Platelet count 250,000/mm3

“Avoid contact sports.” “Do not take any aspirin.” “Eat a diet high in iron.”

“Perform good hand hygiene.”

MULTIPLE RESPONSE

The nurse is monitoring a client with liver failure. Which assessments does the nurse perform when monitoring for bleeding in this client?

(Select all that apply.) Gums

Lung sounds Urine Stool Hair

The nurse is teaching a client who is receiving sodium warfarin (Coumadin). Which topics does the nurse include in the teaching plan?

(Select all that apply.) Foods high in vitamin K

Using acetaminophen (Tylenol) for minor pain Daily exercise and weight management

Use of a safety razor and soft toothbrush Blood testing regimen

Chapter 42: Care of Patients with Hematologic Problems

Test Bank

MULTIPLE CHOICE

The registered nurse is assigning a practical nurse to care for a client who has leukemia. Which instruction does the registered nurse provide

to the practical nurse when delegating this client’s care? a. Evaluate the amount of protein the client eats.

b. Assess the client’s roommate for symptoms of infection. c. Perform effective hand hygiene frequently.

d. Wear a mask when entering the room.

The nurse is teaching a client who has sickle cell disease and was admitted for splenomegaly and abdominal pain. Which instruction does

the nurse include in the client’s discharge teaching? a. “Avoid drinking large amounts of fluids.”

b. “Eat six small meals daily instead of large meals.” c. “Engage in aerobic exercise 3 days a week.”

d. “Receive a yearly influenza vaccination.”

The nurse is planning discharge teaching for a client who has acute myelogenous leukemia (AML). Which instruction does the nurse include

in this client’s discharge plan? a. Avoid contact sports.

b. Refrain from intercourse. c. Apply heat to any bruised areas.

d. Use aspirin for headaches.

The nurse is caring for a client who has autoimmune thrombocytopenic purpura. Which intervention does the nurse implement for this client?

a. Avoid intramuscular injections.

b. Administer prescribed anticoagulants. c. Infuse intravenous normal saline.

d. Monitor for an increase in temperature.

The nurse is teaching a client who is being discharged after stem cell transplantation. Which instruction does the nurse include in this client’s

discharge teaching? a. Eat a diet high in fruits and vegetables.

b. Ask your provider to administer a rubella vaccination. c. Wash your hands frequently.

d. Participate in physical therapy every day.

The nurse prepares to administer a blood transfusion to a client. Which means of identification does the nurse use to ensure that the blood is

administered to the correct client? a. Ask the client whether his or her name is the one on the blood

product tag. b. Ask the client’s spouse if the client is supposed to have a transfusion.

c. Compare the name and ID number on the blood product tag with the name and ID number on the client’s ID band.

d. Compare the unit and room number of the client with the unit and room number listed on the blood product tag.

The nurse is providing health promotion education to a client who has a family history of leukemia. Which factor does the nurse teach this client

to avoid? a. Alcohol consumption

b. Exposure to ionizing radiation c. High-cholesterol diet d. Smoking cigarettes

The nurse is assessing a client who has a factor VIII deficiency. Which clinical manifestation does the nurse expect to assess in this client?

a. Excessive bleeding from a cut b. Chronic lower back pain

c. Nausea and vomiting d. Temperature of 101° F

The nurse is planning care for a client who has leukemia. Which intervention does the nurse include in the plan of care to prevent fatigue?

a. Arrange for a family member to stay with the client. b. Plan care for times when the client has the most energy. c. Schedule for daily physicals and occupational therapy.

d. Plan all activities to occur in the morning to allow for afternoon naps.

The nurse is teaching a client with vitamin B12 deficiency anemia to eat a diet high in this vitamin. Which meal selected by the client indicates

that the client correctly understands the prescribed diet? a. Baked chicken breast, mashed potatoes, glass of milk

b. Eggplant parmesan, cottage cheese, iced tea c. Fried liver and onions, orange juice, spinach salad d. Fettuccine alfredo, green salad, glass of red wine

The nurse is teaching a client who has iron deficiency anemia. Which food choice indicates that the client correctly understands the teaching?

a. Chicken b. Oranges

c. Steak d. Tomatoes

The nurse assesses that a client has a smooth, beefy red tongue. Which intervention does the nurse implement for this client?

a. Administer prescribed oral iron supplements. b. Monitor the daily white blood cell count.

c. Provide a diet high in green leafy vegetables. d. Perform more frequent mouth care.

Which risk factor does the nurse assess for to determine a client’s cause of anemia?

a. Antacid therapy b. Chronic alcoholism

c. Congestive heart failure d. Type 2 diabetes

The nurse is teaching a client who is being discharged to home after bone marrow transplantation. The client asks, “Why is it so important to

protect myself from injury?” How does the nurse respond? a. “Injuries put you at high risk for infection.”

b. “Platelet recovery is slow, which makes you at risk for bleeding.”

c. “Severe trauma could result in rejection of the transplant.” d. “The medications you are taking will make you bruise easily.”

The nurse is preparing a client for surgery. The client states, “I am concerned I might be given blood products during surgery and this would be against my religious beliefs.” How does the nurse respond?

a. “We can use other means to replace blood loss besides blood products.”

b. “Your chance of needing a blood transfusion is small.” c. “The operating team will do what is necessary to save your life.”

d. “You could have family members donate blood for you.”

The nurse is caring for a 20-year-old man who has Hodgkin’s lymphoma in the abdominal and pelvic regions. The client is scheduled for radiation therapy and states, “I want to have children someday, and this procedure

will destroy my chances.” How does the nurse respond? a. “Adoption is always an option.”

b. “Infertility is not seen with this type of radiation therapy.” c. “Sperm production will be permanently disrupted.”

d. “You have the option to store sperm in a sperm bank.”

The nurse is preparing a client with leukemia for a peripheral stem cell transfusion. Which information does the nurse provide the client?

a. “Nausea and vomiting are common after the transfusion.” b. “The transfusion will take about 6 hours.”

c. “You may have numbness in your fingers and toes.” d. “Your urine may be red for a short time.”

The nurse is caring for a client during a sickle cell crisis. Which intervention does the nurse implement for the client? a. Administer acetaminophen (Tylenol) as needed.

b. Administer intravenous fluids to keep the vein open. c. Keep the room temperature at 80° F.

d. Transfuse red blood cells (RBCs).

The nurse is assessing a client with anemia. Which clinical manifestation does the nurse expect to see in this client?

a. Dyspnea with activity b. Hypertension c. Bradycardia

d. Warm, flushed skin

The nurse is transfusing red blood cells to a client who has sickle cell disease. Which laboratory result indicates that the nurse should

discontinue the transfusion? a. Hematocrit level (Hct), 32%

b. Hemoglobin S, 88% c. Serum iron level, 300 mcg/dL

d. Total white blood cell count, 12,000/mm3

A client is newly diagnosed with sickle cell anemia. Which information does the nurse include in the client’s discharge instructions?

a. “Eat a diet high in iron.” b. “Take hydroxyurea (Droxia) every morning.” c. “Be aware of the early symptoms of crisis.”

d. “Do not use any oral contraceptives.”

A client who has sickle cell anemia is admitted to the hospital. The client reports severe pain. Which action does the nurse take first?

a. Administer one unit of packed red blood cells. b. Administer prescribed hydroxyurea (Droxia).

c. Begin intravenous fluids at 250 mL/hr.

d. Prepare for bone marrow transplantation.

The nurse is teaching a client who was recently diagnosed with thrombocytopenia. Which instruction does the nurse include in this

client’s discharge teaching? a. “Drink at least 3 liters of fluid each day.”

b. “Use a soft-bristled toothbrush.” c. “Avoid blowing your nose.”

d. “Use only aspirin when having pain.”

The nurse is teaching a client who has myelodysplastic syndrome. Which instruction does the nurse include in this client’s teaching?

a. “Rise slowly when getting out of bed.” b. “Drink at least 3 liters of liquids per day.”

c. “Wear gloves and socks outdoors in cool weather.” d. “Use a soft-bristled toothbrush.”

The nurse observes that a client, whose blood type is AB-negative, is receiving a transfusion with type O-negative packed red blood cells.

Which action does the nurse take first? a. Report the problem to the blood bank.

b. Assess and record the client’s vital signs. c. Stop the transfusion and keep the IV open.

d. Administer prescribed diphenhydramine (Benadryl).

The nurse is assessing a client with a history of heart failure who is receiving a unit of packed red blood cells. The client’s respiratory rate is 33 breaths/min and blood pressure is 140/90 mm Hg. Which action does

the nurse take first? a. Administer prescribed diphenhydramine (Benadryl).

b. Continue to monitor the client’s vital signs. c. Stop the infusion of packed red blood cells. d. Slow the infusion rate of the transfusion.

A client who is receiving a unit of red blood cells begins to report chest and lower back pain. Which action does the nurse take first?

a. Administer morphine sulfate 1 mg IV. b. Assess the level of the pain.

c. Stop the transfusion.

d. Reposition the client on the right side.

The nurse is preparing to transfuse a third unit of red blood cells to a client. Which laboratory result is the nurse most concerned about?

a. Fibrinogen level less than 100 mg/dL b. Hematocrit of 30%

c. Potassium level of 5.5 mg/dL d. Serum ferritin level of 250 ng/mL

MULTIPLE RESPONSE

The nurse is preparing to administer transfusion therapy to a client. Which interventions does the nurse implement before starting the

infusion? (Select all that apply.) a. Confirm the client’s room number with the blood tag.

b. Check the client’s ABO and Rh types with the blood tag. c. Place a 20-gauge needle or larger in the client’s forearm.

d. Obtain the client’s pulse oximetry reading. e. Assess the client’s temperature.

The nurse is teaching a client who is scheduled to undergo allogeneic bone marrow transplantation. Which statements indicate that the client

correctly understands the teaching? (Select all that apply.) a. “The surgeon will insert the marrow into my femur bone.” b. “Until the marrow transplant takes, I can have visitors.”

c. “The transplant does not start working immediately.” d. “I will need chemotherapy before my transplant.”

e. “Radiation treatments will begin 2 days after transplantation.”

Chapter 43: Assessment of the Nervous System

Test Bank

MULTIPLE CHOICE

The nurse assesses a client who has trauma to the cerebrum. Which clinical manifestation does the nurse expect to observe?

a. Poor coordination b. Memory loss

c. Hyperthermia

d. Slurred speech

The nurse is assessing a client with a frontal lobe brain injury. Which clinical manifestation does the nurse expect to see?

a. Inability to interpret taste sensations b. Inability to interpret sound

c. Impaired judgment d. Impaired learning

The nurse is planning to provide discharge teaching related to cardiac medications to a client who has experienced damage to the left temporal

lobe of the brain. What does the nurse do to assist the client to understand the content of the instruction?

a. Use a larger print size for written materials. b. Ensure that the client is wearing glasses.

c. Point out the color of the medication. d. Sit on the client’s right side.

After performing a physical assessment on a 75-year-old client, the nurse notes that the client has a hypoactive response to a test of deep tendon

reflexes. Which intervention does the nurse include in this client’s plan of care?

a. Assist the client with ambulation. b. Elevate the client’s lower extremities.

c. Apply elastic support hose. d. Massage the client’s legs.

The nurse is discharging an 80-year-old client with diminished touch sensation. Which instruction does the nurse provide to promote client

safety? a. “Walk barefoot only in your home.”

b. “Bathe in warm water to increase your circulation.” c. “Look at the placement of your feet when walking.”

d. “Put throw rugs at the foot of your bed for cushioning.”

A client admitted the previous day for a suspected neurologic disorder becomes increasingly lethargic. Which is the best nursing action?

a. Promote a quiet atmosphere for sleep and rest to treat the client’s sleep deprivation.

b. Explain to the family that this is a normal age-related decline in mental processing.

c. Consult a psychiatrist to treat the client’s hospital-acquired depression.

d. Complete a full neurologic assessment and notify the neurologist.

The nurse is assessing a client’s remote memory. Which statement by the client confirms that remote memory is intact?

a. “Mary had a little lamb whose fleece was white as snow.” b. “I was born on April 3, 1967, in Johnstown Community Hospital.”

c. “Apple, chair, and pencil are the words you just stated.” d. “My sister brought me to the clinic for this appointment.”

During a neurologic examination, a client demonstrates a positive Romberg’s sign with eyes closed, but not with eyes open. Which

condition does the nurse associate with this finding? a. Difficulty with proprioception

b. Peripheral motor disorder c. Impaired cerebellar function

d. Positive pronator drift

The nurse is assessing the deep tendon reflexes of a client with long- standing diabetes mellitus. Which clinical manifestation does the nurse

expect to see? a. Bilateral hypoactive reflexes b. Bilateral hyperactive reflexes c. Asymmetric reflex response

d. Bilateral ankle clonus

During a neurologic assessment of a client, the nurse notes that the client’s arms, wrists, and fingers have become flexed, and internal

rotation and plantar flexion of the legs are evident. How does the nurse document these findings? a. Decorticate posturing b. Decerebrate posturing c. Atypical hyperreflexia

d. Spinal cord degeneration

The nurse is evaluating a client’s physical assessment with the medical history and treatment plan. The nurse notes that the client’s right pupil

appears dilated, with a sluggish pupillary response to light. Which disorder and related treatment does this physical finding correlate with?

a. Coronary artery disease and beta blockers

b. Diabetes mellitus and oral glycemic reducing agents c. Glaucoma and intraocular pressure–reducing eyedrops

d. Myopia and corrective laser surgery

Before electroencephalography, a client asks, “Why will I be asked to take deep breaths during the procedure?” How does the nurse respond? a. “Hyperventilation causes cerebral vasodilatation and increases the

likelihood of seizure activity.” b. “Hyperventilation causes cerebral vasoconstriction and increases

the likelihood of seizure activity.” c. “Deep breathing will keep you relaxed and will lower the seizure

threshold.” d. “Deep breathing will make you hypoxemic, which lowers the seizure

threshold.”

The nurse is caring for a client post-cerebral angiography via the client’s right femoral artery. Which intervention does the nurse implement?

a. Check the right lower extremity pulses. b. Measure orthostatic blood pressure. c. Perform a funduscopic examination.

d. Assess the client’s gag reflex.

The nurse is preparing a client for magnetic resonance angiography. Which question is a priority at this time?

a. “Have you had a recent blood transfusion?” b. “Do you have allergies to iodine or shellfish?”

c. “Do you have a history of urinary tract infections?” d. “Do you currently use oral contraceptives?”

The nurse is caring for a client who had a computed tomography (CT) scan of the head with contrast medium. Which priority intervention does

the nurse implement? a. Maintain bedrest with the head of the bed elevated less than 30

degrees. b. Apply a pressure dressing to the site of injection.

c. Increase fluid intake after the procedure. d. Maintain sedation for 8 hours postprocedure.

The nurse is obtaining the health history of a client scheduled for magnetic resonance imaging (MRI). Which condition requires the nurse

to cancel the MRI? a. Amputated leg

b. Internal insulin pump

c. Intrauterine device d. Atrioventricular (AV) graft

Which priority instruction or precaution does the nurse teach a client who is scheduled for a positron emission tomography scan of the brain? a. “Avoid caffeine-containing substances for 12 hours before the test.” b. “Drink at least 3 liters of fluid during the 24 hours after the test.” c. “Do not take your cardiac medication on the morning of the test.”

d. “Remove your dentures and any metal before the test begins.”

A female client with deteriorating neurologic function states, “I am worried I will not be able to care for my young children.” How does the

nurse respond? a. “Caring for your children is a priority. You may not want to ask for

help, but you have to.” b. “Our community has resources that may help you with some

household tasks so you have energy to care for your children.” c. “You seem distressed. Would you like to talk to a psychologist about

adjusting to your changing status?” d. “Give me more information about what worries you, so we can see if

we can do something to make adjustments.”

The nurse is planning care for an 83-year-old client with age-related changes to his sensory perception. Which nursing action does the nurse

implement to ensure the client’s safety? a. Provide a call button that requires only minimal pressure to

activate. b. Use a clock and a calendar to orient and minimize onset of

dementia. c. Ensure that the path to the bathroom is free from equipment.

d. Admit the client to the room closest to the nursing station.

A client is scheduled for a single-photon emission computed tomography test. Which condition in the client’s history causes the nurse to contact

the provider before the test takes place? a. Peptic ulcers

b. Smoking history c. Liver failure

d. Currently breast feeding

The nurse is teaching a client before magnetic resonance imaging (MRI). Which statement indicates that the client understands the content of the

education?

a. “I need to stay away from heavy metals for the next 48 hours.” b. “My urine will be radioactive for the next 48 hours.”

c. “I must increase my fluids because of the dye used for the MRI.” d. “I can return to my usual activities immediately after the MRI.”

While assessing pain discrimination, a client correctly identifies, with eyes closed, a sharp sensation on the right hand when touched with a pin.

How does the nurse then proceed with the examination? a. Touch the pin on the same area of the left hand.

b. Touch the pin on the right forearm. c. Touch the pin on the right upper arm.

d. Touch the right hand with a drop of cold water.

The nurse is assessing a client scheduled for a lumbar puncture. Which clinical manifestation assessed by the nurse complicates the lumbar

puncture procedure? a. Normal intracranial pressures b. Allergy to iodine or shellfish c. Restlessness and agitation

d. Eating lunch less than 2 hours ago

On assessment of the left plantar reflexes of an adult client, the nurse notes the response shown in the photograph below. What action does the

nurse take after assessing this new finding?

a. Relay this abnormal finding to other members of the health care team.

b. Anticipate the need for cerebral angiography to determine the cause.

c. Examine the family history for a potential genetic disorder. d. Document the finding and continue the assessment.

MULTIPLE RESPONSE

In a client with an injury to the medulla, the nurse monitors for which clinical manifestations secondary to damage of cranial nerves that

emerge from the medulla? (Select all that apply.) a. Loss of smell

b. Impaired swallowing c. Blink reflex

d. Visual changes

e. Inability to shrug shoulders f. Loss of gag reflex

The nurse is assessing a client with a temporal lobe injury. Which clinical manifestations correlate with this injury? (Select all that apply.)

a. Memory loss b. Personality changes

c. Loss of temperature regulation d. Difficulty with sound interpretation

e. Speech difficulties f. Impaired taste

The nurse is administering a medication to a client that stimulates the sympathetic division of the autonomic nervous system. Which clinical

manifestations does the nurse monitor for? (Select all that apply.) a. Decreased heart rate b. Increased heart rate

c. Decreased force of contraction d. Increased force of contraction

e. Decreased respirations

COMPLETION

Immediately after a lumbar puncture, the client begins to vomit and an IV is started with normal saline (0.9% NS). The provider orders a 200- mL bolus over 15 minutes. Using an infusion pump that delivers mL/hr,

the rate at which the nurse sets the pump is mL.

Chapter 44: Care of Patients with Problems of the Central Nervous System: The Brain

Test Bank

MULTIPLE CHOICE

The nurse is caring for a client experiencing migraine headaches who is receiving a beta blocker to help manage this disorder. When preparing a

teaching plan, which instruction does the nurse plan to provide? a. “Take this drug only when you have prodromal symptoms

indicating the onset of a migraine headache.” b. “Take this drug as ordered, even when feeling well, to prevent

vascular changes associated with migraine headaches.”

c. “This drug will relieve the pain during the aura phase soon after a headache has started.”

d. “This medication will have no effect on your heart rate or blood pressure because you are taking it for migraines.”

The nurse is assessing a client with a history of migraines. Which clinical manifestation is an early sign of a migraine with aura?

a. Vertigo b. Lethargy

c. Visual disturbances d. Numbness of the tongue

The nurse is reviewing a client’s prescription for sumatriptan succinate (Imitrex). Which condition in this client’s medical history does the nurse

report to the health care provider? a. Bronchial asthma

b. Gonorrhea c. Prinzmetal’s angina

d. Chronic kidney disease

The nurse is assessing a client with a cluster headache. Which clinical manifestation does the nurse expect to find?

a. Ipsilateral tearing of the eye b. Exophthalmos

c. Abrupt loss of consciousness d. Neck and shoulder tenderness

A client with epilepsy develops stiffening of the muscles of the arms and legs, followed by an immediate loss of consciousness and jerking of all

extremities. How does the nurse document this seizure activity? a. Atonic seizure

b. Absence seizure c. Myoclonic seizure

d. Tonic-clonic seizure

The nurse is assessing a client with a history of absence seizures. Which clinical manifestation does the nurse assess for?

a. Automatisms b. Intermittent rigidity

c. Sudden loss of muscle tone d. Brief jerking of the extremities

The nurse is caring for a client with a history of epilepsy who suddenly begins to experience a tonic-clonic seizure and loses consciousness. What

is the nurse’s priority action? a. Restrain the client’s extremities. b. Turn the client’s head to the side. c. Take the client’s blood pressure.

d. Place an airway into the client’s mouth.

A client is actively experiencing status epilepticus. Which prescribed medication does the nurse prepare to administer?

a. Atropine b. Lorazepam (Ativan) c. Phenytoin (Dilantin)

d. Morphine sulfate

A client with new-onset status epilepticus is prescribed phenytoin (Dilantin). After teaching the client about this treatment regimen, the

nurse assesses the client’s understanding. Which statement indicates that the client understands the teaching?

a. “I must drink at least 2 liters of water daily.”

b. “This will stop me from getting an aura before a seizure.” c. “I will not be able to be employed while taking this medication.”

d. “Even when my seizures stop, I will take this drug.”

The nurse is teaching a client who is newly diagnosed with epilepsy. Which statement by the client indicates a need for further teaching

concerning the drug regimen? a. “I will not drink any alcoholic beverages.”

b. “I will wear a medical alert bracelet.” c. “I will let my doctor know about all of my prescriptions.”

d. “I can skip a couple of pills if they make me ill.”

The nurse assesses for which clinical manifestations in the client with suspected encephalitis?

a. Fever of 101° F (38.3° C) b. Nausea and vomiting

c. Hypoactive deep tendon reflexes d. Pain on flexion of the neck

The nurse is taking the health history of a client suspected of having bacterial meningitis. Which question is most important for the nurse to

ask? a. “Do you live in a crowded residence?”

b. “When was your last tetanus vaccination?” c. “Have you had any viral infections recently?”

d. “Have you traveled out of the country in the last month?”

The nurse is talking to the family of a client who has Parkinson’s disease. Which statement indicates that the family has a good understanding of

the changes in motor movement associated with this disease? a. “I can never tell what she’s thinking. She hides behind a frozen

face.” b. “She drools all the time so I just can’t take her out anywhere.”

c. “I think this disease makes her nervous. She perspires all the time.” d. “She has trouble chewing so I will offer bite-sized portions.”

The nurse is caring for a client with Parkinson’s disease. Which intervention does the nurse implement to prevent respiratory

complications in the client? a. Keep an oral airway at the bedside.

b. Ensure fluid intake of at least 3 L/day.

c. Teach the client pursed-lip breathing techniques. d. Maintain the head of the bed at 30 degrees or greater.

The daughter of a client with Alzheimer’s disease asks, “Will the medication my mother is taking improve her dementia?” How does the

nurse respond? a. “It will help your mother live independently once more.”

b. “It is used to halt the advancement of Alzheimer’s disease but will not cure it.”

c. “It will provide a steady improvement in memory but not in problem solving.”

d. “It will not improve dementia but can help control emotional responses.”

A client with Alzheimer’s disease is admitted to the hospital. Which psychosocial assessment is most important for the nurse to complete?

a. Ability to recall past events b. Ability to perform self-care

c. Reaction to a change of environment d. Relationship with close family members

The nurse is caring for a hospitalized client with Alzheimer’s disease who has a history of agitation. Which intervention does the nurse implement

to help prevent agitation and aggressive behavior in this client? a. Provide undisturbed sleep. b. Orient the client to reality.

c. Leave the television turned on. d. Administer hypnotic drugs as needed.

A hospitalized client with late-stage Alzheimer’s disease says that breakfast has not been served. The nurse witnessed the client eating

breakfast earlier. Which statement made to this client is an example of validation therapy?

a. “I see you are still hungry. I will get you some toast.” b. “You are confused about mealtimes this morning.”

c. “You ate your breakfast 30 minutes ago.” d. “You look tired. Maybe a nap will help.”

A client is prescribed levetiracetam (Keppra). Which laboratory tests does the nurse monitor for potential adverse effects of this medication?

a. Serum electrolyte levels b. Kidney function tests

c. Complete blood cell count

d. Antinuclear antibodies

The caregiver of a client with advanced Alzheimer’s disease states, “She is always wandering off. What can I do to manage this restless

behavior?” How does the nurse respond? a. “Allow for a 45-minute daytime nap.”

b. “Take the client for frequent walks throughout the day.” c. “Using a Geri-chair may decrease agitation.”

d. “Give a mild sedative during periods of restlessness.”

A client who has Alzheimer’s disease is being discharged home. What safety instructions does the nurse include in the teaching plan for the

client’s caregiver? a. “Keep exercise to a minimum.”

b. “Place a padded throw rug at the bedside.” c. “Install deadbolt locks on all outside doors.” d. “Keep the lights off in the bedroom at night.”

The nurse is assessing a client with Huntington’s disease. Which motor

changes does the nurse monitor for in this client? a. Shuffling gait

b. Jerky hand movements c. Continuous chewing motions

d. Tremors of the hands during fine motor tasks

The nurse is planning to bathe a client diagnosed with meningococcal meningitis. In addition to gloves, what personal protective equipment

does the nurse use? a. Particulate respirator

b. Isolation gown c. Shoe covers

d. Surgical mask

A client diagnosed with the Huntington gene but who has no symptoms asks for options related to family planning. Which is the nurse’s best

response? a. “Most clients with the Huntington gene do not pass on Huntington

disease to their children.” b. “I understand that they can diagnose this disease in embryos.

Therefore you could select a healthy embryo from your fertilized eggs for implantation to avoid passing on Huntington disease.”

c. “The need for family planning is limited because one of the hallmarks of Huntington disease is infertility.”

d. “Tell me more specifically what information you need about family planning so that I can direct you to the right information or health care

provider.”

The nurse is caring for a client who has chronic migraine headaches. Which complementary health therapy does the nurse suggest? a. “Place a hot compress on your forehead at the onset of the

headache.” b. “Wear dark sunglasses when you are in brightly lit spaces.”

c. “Lie down in a darkened room when you experience a headache.” d. “Do not sleep longer than 6 hours at one time.”

MULTIPLE RESPONSE

The nurse is planning care for a client with epilepsy. Which precautions does the nurse implement to ensure the safety of the client while in the

hospital? (Select all that apply.) a. Have suction equipment at the bedside. b. Place a padded tongue at the bedside.

c. Permit only clear oral fluids.

d. Keep bed rails up at all times. e. Maintain the client on strict bedrest. f. Ensure that the client has IV access.

The nurse is teaching a client with chronic headaches about headache triggers. Which statements does the nurse include in the client’s teaching

plan? (Select all that apply.) a. “Increase your intake of caffeinated beverages.” b. “Increase your intake of fruits and vegetables.”

c. “Avoid all alcoholic beverages.” d. “Avoid drinking red wine.”

e. “Incorporate physical exercise into your daily routine.” f. “Incorporate an occasional fast into your plan.”

The nurse is assessing the results of diagnostic tests on a client’s cerebrospinal fluid (CSF). Which values and observations does the nurse

correlate as most indicative of viral meningitis? (Select all that apply.) a. Clear

b. Cloudy c. Normal protein level

d. Increased protein level e. Normal glucose level

f. Decreased glucose level

Chapter 45: Care of Patients with Problems of the Central Nervous System: The Spinal Cord

Test Bank

MULTIPLE CHOICE

The nurse is providing health education at a community center. Which instruction does the nurse include as part of client education for the

prevention of low back pain? a. “Participate in a regular exercise program.”

b. “Purchase a soft mattress for sleeping comfort.” c. “Wear high-heeled shoes only for special occasions.”

d. “Keep your weight within 20% of your ideal body weight.”

The nurse is caring for a client who has low back pain (LBP) from a work-related injury. Which measures does the nurse incorporate into the

client’s plan of care? a. Apply moist heat continuously to the affected area.

b. Use ice packs or ice massage for 1 to 2 hours over the affected area. c. Apply heat packs for 20 to 30 minutes at least four times daily.

d. Advise the client to avoid hot baths or showers.

A client who has a herniated disk is being discharged after a percutaneous endoscopic discectomy. Which postprocedure instructions

does the nurse provide before discharge? a. “You should begin an exercise routine which includes walking every

day.” b. “You must sleep in a supine position until the bandage is removed.”

c. “You may feel numbness or tingling in the legs for 24 hours.” d. “You will need to wear a lumbar brace for 1 week.”

The nurse is assessing a client who had a discectomy 6 hours ago. Which client complaint requires priority action by the nurse?

a. “I am feeling tired.” b. “My mouth is so dry.”

c. “I can’t seem to relax and rest.” d. “I am unable to urinate.”

The nurse is providing discharge teaching to a client after a lumbar laminectomy. For which complication does the nurse instruct the client

to return to the hospital? a. Pain at the incision site

b. Decreased appetite c. Slight redness and itching at the incision site

d. Clear drainage from the incision site

The nurse is caring for a client who has undergone a spinal fusion. Which specific postoperative instructions does the nurse give this client?

a. “You may lift items up to 10 pounds.” b. “Wear your brace when you are out of bed.”

c. “You must remain on bedrest for 48 hours after surgery.” d. “You will need to take steroids to prevent rejection of the bone

graft.”

A client who suffered a spinal cord injury at level T5 several months ago develops a flushed face and blurred vision. On taking vital signs, the

nurse notes the blood pressure to be 184/95 mm Hg. Which is the nurse’s first action?

a. Palpate the area over the bladder for distention. b. Place the client in the Trendelenburg position.

c. Administer oxygen via a nasal cannula. d. Perform bilateral carotid massage.

Emergency medical services arrive to the emergency department with a client who has a cervical spinal cord injury. Which priority assessment

does the emergency department nurse perform at this time? a. Level of consciousness and orientation

b. Heart rate and rhythm c. Muscle strength and reflexes

d. Respiratory pattern and airway

The nurse is caring for a client who has a vertebral fracture. Which intervention does the nurse implement to prevent deterioration of the

client’s neurologic status? a. Reorient the client to time, place, and person.

b. Administer the Mini-Mental State Examination.

c. Immobilize the affected portion of the spinal column. d. Reposition the client every 2 hours.

A client who experienced a spinal cord injury 1 hour ago is brought to the emergency department. Which prescribed medication does the nurse

prepare to administer to this client? a. Intrathecal baclofen (Lioresal) b. Methylprednisolone (Medrol)

c. Atropine sulfate d. Epinephrine (Adrenalin)

The nurse is assessing a client with a spinal cord injury at the T5 level. Which clinical manifestation alerts the nurse to the presence of a

complication of this injury? a. Rhinorrhea and epiphora

b. Fever and cough c. Agitation and restlessness

d. Hip and knee pain

The nurse notes reddened areas over the hips and sacrum of a client with paraplegia from a spinal cord injury. Which action does the nurse

implement? a. Massage the reddened areas with a barrier cream.

b. Perform hip flexion and extension range-of-motion (ROM) exercises.

c. Reposition the client so that the reddened area does not bear weight. d. Ensure that the client sits in a chair at least once each shift.

The nurse is caring for a client with a lower motor neuron lesion who wishes to achieve bladder control. Which intervention does the nurse

implement to effectively stimulate the initiation of voiding for this client? a. Stroking the inner aspect of the thigh

b. Intermittent catheterization c. Digital anal stimulation d. The Valsalva maneuver

A client who has a lower motor neuron injury experiences a flaccid bowel elimination pattern. Which action does the nurse implement to assist in

relieving this client’s constipation? a. Pouring warm water over the perineum b. Tapping the abdomen from left to right c. Administering daily tap water enemas

d. Implementing a consistent daily time for elimination

A client with paraplegia is scheduled to participate in a rehabilitation program. The client states, “I do not understand the need for

rehabilitation; the paralysis will not go away and it will not get better.” How does the nurse respond?

a. “If you do not want to participate in the rehabilitation program, I will cancel the order.”

b. “Your doctor has helped many clients with your injury and has ordered a rehabilitation program to help you.”

c. “The rehabilitation program will teach you how to maintain the functional ability you have and prevent further disability.”

d. “When new discoveries are made regarding paraplegia, people in rehabilitation programs will benefit first.”

The nurse is teaching a client who has a spinal cord injury how to prevent respiratory problems at home. Which statement indicates that

the client correctly understands the teaching? a. “I will use my incentive spirometer every 2 hours while I’m awake.”

b. “I will not drink thick fluids to prevent choking.” c. “I will take cough medicine to prevent excessive coughing.” d. “I will position myself on my right side so I don’t aspirate.”

The nurse assesses for which clinical manifestation in a client with multiple sclerosis (MS) of the relapsing type?

a. Absence of periods of remission b. Attacks becoming increasingly frequent c. Absence of active disease manifestations

d. Gradual neurologic symptoms without remission

The nurse is assessing a client with an early onset of multiple sclerosis (MS). Which clinical manifestation does the nurse expect to see?

a. Hyperresponsive reflexes b. Excessive somnolence

c. Nystagmus d. Heat intolerance

A client presents with an acute exacerbation of multiple sclerosis. Which prescribed medication does the nurse prepare to administer?

a. Baclofen (Lioresal) b. Interferon beta-1b (Betaseron) c. Dantrolene sodium (Dantrium) d. Methylprednisolone (Medrol)

A client with multiple sclerosis is being treated with fingolimod (Gilenya). Which clinical manifestation alerts the nurse to an adverse

effect of this medication? a. Periorbital edema b. Black tarry stools

c. Bradycardia d. Vomiting after meals

The nurse is preparing a client who has multiple sclerosis (MS) for discharge home from a rehabilitation center. The client has been prescribed cyclophosphamide (Cytoxan) and methylprednisolone

(Medrol). Which instruction does the nurse include in the teaching plan for the client?

a. “Take warm baths to promote muscle relaxation.” b. “Avoid crowds and people with colds.”

c. “Use physical aids such as walkers as little as possible.” d. “Stop using these medications when your symptoms improve.”

Early manifestations of amyotrophic lateral sclerosis (ALS) and multiple

sclerosis (MS) are somewhat similar. Which clinical feature of ALS distinguishes it from MS?

a. Dysarthria b. Dysphagia

c. Muscle weakness d. Impairment of respiratory muscles

Which neurologic test or procedure requires the nurse to determine whether an informed consent has been obtained from the client before

the test or procedure? a. Measurement of sensation using the pinprick method

b. Computed tomography of the cranial vault c. Lumbar puncture for cerebrospinal fluid (CSF) sampling

d. Venipuncture for autoantibody analysis

A client is scheduled for magnetic resonance imaging (MRI). Which action does the nurse implement before the test?

a. Ensure that the person does not eat for 8 hours before the procedure.

b. Discontinue all neuroactive medications 3 hours before the procedure.

c. Make sure that the client has an identification bracelet that cannot be removed.

d. Replace the client’s gown with metal snaps with one that has cloth

ties.

The nurse is teaching a client who has an unstable thoracic vertebral fracture and is being treated with immobilization before surgery. Which

statement does the nurse include in the client’s teaching? a. “You will need to apply an immobilizing brace snugly around your

waist when out of bed.” b. “You will remain strapped to the transport back board until the

surgical room is ready.” c. “Keep your spine in alignment by not sitting up, arching your back,

or twisting in bed.” d. “An incentive spirometer will prevent you from having atelectasis

and pneumonia after surgery.”

The nurse is planning care for a client who has a spinal cord injury. Which interdisciplinary team member does the nurse consult with to

assist the client with activities of daily living? a. Social worker

b. Physical therapist c. Occupational therapist

d. Case manager

The nurse is discussing advanced directives with a client who has amyotrophic lateral sclerosis (ALS). The client states, “I do not want to

be placed on a mechanical ventilator.” How does the nurse respond? a. “You will need to discuss that with your family and health care

provider.” b. “Why are you afraid of being placed on a breathing machine?” c. “What would you like to be done if you begin to have difficulty

breathing?” d. “You will be on the ventilator only until your muscles get stronger.”

MULTIPLE RESPONSE

The nurse is assessing a client’s coping strategies after suffering a traumatic spinal cord injury. Which information related to this

assessment is important for the nurse to obtain? (Select all that apply.) a. Spiritual or religious beliefs

b. Level of pain c. Family support

d. Level of independence e. Annual income

f. Previous coping strategies

The nurse is teaching a client with a spinal cord tumor about the treatment plan. Which statements indicate that the client correctly

understands the teaching? (Select all that apply.) a. “Because my symptoms occurred so quickly, I am likely to be cured

quickly by surgery.” b. “Radiation therapy can shrink the tumor but radiation can cause

more problems, too.” c. “I am glad you are here to turn me. Lying in one position for a long

time makes my pain worse, even if turning is uncomfortable.” d. “I have put my affairs in order and purchased a burial plot because

this type of cancer is almost always fatal.” e. “My family is making some changes at home for me, including

moving my bedroom downstairs.”

The nurse is teaching a male client with a spinal cord injury at T4 (thoracic) about the sexual effects of this injury. Which statement by the

client indicates correct understanding of the teaching? (Select all that apply.)

a. “I will not be able to have an erection because of my injury.” b. “Ejaculation may not be as predictable as before.”

c. “I will explore other ways besides intercourse to please my partner.” d. “I may urinate with ejaculation but this will not cause an infection.”

e. “I should be able to have an erection with stimulation.”

Chapter 46: Care of Patients with Problems of the Peripheral Nervous System

Test Bank

MULTIPLE CHOICE

The nurse recognizes which pathophysiologic feature as a hallmark of Guillain-Barré syndrome?

a. Nerve impulses are not transmitted to skeletal muscle. b. The immune system destroys the myelin sheath.

c. The distal nerves degenerate and retract. d. Antibodies to acetylcholine receptor sites develop.

The nurse assesses a client who has Guillain-Barré syndrome. Which clinical manifestation does the nurse expect to find in this client?

a. Ophthalmoplegia and diplopia b. Progressive weakness without sensory involvement

c. Progressive, ascending weakness and paresthesia d. Weakness of the face, jaw, and sternocleidomastoid muscles

The nurse reviews laboratory data for a client who has Guillain-Barré syndrome (GBS). Which result does the nurse correlate with this disease

process? a. Increased cerebrospinal fluid (CSF) protein level b. Decreased serum protein electrophoresis results

c. Increased antinuclear antibodies d. Decreased immune globulin G (IgG) levels

The intensive care nurse is caring for a client who has Guillain-Barré syndrome. The nurse notes that the client’s vital capacity has declined to 12 mL/kg, and the client is having difficulty clearing secretions. Which is

the nurse’s priority action? a. Place the client in a high Fowler’s position. b. Prepare the client for elective intubation. c. Administer oxygen via a nasal cannula.

d. Auscultate for breath sounds.

A client who has Guillain-Barré syndrome is scheduled for plasmapheresis. Before the procedure, which clinical manifestation does the nurse use to determine patency of the client’s arteriovenous shunt?

a. Palpable distal pulses b. A pink, warm extremity c. The presence of a bruit

d. Shunt pressure higher than 25 mm Hg

The nurse assesses a client with Guillain-Barré syndrome during plasmapheresis. Which complication does the nurse monitor for during

this procedure? a. Tachycardia b. Hypovolemia c. Hyperkalemia d. Hemorrhage

The nurse teaches a client with Guillain-Barré syndrome (GBS) about the recovery rate of this disorder. Which statement indicates that the

client correctly understands the teaching? a. “I need to see a lawyer because I do not expect to recover from this

disease.” b. “I will have to take things slowly for several months after I leave the

hospital.”

c. “I expect to be able to return to work in construction soon after I get discharged.”

d. “I wonder if my family will be able to manage my care now that I am paralyzed.”

The nurse assesses a client who has myasthenia gravis. Which clinical manifestation does the nurse expect to observe in this client?

a. Inability to perform the six cardinal positions of gaze b. Lateralization to the affected side during the Weber test

c. Absent deep tendon reflexes d. Impaired stereognosis

The nurse is assessing laboratory results for a client with myasthenia gravis (MG). Which results does the nurse correlate with this disease

process? a. Elevated serum calcium level

b. Decreased thyroid hormone level c. Decreased complete blood count

d. Elevated acetylcholine receptor antibody levels

A client suspected to have myasthenia gravis is scheduled for the Tensilon (edrophonium chloride) test. Which prescribed medication does

the nurse prepare to administer if complications of this test occur? a. Epinephrine

b. Atropine sulfate c. Diphenhydramine

d. Neostigmine bromide

The nurse is caring for a client who has myasthenia gravis. Which nursing intervention does the nurse implement to reduce muscle

weakness in this client? a. Administer a therapeutic massage.

b. Collaborate with the physical therapist. c. Perform passive range-of-motion exercises.

d. Reposition the client every 2 hours.

The nurse is assessing a client who is experiencing a myasthenia crisis. Which diagnostic test does the nurse anticipate being ordered?

a. Babinski reflex test b. Tensilon test

c. Cholinesterase challenge test

d. Caloric reflex test

A client who has myasthenia gravis is receiving atropine for a cholinergic crisis. Which intervention does the nurse implement for this client?

a. Suction the client to remove secretions. b. Turn and reposition the client every 2 hours. c. Measure urinary output every 30 minutes.

d. Administer prescribed anticholinergic drugs as needed.

The nurse instructs a client who has myasthenia gravis to take prescribed medications on time and to eat meals 45 to 60 minutes after taking anticholinesterase drugs. The client asks why the timing of meals

is so important. Which is the nurse’s best response? a. “This timing allows the drug to have maximum effect, so it is easier

for you to chew, swallow, and not choke.” b. “This timing prevents your blood sugar level from dropping too low

and causing you to be at risk for falling.” c. “These drugs are very irritating to your stomach and could cause

ulcers if taken too long before meals.” d. “These drugs cause nausea and vomiting. By waiting a while after

you take the medication, you are less likely to vomit.”

A client who has myasthenia gravis is recovering after a thymectomy. Which complication does the nurse monitor for in this client?

a. Sudden onset of shortness of breath b. Swelling of the lower extremities

c. Lower abdominal tenderness d. Decreased urinary output

A client with myasthenia gravis is preparing for discharge. Which instructions does the nurse include when educating the client’s family

members or caregiver? a. Technique for therapeutic massage to the lower extremities

b. Administration of morphine sulfate via an IV pump c. Instructions for preparing thin, puréed foods

d. Cardiopulmonary resuscitation (CPR)

The nurse teaches a client who has autonomic dysfunction about injury prevention. Which statement indicates that the client correctly

understands the teaching? a. “I will change positions slowly.”

b. “I will avoid wearing cotton socks.” c. “I will use an electric razor.”

d. “I will use a heating pad on my feet.”

The nurse is planning discharge teaching for a client who has peripheral neuropathy of the lower extremities. Which instruction does the nurse

include in the teaching plan? a. “Cut all calluses and corns from your feet as soon as you notice

them.” b. “Your balance will be steadier if you go barefoot while at home.”

c. “Use a thermometer to check the temperature of bath water.” d. “Avoid using lotion on the feet and legs.”

The nurse is caring for a client who has undergone peripheral nerve repair. Which priority assessment does the nurse perform

postoperatively? a. Evaluate extremity mobility.

b. Assess the skin surrounding the cast. c. Test distal extremities for sensation.

d. Auscultate bowel sounds.

The nurse is assessing a client with trigeminal neuralgia. Which clinical manifestation does the nurse expect to observe?

a. Excruciating pain b. Decreased mobility

c. Controllable facial twitching d. Increased talkativeness

The nurse is assessing a client who had a dissection of all branches of the right trigeminal nerve. When asked to wrinkle his forehead, the client

wrinkles only the left side. Which is the nurse’s best action? a. Place the client in high Fowler’s position.

b. Document the finding. c. Assess the corneal reflex.

d. Notify the health care provider.

A client with trigeminal neuralgia is about to undergo surgery for pain relief. The client asks, “How will this surgery relieve my pain?” How

does the nurse respond? a. “The surgeon will cut the connection between the cranial nerves.” b. “The surgeon will use an electrode to bypass the trigeminal nerve

conduction.” c. “An incision is made into the nerve itself, and an anesthetic is

applied to the area.” d. “A small artery compressing the nerve will be relocated.”

The nurse is teaching a client who is receiving carbamazepine (Tegretol) for chronic trigeminal neuralgia. Which statement indicates that the

client correctly understands the teaching? a. “This drug will prevent seizures, which can occur because of

trigeminal disease.” b. “I expect to have surgery soon, so I can stop taking this drug now.” c. “This medication is very successful in relieving pain. I am glad to be

taking it.” d. “I will avoid drinking alcohol because it can add to the side effects of

this medicine.”

The nurse teaches a client who has Guillain-Barré syndrome (GBS) about pain management. Which statement indicates that the client

correctly understands the teaching? a. “I can use the button on the pump as often as I want to get more

pain medication.” b. “Aspirin will provide the best relief from my pain associated with

this disease.” c. “A combination of morphine and distraction helps bring me relief

right now.” d. “I should not have any pain as a result of impaired motor and

sensory neurons.”

The nurse is obtaining a health history for a 45-year-old woman with Guillain-Barré syndrome (GBS). Which statement by the client does the

nurse correlate with the client’s diagnosis? a. “My neighbor also had Guillain-Barré syndrome.”

b. “I had a viral infection about 2 weeks ago.” c. “I am an artist and work with oil paints.”

d. “I have a history of a cardiac dysrhythmia.”

MULTIPLE RESPONSE

A client has just undergone surgery for peripheral nerve trauma. Which interventions does the nurse include in the client’s plan of care? (Select

all that apply.) a. Immobilization of the affected area with a splint

b. Rotation of cold and heat therapy c. Occupational therapy

d. Skin care, including hygiene and ointments e. High-fat, low-protein diet

The nurse is preparing a staff in-service program related to restless legs syndrome (RLS). Which potential risk factors of this syndrome does the

nurse include? (Select all that apply.) a. Skin rashes

b. Polyneuropathies c. Muscle atrophy

d. Diabetes mellitus type 2 e. Hypercalcemia

The nurse is preparing to send a cerebrospinal fluid sample to the laboratory. Which actions does the nurse implement during this

procedure? (Select all that apply.) a. Use Standard Precautions.

b. Wear sterile gloves when handling the specimen. c. Place the specimen on ice.

d. Send the specimen in a sealed bag displaying a biohazard symbol. e. Confirm the specimen label with the client’s identification band.

Chapter 47: Care of Critically Ill Patients with Neurologic Problems

Test Bank

MULTIPLE CHOICE

The nurse is obtaining a health history for a client admitted to the hospital after experiencing a brain attack. Which disorder does the nurse

identify as a predisposing factor for an embolic stroke? a. Seizures

b. Psychotropic drug use c. Atrial fibrillation

d. Cerebral aneurysm

A client with aphasia presents to the emergency department with a suspected brain attack. Which clinical manifestation leads the nurse to

suspect that this client has had a thrombotic stroke? a. Two episodes of speech difficulties in the last month

b. Sudden loss of motor coordination c. A grand mal seizure 2 months ago

d. Chest pain and nuchal rigidity

The nurse is caring for an 80-year-old client who presented to the emergency department in a coma. Which question does the nurse ask the client’s family to help determine whether the coma is related to a brain

attack? a. “How many hours does your mother usually sleep at night?” b. “Did your mother complain recently of weakness in her lower

extremities?” c. “Is any history of seizures known among your mother’s immediate

family?” d. “Does your mother drink any alcohol or take any medications?”

The nurse is assessing a client who had a stroke in the right cerebral hemisphere. Which neurologic deficit does the nurse assess for in this

client? a. Impaired proprioception

b. Aphasia c. Agraphia

d. Impaired olfaction

A client who had a stroke combs her hair only on the right side of her head and washes only the right side of her face. How does the nurse

interpret these actions? a. Poor left-sided motor control

b. Paralysis or contractures on the right side

c. Limited visual perception of the left fields d. Unawareness of the existence of her left side

The nurse notes that the left arm of a client who has experienced a brain attack is in a contracted, fixed position. Which complication of this

position does the nurse monitor for in this client? a. Shoulder subluxation b. Flaccid hemiparesis c. Pathologic fracture d. Neglect syndrome

The nurse is caring for a client who has experienced a stroke. Which nursing intervention for nutrition does the nurse implement to prevent

complications from cranial nerve IX impairment? a. Turn the client’s plate around halfway through the meal.

b. Place the client in high Fowler’s position. c. Order a clear liquid diet for the client.

d. Verbalize the placement of food on the client’s plate.

A client who had a brain attack was admitted to the intensive care unit yesterday. The nurse observes that the client is becoming lethargic and is unable to articulate words when speaking. What does the nurse do next?

a. Check the client’s blood pressure and apical heart rate. b. Elevate the back rest to 30 degrees and notify the health care

provider. c. Place the client in a supine position with a flat back rest, and

observe. d. Assess the client’s white blood cell count and differential.

The nurse is caring for a client who had a stroke. Which nursing intervention does the nurse implement during the first 72 hours to

prevent complications? a. Administer prescribed analgesics to promote pain relief.

b. Cluster nursing procedures together to avoid fatiguing the client. c. Monitor neurologic and vital signs closely to identify early changes

in status. d. Position with the head of the bed flat to enhance cerebral perfusion.

A client who first experienced symptoms related to a confirmed thrombotic stroke 2 hours ago is brought to the intensive care unit. Which prescribed medication does the nurse prepare to administer?

a. Tissue plasminogen activator b. Heparin sodium

c. Gabapentin (Neurontin) d. Warfarin (Coumadin)

A client who had a stroke is receiving clopidogrel (Plavix). Which adverse effect does the nurse monitor for in this client?

a. Repeated syncope b. New-onset confusion

c. Spontaneous ecchymosis d. Abdominal distention

The nurse is caring for a client who is immobile from a recent stroke. Which intervention does the nurse implement to prevent complications

in this client? a. Position the client with the unaffected side down.

b. Apply sequential compression stockings. c. Instruct the client to turn the head from side to side.

d. Teach the client to touch and use both sides of the body.

A client has experienced a stroke resulting in damage to Wernicke’s area.

Which clinical manifestation does the nurse monitor for? a. Inability to comprehend spoken words b. Communication with rote speech only

c. Slurred speech d. Inability to make sounds

A client who has had a stroke with left-sided hemiparesis has been referred to a rehabilitation center. The client asks, “Why do I need

rehabilitation?” How does the nurse respond? a. “Rehabilitation will reverse any physical deficits caused by the

stroke.” b. “If you do not have rehabilitation, you may never walk again.”

c. “Rehabilitation will help you function at the highest level possible.” d. “Your doctor knows best and has ordered this treatment for you.”

The nurse is teaching bladder training to a client who is incontinent after a stroke. Which instruction does the nurse include in this client’s

teaching? a. “Decrease your oral intake of fluids to 1 liter per day.” b. “Use a Foley catheter at night to prevent accidents.”

c. “Plan to use the commode every 2 hours during the day.” d. “Hold your bladder as long as possible to restore bladder tone.”

The nurse is caring for a client admitted to the intensive care unit after incurring a basilar skull fracture. Which complication of this injury does

the nurse monitor for? a. Aspiration

b. Hemorrhage c. Pulmonary embolus

d. Myocardial infarction

A client who has a head injury is transported to the emergency department. Which assessment does the emergency department nurse

perform immediately? a. Pupil response b. Motor function

c. Respiratory status d. Short-term memory

The nurse is caring for a client who has a moderate head injury. The client’s sister asks, “Will my brother return to his normal functioning

level when his brain heals?” How does the nurse respond? a. “You should expect a full recovery in all ways by the time of

discharge.” b. “Usually, someone with this type of injury returns to baseline within

6 months.” c. “Your brother may experience many changes in personality and

cognitive abilities.” d. “Learning complex new skills may be more difficult, but you can

expect other functions to return to normal.”

A client who has a severe head injury is placed in a drug-induced coma. The client’s husband states, “I do not understand. Why are you putting

her into a coma?” How does the nurse respond? a. “These drugs will prevent her from experiencing pain when

positioning or suctioning is required.” b. “This medication will help her remain cooperative and calm during

the painful treatments.” c. “This medication will decrease the activity of her brain so that

additional damage does not occur.” d. “This medication will prevent her from having a seizure and will

reduce the need for monitoring intracranial pressure.”

The nurse is preparing to administer prescribed mannitol (Osmitrol) to a client with a severe head injury. Which precaution does the nurse take

before administering this medication? a. Draw up the medication using a filtered needle.

b. Have injectable naloxone (Narcan) prepared and ready at the bedside.

c. Prepare to hyperventilate the client before drug administration. d. Discontinue a barbiturate-induced coma before drug

administration.

A client with a head injury is being given midazolam (Versed) while on mechanical ventilation. Which action does the nurse implement for this

client? a. Monitor for seizures.

b. Assess for urinary output. c. Provide a clear liquid diet. d. Administer an analgesic.

The nurse is caring for a client who is disoriented as the result of a stroke. Which action does the nurse implement to help orient this client?

a. Ask the family to bring in pictures familiar to the client. b. Turn on the television to a 24-hour news station.

c. Maintain a calm and quite environment by minimizing visitors. d. Provide auditory and visual stimulation simultaneously.

The nurse is planning the discharge of a client who has sustained a moderate head injury and is experiencing personality and behavior

changes. The client’s wife states, “I am concerned about how different he is. What can I do to help with the transition back to our home?” How

does the nurse respond? a. “Be firm and let him know when his behavior is unacceptable.”

b. “Minimizing the number of visitors will help stabilize his personality.”

c. “Developing a routine will help provide him with a structured environment.”

d. “He will return to his normal emotional functioning in 6 to 12 months.”

The nurse assesses periorbital edema and ecchymosis around both eyes of a client who is 6 hours postoperative for craniotomy. Which

intervention does the nurse implement for this client? a. Position the client with the head of the bed flat.

b. Apply an ice pack to the affected area. c. Assess arterial blood pressure. d. Notify the health care provider.

The nurse is assessing a client who was recently diagnosed with a meningioma. Which statement indicates that the client correctly

understands the diagnosis? a. “This is the worst type of brain tumor, and surgery is not an

option.” b. “My tumor can be removed, but I can still have damage because of

pressure in my brain.” c. “Even after the surgery, I will need chemotherapy to decrease the

spread of the tumor.” d. “Radiation is never used on brain tumors because of possible nerve

damage.”

MULTIPLE RESPONSE

A client is admitted for evaluation of a cerebral tumor. Which clinical manifestations does the nurse assess this client for?

a. Hemiplegia b. Aphasia

c. Hearing loss d. Behavior changes

e. Nystagmus

COMPLETION

The nurse is preparing to administer a prescribed dose of intravenous dexamethasone (Decadron) to a client after craniotomy. The pharmacy

supplies dexamethasone 40 mcg in 20 mL normal saline to be administered over 15 minutes. The nurse sets the IV pump at a rate of

mL/hr.

Chapter 48: Assessment of the Eye and Vision

Test Bank

MULTIPLE CHOICE

Why is the optic disc considered to be a blind spot? a. This area does not contain photoreceptors.

b. Light rays are unable to focus on this location. c. Blood vessels form a meshwork and interfere with vision.

d. This area is heavily pigmented and light rays are absorbed.

During assessment of an older adult, which finding does the nurse immediately report to the health care provider?

a. Yellowing or bluing of the sclera b. Lack of discrimination between green and violet

c. An opaque, bluish-white ring within the outer edge of the cornea d. Pupil constriction in response to light occurring in 2 seconds

Which teaching is essential for a client who is going to have intraocular pressure measurement with a slit lamp?

a. “The test causes temporary blindness.” b. “The test is quick and a local anesthetic is used.”

c. “The test does cause a little pain, but it is over quickly.” d. “The test causes some tearing, but no pain.”

The nurse performs an assessment of a client’s extraocular movement

and notes no difficulty. Which additional assessment data assist in confirming this finding?

a. No episodes of double vision b. Synchronized blinking movements

c. No reports of headaches and dizziness d. Both pupils constricting equally in response to light

A client has paralysis of the right medial rectus muscle of the right eye. Which assessment finding assists the nurse in validating this diagnosis?

a. Client is unable to turn the eye in toward the nose. b. Client is unable to lift the upper eyelid.

c. Client cannot look downward. d. Client cannot look upward.

The nurse is assessing extraocular eye movements (EOMs) in an older adult client and finds that the client is unable to sustain an upward gaze

for longer than 2 seconds. What does the nurse do next? a. Repeat the test while holding the client’s head in a fixed position.

b. Perform a cover-uncover eye test. c. Document the finding and continue assessing.

d. Assess for additional signs of impending brain attack.

The nurse is assessing an older adult client whose irises no longer fully dilate. What is the best intervention for the nurse to suggest?

a. “Wear dark glasses whenever you are outside.” b. “Use eyedrops on a regular basis to prevent dryness.”

c. “Avoid rubbing your eyes to prevent corneal abrasions.” d. “Turn up room lights when reading or doing close work.”

The nurse is performing vision screenings. Which client is at greatest risk for developing vision problems?

a. Postpartum woman with no complications b. Young client who has diabetes mellitus

c. Middle-aged adult who takes aspirin daily d. Older client with chronic dry eye syndrome

A client relates that the vision in the left eye is greatly decreased from the day before. What does the nurse do first?

a. Assess current medications. b. Patch the left eye.

c. Notify the ophthalmologist. d. Perform an in-depth interview.

During assessment, the nurse notes that a client’s right pupil is 2 mm larger than the left pupil. Which is the nurse’s first action?

a. Ask the client how long this condition has been present. b. Attempt to elicit a red reflex in both eyes. c. Document the finding as the only action.

d. Identify the medications that the client is taking.

The nurse is assessing the blink reflex in a client who is blind. Which is the best technique to use?

a. Ask the client to blink first with one eye and then with the other. b. Expel a syringe of air toward the client’s eyes.

c. Shine a bright light at the client’s pupils one at a time. d. Suddenly bring a finger toward the client’s face.

The nurse is performing an eye assessment on a client. Which finding confirms normal accommodation during visual assessment? a. Both pupils constrict when a light is shined at one eye.

b. The client blinks in response to a threatening movement. c. Both pupils constrict when focusing on an object being moved in

toward the nose. d. The client is able to hold an upward gaze without moving the head

for 15 seconds.

The nurse is assessing a client for the possibility of a lens opacity. Which assessment finding confirms this problem?

a. Increased intraocular pressure b. Absence of a red reflex

c. Decreased central vision d. Positive corneal staining

A client is scheduled for electroretinography. Which statement indicates that the client understands the teaching about this procedure? a. “I will wear dark glasses in sunlight to prevent eye pain.”

b. “I am going to drink at least 3 liters of water to flush the dye out of my system.”

c. “I will avoid rubbing my eyes until the anesthetic drops have worn off.”

d. “I will not drive for the first 24 hours after the procedure.”

The nurse is evaluating a client’s technique for instilling eyedrops. Which behavior indicates that the client needs more teaching?

a. Closing they eye after the drops are in b. Touching the eye with the tip of the dropper

c. Allowing the drops to spread across the eye surface d. Getting the drops into the conjunctival pocket

The nurse is educating a client about the instillation of eyedrops. Which client statement indicates the need for additional teaching?

a. “Squeezing my eye tightly after I put the drops in may force the drops out of my eye too quickly.”

b. “If the drops are kept in the refrigerator, I will be able to tell when they are in my eye because they will feel cold.”

c. “My sister has the same prescription, so we can use the same bottle of eyedrops.”

d. “I will wash my hands before I use these eyedrops.”

A client with presbyopia asks her nurse about corrective lenses. Which is the nurse’s best response?

a. “This type of problem cannot be helped with corrective lenses.” b. “Corrective lenses are needed for both near and distance vision.”

c. “Corrective lenses can be used for reading and close work.” d. “Corrective lenses are needed for distance only.”

A teenager is admitted to the emergency department with a possible fracture of the left orbit after getting hit in the face with a baseball. All tests are negative and the client is being discharged. Which is important

for the nurse to teach the client? a. “Keep an eye patch on the eye for 48 hours.”

b. “Always wear protective equipment to prevent eye damage.” c. “Take aspirin if a headache should occur.” d. “Do not do any heavy lifting for a week.”

An anxious adult client asks why she needs to have intraocular pressure tested every year. What is the best response from the nurse?

a. “Many changes can occur because of aging.” b. “If the pressure is too low, you will be blind.”

c. “If the pressure is too high, blood will not flow through the eye.” d. “Loss of vision can occur if the pressure is too high or too low.”

A client is told that he has 20/10 vision when tested on the Snellen chart. How does the nurse explain this finding to the client?

a. “You can read at 10 feet what others can read at 20 feet.” b. “You can read at 20 feet what others can read at 10 feet.”

c. “This demonstrates normal vision.” d. “You are considered legally blind.”

The nurse is assessing a client’s eyes. Which is the first step for the nurse in this procedure?

a. Explain the procedure. b. Wash the hands.

c. Assess for infections. d. Use the Snellen chart.

The nurse is triaging clients in the emergency department. Which clients require immediate attention by an ophthalmologist?

a. Older client with an intraocular pressure (IOP) of 15 b. Confused client in need of an ophthalmoscopic examination

c. Young client with dry drainage from one eye d. Middle-aged client with recent onset of eye pain

MULTIPLE RESPONSE

The nurse is assessing the eye changes in an older adult. Which changes lead the nurse to consult with the health care provider? (Select all that

apply.) a. Increasing difficulty perceiving greens, blues, and violets

b. Increasing redness in the eyes c. Acute pain in the eyes

d. Sudden change in acuity e. Need for additional lighting for reading

f. Need to hold newspaper farther away to read

OTHER

The nurse is administering ophthalmic drops to a client with an eye infection. Put the following nursing interventions in order, from first to

last. (Separate letters by a comma and space as follows: a, b, c, d.) Recheck the five Rs and the expiration date of the drug.

Put on gloves. Have the client tilt the head backward.

Wash your hands. Pull the lower eyelid downward and instill the medication into the

conjunctival sac. Instruct the client to close the eyes gently without squeezing the eyelids

together.

A client is scheduled for a fluorescein angiography. Place the nurse’s activities in order, from highest to lowest priority. (Separate letters by a

comma and space as follows: a, b, c, d.) Start an intravenous access. Instill mydriatic eyedrops.

Have the consent form signed. Have the client drink fluids.

Inject fluorescein dye. Have the client wear dark glasses.

Chapter 49: Care of Patients with Eye and Vision Problems

Test Bank

MULTIPLE CHOICE

A client is using an ophthalmic beta-blocking agent for the treatment of glaucoma. Which instruction does the nurse give to the client to prevent

orthostatic hypotension? a. “Change positions quickly after administering the drops.”

b. “Take your pulse at least four times daily.”

c. “Apply pressure to the inside corner of your eye when administering the drops.”

d. “Lay down for 10 minutes after administering the drops.”

Which is the most important information for the nurse to teach a client who is receiving cycloplegic drug therapy?

a. “Do not drive or operate machinery until the drug wears off.” b. “Use at least a 30 SPF sunscreen agent when going outdoors.”

c. “Remain on bedrest for 24 hours in a prone position.” d. “Turn up the lights because acuity will be decreased in low-light

environments.”

An older adult client who has a mature cataract in the right eye states, “Now I have lost the sight in my right eye because I waited too long for

treatment.” How does the nurse best respond to the client? a. “Yes, this type of blindness could have been prevented by earlier

treatment.” b. “It is fortunate you came for treatment in time to save the sight of

your other eye.” c. “Nothing you could have done would have made any difference.” d. “Surgery can still save the sight in your eye with removal of the

cataract.”

Which statement indicates that the client understands teaching about the use of aspirin post–cataract surgery?

a. “It may increase intraocular pressure after cataract surgery.” b. “It changes the ability of the blood to clot and increases the risk of

bleeding.” c. “It reduces inflammation and might mask any symptoms of

infection.” d. “It can cause nausea and vomiting and may increase intraocular

pressure.”

Which assessment alerts the nurse to the possible presence of a cataract in a client?

a. Loss of central vision b. Loss of peripheral vision

c. Dull aching in the eye and brow areas d. Blurred vision and reduced color perception

A client is recovering from cataract surgery and needs medication to prevent a potential eye infection. Which drug does the nurse question

administering to the client?

a. Tobramycin (Tobrex) b. Apraclonidine (Iopidine) c. Gentamicin (Genoptic) d. Ciprofloxacin (Ciloxan)

Which statement indicates that a client understands why his cataract surgery is being done first on the eye with the poorest vision?

a. “Insurance reimbursement dictates the timing of surgeries.” b. “The eye with poorer vision is at greater risk for permanent

damage.” c. “The pressure in the poorer eye could increase, causing permanent

damage.” d. “If a complication arises in that eye, I will still have some vision in

the better eye.”

The nurse is teaching a client about home care after cataract surgery. Which statement indicates that the client requires further teaching?

a. “I am glad that I don’t need an eye patch after the surgery.” b. “I will try a cool compress to decrease the swelling around the

operated eye.” c. “Dark sunglasses will be necessary when I am in the sun.”

d. “Pain, nausea, and vomiting are normal after this surgery.”

A client has been educated about activities that can increase intraocular pressure. Which statement indicates that the client requires further

teaching? a. “I will avoid wearing tight shirt collars and ties.”

b. “I will take stool softeners daily to prevent straining.” c. “I will try not to sneeze, cough, or blow my nose.”

d. “I will not put my arms above my head.”

The nurse assesses a client post–cataract surgery and finds white, dry, crusty drainage on the client’s eyelid and lashes. What does the nurse do

next? a. Obtain a specimen of the drainage for culture.

b. Clean away the drainage and apply the prescribed drops. c. Contact the physician for an antibiotic order.

d. Arrange for the client to be seen by the ophthalmologist today.

The nurse is assessing a client who wishes to be considered as a potential donor for corneal transplantation. Which medical diagnosis at the time

of death excludes the client from consideration? a. Small cell lung cancer

b. Chronic heart failure c. Profound nearsightedness d. History of detached retina

The nurse assesses several clients. Which one is most likely to have secondary open-angle glaucoma?

a. Client with gradual onset of blurred vision b. Client who has recently had eye surgery

c. Client who sees halos around lights d. Client with reactive pupils and clear sclera

Which statement made by a client after corneal transplantation indicates a need for further teaching?

a. “I will wear an eye shield at night for at least 1 month.” b. “I will avoid bending at the waist and straining when moving my

bowels.” c. “I won’t worry if I have increased tearing, because it is normal.” d. “I’ll notify the ophthalmologist if any signs of rejection occur.”

Which clinical manifestation alerts the nurse to the possibility of a vitreous humor hemorrhage? a. Presence of a red reflex

b. Reddened whites of the eye c. Red haze or floaters in the line of vision d. Swelling of the upper and lower eyelids

The nurse is providing discharge teaching for a client with posterior uveitis. Which is the most important precaution for the nurse to teach

the client? a. Correct technique for eyedrop instillation

b. Clinical manifestations of retinal hemorrhage c. Correct technique for insertion of contact lenses

d. Proper timing of opioid analgesics

A client with macular degeneration would like to watch television. Where does the nurse place the television for best visualization of the screen?

a. As close to the client’s face as possible b. As far away as possible, with low lights

c. Directly in front of the client d. On either side of the client

In the emergency department, the nurse is caring for a client diagnosed with a hyphema. Which statement by the client indicates a need for

further teaching? a. “When I get home, I can lie flat in bed and turn from side to side.”

b. “For a few days, I cannot even read a book or watch television.” c. “I will need to protect the eye with a patch and shield.”

d. “I need to stay on bedrest and will try not to make any sudden movement.”

A client has just returned from having surgery, and sulfahexafluoride gas was used intraocularly. How does the nurse position the client?

a. Completely supine, with sandbags beside the head b. On the nonoperative side in the Trendelenburg position

c. On the operative side in the Trendelenburg position d. On the abdomen, with the affected eye up

A client comes to the emergency department with periorbital ecchymosis of the right eye. Which is the nurse’s priority action?

a. Apply an ice pack to the affected eye.

b. Patch the eye to prevent eye movement. c. Assess the client’s vision in both eyes.

d. Irrigate the affected eye with normal saline.

The nurse is teaching a client how to apply eye medication. Which is the correct technique for applying ointment into the eye?

a. From the middle out b. From the inner canthus to the outer canthus c. From the outer canthus to the inner canthus

d. Against the inner aspect of the eyelid

A client has conjunctivitis in both eyes and is being treated with topical antibiotics. Which statement by the client indicates a need for further

teaching? a. “I’ll avoid sharing washcloths or towels with other family

members.” b. “I will wash my hands after applying the eye ointment to each eye.” c. “I will call the ophthalmologist if the drainage continues after the

antibiotics are started.” d. “I’ll use the same tube of topical ointment for each infected eye.”

A client is having intraocular pressure measured for both eyes. Which response by the client best indicates that the client understands why this

is necessary every year? a. “Elevated eye pressure can cause high blood pressure.”

b. “If eye pressure is too high, your eyes will dry out.” c. “Elevated eye pressure can press on blood vessels in the eye.”

d. “Increased eye pressure causes the tear ducts to become blocked.”

A client just underwent a keratoplasty. Which activity does the nurse suggest that the client begin possibly 1 week after surgery?

a. Continue with salsa dance lessons. b. Jog only one-half mile versus the usual 2 miles.

c. Return to employment as a receptionist. d. Help the family move furniture from room to room.

MULTIPLE RESPONSE

A client with acute-angle glaucoma has several medications ordered.

Which medications does the nurse question? (Select all that apply.) a. Acetazolamide (Diamox)

b. Pilocarpine (Pilocar) c. Atropine (Isopto Atropine)

d. Latanoprost (Xalatan) e. Timolol (Timoptic)

f. Epinephrine

The nurse is teaching a postoperative client who had a keratoplasty. Which responses by the client require further teaching about safety in

the home? (Select all that apply.) a. “We use throw rugs in the bathroom.”

b. “Our neighbors will be bringing food for a week.” c. “We may have two extension cords in the living room.”

d. “Most of the furniture is placed against the wall, except for one rocking chair.”

e. “Every room has at least one window.” f. “The hallway has low lighting.”

A blind client is admitted to the hospital unit. Orientation to the unit includes which information? (Select all that apply.)

a. Introduce the staff to the client. b. Describe the room to the client using one reference point.

c. Walk the client to the bathroom and describe it.

d. Tell the client to use the call light if he or she wants to go to the bathroom.

e. Explain the routine of the unit and how to operate the bed controls. f. Assist in putting the client’s belongings away.

OTHER

A client has an eye prosthesis and needs to have it inserted into the eye socket. Place the following steps of how to insert an eye prosthesis in the

correct order. (Select in order of priority.) Wash your hands.

Explain the procedure to the client. Remove the prosthesis from its container and rinse it with tepid water.

Cover the work area with a cloth or towel. Don gloves.

Place the prosthesis between the thumb and forefinger of your dominant hand with the notched end of the prosthesis closest to the client’s nose.

Insert the prosthesis with the top edge slipping under the upper lid. Lift the client’s upper lid using your nondominant hand.

Retract the lower lid slightly until the bottom edge of the prosthesis slips behind it.

Release your hand slowly. Gently release the upper eyelid.

Chapter 50: Assessment of the Ear and Hearing

Test Bank

MULTIPLE CHOICE

The nurse notes that a client’s tympanic membrane moves in response to air injected into the external canal. What is the nurse’s best action?

a. Notify the health care provider. b. Document the finding.

c. Prepare to wash the external ear canal. d. Immediately remove the otoscope.

The nurse is performing an ear assessment on an older adult. Which assessment finding does the nurse document in the client’s chart as an

expected age-related change? a. Coarse hair is seen in the auditory canal.

b. Tympanic membrane is intact and bulging. c. Impacted cerumen is present in the auditory canal.

d. Small, painless nodules are noted on the helix of the pinna.

Which client is at highest risk for hearing loss? a. Client with heart failure receiving digoxin (Lanoxin), 0.125 mg

orally daily b. Client with asthma receiving high-dose methylprednisolone (Solu-

Medrol) therapy c. Client with osteomyelitis receiving IV gentamicin (Garamycin)

d. Client with hyperkalemia being treated with intravenous glucose and insulin

The nurse is caring for an older adult client with sensorineural hearing loss. Which assessment finding does the nurse correlate with the client’s

health history? a. History of frequent ear infections

b. Swims frequently c. Worked in a sawmill for the last 20 years

d. Had a tumor removed from his left eardrum last year

The nurse is caring for a client who will undergo electronystagmography testing the following day. Which instruction does the nurse provide for

the client? a. “You should drink only caffeine-free beverages the day of and the

day before the test.” b. “Do not chew gum or clean your ears for 24 hours after the test is

completed.” c. “You may feel flushed as the contrast dye is injected through your

IV for the test.” d. “You will be sedated for the test, so you need someone to drive you

home.”

The nurse is caring for a client who may have an ear infection. Which intervention is used to prevent spread of the infection to other clients?

a. A new sterile otoscope speculum is used to examine each of the client’s ears.

b. The nurse washes his hands after removing hearing aids from the client’s ears.

c. Hearing aids are cleaned with alcohol before they are re-inserted into the client’s ears.

d. The tuning fork is cleaned with hydrogen peroxide before and after use with the client.

The nurse is caring for a client who is hard of hearing. Which intervention best helps the client with communication?

a. Speaking loudly and adding extra inflections to the tone of voice b. Bending over the client so that he or she can see the nurse’s lips

more easily

c. Closing the door to the room and making sure that lighting is adequate

d. Asking the client’s spouse to answer questions that are not heard by the client

Which statement indicates that a client needs additional teaching about protecting the ears and preventing hearing loss?

a. “I will start a smoking cessation program and will take a multivitamin every day.”

b. “I will wear earplugs whenever I cut the grass or use my snow blower.”

c. “I will blow my nose gently, one nostril at a time, whenever I get a cold or the flu.”

d. “I will take Motrin (ibuprofen) instead of Tylenol (acetaminophen) for pain.”

Which is the best assessment question for the nurse to ask a client with tinnitus?

a. “How exactly do you clean your ears?” b. “Have you had your hearing checked lately?”

c. “Do you have ringing in both ears or in only one ear?” d. “Does the ringing make it hard for you to sleep at night?”

The nurse notes reddened areas behind both ears. What does the nurse ask the client?

a. “Do you wear eyeglasses?” b. “Do you have any allergies?”

c. “Do you use dandruff shampoo?” d. “Have you been around anyone with lice?

Which statement indicates that a client needs additional teaching about ear hygiene?

a. “I will wash my hands before I put in my earplugs at work.” b. “I will clean my ears with plain warm water and a washcloth every

day.” c. “I will use a cotton swab to get the extra water out of my ears after I

swim.” d. “I can rinse my ears with half-strength hydrogen peroxide if ear wax

builds up.”

Several clients come to the emergency department following an accident. Which client does the nurse assess first?

a. Client with clear watery drainage from the ear canals

b. Client who reports tinnitus and pain in the right ear c. Client with a deep, 1-inch laceration to the pinna

d. Client who has had severe difficulty hearing since the accident

A client asks the nurse why there is “waxy yellow stuff” on the cotton swab when he cleans his ears. Which is the nurse’s best response?

a. “The yellow ear wax helps transmit sound to your middle ear.” b. “The yellow ear wax indicates that you have an infection in your

ears.” c. “The yellow ear wax helps protect and lubricate the inside of your

ear canal.” d. “The yellow ear wax builds up when you don’t clean your ears often

enough.”

A client is scheduled for a caloric test to evaluate the vestibular portion of the inner ear. Which statement by the client leads the nurse to

conclude that more teaching is necessary? a. “I can eat a hearty breakfast before the procedure.”

b. “I will have to stay in bed after the procedure to prevent nausea.” c. “Warm water will be infused into my affected ear.”

d. “I may experience dizziness after the water is inserted.”

MULTIPLE RESPONSE

The nurse is caring for an older client who presents with dizziness and difficulty hearing. Which of the nurse’s assessment findings will require collaboration with the client’s primary health care provider? (Select all

that apply.) a. Tympanic membrane is retracted, with multiple air bubbles. b. The client reports inability to hear high-frequency voices and

sounds. c. Clear watery drainage is present in the ear canal and is positive for

glucose. d. Tympanic membrane is shiny and translucent, with light reflex

noted. e. Hearing test indicates positive Rinne test, with AC > BC noted

bilaterally. f. The client reports dizziness after taking naproxen (Aleve) for

arthritis pain.

Which statements by a client alert the nurse that the client may have some psychosocial issues with impaired hearing? (Select all that apply.)

a. “I get so angry when I cannot hear what my daughter says.” b. “When I use my hearing aids, I hear the choir so clearly.”

c. “I don’t mind sitting in my chair all day long and not playing bingo.”

d. “My family never seems to visit anymore because their voices all seem so distant.”

e. “No one asks my opinion because I cannot hear their question.” f. “My grandchildren do not think that I am funny anymore because I

cannot hear their jokes.”

OTHER

A client is being taught how to safely irrigate ears to remove cerumen. What is the correct order of self-ear irrigation? (Separate letters by a

comma and space as follows: a, b, c, d.) Fill the syringe with lukewarm water. Hold the head at a 30-degree angle.

Insert the tip of the syringe carefully into the ear canal and aim toward the canal roof.

Tilt the head at a 90-degree angle to remove excess fluid. Use one hand to hold the syringe and the other to push the plunger.

Repeat the procedure on the opposite ear. Continue the procedure until at least a cup of fluid has flowed into and

out of the ear. The ear should fill with fluid and the water will flow out with cerumen.

Chapter 51: Care of Patients with Ear and Hearing Problems

Test Bank

MULTIPLE CHOICE

The nurse is caring for a client with external otitis. Which assessment finding indicates to the nurse that the client’s infection has worsened?

a. The client now reports tinnitus and vertigo at night. b. The client now has a positive Rinne test, with AC > BC.

c. The tympanic membrane is pearly gray with white patches. d. The auricular lymph nodes have increased in size over the last 24

hours.

The nurse is caring for a client with a furuncle on the pinna at the opening of the ear canal. The nurse is reviewing home care instructions with the client. Which statement by the client indicates that additional

teaching is needed? a. “I will put the bacitracin ointment on the sore three times a day.” b. “I will gently squeeze the sore to drain the liquid out once a day.”

c. “I will take Tylenol (acetaminophen) if my ear starts to hurt a lot.” d. “I will put a warm compress on the sore for 15 minutes three times a

day.”

The nurse is caring for a client with otitis media. The client reports that the pain was severe during the night but was gone upon awakening in the

morning. Which finding does the nurse expect to observe during the client’s physical assessment?

a. The tympanic membrane is bluish-gray. b. Purulent fluid is present in the ear canal.

c. The pinna and the tragus are reddened and swollen. d. Sounds are lateralized toward the affected ear.

The nurse is caring for a client with otitis media and notes purulent drainage in the ear canal during the physical assessment. Which is the

nurse’s priority intervention? a. Obtain a specimen of the drainage for culture.

b. Irrigate the ear canal with sterile normal saline. c. Gently examine the client’s ear with an otoscope.

d. Place a cotton ball in the ear canal to absorb the drainage.

A client with a ruptured tympanic membrane asks the nurse whether

hearing will be affected permanently. Which is the nurse’s best response? a. “Possibly. The eardrum usually heals in 1 to 2 weeks. Any persistent

hearing problem should be evaluated.” b. “No. Antibiotics will help resolve the infection and cure your hearing

impairment.” c. “Yes. It will be important for you to be fitted with a hearing aid as

soon as possible.” d. “Yes. Any time the eardrum is ruptured it will form a scar, which

will cause some degree of permanent hearing loss.”

The nurse is caring for a client with Ménière’s disease. The client asks the nurse how to prevent another acute episode from occurring. Which is

the nurse’s best response? a. “Stop or reduce cigarette smoking.”

b. “Use aspirin rather than acetaminophen (Tylenol) for pain.” c. “Reduce the quantity of saturated fats in your diet.”

d. “Avoid crowds and people with upper respiratory infection.”

When performing a client’s physical assessment, the nurse notes that the client has conductive hearing loss. Which finding does the nurse expect

to see in the client’s medical history? a. History of diabetes with peripheral neuropathy

b. Frequent episodes of otitis media during childhood c. History of frequent impactions of cerumen in the ear canals

d. History of osteomyelitis treated with IV gentamicin (Garamycin)

The nurse is caring for a client with Ménière’s disease. What does the nurse recommend to the client to reduce the symptoms of vertigo?

a. “Take salt and potassium supplements daily.” b. “Drink at least eight glasses of water every day.”

c. “Blow your nose hard when dizziness first begins.” d. “When dizziness begins, lie down and keep your head still.”

A client is being started on scopolamine (Transderm Scop) for vertigo. What does the nurse tell the client regarding this medication? a. “You may drive your car while taking this medication.”

b. “Concentration on your college courses will not be affected.” c. “It is recommended that you limit activities requiring a detailed

focus.” d. “You should be able to continue your job as a crane operator.”

The nurse is providing discharge instructions for a client who will be going home following tympanoplasty surgery. Which statement by the

client indicates that additional teaching is needed? a. “I will wear earplugs whenever I am in noisy areas.”

b. “I will occlude only one nostril when I blow my nose.” c. “I will wait 3 weeks before I resume my aerobics workouts.”

d. “I will use a cotton swab to clean drainage from inside my ear.”

The nurse is assessing several clients with hearing loss. Which client does the nurse recommend should investigate hearing aids?

a. Client who has smoked two packs of cigarettes a day for 30 years b. Client who had chronic middle ear infections during childhood

c. Client with constant tinnitus that becomes worse at night d. Client who worked as a security guard at rock concerts for 10 years

The nurse is caring for a newly deaf client who is learning to read lips. Which client statement indicates that additional teaching is needed?

a. “After I practice lip reading for a while, I won’t need to worry about using sign language anymore.”

b. “I will have a harder time lip reading when I am not familiar with the topic of the conversation.”

c. “Focusing so much on lip reading will make me tired, so I will try to keep conversations short.”

d. “I may not be able to lip read very well when the other person has a beard or when light in the room is inadequate.”

A client is being discharged after a tympanoplasty. Which instruction about cephalexin (Keflex) does the nurse provide to this client? a. “Be sure to finish all the Keflex pills, even if you feel fine.”

b. “The Keflex may turn your urine an orange color while you are taking it.”

c. “Take the Keflex on an empty stomach and stay upright for 30 minutes afterward.”

d. “Use sunscreen and avoid exposure to sunlight while you are taking Keflex.”

A client tells the nurse, “Bumps have developed in my ear canals from my hearing aid.” Which is the nurse’s best recommendation for the

client? a. “Clean your hearing aid with rubbing alcohol every evening and let

it dry overnight.” b. “Apply a small amount of benzoyl peroxide cream to the inside of

your ear canals before you insert your hearing aid.” c. “Clean your hearing aid with mild soap and water and make sure

that it is completely dry before inserting it in your ears.” d. “Clean your ears with half-strength hydrogen peroxide twice a day

before you put in your hearing aid and after you take it out.”

The nurse teaches a client’s wife how to administer eardrops to the client. Which statement by the client’s wife indicates that additional

teaching is needed? a. “I will make sure that the eardrops are at room temperature before

using them.” b. “I will wash my hands before and after giving my husband the

eardrops.” c. “After I put the drops in, I will gently tug on the outer ear to make

sure that they go into the ear canal.” d. “I will have my husband lay on his back with his chin up when I

give him the eardrops.”

Which recommendation does the nurse provide for the client with Ménière’s disease who has periodic spells of vertigo?

a. “Avoid wearing high-heeled shoes.” b. “Put brightly colored rugs on the floor for visibility.”

c. “Step on a sturdy chair to get items from high shelves.” d. “Wait to drive a car until after you have taken your Benadryl.”

Which is the best approach for the nurse to use to obtain a history from a client with sudden hearing loss? a. Question the client’s family.

b. Write out the questions for the client to answer. c. Obtain the information from the client’s old chart.

d. Check with the client’s primary health care provider.

The nurse is caring for a client who has just been diagnosed with profound hearing loss. The client tells the nurse, “The doctors must have made a mistake. There’s no way I can be deaf!” Which is the nurse’s best

response? a. “Why do you think that the doctors made a mistake?”

b. “I can tell that you are anxious and scared about your hearing.” c. “Lots of people lead productive lives after losing their hearing.”

d. “The doctors did extensive tests to make sure that the diagnosis was correct.”

The client requires a hearing aid but tells the nurse that he cannot afford to pay for it right now. What is the nurse’s best response?

a. “Your insurance company should pay some of the cost.” b. “The hospital can set up a payment plan for the new hearing aid.” c. “I’ll ask the social worker about organizations that help pay for

hearing aids.” d. “You can check around to see who has the lowest price.”

The nurse is caring for an older client whose ear canals are impacted with hard cerumen. Which action by the nurse is best to remove the

cerumen? a. Instruct the client to put a few drops of mineral oil into each ear

every evening and to schedule the irrigation for 3 days later. b. Aim the irrigation fluid directly at the center of the cerumen to

facilitate dissolving the impaction. c. Administer 10 mg of prochlorperazine (Compazine) to prevent

nausea during irrigation of the ears. d. Irrigate the ears with 35 to 40 mL of sterile normal saline and repeat

as needed until the cerumen is cleared.

MULTIPLE RESPONSE

Which of the nurse’s assessment findings will require collaboration with the client’s primary health care provider? (Select all that apply.)

a. Purulent drainage from the ear canal b. Hearing loss with nausea and vertigo

c. Ringing in the ears after attending a loud rock concert d. Presence of cerumen blocking 50% of the ear canal

e. Increasing hearing loss since starting furosemide (Lasix)

f. Temperature of 101.7° F following a stapedectomy 3 days ago

Which client statement indicates that the client understands teaching about stapedectomy surgery? (Select all that apply.)

a. “My hearing will get worse before it gets better.” b. “I will have to miss 6 weeks of swim team practice.”

c. “I will see the doctor 1 week after surgery to have my stitches removed.”

d. “Foods may taste funny for a short time after surgery.” e. “I may get dizzy and feel like the room is spinning after surgery.”

f. “I can blow my nose to relieve the feeling of fullness in my ear after surgery.”

A client has mastoiditis. The nurse assesses most carefully for which manifestations? (Select all that apply.)

a. Red and bulging eardrum b. A crackling sound upon yawning

c. Enlarged lymph nodes behind the ear d. Low-grade fever and malaise

e. Diminished hearing f. Loss of appetite

OTHER

The nurse is instilling eardrops into a client’s ear. Place the following in order according to best practice. (Separate letters by a comma and space

as follows: a, b, c, d.) Ask the client to move the head gently back and forth five times.

Wash your hands. Wear gloves to remove any packing from the ear.

Perform an otoscopic examination to see if the eardrum is intact. Irrigate the ear if needed to remove cerumen.

Tilt the client’s head in the opposite direction of the affected ear and place the drops in the affected ear.

Warm the bottle of eardrops in a bowl of warm water for 5 minutes. Wash your hands again.

Insert a ball of cotton in the ear as packing.

COMPLETION

The nurse is caring for a 132-lb client with an ear infection who is to receive amoxicillin, 40 mg/kg/day in divided doses every 8 hours. The

nurse will administer mg/dose of amoxicillin to the client.

Chapter 52: Assessment of the Musculoskeletal System

Test Bank

MULTIPLE CHOICE Which postoperative order does the nurse clarify with the surgeon before

discharging the client who just had arthroscopic surgery on the right knee?

a. Keep the right leg elevated on a soft pillow for 12 hours. b. Maintain non–weight bearing by right leg for 48 hours.

c. Use ice on the knee for 24 hours. d. Administer two tablets of oxycodone/APAP (Tylox) every 4 hours for

pain.

An occupational therapist is treating a client with rheumatoid arthritis. Which assessment finding in the client does the nurse share with the

occupational therapist? a. Difficulty sleeping because of pain in the knees and elbows

b. Difficulty tying shoelaces and doing zippers on clothing c. Swollen knees with crepitus and limited range of motion

d. Generalized joint stiffness that is worse in the early morning

The nurse is caring for a client who is to have a computed tomography (CT) scan of the leg. Which assessment question does the nurse ask the

client before the procedure? a. “Do you have any metal clips, plates, or pins in your body?” b. “Have you had anything to eat or drink in the last 6 hours?”

c. “Do you have someone to drive you home after the procedure?” d. “Do you have any allergies to shrimp, scallops, or other seafood?”

The nurse is assessing a client who reports severe knee pain after a fall. Which question does the nurse ask to determine the radiation of the

pain? a. “What makes the pain better or worse?”

b. “Are you able to bear any weight on the knee at all?” c. “Does the pain move to another area from your knee?” d. “How would you rate the pain on a scale of 1 to 10?”

Which instruction does the nurse give to the client before he or she has electromyography (EMG)?

a. “Make sure that you have someone to drive you home after the test.”

b. “Do not eat or drink anything for at least 6 hours before the test.” c. “You will have to avoid heavy lifting for 24 hours following the test.”

d. “Do not take your cyclobenzaprine (Flexeril) on the 2 days before the test.”

The nurse is caring for a client with prostate cancer. Which laboratory finding indicates to the nurse that the cancer has metastasized to the

bone? a. Serum calcium, 21.6 mg/dL b. Creatine kinase, 55 U/mL

c. Alkaline phosphatase, 45 IU/mL d. Lactate dehydrogenase, 120 U/L

The nurse is caring for a client who presents with achy jaw pain. Which assessment technique does the nurse use to determine whether the client

has inflammation of the temporomandibular joint (TMJ)? a. Checking for decayed, fractured, loose, or missing teeth

b. Observing the jaw joint as the client chews a piece of food c. Palpating the joint during movement for tenderness or crepitus

d. Observing for asymmetric joint protrusion when the client’s mouth

is closed

The nurse is caring for a client who is able to flex the right arm forward without difficulty or pain but is unable to abduct the arm because of pain

and muscle spasms. Which condition does the nurse suspect based on these assessment findings?

a. Dislocated elbow b. Lesion in the rotator cuff

c. Osteoarthritis of the shoulder d. Atrophy of the supraspinatus muscle

The nurse is assessing a client who is suspected of having muscular dystrophy. Which statement by the client indicates that more teaching may be needed about the creatine kinase (CK) test that the health care

provider has ordered? a. “The Lasix that I took this morning may affect the test results.”

b. “The CK test is 90% accurate in demonstrating muscle trauma or injury.”

c. “The level of CK will be decreased with skeletal muscle disease.” d. “When muscle is damaged, CK isoenzymes are released over time.”

A client has cancer and a pacemaker, and suffers from claustrophobia. Which diagnostic test is the best indicator of the client’s bone metastasis?

a. Magnetic resonance imaging (MRI) b. Arthrogram c. Ultrasound

d. Thallium bone scan

Which client does the nurse assess first at the start of the nursing shift? a. Client wanting to know information about a magnetic resonance

imaging (MRI) test scheduled in 3 hours b. Client who is verbalizing mild discomfort after an electromyography

(EMG) c. Client who reports increased pain and swelling after an arthroscopy

d. Client who refuses to drink more fluids after a nuclear medicine scan

MULTIPLE RESPONSE The nurse is performing a medical history and physical assessment on an older client. Which common findings in the older client are related to the

musculoskeletal system? (Select all that apply.) a. Decrease in bone density

b. Decrease in falls due to lack of activity c. Atrophy of the muscle tissue d. Decrease in bone prominence

e. Degeneration of cartilage f. Reduced range of motion of the joints

Chapter 53: Care of Patients with Musculoskeletal Problems

Test Bank

MULTIPLE CHOICE

The RN has assigned a client with severe osteoporosis to an LPN. Which

information about the care of the client is most important for the RN to provide the LPN?

a. Provide passive range of motion (ROM) to all weight-bearing joints. b. Position the client upright to promote lung expansion.

c. Place a pillow between the client’s knees when in the side-lying position.

d. Use a lift sheet to reposition the client.

Which instruction is most important for the RN to provide to the nursing assistant assigned to care for a client with primary osteoporosis?

a. “Clean up clutter in the room.” b. “Encourage the client to bathe herself or himself."

c. “Monitor urinary output.” d. “Perform passive range-of-motion exercises.”

Which instruction does the nurse include in the discharge teaching plan of a client who has osteoporosis? a. “Avoid using scatter rugs.”

b. “Avoid weight-bearing exercises.” c. “Use a cane when walking outside.”

d. “Reduce the amount of protein in your diet.”

After the administration of each dose of zoledronic acid (Zometa), it is most important for the nurse to determine which finding?

a. Capillary refill b. Pain relief

c. Level of consciousness d. Urine output

Which nursing intervention is most effective in preventing transfer of an organism from the wound of a client with osteomyelitis to other clients?

a. Contact Precautions b. Restriction of visitors

c. Irrigating the wound as needed d. Leaving the wound open to air

While caring for a client who has chronic osteomyelitis and wound drainage, which intervention is most important for the nurse to

implement? a. Cover the wound with a dressing.

b. Teach about the cause of the infection. c. Monitor the erythrocyte sedimentation rate (ESR).

d. Prepare the client for hyperbaric oxygenation.

Which exercise does the nurse recommend to a client at risk for osteoporosis?

a. High-impact aerobics 45 minutes once weekly b. Walking 30 minutes three times weekly c. Jogging 30 minutes four times weekly

d. Bowling for 1 hour twice weekly

An adult client’s susceptibility to osteoporosis is caused by which aspect of his or her history?

a. Fractured arm at age 16 b. Active smoking

c. Vitamin D supplements d. Weight lifting

Which client does the nurse assess more carefully for risk of developing primary osteoporosis?

a. African-American client b. Resident of a nursing home

c. Client who eats meat with every meal d. Client who drinks 6 cups of coffee daily

A client’s susceptibility to osteomalacia is related to which risk factor? a. Calcium level of 11 mg/dL b. Diet high in milk and soy

c. Phosphate level of 1.0 mg/dL d. Taking vitamin D supplements

A female client who is a carrier of the gene for Duchenne’s muscular dystrophy asks whether any of her daughters will have this disease.

Which is the nurse’s best response? a. “Both parents must have the defective gene.”

b. “Your daughter cannot get the disease.” c. “Your daughters have a 50% chance of developing the disease.”

d. “Your daughters will become carriers of the gene.”

When preparing to care for a client with a family history of Paget’s disease, it is most important for the nurse to include education in which

area? a. Avoidance of infections

b. Exercise program c. Nutrition high in vitamin C

d. Need for genetic testing

The mother of a 16-year-old client diagnosed with Ewing’s sarcoma expresses concern that her son seems to be angry at everyone in the

family. How does the nurse respond? a. “You need to set limits with your son.”

b. “This is a normal stage in the grieving process.” c. “He will be back to normal when he leaves the hospital.”

d. “This is typical behavior for a teenager.”

A client diagnosed with primary bone sarcoma of the leg is scheduled for tumor removal. The client expresses fear of loss of function. Which is the

nurse’s best response? a. “It is normal to feel this way.”

b. “Physical therapy will assist you to regain function.” c. “This surgery is better than an amputation.” d. “This surgery is necessary to save your life.”

A client newly diagnosed with Ewing’s sarcoma is most likely to exhibit which laboratory finding?

a. Elevated red blood cells (RBCs) b. Elevated alkaline phosphatase (ALP)

c. Decreased erythrocyte sedimentation rate (ESR) d. Decreased serum lactate dehydrogenase (LDH)

A client is prescribed alendronate (Fosamax). Which statement indicates that the client understands teaching about this drug?

a. “I should take this drug with a full glass of water.” b. “I need to lie down for 30 minutes after taking it.”

c. “This drug should be taken after a meal.” d. “This drug needs to be taken at the same time as calcium.”

A client is seen at the clinic with the medical diagnosis of osteomalacia. When taking the client’s history, what does the nurse assess for?

a. Arm and leg strength b. Dietary intake of vitamin D c. Dietary intake of calcium

d. Exercise habits

The nurse has educated a client on Paget’s disease. Which statement by the client indicates good understanding of causative factors?

a. “It is caused by lack of calcium in my diet.” b. “I probably had a fracture that caused it.”

c. “This disease occurs because of lack of exercise.” d. “I may have a genetic predisposition.”

A client has severe Paget’s disease. Which factor has the highest priority when the nurse intervenes in the care of this client?

a. Dietary education

b. Exercise program c. Genetic testing d. Relief of pain

Which assessment finding relates most directly to a diagnosis of chronic osteomyelitis?

a. Erythema of the affected area b. Swelling around the affected area

c. Temperature higher than 101° F (38° C) d. Ulceration of the skin

When providing care for a client who has had a débridement for osteomyelitis, which intervention is most important for the nurse to

implement? a. Assess the white blood cell count.

b. Assess circulation in the distal extremities. c. Administer pain medication.

d. Monitor temperature.

A client who has been diagnosed with osteomyelitis is beginning antibiotic therapy. Which information does the nurse include in the

client’s teaching plan? a. Needing a consultation with a surgeon

b. Continuing on Contact Isolation at home c. Remaining in the hospital for the rest of the treatment

d. Receiving antibiotic treatment at home from the home health nurse

Two hours after limb salvage surgery for a client with left leg bone sarcoma, the nurse notes that the toes of the left foot are more

edematous, are cooler to the touch, and have a slower capillary refill. Which action does the nurse take first? a. Apply ice to the distal extremity.

b. Check the splint for proper placement. c. Elevate the left foot.

d. Loosen the pressure dressing.

The nurse is caring for a client with a lesion in the area of the tibia that is swollen and tender. Which client problem is the highest priority for

nursing care? a. Need for increased calories related to increased metabolism

b. Pain management related to physical injury c. Compromised self-care related to weakness d. Safety risk related to skeletal impairment

Which client is at highest risk for the development of plantar fasciitis? a. Young adult runner b. Adolescent swimmer

c. Older adult client who walks with a cane d. Adult client confined to a wheelchair

The nurse is caring for a client with rheumatoid arthritis. For which condition does the nurse assess most carefully?

a. Dupuytren’s contracture b. Hallux valgus

c. Morton’s neuroma d. Plantar fasciitis

The nurse is assessing a client with Paget’s disease. Which assessment finding leads the nurse to notify the health care provider immediately?

a. Client is 5 feet in height and weighs 130 pounds. b. Long bones of the legs and arms are bowing.

c. Base of the skull is enlarged with changes in vital signs.

d. Mild pain is present in the area of the hips and pelvis.

MULTIPLE RESPONSE

The nurse reviews the health history of a client with acute osteomyelitis. Which findings might have contributed to the diagnosis? (Select all that

apply.) a. Recent dental work

b. Urinary tract infection c. Pregnancy

d. Age e. Hemodialysis

f. Gastrointestinal infection

The nurse is performing a medical history and physical assessment for a client. Which assessment findings lead the nurse to conclude that the

client is at risk for development of osteoporosis? (Select all that apply.) a. Client is a white woman with a body mass index (BMI) of 19.4.

b. Client fractured her wrist badly in a fall last year. c. Client drinks at least four cans of diet cola every day.

d. Client does tai chi exercises for 45 minutes every morning. e. Client has smoked two packs of cigarettes a day for 40 years.

f. Client has taken estrogen (Premarin) 0.625 mg daily since menopause.

Chapter 54: Care of Patients with Musculoskeletal Trauma

Test Bank

MULTIPLE CHOICE

A client has a fracture and is being treated with skeletal traction. Which assessment causes the nurse to take immediate action?

a. The client’s blood pressure is 130/86 mm Hg. b. The traction weights are resting on the floor.

c. Slight oozing of clear fluid is noted at the pin site. d. Capillary refill of the extremity is less than 3 seconds.

A client has been diagnosed with carpal tunnel syndrome. Which intervention does the nurse question in the treatment of this injury?

a. Ibuprofen 600 mg three times a day with meals b. Weekly injections of a corticosteroid by the physician

c. Morphine 30 mg to be taken orally every 4 hours d. Use of a hand brace or splint during the day

The nurse is caring for a client with a fractured femur. Which factor in the client’s history may impede healing of the fracture?

a. A sedentary lifestyle b. A history of smoking

c. Oral contraceptive use d. Paget’s disease

A client who has sustained a crush injury to the right lower leg reports numbness and tingling of the affected extremity. The skin of the right leg

appears pale. Which is the nurse’s first intervention? a. Assess pedal pulses.

b. Apply oxygen by nasal cannula. c. Increase the IV flow rate.

d. Document the finding.

While assessing an older adult client admitted 2 days ago with a

fractured hip, the nurse notes that the client is confused, tachypneic, and restless. Which is the nurse’s first action? a. Administer oxygen via nasal cannula. b. Apply restraints and ask for a sitter.

c. Slow the IV flow rate. d. Discontinue the pain medication.

The nurse is caring for several clients with fractures. Which client does the nurse consider at highest risk for developing deep vein thrombosis?

a. Middle-aged woman with a fractured ankle taking aspirin for rheumatoid arthritis

b. Young adult male athlete with a fractured clavicle c. Female with type 2 diabetes with fractured ribs

d. Older man who smokes and has a fractured pelvis

The nurse is rounding on assigned orthopedic clients. The client with which type of fracture requires immediate interventions to prevent

infection? a. Fractured clavicle

b. Open fracture of the tibia c. Simple fracture of the wrist

d. Compression fracture of a vertebra

The nurse is performing an assessment on a client admitted with a fractured left humerus. When the client moves the extremity, the nurse

notes the presence of a grating sound. Which is the nurse’s best intervention?

a. Immobilize the arm. b. Perform range of motion.

c. Monitor for other signs of infection. d. Administer steroids.

The nurse is caring for a client with a pelvic fracture. Which is the nurse’s priority action to prevent complications?

a. Monitor temperature daily. b. Insert a urethral catheter.

c. Monitor blood pressure frequently. d. Turn the client every 2 hours.

A client who had a plaster cast applied to the right arm 3 weeks ago presents to the clinic with an erythrocyte sedimentation rate (ESR) that

has increased from 15 to 25 mm/hr. Which is the nurse’s best action? a. Repeat this laboratory assessment in 4 hours.

b. Have the cast reapplied. c. Evaluate temperature and vital signs.

d. Obtain blood for a platelet count.

The home care nurse is visiting a client with diabetes who has a new cast on the arm. On assessment, the nurse finds the client’s fingers to be pale,

cool, and slightly swollen. Which is the nurse’s first intervention? a. Elevate the arm above the level of the heart.

b. Encourage active and passive range of motion. c. Apply heat to the affected hand.

d. Place a window or bivalve the cast.

A client who had a wrist cast applied 3 days ago calls from home, reporting that the cast is loose enough to slide off. How does the nurse

respond? a. “Keep your arm above the level of your heart.”

b. “As your muscles atrophy, the cast is expected to loosen.” c. “Wrap an elastic bandage around the cast to prevent it from

slipping.” d. “You need a new cast now that the swelling is decreased.”

The nurse is assessing a client with a body cast. Which assessment finding indicates a complication that must be reported to the health care

provider? a. Blood pressure, 130/85 mm Hg

b. Urinary output, 40 mL/hr c. Redness around the edges of the cast

d. Vomiting after meals

A client for whom skeletal traction is planned asks for an explanation regarding the purpose of this type of traction. Which is the nurse’s best

response? a. “It aids in realigning the bone.”

b. “It prevents low back pain.” c. “It decreases muscle spasms that occur with a fracture.” d. “It prevents injury to the skin as a result of the fracture.”

The nurse notes that the skin around the client’s skeletal traction pin site is swollen, red, and crusty, with dried drainage. Which is the nurse’s

priority intervention? a. Decrease the traction weight.

b. Apply a new dressing. c. Cleanse the area, scrubbing off the crusty areas.

d. Culture the drainage.

The nurse is caring for a client with an external fixator in place on the leg. What does the nurse assess for first?

a. Alteration in skin integrity b. Impaired motor action

c. Acute pain d. Signs of infection

The nurse is teaching a client who has left leg weakness to walk with a cane. Which gait training technique is correct?

a. Place the cane in the client’s left hand and move the cane forward, followed by moving the left leg one step forward.

b. Place the cane in the client’s left hand and move the cane forward, followed by moving the right leg one step forward.

c. Place the cane in the client’s right hand and move the cane forward, followed by moving the left leg one step forward.

d. Place the cane in the client’s right hand and move the cane forward, followed by moving the right leg one step forward.

A client asks why a plaster cast is not applied to the fractured clavicle. Which is the nurse’s best response?

a. “Plaster will make the area too heavy for movement.” b. “A splint or a bandage is sufficient to keep the bones in alignment.”

c. “Cloth braces are less likely to disrupt circulation.” d. “Fractures to the upper body always heal more quickly.”

The nurse assesses a client with a below-knee amputation. Which assessment of the skin flap requires immediate action?

a. Pink and warm to the touch b. Pale and cool to the touch

c. Dark pink and dry to the touch d. Pink and slightly moist to the touch

A client who has had an above-knee amputation of the right leg reports pain in the right foot. Which priority medication does the nurse

administer? a. IV morphine

b. 650 mg of acetaminophen c. IV calcitonin

d. 600 mg of ibuprofen

The nurse is caring for a client 3 days after a below-knee amputation. Which is a priority intervention?

a. Muscle-strengthening exercises b. Use of a very soft bed mattress

c. Placing a pillow between the client’s knees d. Placing the client in high Fowler’s position

The nurse is caring for clients with above-knee amputations. Which client does the nurse treat first?

a. Client who reports phantom limb pain b. Client who reports cramping

c. Client who does not want to move the leg d. Client with regional pain syndrome

The nurse is caring for a client after arthroscopy surgery. Which intervention is a postoperative priority for this client?

a. Passive range of motion on the involved knee b. Active range of motion on the involved knee

c. Straight leg raises with the involved leg d. Immobilization of the leg

The client is being assessed for rotator cuff injury. Which physical assessment finding is consistent with this type of injury?

a. The client is unable to maintain adduction of the affected arm at the shoulder for longer than 30 seconds.

b. The client is able to raise the affected arm to shoulder height but feels pain on doing this maneuver.

c. The client is unable to initiate or maintain abduction of the affected arm at the shoulder.

d. The client has referred pain to the shoulder and arm opposite the affected shoulder.

The nurse is caring for a client with a fractured fibula. Which assessment prompts immediate action by the nurse?

a. Reported pain of 4 on a scale of 0 to 10 b. Numbness and tingling in the extremity

c. Swollen extremity where the injury occurred d. Reports of being cold in bed

A client has a fractured humerus. Which dietary choice indicates that the client understands the nutrition needed to assist in healing the fracture?

a. Skim milk, vitamin D supplements, and fish b. Soy milk, vitamin B supplements, and bacon, lettuce, and tomato

sandwich c. Whole milk, vitamin A supplements, and vegetable lasagna

d. Low-fat milk, vitamin C supplements, and roast beef

The nurse is caring for an older adult client with multiple fractures. How does the nurse manage pain in this client?

a. Meperidine (Demerol) injections every 4 hours rather than PRN b. Patient-controlled analgesia (PCA) pump with morphine

c. Ibuprofen (Motrin) 600 mg every 4 hours d. IV morphine PRN

A client has an arm cast and reports that it feels really tight and the fingers are puffy. What is the nurse’s best response?

a. “Elevate your arm on two pillows and apply ice to the cast.” b. “Continue to take ibuprofen (Motrin) until the swelling subsides.”

c. “It is normal for a new cast to feel a little tight for the first few days.”

d. “Please come to the clinic today to have your arm checked by the

health care provider.”

A client who had a long-leg cast applied last week reports to the clinic nurse, “I can’t seem to catch my breath and I feel a bit lightheaded.”

Which is the priority action of the nurse? a. Listen to the client’s lungs and check the client’s blood glucose level. b. Give the client 2 L of oxygen via nasal cannula and check vital signs. c. Check the client’s pulse oximetry and arrange emergency transfer to

the hospital. d. Reassure the client that it takes much more effort to move with a

long-leg cast.

A client has left-sided weakness. Which action by the client indicates that additional teaching about proper cane use is needed?

a. Holding the cane in the right hand b. Advancing the cane while the right leg moves forward

c. Stepping forward first with the right leg when ambulating d. Flexing the elbow 15 to 20 degrees when holding the cane

The nurse is caring for an older adult client who had leg amputation surgery the previous day. During the admission assessment, the client tells the nurse, “I don’t want to live with only one leg, so I should have

died during the surgery.” Which is the nurse’s best response? a. “Your vital signs are good, and you are doing just fine right now.” b. “Your children are waiting outside and do not want to lose their

parent.” c. “Remember that you are still the same person inside, with a missing

body part.” d. “You will be able to do some of the same things as before you

became disabled.”

MULTIPLE RESPONSE

A client has a fractured tibia and is asking the nurse about external fixation. What are some advantages for the use of external fixation for

the immobilization of fractures? (Select all that apply.) a. Leads to minimal blood loss b. Allows for early ambulation

c. Decreases the risk of infection d. Increases blood supply to tissues

e. Provides visualization of bone ends f. Promotes healing

An older woman is admitted after falling down the stairs. Which assessment findings require immediate intervention? (Select all that

apply.) a. Blood pressure, 80/50 mm Hg

b. Potassium, 6.0 mEq/L c. Dark brown urine

d. Heart rate, 90 beats/min e. Urine output, 50 mL/hr

A client with a new fracture reports pain in the site of the fracture. An opioid pain medication was administered 20 minutes ago. Which is the

nurse’s best intervention? (Select all that apply.) a. Administration of additional opioids

b. Elevation of the extremity c. Application of ice

d. Application of heat e. Keeping the extremity in a dependent position

Chapter 55: Assessment of the Gastrointestinal System

Test Bank

MULTIPLE CHOICE

The nurse is caring for a client who is receiving radiation treatment for oral cancer. Which problem does the nurse anticipate for this client?

a. Failure to absorb nutrients from the stomach b. Inability to digest protein

c. Impaired ability to soften and break down food d. Difficulty swallowing food

Which question best assists the nurse in assessing a client with acute diarrhea?

a. “Have you traveled outside the country recently?” b. “Have you had a colonoscopy lately?”

c. “Do you have any trouble swallowing?” d. “Do you have any allergies?”

A client has been taking naproxen (Naprosyn) for several months. Which assessment question is important for the nurse to ask?

a. “Have you experienced any constipation?” b. “Have you had any stomach pain or indigestion?”

c. “Have you had any difficulty swallowing?” d. “Have you noticed any weight loss lately?”

The nurse is screening clients at a health fair. Which client is at highest risk for the development of colon cancer?

a. Older white client with irritable bowel syndrome b. Middle-aged African-American client who smokes cigars

c. Middle-aged Asian client who travels and eats out frequently d. Older American Indian client taking hormone replacement therapy

When performing an assessment, the nurse detects a fruity odor on the client’s breath. What does the nurse do next?

a. Assess the client’s blood sugar level. b. Assess the client’s stool for occult blood.

c. Instruct the client in oral hygiene techniques. d. Assess the client for petechiae, itching, and jaundice.

The nurse is preparing to perform an abdominal assessment on a client with suspected cholecystitis. In what sequence does the nurse palpate the

client’s abdomen? a. Palpate the lower quadrants only. b. Palpate the upper quadrants last. c. Palpate the upper quadrants only.

d. Defer palpation and use percussion only.

The nurse is caring for a client who has just returned from abdominal surgery. When auscultating the client’s abdomen, the nurse does not hear

any bowel sounds. Which is the nurse’s best action? a. Notify the health care provider.

b. Percuss the abdomen. c. Document the finding.

d. Insert a nasogastric tube.

The nurse assesses dullness at the left anterior axillary line. The nurse is concerned about which condition that the client may have?

a. Cirrhosis b. Splenomegaly

c. Bowel obstruction d. Abdominal aortic aneurysm

The nurse finds a positive Blumberg’s sign in a client with abdominal pain. Which action does the nurse plan?

a. Have the client be NPO in preparation for surgery. b. Document this normal finding in the client’s record.

c. Immediately auscultate the client’s abdomen for bowel sounds. d. Repeat the maneuver with the client in a supine position, with the

knees flexed.

Which laboratory finding does the nurse expect to find on assessment of a client with advanced cirrhosis?

a. Amylase, 129 IU/L; alkaline phosphate, 45 U/L b. Reticulocyte count, 1%; magnesium, 1.5 mEq/L

c. Hemoglobin, 14 g/dL; direct bilirubin, 0.2 mg/dL d. Prothrombin time (PT), 17.5 seconds; albumin, 1.6 g/dL

The nurse is caring for a client who just completed an upper GI radiographic series with oral barium contrast. Which instructions does

the nurse provide to the client?

a. “Drink plenty of fluids over the next few days.” b. “Do not eat or drink anything for 6 hours after the test.” c. “You may not drive or operate heavy machinery today.”

d. “Do not take any blood thinners for 24 hours after the test.”

The client is scheduled for a colonoscopy. Which statement indicates that the client needs additional teaching about the procedure?

a. “I may have gas and abdominal cramps after the test.” b. “I will take strong laxatives the afternoon before the test.”

c. “I will take my Coumadin with a sip of water tomorrow morning.” d. “I will take nothing by mouth after midnight on the day of the test.”

The nurse is preparing the client for a computed tomography (CT) scan of the abdomen with IV contrast. Which question does the nurse ask the

client before the examination? a. “Are you allergic to shrimp, scallops, or shellfish?”

b. “Have you had anything to eat or drink in the past 12 hours?” c. “Did you finish taking all the prescribed laxatives?”

d. “Can you tolerate being tilted from side to side?”

An abdominal ultrasound is scheduled for the client. Which statement by the client indicates that the nurse’s teaching about the procedure was

effective? a. “The IV contrast may burn when it is injected.”

b. “I will drive myself home after the test is completed.” c. “I will empty my bladder completely before the test.”

d. “I may have to take a laxative to pass the barium afterward.”

While a health history is obtained from a client with a new diagnosis of advanced pancreatic cancer, the client begins to cry. Which is the nurse’s

best response? a. “I am so sorry for making you cry!”

b. “I will step out for a few minutes until you feel better.” c. “I can see that you are upset about this. It is all right to cry.”

d. “I can see that I am upsetting you. Let’s move on to something else.”

The nurse is performing an abdominal assessment on an older client. Which assessment finding does the nurse expect as a normal

consequence of aging?

a. Increased salivation and drooling b. Hyperactive bowel sounds and loose stools

c. Increased gastric acid production and heartburn d. Impaired sensation to defecate and constipation

The nurse is caring for a client who just had an esophagogastroduodenoscopy (EGD) completed. The client tells the nurse that her mouth is very dry after the procedure. Which is the

nurse’s best action? a. Keep the client NPO (nothing by mouth).

b. Check the client’s gag reflex. c. Offer the client sips of clear liquids.

d. Provide the client with a few ice chips.

A client has a family history of colon cancer. Which laboratory tests are ordered to rule out colon cancer?

a. Cholesterol b. Serum lipase

c. Carcinoembryonic antigen d. Xylose absorption

MULTIPLE RESPONSE

The nurse performs percussion of a client’s abdomen. Which findings may the nurse determine with this assessment technique? (Select all that

apply.) a. Hepatomegaly b. Kidney stones

c. Ascites d. Large mass below the liver

e. Biliary colic f. Ileus

A client reports that he has been passing black stools for the last few days. Which findings from the client’s health history does the nurse

consider as possible causes? (Select all that apply.) a. Cirrhosis

b. Cholecystitis c. Hemorrhoids d. Diverticulitis

e. Long-term use of NSAIDs f. Use of iron supplements

A client has jaundice and ascites. Which laboratory values indicate hepatic disease? (Select all that apply.)

a. Albumin, 2.0 g/dL b. Potassium, 3.0 mEq/L

c. Alanine aminotransferase (ALT), 45 IU/L d. Aspartate aminotransferase (AST), 45 U/L e. Unconjugated (indirect) bilirubin, 1 mg/dL

f. Ammonia, 120 mg/dL

Chapter 56: Care of Patients with Oral Cavity Problems

Test Bank

MULTIPLE CHOICE The nurse is caring for a client who is being discharged following surgery

for oral cancer. Which sign is the client instructed to watch for that

indicates possible metastasis of the cancer? a. Fragile gums that bleed easily

b. White patches on the tongue and the back of the throat c. Painful ulcerated lesions on the gums or inside of the cheek d. Small hard lumps on the side of the neck or under the chin

The nurse is caring for a female client who has just undergone excision of a parotid gland tumor. The client tells the nurse that she is

experiencing facial weakness on the operative side. Which is the nurse’s best response?

a. “You may be experiencing a slight stroke, and I will notify the doctor.”

b. “This is a temporary condition that will resolve once radiation treatment is begun.”

c. “You are experiencing weakness because the facial nerve was irritated during the surgery.”

d. “You probably have a pinched nerve after lying on the operating room table for so long.”

The nurse is caring for a client who is receiving radiation therapy for treatment of oral cancer. The client reports a constant dry mouth. Which

is the nurse’s best response? a. “Massage the area just over the lower jaw twice a day.”

b. “Use lemon and glycerin swabs to clean your mouth and help keep it

moist.” c. “Suck on lemon slices to help increase saliva production.” d. “Rinse your mouth out often with warm saline or sodium

bicarbonate solution.”

The nurse is caring for a client who has just received a diagnosis of advanced oral cancer and has learned that a glossectomy with jaw resection will have to be scheduled. The client states to the nurse, “I would rather die than have half of my face removed. My life is over.” Which is the best description of the client’s response to the diagnosis?

a. Refusal of any more treatment b. Grief over the diagnosis

c. Acceptance of the diagnosis d. Denial about the diagnosis

The nurse is caring for a client who has just received a diagnosis of advanced oral cancer that will require extensive surgery. Which statement by the client indicates that the diagnosis is accepted?

a. “The biopsy test results will be confirmed again next week.” b. “Of all the bad things to happen to me, now I have cancer on top of

it all.” c. “If I can live long enough to see my son get married, everything will

be alright.” d. “I don’t like it, but I have cancer and that’s the way it is.”

The nurse is caring for a client who just had a radical jaw and neck resection. The nurse is developing a teaching plan for the client and

spouse about care after discharge from the hospital. Which is an effective teaching objective for this client and spouse?

a. The client’s spouse will be able to change the client’s tracheostomy ties correctly after three teaching sessions.

b. The client and spouse will verbalize the signs of readiness for oral feedings following placement of the tracheostomy.

c. The client’s spouse will correctly administer the client’s tube feedings twice daily.

d. The client and spouse will understand incision care and the importance of infection prevention.

The nurse is caring for a client who just had a radical jaw and neck resection for oral cancer. The nurse has just completed teaching for the

spouse and client about tracheostomy care. Which notation in the client’s chart is the most accurate documentation of the teaching that occurred? a. “The client and spouse were instructed regarding management of

mucous plugs and thick secretions.” b. “Information about home oxygen therapy and equipment was

provided for the client and spouse.”

c. “The client and spouse were shown how to suction the tracheostomy and change the ties.”

d. “Correct suctioning procedure was demonstrated, and the client’s spouse verbalized two instances when suctioning needs to occur.”

The nurse is caring for a client who will be going home after a radical jaw and neck resection. The client’s spouse will be the primary caregiver

at home and will need to care for the client’s feeding tube and tracheostomy. Which skill is the highest priority for the nurse to teach

the client’s spouse before discharge from the hospital? a. Monitoring the incision lines for infection or leakage of saliva

b. Assessing the client for readiness to resume oral feedings c. Cleaning the tracheostomy and suctioning as needed

d. Administering tube feedings and cleaning the feeding tube site

The spouse of a client has just completed tracheostomy care for the first time, with minimal assistance from the nurse. Which statement offers the

most constructive feedback from the nurse? a. “I see that you had a tough time, but you will do better with

practice.” b. “You were able to clean the inner cannula well. Now, let’s change the

ties again.”

c. “You seem to have had a tough time because it was your first attempt.”

d. “You seem to understand what I said. Is there anything else I can help you with?”

The nurse is caring for a client who has just completed treatment for basal cell carcinoma on the lower lip. The client says to the nurse,

“Cigarettes are ruining my life. I’ll do anything to quit smoking.” Which is the nurse’s best response?

a. “Here is some information about smoking cessation programs in the area. Let’s discuss the options.”

b. “Here are some pamphlets that show the financial benefits of quitting smoking.”

c. “If you quit smoking, your risk for developing cancer again will decrease dramatically.”

d. “Your chest x-ray is still clear, so you could prevent permanent lung damage if you quit smoking now.”

The nurse is leading a teaching session about methods to decrease the risk of mouth cancer. Which client statement indicates that the nurse was

successful in teaching the information? a. “I will chew tobacco rather than smoke it.”

b. “I will use sugar rather than artificial sweeteners.” c. “I will regularly use a lip balm that contains sunscreen.”

d. “I will use a tanning salon rather than sunbathing at the beach.”

The nurse is caring for a client who will be taking nystatin (Mycostatin) for treatment of oral candidiasis. Which instructions does the nurse

provide for the client before administering the medication? a. “Let the tablet dissolve slowly in your mouth.”

b. “Take the medicine with a snack or a light meal.” c. “Swallow the pills whole, followed by a full glass of water.” d. “Swish the liquid around your mouth before swallowing it.”

The nurse is caring for a client who has undergone a radical jaw and neck resection. The client tells the nurse that the area feels very swollen and painful. Which is the best intervention for the nurse to make this

client more comfortable? a. Frequently suction the client’s mouth and airway.

b. Apply warm moist compresses to the area. c. Elevate the head of the client’s bed to semi-Fowler’s.

d. Administer ibuprofen (Motrin) 600 mg every 6 hours around the clock.

The nurse is caring for a client who will be undergoing a radical jaw and throat resection for oral cancer. Which statement by the client indicates

that further teaching is needed? a. “I will have a temporary tracheostomy placed in my neck to help me

breathe.” b. “I will not be able to get out of bed for 3 days after surgery.”

c. “The doctor will put in a feeding tube for nutrition until I can swallow and eat.”

d. “My speech may be slurred for a long time after the surgery.”

The nurse is caring for a client who has just undergone surgery for oral cancer. What advice does the nurse give the client to assist in maintaining

the airway? a. “Limit your fluids to 3 cups of water a day.”

b. “Take deep breaths, hold, then cough to mobilize any secretions.” c. “Lying flat in bed will be more comfortable for breathing.”

d. “Usually suctioning is not needed after oral surgery.”

The nurse is caring for a client who has just undergone a partial glossectomy and partial mandibulectomy for oral cancer. Which is the

highest priority for this client? a. Maintenance of the airway

b. Ability to communicate

c. Adequate body image d. Pain management

The nurse is assessing a client during a routine physical examination. Which statement made by the client concerning the risk of oral cancer

indicates that further teaching is needed? a. “I will brush my teeth and floss regularly.” b. “I will begin a smoking cessation program.”

c. “I can still use chewing tobacco since I stopped smoking.” d. “I will limit my intake of alcoholic beverages.”

The nurse is caring for a client with stomatitis. Which statement does the nurse include in teaching about oral care for this client?

a. “Rinse your mouth out twice a day with mouthwash.” b. “Clean your mouth frequently during the day with a gentle foam

sponge.” c. “Use lemon-glycerin swabs to clean your mouth after meals and at

bedtime.” d. “Suck on ice cubes to minimize the discomfort.”

MULTIPLE RESPONSE

The nurse is performing oral health screenings at a local community center. Which clients are at higher risk for developing oral cancer?

(Select all that apply.) a. Female who has taken oral contraceptives for the last 4 years

b. Adult client with a history of alcoholism c. Adult client who regularly eats spicy foods

d. Middle-aged male who smokes a pipe e. Adult client who goes to a tanning salon weekly

f. Client who frequently chews gum

The nurse is assessing a client with a salivary gland tumor for facial nerve involvement. Which movements does the nurse ask the client to

perform? (Select all that apply.) a. Open the mouth wide.

b. Raise the eyebrows. c. Smile or frown. d. Pucker the lips. e. Blow the nose.

f. Puff out the cheeks.

The nurse is caring for a client with stomatitis. Which items reported during the history of the client may contribute to the reoccurrence of this

condition? (Select all that apply.) a. Drinking two glasses of wine nightly b. Smoking three cigarettes each day

c. Vitamin A deficiency d. Drinking four cups of coffee daily

e. Fruits and nuts as the mainstay of the diet f. Vitamin C deficiency

Chapter 57: Care of Patients with Esophageal Problems

Test Bank

MULTIPLE CHOICE

What is the pH range of the distal esophagus? a. 1.5 to 2.0 b. 3.0 to 4.5 c. 4.5 to 6.0 d. 6.0 to 7.0

An obese client has reflux and asks how being overweight could cause this condition. Which response by the nurse is best?

a. “You eat more food, more often, than nonobese people do.” b. “The weight adds extra pressure, which helps push stomach

contents up.” c. “Obese people tend to eat more high-fat food, which presents a

risk.” d. “Obesity is not related to reflux, but losing weight would be

healthy.”

Which client does the nurse assess most carefully for the development of gastroesophageal reflux disease?

a. Client with atrial fibrillation who drinks decaffeinated coffee b. Client who has lost 20 pounds through diet and exercise

c. Diabetic client taking oral hypoglycemic agents d. Postoperative client who has a nasogastric (NG) tube

A client with esophageal reflux who experiences regurgitation while lying flat is at risk for which complication?

a. Erosion b. Bleeding

c. Aspiration d. Odynophagia

A client just experienced an episode of reflux with regurgitation. What assessment by the nurse is the priority? a. Auscultate the lungs for crackles.

b. Inspect the oral cavity. c. Check the oxygen saturation.

d. Teach the client to sleep sitting up.

The health care provider is prescribing medication to treat a client’s severe gastroesophageal reflux disease (GERD). Which medication does

the nurse anticipate teaching the client about? a. Magnesium hydroxide (Gaviscon)

b. Ranitidine (Zantac) c. Nizatidine (Axid)

d. Omeprazole (Prilosec)

A client is undergoing diagnostic testing for gastroesophageal reflux disease (GERD). Which test does the nurse tell the client is best for

diagnosing this condition? a. Endoscopy

b. Schilling test c. 24-Hour ambulatory pH monitoring

d. Stool testing for occult blood

A client has Barrett’s esophagus. Which client assessment by the nurse requires consultation with the health care provider?

a. Sleeping with the head of the bed elevated b. Coughing when eating or drinking

c. Wanting to eat several small meals during the day d. Chewing antacid tablets frequently during the day

The nurse is teaching a client about self-management of gastroesophageal reflux. Which statement by the nurse is most

appropriate? a. “Eat four to six small meals each day.”

b. “Eat a small evening snack 1 to 2 hours before bed.” c. “No specific foods or spices need to be cut from your diet.”

d. “You may include orange or tomato juice with your breakfast.”

The nurse is in the room of a client who is sleeping in bed. The client experiences an episode of reflux with regurgitation. Which action does

the nurse take first? a. Have the client roll to the side.

b. Raise the head of the client’s bed. c. Auscultate the client’s lung sounds.

d. Call the Rapid Response Team.

A client with severe gastroesophageal reflux disease (GERD) is still having symptoms of reflux despite taking omeprazole, (Prilosec) 20 mg

daily. What does the nurse do next? a. Document the finding in the client’s chart.

b. Obtain an order for omeprazole twice daily. c. Instruct the client to double the daily dose.

d. Tell the client to take antacids with omeprazole.

A client is admitted to the cardiac monitoring unit for a suspected

myocardial infarction. The client reports long-standing nighttime reflux, and the health care provider orders nizatidine (Axid) 150 mg twice a day.

Which action by the nurse is most appropriate? a. Consult with the health care provider because the dose is too high. b. Check the client’s kidney function tests before administering the

drug. c. Ask the pharmacist to recommend another histamine receptor

agonist. d. Give the medication as ordered and monitor for effectiveness.

A client who has undergone Nissen fundoplication for gastroesophageal reflux disease (GERD) is ready for discharge home. Which statement

made by the client indicates understanding of the disease? a. “I will no longer need any medication for my GERD.”

b. “I will avoid spicy foods because they can irritate the suture line.” c. “I should take anti-reflux medications when I eat a large meal.”

d. “I will need to continue to watch my diet and may still need medication.”

Which symptom indicates a need for immediate intervention in a client with a rolling hernia?

a. Reflux b. Crackles in the lungs

c. Distended and firm abdomen

d. Two episodes of diarrhea

Which statement indicates that the client understands the management of his or her sliding hiatal hernia?

a. “I will lie flat for 30 minutes after each meal.” b. “I will remain upright for several hours after each meal.”

c. “I will have my blood count done in 2 weeks to check for anemia.” d. “I will sleep at night while lying on my left side to prevent reflux.”

A client has returned to the nursing unit after a sliding hernia repair. Which action by the nurse is most important in preventing

complications? a. Range of motion to the lower extremities

b. Elevating the head of the bed to 30 degrees c. Monitoring input and output d. Assessing for bowel sounds

A client who has undergone a fundoplication wrap for hernia repair has returned from the postanesthesia care unit with a nasogastric tube

draining dark brown fluid. Which is the nurse’s priority action? a. Assess the placement of the tube.

b. Document the finding and continue to monitor. c. Clamp the nasogastric tube for 30 minutes.

d. Irrigate the nasogastric tube with normal saline.

A client had an open fundoplication 2 days ago. Which assessment by the nurse indicates that an important National Patient Safety Goal is being

met for this client? a. The client uses the spirometer during the shift.

b. The client’s pain is monitored and treated. c. The client has vital signs taken routinely.

d. The client verbalizes understanding of the discharge teaching.

A client who has undergone an open fundoplication hernia repair is preparing for discharge. Which information is most important for the

nurse to include in discharge instructions? a. “You can take laxatives for constipation.”

b. “Eat three normal-sized meals daily.” c. “Notify your health care provider if you get a cough.”

d. “You can go back to work in about a week.”

A client is admitted with progressive dysphagia. What intervention by the nurse takes priority?

a. Weigh the client daily. b. Instruct the client on a high-protein diet.

c. Assess and treat the client’s pain. d. Administer antitussive medications.

A client 2 hours post–esophageal dilation develops increasing pain in the throat. Which is the best action of the nurse?

a. Administer an analgesic. b. Document the finding. c. Reposition the client.

d. Assess the client for perforation.

Which factor places a client at risk for esophageal cancer? a. High-stress occupation

b. Preference for high-fat foods c. 20-pack-year smoking history

d. History of myocardial infarction

The nurse is performing an assessment of a client with suspected esophageal cancer. Which statement made by the client does the nurse

correlate with advanced disease? a. “I have difficulty swallowing solids.”

b. “I usually have a sticking feeling in my throat.” c. “I have difficulty swallowing soft foods.”

d. “I have difficulty swallowing liquids.”

A client has esophageal cancer. Which intervention by the nurse takes priority?

a. Maintaining nutritional intake b. Allowing grieving

c. Preventing aspiration d. Managing pain relief

A client with esophageal cancer and dysphagia states that it has become more difficult to swallow, and the client has experienced several choking episodes during meals. Which strategy would the nurse recommend to

assist this client in obtaining adequate nutrition? a. Monitor caloric intake and weigh the client daily.

b. Instruct the client to drink only clear liquids. c. Tell the client that artificial feeding will now be required.

d. Encourage the client to eat semisoft foods and thickened liquids.

A client with esophageal cancer is receiving radiation therapy. Which finding alerts the nurse to a possible complication in this client?

a. Redness of the skin at the site of radiation b. Worsening of dysphagia or odynophagia

c. Development of nausea or vomiting d. A profound feeling of tiredness

A client has undergone an esophagogastrostomy for cancer of the esophagus. How will the nurse best support the client’s respiratory

status? a. Assessing the client’s breath sounds every 4 hours

b. Performing chest physiotherapy every 6 hours c. Maintaining the client in a supine position

d. Administering analgesia regularly

The nurse notes frank red blood in the drainage container from the nasogastric (NG) tube of a client who is 2 days post-

esophagogastrostomy. Which is the nurse’s priority intervention? a. Irrigate the NG tube with cold saline. b. Document the drainage in the chart.

c. Reposition the tube in the opposite nostril. d. Assess the client’s vital signs and abdomen.

An older client is 1 day post-esophagectomy. The nurse finds the client short of breath with a heart rate of 120 beats/min. Which action by the

nurse takes priority? a. Assess the client’s lungs and oxygen saturation. b. Ask the client to rate pain, and treat if needed. c. Help the client change to a side-lying position.

d. Increase the client’s supplemental oxygen.

What does the nurse teach the client with esophageal diverticula about dietary needs?

a. “Eat soft foods and smaller meals.” b. “Only eat puréed foods.”

c. “Avoid drinking liquids with meals.” d. “Avoid dairy products.”

A client is admitted with a chemical injury to the esophagus after ingestion of an alkaline substance. The client states, “I am having trouble breathing because of these air bubbles in my neck.” Which action by the

nurse is most appropriate? a. Continue assessing the client while another nurse calls the health

care provider. b. Ask the client to rate the pain and prepare to administer pain

medication. c. Have the client cough and deep breathe, then assess his or her lung

sounds. d. Give the client small sips of water to see whether he or she has

dysphagia.

A client has been diagnosed with early esophageal cancer. The nurse plans care by implementing measures designed to address which priority

concern? a. Nutritional support b. Pulmonary toileting

c. Fluid and electrolyte balance d. Educational needs

MULTIPLE RESPONSE The nurse is obtaining the history of a client with a sliding hernia. Which

symptoms does the nurse expect to see in this client? (Select all that apply.)

a. Reflux b. Bleeding

c. Dysphagia d. Belching

e. Breathlessness f. Vomiting

Which interventions can the nurse delegate to unlicensed personnel when caring for a client with esophageal cancer? (Select all that apply.)

a. Maintaining intake and output b. Maintaining calorie count

c. Administering tube feeding d. Obtaining vital signs

e. Teaching changes in daily activities f. Changing the incision dressing

Which referrals does the nurse make for an older adult client who is being discharged with esophageal cancer? (Select all that apply.)

a. IV infusionist b. Home health aide

c. Medicare or Medicaid d. Meals on Wheels

e. Housecleaning service f. Transportation to and from treatment

Chapter 58: Care of Patients with Stomach Disorders

Test Bank

MULTIPLE CHOICE

The nurse is caring for a client who is at risk for developing gastritis. Which finding from the client’s history leads the nurse to this

conclusion? a. Client is lactose intolerant and cannot drink milk.

b. Client recently traveled to Mexico and South America. c. Client works at least 60 hours per week in a stressful job.

d. Client takes naproxen sodium (Naprosyn) 500 mg daily for arthritis pain.

The nurse is caring for a female client who has just received a prescription for misoprostol (Cytotec). Which instructions does the nurse

provide to the client regarding this medication? a. “You may dissolve the contents of the capsule in warm water if it is

difficult for you to swallow.” b. “Take this medication on an empty stomach just before going to bed

every evening.” c. “You will need to stop taking your magnesium hydroxide (Mylanta)

now that you are on this drug.” d. “You should add extra fiber to your diet because this medication

may cause constipation.”

The nurse is caring for a client with a gastric ulcer who suddenly develops sharp mid-epigastric pain. The nurse notes that the client’s

abdomen is hard and very tender to light palpation. Which is the priority action of the nurse?

a. Place the client in a knee-chest position. b. Prepare the client for emergency surgery.

c. Insert a nasogastric (NG) tube to low intermittent suction. d. Assess the client’s pain and administer analgesics.

The nurse is caring for a client with peptic ulcer disease. The client vomits a large amount of undigested food after breakfast. Which

intervention does the nurse prepare to provide for the client? a. Administer a soap suds cleansing enema.

b. Change the client’s diet to clear liquids only. c. Insert a nasogastric (NG) tube to low intermittent suction.

d. Administer prochlorperazine (Compazine) 10 mg IM.

The home care nurse is caring for a client who has recently undergone a subtotal gastrectomy. The nurse notes that the client’s tongue is shiny

and beefy red. Which assessment question does the nurse ask the client regarding this finding?

a. “Have you been taking your multivitamin every day?” b. “How much weight have you lost since your surgery?”

c. “Have you been experiencing heartburn or nausea after eating?” d. “What kind of mouthwash do you use after you brush your teeth?”

The nurse is providing discharge teaching for a client who has peptic ulcer disease caused by Helicobacter pylori infection. Which statement

by the client indicates that additional teaching is needed? a. “I will avoid drinking coffee, even if it is decaffeinated.”

b. “I will take a multivitamin every morning with breakfast.” c. “I will go to my tai chi class to wind down after a busy day.”

d. “I will take my medication every day until my heartburn is gone.”

The nurse is teaching a health promotion class about preventing cancer. Which statement by a student indicates understanding of gastric cancer

development? a. “I should skip my morning bacon and egg sandwich to reduce my

risk of gastric cancer.” b. “I have been lactose intolerant for many years, so I should have a

yearly test for gastric cancer.” c. “I should switch from regular to decaffeinated coffee to reduce my

risk of gastric cancer.” d. “I am at low risk for developing gastric cancer because I am a

vegetarian and I eat only organic produce.”

An older client is admitted to the hospital with acute gastritis. The health care provider orders magnesium hydroxide (Mylanta) 1 hour and 3 hours after meals and at bedtime. Which action by the nurse is most

appropriate? a. Check the client’s renal function studies before giving the drug.

b. Call the health care provider and ask for a different antacid for the client.

c. Assess the client’s pain and treat pain if present.

d. Assist the client in ordering bland food from the menu.

The nurse is caring for a client with chronic gastritis. The client asks the nurse how to prevent another flare-up of gastritis. Which is the nurse’s

best response? a. “Join a support group to help you stop smoking.”

b. “Take a multivitamin with iron and folic acid every day.” c. “Make sure to include plenty of fresh vegetables in your diet.”

d. “Make sure that your weight stays within normal limits.”

The nurse is caring for a client with peptic ulcer disease. Which assessment finding indicates to the nurse that the client most likely has

an ulcer in the stomach rather than in the duodenum? a. Body mass index (BMI) is 16.6.

b. Stool is positive for occult blood. c. Client has had four ulcers in the last 5 years.

d. Hemoglobin is 13 g/dL and hematocrit is 42%.

The nurse is caring for a client who has been brought to the emergency department with upper GI bleeding. The client is vomiting copious amounts of bright red blood. Which is the nurse’s priority action?

a. Ensure that the client has a patent airway. b. Start a normal saline IV infusion.

c. Gather equipment to start a saline lavage. d. Assess the client for causative factors.

The nurse is caring for a client who has just arrived in the emergency department reporting epigastric pain. The client says that emesis earlier in the day looked like coffee grounds. What does the nurse prepare to do

for the client first? a. Check the client’s stool for occult blood.

b. Insert 18-gauge IV lines with normal saline infusions. c. Insert a nasogastric tube and prepare for gastric lavage.

d. Determine whether the client has a history of ulcers.

The nurse is caring for a client who presents with chronic epigastric pain, heartburn, and anorexia. The client asks the nurse how the doctor can best determine whether the symptoms are caused by gastritis. Which

is the nurse’s best response? a. “You will be asked to drink a barium solution while x-rays are taken

of your stomach.”

b. “The doctor will take a look inside your stomach using a tube with a light on the end of it.”

c. “A CT scan of your abdomen will show whether inflammation is present in your stomach.”

d. “A blood sample will be sent to the laboratory to determine whether you have a stomach infection or bleeding.”

The nurse is caring for a client with a history of heart failure and chronic gastritis. The client tells the nurse about taking 2 teaspoons of sodium

bicarbonate every night before going to bed to prevent heartburn. Which is the nurse’s best response?

a. “You should let the doctor know right away if you develop diarrhea.”

b. “I will let your doctor know so a safer antacid can be prescribed for you.”

c. “Do not take that with milk, because the combination can cause kidney stones.”

d. “Make sure that you mix the sodium bicarbonate with at least 8 ounces of water.”

The nurse is caring for a client with suspected upper GI bleeding. The nurse inserts a nasogastric (NG) tube for gastric lavage and checks

placement of the tube in the stomach. When fluid is aspirated from the tube, the pH is found to be 6. Which is the priority action of the nurse?

a. Obtain an order for a stat chest x-ray. b. Auscultate over the lung fields bilaterally.

c. Assess whether the tube is coiled in the client’s throat. d. Auscultate over the epigastric area while instilling air.

A client has been taking an antacid for several weeks without improvement in symptoms. Which response by the nurse is most helpful?

a. “Tell me exactly how you take your antacid.” b. “Would you be willing to try a more expensive medication?”

c. “Are you sure you are taking this exactly as ordered?” d. “Let’s ask the health care provider if the dose can be doubled.”

The nurse is caring for a client with gastritis who will undergo a nuclear medicine GI bleeding study in the morning. What instruction for

preparation does the nurse give the client? a. “You cannot eat anything after midnight.”

b. “You should drink several glasses of water in the morning.” c. “You must make arrangements for transportation home.”

d. “No special preparations are required for this test.”

The nurse is caring for a client who has recently undergone a partial gastrectomy. The client reports becoming dizzy and sweaty with heart palpitations about 2 hours after eating. The client is now afraid to eat

anything. Which is the nurse’s best response? a. “Drink at least 6 ounces of fluid before each meal.”

b. “Try a clear liquid diet for the next few days.” c. “You probably should avoid dairy products.”

d. “Limit carbohydrate intake with meals.”

The nurse is caring for a client who recently has undergone a partial gastrectomy. Two hours after eating lunch, the client becomes dizzy,

diaphoretic, and confused. Which is the nurse’s priority action? a. Check the client’s blood sugar level. b. Increase the client’s IV infusion rate. c. Auscultate the client’s bowel sounds.

d. Place the client in high Fowler’s position.

The nurse is caring for a client with advanced gastric cancer who is scheduled for palliative surgery to relieve gastric outlet obstruction. The client asks the nurse why he should bother having the surgery, because

he will not be cured. Which is the nurse’s best response?

a. “It will allow the doctors to determine more accurately how long you have to live.”

b. “It will relieve the obstruction so you will be more comfortable and able to eat again.”

c. “It will remove much of the tumor so that chemotherapy will be more effective.”

d. “It will help prevent the tumor from spreading to other parts of your body.”

The nurse is caring for a client who will be discharged from the hospital following surgery for advanced gastric cancer. The client’s daughter verbalizes the fear that she will not be able to manage her parent’s symptoms adequately at home. Which is the nurse’s best response?

a. “The nurses have taught you everything you need to know to care for your parent.”

b. “Don’t worry, the pain pills will keep your parent comfortable until the end.”

c. “I will ask the social worker to arrange for a hospice nurse to help you at home.”

d. “I will ask the health care provider to review the care instructions with you again.”

The nurse is caring for a client who reports persistent epigastric pain, heartburn, and nausea, despite faithfully taking ranitidine (Zantac),

aluminum hydroxide (Amphojel), and metronidazole (Flagyl) as prescribed. Which is the nurse’s best response?

a. “Is your pain better or worse after you eat?” b. “Have you tried elevating the head of your bed at night?”

c. “Have you been taking the Amphojel and Flagyl together?” d. “Have you been experiencing foul-smelling diarrhea lately?”

A client with Zollinger-Ellison syndrome will be admitted to the medical unit. Which intervention does the nurse include in the client’s nursing

plan of care? a. Performing a urine test for ketones every morning before breakfast

b. Performing perineal care and applying a moisture barrier twice daily

c. Assessing the abdomen for fluid wave and shifting dullness every 8 hours

d. Keeping 2 units of packed red blood cells on hold at all times

A client has returned to the nursing unit after esophagogastroduodenoscopy (EGD). Which action by the nurse takes

priority? a. Keep the client on strict bedrest for 8 hours.

b. Delegate taking vital signs to the nursing assistant. c. Increase the IV rate to flush the kidneys.

d. Assess the client’s gag reflex.

The nurse is caring for a client who has received multiple serious injuries in a motor vehicle accident. The client asks the nurse why ranitidine

(Zantac) is prescribed because she does not have any abdominal pain. Which is the nurse’s best response?

a. “It will help prevent the development of a stomach ulcer from the stress of your injuries.”

b. “It will help prevent aspiration pneumonia when you are anesthetized during surgery tomorrow.”

c. “It will help your throat heal after it was irritated from the nasogastric tube.”

d. “It will help prevent nausea and vomiting from the narcotic pain medications that you are taking.”

The nurse is to insert a nasogastric (NG) tube for a client with upper GI bleeding. Which instruction does the nurse give to the client before

starting the procedure? a. “You may take some sips of water when I begin to insert the tube

into your nose.” b. “Please hold your breath when I insert the tube through your nose.” c. “Tilt your head down to your chest when the tube gets to the back of

your throat.” d. “The distance from the end of your nose to your navel tells me which

size tube to use.”

MULTIPLE RESPONSE

The nurse is caring for a client who has recently undergone a partial gastrectomy. The client asks the nurse which foods would be best for him to have for breakfast. Which menu items does the nurse recommend for

the client? (Select all that apply.) a. Blueberry pancakes with maple syrup

b. A half-grapefruit with a blueberry muffin c. Plain bagel with margarine or butter

d. Raisin bran with milk and artificial sweetener e. Scrambled eggs with cheese and a slice of bacon f. One half cup of cottage cheese with canned pears

The nurse is caring for a client with a nasogastric (NG) tube after an episode of GI bleeding. Which interventions are included in the nursing

care plan? (Select all that apply.) a. Monitor and record intake and output every 8 hours.

b. Monitor hemoglobin and hematocrit laboratory values. c. Ensure that suction is set on high continuous for Levin tubes. d. Measure the client’s girth and/or assess for distention daily. e. Pin the tube to the client’s gown, so it cannot be dislodged.

f. Check vital signs and orthostatic blood pressure every 4 hours and PRN.

The nurse is caring for a client who will undergo a gastrectomy the following day. Which interventions are included in the postoperative

plan of care for the client? (Select all that apply.) a. Monitor and record accurate intake and output (I&O).

b. Remind the client to use the incentive spirometer twice daily. c. Change abdominal dressings daily using medical asepsis.

d. Remind the client daily to use patient-controlled analgesia (PCA) before pain becomes severe.

e. Keep the head of the client’s bed elevated whenever possible. f. Irrigate the nasogastric tube with normal saline every 8 hours PRN.

Chapter 59: Care of Patients with Noninflammatory Intestinal Disorders

Test Bank

MULTIPLE CHOICE

A client has irritable bowel syndrome. Which menu selections by this client indicate good understanding of dietary teaching?

a. Tuna salad on white bread, cup of applesauce, glass of diet cola b. Broiled chicken with brown rice, steamed green beans, glass of apple

juice c. Grilled cheese sandwich, small ripe banana, cup of hot tea with

lemon d. Grilled steak, green beans, dinner roll with butter, cup of coffee with

cream

The nurse is performing a physical examination on a client. Which assessment finding leads the nurse to check the client’s abdomen for the

presence of an acquired umbilical hernia? a. Body mass index (BMI) of 41.9

b. Cholecystectomy last year c. History of irritable bowel syndrome

d. Daily dose of lansoprazole (Prevacid) 30 mg orally

The nurse notes a bulge in a client’s groin that is present when the client stands and disappears when the client lies down. Which conclusion does

the nurse draw from these assessment findings? a. Reducible inguinal hernia b. Indirect umbilical hernia

c. Strangulated ventral hernia

d. Incarcerated femoral hernia

The nurse is caring for a client with an umbilical hernia who reports increased abdominal pain, nausea, and vomiting. The nurse notes high- pitched bowel sounds. Which conclusion does the nurse draw from these

assessment findings? a. Bowel obstruction; client should be placed on NPO status. b. Perforation of the bowel; client needs emergency surgery.

c. Adhesions in the hernia; client needs elective surgery. d. Hernia is dangerously enlarged; client needs a nasogastric (NG)

tube.

The nurse is teaching a client how to use a truss for a femoral hernia. Which statement by the client indicates the need for further teaching?

a. “I will put on the truss before I go to bed each night.” b. “I will put some powder under the truss to avoid skin irritation.”

c. “The truss will help my hernia because I can’t have surgery.” d. “If I have abdominal pain, I will let my health care provider know

right away.”

The nurse is providing preoperative teaching for a client who will undergo herniorrhaphy surgery. Which instruction does the nurse give

to the client? a. “Eat a low-residue diet for the first week after surgery.”

b. “Change the dressing every day until the staples are removed.” c. “Take acetaminophen (Tylenol) 1000 mg every 4 hours for pain.” d. “Cough and deep breathe every 2 hours for the first week after

surgery.”

The nurse is performing a physical assessment for a client who underwent a hemorrhoidectomy the previous day. The nurse notes that the client has lower abdominal distention accompanied by dullness to

percussion over the distended area. Which is the nurse’s priority action? a. Assess the client’s vital signs.

b. Determine the last time the client voided. c. Insert a rectal tube to facilitate passage of flatus.

d. Document the findings in the client’s chart.

The nurse is screening clients at a community health fair. Which client is at highest risk for development of colorectal cancer?

a. Young adult who drinks eight cups of coffee every day

b. Middle-aged client with a history of irritable bowel syndrome c. Older client with a BMI of 19.2 who works 65 hours per week

d. Older client who travels extensively and eats fast food frequently

The nurse is performing a physical assessment of a client with a new diagnosis of colorectal cancer. The nurse notes the presence of visible

peristaltic waves and, on auscultation, hears high-pitched bowel sounds. Which conclusion does the nurse draw from these findings?

a. The tumor has metastasized to the liver and biliary tract. b. The tumor has caused an intussusception of the intestine.

c. The growing tumor has caused a partial bowel obstruction. d. The client has developed toxic megacolon from the growing tumor.

The nurse is caring for a client who is scheduled to have fecal occult blood testing. Which instructions does the nurse give to the client?

a. “You must fast for 12 hours before the test.” b. “You will be given a cleansing enema the morning of the test.”

c. “You must avoid eating meat for 48 hours before the test.” d. “You will be sedated and will require someone to accompany you

home.”

A client who has had fecal occult blood testing tells the nurse that the test was negative for colon cancer and wishes to cancel a colonoscopy scheduled for the next day. Which is the nurse’s best response?

a. “I will call and cancel the test for tomorrow.” b. “You need two negative fecal occult blood tests.”

c. “This does not rule out the possibility of colon cancer.” d. “You should wait at least a week to have the colonoscopy.”

The nurse is caring for a client who has been newly diagnosed with colon cancer. The client has become withdrawn from family members. Which

strategy does the nurse use to assist the client at this time? a. Ask the health care provider for a psychiatric consult for the client.

b. Explain the improved prognosis for colon cancer with new treatment.

c. Encourage the client to verbalize feelings about the diagnosis. d. Allow the client to remain withdrawn as long as he or she wishes.

The nurse is caring for a client with colon cancer and a new colostomy. The client wishes to talk with someone who had a similar experience.

Which is the nurse’s best response?

a. “Most people who have had a colostomy are reluctant to talk about it.”

b. “I will make a referral to the United Ostomy Associations of America.”

c. “You can get all the information you need from the enterostomal therapist.”

d. “I do not think that we have any other clients with colostomies on the unit right now.”

The nurse is caring for a client who has suffered abdominal trauma in a motor vehicle crash. Which laboratory finding indicates that the client’s

liver was injured? a. Serum lipase, 49 U/L

b. Serum amylase, 68 IU/L c. Serum creatinine, 0.8 mg/dL

d. Serum transaminase, 129 IU/L

The nurse is caring for a client who is brought to the emergency department following a motor vehicle crash. The nurse notes that the client has ecchymotic areas across the lower abdomen. Which is the

priority action of the nurse? a. Measure the client’s abdominal girth.

b. Assess for abdominal guarding or rigidity. c. Check the client’s hemoglobin and hematocrit.

d. Ask whether the client was riding in the front or back seat of the car.

A client tells the nurse that her husband is repulsed by her colostomy and refuses to be intimate with her after surgery. Which is the nurse’s best

response? a. “Let’s talk to the ostomy nurse to help you and your husband work

through this.” b. “You could try to wear longer lingerie that will better hide the

ostomy appliance.” c. “You should empty the pouch first so it will be less noticeable for

your husband.” d. “If you are not careful, you can hurt the stoma if you engage in

sexual activity.”

The nurse is caring for a client who just had colon resection surgery with a new colostomy. Which teaching objective does the nurse include in the

client’s plan of care? a. Understanding colostomy care and lifestyle implications

b. Learning how to change the appliance independently c. Demonstrating the correct way to change the appliance by discharge

d. Not being afraid to handle the ostomy appliance tomorrow

The nurse is caring for a client who has been diagnosed with a bowel obstruction. Which assessment finding leads the nurse to conclude that

the obstruction is in the small bowel? a. Potassium of 2.8 mEq/L, with a sodium value of 121 mEq/L

b. Losing 15 pounds over the last month without dieting c. Reports of crampy abdominal pain across the lower quadrants

d. High-pitched, hyperactive bowel sounds in all quadrants

A client post-hemorrhoidectomy feels the need to have a bowel movement. Which action by the nurse is best? a. Have the client use the bedside commode.

b. Stay with the client, providing privacy. c. Make sure toilet paper and the call light are in reach.

d. Plan to send a stool sample to the laboratory.

A client is brought to the emergency department after being shot in the abdomen and is hemorrhaging heavily. Which action by the nurse is the

priority? a. Draw blood for type and crossmatch.

b. Start two large IVs for fluid resuscitation. c. Obtain vital signs and assess skin perfusion.

d. Assess and maintain a patent airway.

A client with a mechanical bowel obstruction reports that abdominal pain, which was previously intermittent and colicky, is now more

constant. Which is the priority action of the nurse? a. Measure the abdominal girth.

b. Place the client in a knee-chest position. c. Medicate the client with an opioid analgesic.

d. Assess for bowel sounds and rebound tenderness.

The nurse is teaching self-care measures for a client who has hemorrhoids. Which nursing intervention does the nurse include in the

plan of care for the client? a. Instruct the client to use dibucaine (Nupercainal) ointment

whenever needed. b. Teach the client to choose low-fiber foods to make bowels move

more easily. c. Tell the client to take his or her time on the toilet when needing to

defecate. d. Encourage the client to dab with moist wipes instead of wiping with

toilet paper.

The nurse is caring for a client who is to receive 5-fluorouracil (5-FU) chemotherapy IV for the treatment of colon cancer. Which assessment

finding leads the nurse to contact the health care provider? a. White blood cell (WBC) count of 1500/mm3

b. Presence of fatigue with a headache c. Presence of slight nausea and no appetite

d. Two diarrhea stools yesterday

A client who has had a colostomy placed in the ascending colon expresses concern that the effluent collected in the colostomy pouch has remained

liquid for 2 weeks after surgery. Which is the nurse’s best response? a. “This is normal for your type of colostomy.”

b. “I will let the health care provider know, so that it can be assessed.” c. “You should add extra fiber to your diet to stop the diarrhea.”

d. “Your stool will become firmer over the next few weeks.”

A middle-aged male client has irritable bowel syndrome that has not

responded well to diet changes and bulk-forming laxatives. He asks the nurse about the new drug lubiprostone (Amitiza). What information

does the nurse provide him? a. “This drug is investigational right now for irritable bowel

syndrome.” b. “Unfortunately, this drug is approved only for use in women.”

c. “Lubiprostone works well only in a small fraction of irritable bowel cases.”

d. “Let’s talk to your health care provider about getting you a trial prescription.”

The nurse is caring for a client who has undergone removal of a benign colonic polyp. The client asks the nurse why a follow-up colonoscopy is

necessary. Which is the nurse’s best response? a. “You are at risk for developing more polyps in the future.”

b. “You may have other cancerous lesions that could not be seen right now.”

c. “The doctor can remove only a few of the polyps during each colonoscopy.”

d. “This test will ensure that you have healed where the polyp was removed.”

MULTIPLE RESPONSE

The nurse is helping a student prepare to insert a nasogastric tube for an adult client with a bowel obstruction. Which actions by the student

indicate to the nurse that a review of the procedure is needed? (Select all that apply.)

a. Gathering supplies, including an 8 Fr Levin tube, sterile gloves, tape, and water-soluble lubricant

b. Performing hand hygiene and positioning the client in high Fowler’s position, with pillows behind the head and shoulders

c. Attaching a 60-mL irrigation syringe to the end of the nasogastric tube before inserting it into the nose

d. Instructing the client to extend the neck against the pillow once the nasogastric tube has reached the oropharynx

e. Checking for correct placement by checking the pH of the fluid aspirated from the tube

f. Securing the nasogastric tube by taping it to the client’s nose and pinning the end to the pillowcase

g. Connecting the nasogastric tube to intermittent medium suction with an anti-reflux valve on the air vent

The nurse is providing discharge teaching for a client who has undergone colon resection surgery with a colostomy. Which statements by the client

indicate that the instruction was understood? (Select all that apply.) a. “I will change the ostomy appliance daily and as needed.”

b. “I will use warm water and a soft washcloth to clean around the stoma.”

c. “I will start bicycling and swimming again once my incision has healed.”

d. “I will notify the doctor right away if any bleeding from the stoma occurs.”

e. “I will check the stoma regularly to make sure that it stays a deep red color.”

f. “I will avoid dairy products to reduce gas and odor in the pouch.” g. “I will cut the flange so it fits snugly around the stoma to avoid skin

breakdown.”

COMPLETION

A client is to receive 12 mg/kg of 5-fluorouracil (5-FU) chemotherapy IV for treatment of colon cancer. The client weighs 132 lb. The client will

receive milligrams of 5-FU.

Chapter 60: Care of Patients with Inflammatory Intestinal Disorders

Test Bank

MULTIPLE CHOICE

The nurse conducts a physical assessment for a client with abdominal pain. Which finding leads the nurse to suspect appendicitis?

a. Severe, steady right lower quadrant (RLQ) pain b. Abdominal pain that started a day after vomiting began

c. Abdominal pain that increases with knee flexion d. Marked peristalsis and hyperactive bowel sounds

The nurse conducts a physical assessment for a client with severe right lower quadrant (RLQ) abdominal pain. The nurse notes that the

abdomen is rigid and the client’s temperature is 101.1° F (38.4° C). Which laboratory value does the nurse bring to the attention of the

health care provider as a priority? a. A “left shift” in the white blood cell count

b. White blood cell count, 22,000/mm3 c. Serum sodium, 149 mEq/L

d. Serum creatinine, 0.7 mg/dL

The nurse is caring for an older client with Salmonella food poisoning. Which is the priority action of the nurse?

a. Monitor vital signs. b. Maintain IV fluids.

c. Provide perineal care.

d. Initiate Isolation Precautions.

The nurse is caring for a client who is having approximately 20 foul- smelling stools each day. Laboratory Gram stain testing indicates the

presence of white blood cells (WBCs) and red blood cells (RBCs) in the stool. Which organism does the nurse expect to see in the culture report?

a. Helicobacter pylori b. Campylobacter jejuni

c. Clostridium botulinum d. Norwalk virus

The nurse is caring for a client who has acute viral gastroenteritis. Which dietary instruction does the nurse provide to the client?

a. “Drink plenty of fluids to prevent dehydration.” b. “You can have only clear liquids to drink.” c. “Milk products will give you extra protein.”

d. “You can have sips of cola or tea to relieve nausea.”

The nurse provides discharge teaching for a client who was hospitalized for Salmonella food poisoning. Which client statement indicates that

additional teaching is needed? a. “I will let my husband do the cooking for my family.”

b. “I will take the ciprofloxacin (Cipro) until the diarrhea has resolved.”

c. “I will wash my hands with antibacterial soap before and after each meal.”

d. “I will make sure that my dishes go straight into the dishwasher after each meal.”

The nurse is caring for a client who is hospitalized with exacerbation of Crohn’s disease. What does the nurse expect to find during the physical

assessment? a. Positive Murphy’s sign with rebound tenderness

b. Dullness in the lower abdominal quadrants c. High-pitched, rushing bowel sounds in the right lower quadrant

d. Abdominal cramping that the client says is worse at night

A female client is admitted with an exacerbation of ulcerative colitis. Which laboratory value does the nurse correlate with this condition?

a. Potassium, 5.5 mEq/L b. Hemoglobin, 14.2 g/dL

c. Sodium, 144 mEq/L d. Erythrocyte sedimentation rate (ESR), 55 mm/hr

The nurse helps a client with diverticular disease choose appropriate dinner options. Which menu selections are most appropriate?

a. Roasted chicken, rice pilaf, cup of coffee with cream b. Spaghetti with meat sauce, fresh fruit cup, hot tea with lemon c. Chicken Caesar salad, cup of bean soup, glass of low-fat milk

d. Baked fish with steamed asparagus, dinner roll with butter, glass of apple juice

The nurse is teaching a client how to care for a new ileostomy. Which client statement indicates that additional teaching is needed?

a. “I will consult the pharmacist before filling any new prescriptions.” b. “I will empty the ostomy pouch when it is half-filled with stool or

gas.” c. “I will wash my hands with antibacterial soap before and after

ostomy care.” d. “I will call my health care provider if I have not had ostomy

drainage for 3 hours.”

The nurse is caring for a teenage girl with a new ileostomy. She tells the nurse tearfully that she cannot go to the prom with an ostomy. Which is

the nurse’s best response? a. “You should get your prom dress one size larger to hide the ostomy

appliance.” b. “You should avoid broccoli and carbonated drinks so that the pouch

won’t fill with air under your dress.” c. “Let’s talk to the enterostomal therapist (ET) about options for

ostomy supplies and dress styles so that you can look beautiful for the prom.”

d. “You can remove the pouch from your ostomy appliance when you are at the prom so that it is less noticeable.”

The nurse is caring for a client who had ileostomy surgery 10 days ago. The client verbalizes concerns that the effluent has not become formed

and is still liquid green. Which is the nurse’s best response? a. “I will call your health care provider right away because the stool

should be semi-solid by now.” b. “Your stools will firm up in a few weeks as your body gets used to

the ileostomy.” c. “You should eat a high-fiber diet to help make the stool bulkier and

more solid.” d. “You can add buttermilk or yogurt to your diet and avoid

carbonated soft drinks.”

The nurse is caring for a client with perineal excoriation caused by diarrhea from acute gastroenteritis. Which client statement indicates

that additional teaching about perineal care is needed? a. “I will rinse my rectal area with warm water after each stool and

then apply zinc oxide ointment.” b. “I will clean my rectal area thoroughly with toilet paper after each

stool and then apply aloe vera gel.” c. “I will take a sitz bath three times a day and then pat my rectal area

gently but thoroughly to make sure I am dry.” d. “I will clean my rectal area with a soft cotton washcloth and then

apply vitamin A and D ointment.”

A client underwent the first stage of a restorative proctocolectomy with ileo-anal anastomosis (RPC-IPAA). What topic is a high priority for the

nurse to teach? a. Perineal care b. Ostomy care

c. Nutrition therapy d. Relaxation techniques

The nurse is caring for a client with severe ulcerative colitis who has been prescribed adalimumab (Humira). Which client statement indicates

that additional teaching about the medication is needed?

a. “I will avoid large crowds and people who are sick.” b. “I will take this medication with food or milk.”

c. “Nausea and vomiting are common side effects.” d. “I will wash my hands after I play with my dog.”

The nurse is caring for an older client with gastroenteritis. Which order does the nurse consult with the health care provider about?

a. IV 0.45% NS at 50 mL/hr b. Clear liquids as tolerated

c. Diphenoxylate hydrochloride/atropine sulfate (Lomotil) orally, after each loose stool

d. Acetaminophen (Tylenol), 325-650 mg orally every 4 hr PRN pain

The nurse is caring for a client who is taking mesalamine (5- aminosalicylic acid) (Asacol, Rowasa) for ulcerative colitis. The client has

trouble swallowing the pill. Which action by the nurse is most appropriate?

a. Crush the pill carefully and administer it to the client in applesauce or pudding.

b. Empty the contents of the capsule into applesauce or pudding for administration.

c. Contact the client’s health care provider to request an order for Asacol suspension.

d. Contact the client’s health care provider to request an order for

Rowasa enemas instead.

The nurse is caring for a client with ulcerative colitis and severe diarrhea. Which nursing assessment is the highest priority?

a. Skin integrity b. Blood pressure

c. Heart rate and rhythm d. Abdominal percussion

The nurse is preparing to begin teaching the client about how to care for a new ileostomy. Which consideration is the highest priority for the nurse

when planning teaching for this client? a. Informing the client about what to expect with basic ostomy care b. Starting the teaching after the client has received pain medication c. Starting the teaching when the client is ready to look at the stoma

d. Making sure that all needed supplies are ready at the client’s bedside

The nurse is caring for a client with Crohn’s disease and colonic strictures. Which assessment finding requires the nurse to consult the

health care provider immediately? a. Distended abdomen

b. Temperature of 100.0° F (37.8° C) c. Traces of blood in the stool

d. Crampy lower abdominal pain

The nurse is caring for a client with Crohn’s disease who has developed a fistula. Which nursing intervention is the highest priority?

a. Monitor the client’s hematocrit and hemoglobin. b. Position the client to allow gravity drainage of the fistula.

c. Check and record blood glucose levels every 6 hours. d. Encourage the client to consume a diet high in protein and calories.

A client with Crohn’s disease has a draining fistula. Which finding leads the nurse to intervene most rapidly? a. Serum potassium of 2.6 mEq/L

b. The client not wanting to eat anything c. White blood cell count of 8200/mm3 d. The client losing 3 pounds in a week

The nurse has completed the teaching session for a client with a new colostomy. Which feedback statement by the nurse is the most

appropriate? a. “I realize that you had a tough time today, but it will get easier with

practice.” b. “You cleaned the stoma well. Now you need to practice putting on

the appliance.” c. “You seem to understand what I taught you today. What else can I

help you with?” d. “You seem uncomfortable. Do you want your daughter to care for

your ostomy?”

The nurse reviews a health teaching for a client with Crohn’s disease. Which instruction does the nurse provide for the client? a. “You should have a colonoscopy every few years.”

b. “You should eat a diet that is high in protein and fiber.” c. “You should avoid heavy lifting and tight-fitting clothes.”

d. “You should take the Asacol whenever you have loose stools.”

A client has an anorectal abscess. Which teaching topic does the nurse address as the priority?

a. Perineal hygiene b. Comfort measures c. Nutrition therapy

d. Antibiotic use

A nurse is caring for a client hospitalized with botulism. The nurse obtains the following vital signs: temperature—99.8° F (37.6° C), pulse— 100, respiratory rate—10 and shallow, and blood pressure—100/62 mm

Hg. What action by the nurse is most appropriate? a. Allow the client rest periods without interruption.

b. Stay with the client while another nurse calls the physician. c. Check the client’s IV rate and document all findings.

d. Help the client order appropriate food items from the menu.

The nurse is preparing a client with diverticulitis for discharge from the hospital. Which statement by the client indicates that additional teaching

is needed? a. “I will ride my bike or take a long walk at least three times a week.”

b. “I will try to include at least 25 g of fiber in my diet every day.”

c. “I will take a senna laxative at bedtime to avoid becoming constipated.”

d. “I will use my legs rather than my back muscles when I lift heavy objects.”

The nurse has taught self-care measures to a client with an anal fissure. Which action by the client requires the nurse to do additional teaching?

a. Taking warm sitz baths several times daily b. Administering daily enemas to prevent constipation

c. Using bulk-producing agents to aid elimination d. Self-administering anti-inflammatory suppositories

A client is brought to the emergency department with an abrupt onset of vomiting, abdominal cramping, and diarrhea 2 hours after eating food at a picnic. Which infectious microorganism does the nurse suspect as the

probable cause? a. Salmonella

b. Giardia lamblia c. Staphylococcus aureus d. Clostridium botulinum

The nurse is caring for a client with Giardia lamblia infection. Which medication does the nurse anticipate teaching the client about?

a. Metronidazole (Flagyl) b. Ciprofloxacin (Cipro)

c. Sulfasalazine (Azulfidine) d. Ceftriaxone (Rocephin)

The nurse is caring for a client who has food poisoning that may be the result of Clostridium botulinum infection. Which is the priority nursing

assessment for this client? a. Heart rate and rhythm

b. Bowel sounds and heart tones c. Fluid balance and urine output

d. Oxygen saturation and respiratory rate

The nurse is caring for a client with a parasitic gastrointestinal infection. What statement by the client indicates a need for further teaching?

a. “I will have my housekeeper keep my toilet very clean.” b. “I need to shower or bathe every day.” c. “I need to have my well water tested.”

d. “My sexual partner needs to have a stool test.”

MULTIPLE RESPONSE

The client asks the nurse how to avoid becoming ill with Salmonella infection again. Which are appropriate responses from the nurse? (Select

all that apply.) a. “Wash leafy vegetables carefully before eating or cooking them.”

b. “Do not ingest water from the garden hose or the pool.” c. “Wash your hands before and after using the bathroom.”

d. “Be sure meat is cooked to the proper temperature.” e. “Avoid eating eggs that are sunny side up or undercooked.” f. “When eating outdoors, be sure to keep flies off your food.”

A nurse is teaching a community group ways to prevent Escherichia coli infection. Which statements made by the nurse are accurate? (Select all

that apply.) a. “Wash your hands after any contact with animals.”

b. “It is not necessary to buy a meat thermometer.” c. “Stay away from people who are ill with diarrhea.”

d. “Use separate cutting boards for meat and vegetables.” e. “Avoid swimming in backyard pools and using hot tubs.”

A nurse is teaching a community group about food poisoning and gastroenteritis. Which statements by the nurse are accurate? (Select all

that apply.) a. Rotavirus is more common among infants and younger children. b. Escherichia coli diarrhea is transmitted by contact with infected

animals. c. Don’t drink water when swimming to prevent E. coli infection.

d. All clients with botulism require hospitalization. e. Parasitic diseases may not show up for 1 to 2 weeks after infection.

COMPLETION

The nurse is to administer an infliximab (Remicade) infusion to a client who weighs 110 lb. The client is to receive 5 mg/kg of the drug, which is

available as a 100-mg/10 mL solution. The nurse will draw up milliliters of solution for the client’s infusion.

Chapter 61: Care of Patients with Liver Problems

Test Bank

MULTIPLE CHOICE

The nurse correlates which data in the client’s history as a predisposing factor for Laënnec’s cirrhosis?

a. Gallstones b. Alcohol abuse c. Viral hepatitis d. Heart disease

A client is admitted with cirrhosis and hepatopulmonary syndrome. Which clinical manifestation does the nurse monitor for progression or

resolution of this problem? a. Right upper quadrant pain

b. Crackles on auscultation c. Skin and scleral jaundice

d. Nausea and vomiting

A client is admitted with end-stage cirrhosis and severe vomiting. Which problem should the nurse monitor the client most carefully for?

a. Intrahepatic bile stasis b. Bleeding esophageal varices

c. Decreased excretion of bilirubin d. Accumulation of ascites in the abdomen

A client has cirrhosis and has developed ascites and edema. Which laboratory value does the nurse correlate with this condition?

a. Blood glucose, 120 mg/dL b. Serum sodium, 135 mEq/L c. Serum albumin, 2.1 g/dL

d. Blood urea nitrogen, 18 mg/dL

The client with end-stage cirrhosis presents with GI bleeding, combativeness, and confusion. The nurse anticipates an order to

administer which medication? a. Omeprazole (Prilosec)

b. Somatostatin (Octreotide) c. Propranolol (Inderal) d. Lactulose (Heptalac)

The nurse is reviewing a client’s history. Which statement by the client indicates a need for health teaching?

a. “I drink 1 to 2 glasses of red wine a week.” b. “Because of my arthritis, I take a lot of Tylenol.”

c. “One of my cousins died of liver cancer 10 years ago.” d. “I got a hepatitis vaccine before traveling last year.”

The nurse is assessing a client with mild liver disease. Which assessment does the nurse perform to detect the presence of ascites in this client?

a. Measure lower extremities to assess for edema. b. Inspect and palpate the abdomen for distention.

c. Palpate the abdomen in assessing for a fluid wave. d. Percuss the liver while listening for dullness.

The nursing care plan specifies obtaining abdominal girth measurements each shift. The nurse takes the measurement, but when compared with the previous measurement, the new finding is several millimeters off.

Which action by the nurse is best? a. Document the finding in the client’s chart.

b. Look to see when the client last had a dose of diuretic. c. Ensure that the client’s abdomen and flanks are marked with pen.

d. Obtain the measurement while the client sits upright.

The nurse recognizes that fetor hepaticus is consistent with which assessment finding?

a. Purpuric lesions on the extremities b. A fruity or musty breath odor c. Warm and bright red palms

d. Jaundice of the sclera

The nurse is assessing a client for asterixis. Which instruction to the client is most appropriate?

a. “Close your eyes and take turns touching your nose with your fingers.”

b. “Sit on the edge of the bed and hold your legs straight out for 30 seconds.”

c. “Extend your arm, flex your wrist upward, and extend your fingers.”

d. “Say ‘EEEEE’ while I listen to your lungs in the back on both sides.”

Which laboratory data does the nurse correlate with advanced disease in a client with cirrhosis?

a. Elevated serum protein level b. Elevated serum ammonia level

c. Decreased serum ammonia level d. Decreased lactate dehydrogenase level

Which laboratory findings does the nurse recognize as potentially causing complications of liver disease?

a. Elevated aspartate transaminase (AST) and lactate dehydrogenase (LDH) levels

b. Elevated prothrombin time and international normalized ratio (INR)

c. Decreased serum albumin and serum globulin levels d. Decreased serum alkaline phosphatase and alanine

aminotransferase (ALT) levels

The nurse monitors for which clinical manifestation in a client with a decreased fecal urobilinogen concentration?

a. Clay-colored stools b. Petechiae

c. Asterixis d. Melena

A client has cirrhosis. Which nursing intervention would be most effective in controlling ascites?

a. Monitoring intake and output b. Providing a low-sodium diet c. Increasing oral fluid intake

d. Weighing the client daily

A client had a paracentesis 1 hour ago. Which assessment finding requires action by the nurse? a. Urine output of 20 mL/hr

b. Systolic blood pressure increase of 10 mm Hg c. Respiratory rate drop from 18 to 14

d. A 3-pound drop in weight

A client just had a paracentesis. Which nursing intervention is a priority for this client?

a. Monitor urine output. b. Maintain bedrest as per protocol.

c. Position the client flat in bed. d. Secure the trocar to the abdomen with tape.

A client just returned to the nursing unit after having a trans-jugular intrahepatic portal-systemic shunt (TIPS) procedure. Which clinical

finding does the nurse expect to observe in this client? a. Decreased level of consciousness

b. Decreased urinary volume c. Increased blood pressure

d. Increased abdominal girth

A client is hemorrhaging from bleeding esophageal varices and has an esophagogastric tube. Which nursing intervention is the priority?

a. Keep the client sedated to prevent tube dislodgement. b. Maintain balloon pressure at between 15 and 20 mm Hg.

c. Irrigate the gastric lumen with normal saline. d. Maintain the client’s airway.

A client with an esophagogastric tube suddenly experiences acute respiratory distress. Which is the nurse’s first action?

a. Call the health care provider. b. Cut the balloon ports and remove the tube. c. Place the client upright and apply oxygen.

d. Reduce the balloon pressure slightly.

A client is bleeding from esophageal varices. The health care provider is arranging sclerotherapy for the client. Before the client goes to

interventional radiology, the nurse prepares to administer which medication?

a. Terlipressin (Glypressin) b. Enoxaparin (Lovenox) c. Lactulose (Heptalac)

d. Spironolactone (Aldactone)

A client is receiving an infusion of vasopressin (Pitressin) to treat bleeding esophageal varices. Which client complaint indicates to the

nurse that a serious adverse effect of the drug may be occurring? a. Acute nausea and vomiting

b. A pounding frontal headache c. Vertigo and syncope

d. Midsternal chest pain

A client with severe esophageal varices is scheduled for trans-jugular intrahepatic portal-systemic shunt (TIPS) insertion. The nurse

determines that teaching has been effective when the client makes which statement?

a. “I will be discharged home after I wake up completely.” b. “The procedure may be painful because I get only light sedation.”

c. “My liver will function normally within 8 hours of placement of the shunt.”

d. “I will be monitored closely for a while after the procedure is over.”

The nursing assistant is helping a client who has advanced cirrhosis with a bath and other hygiene. Which action by the assistant requires

intervention by the registered nurse? a. Helping the client apply lotion to dry skin areas

b. Giving the client a basin of warm water and soap to use c. Providing a soft toothbrush for oral care

d. Helping the client keep nails trimmed short

The nurse reviews laboratory results for a client with cirrhosis and finds the following: hematocrit, 72%; blood urea nitrogen (BUN), 42 mg/dL; and sodium, 166 mEq/L. Which action by the nurse is most appropriate?

a. Check the client’s blood pressure and pulse. b. Increase the client’s oral fluid intake.

c. Call the health care provider. d. Document the results in the chart.

A thin, cachectic-appearing client has hepatic portal-systemic encephalopathy (PSE). The family expresses distress that the client is

receiving so little protein in the diet. Which explanation by the nurse is most appropriate?

a. “A low-protein diet will help the liver rest and will restore liver function.”

b. “Less protein in the diet will help with the confusion.” c. “Despite looking so thin, protein will not help with weight gain.”

d. “Less protein is needed to prevent fluid from leaking into the abdomen.”

A client is refusing to take lactulose (Heptalac) because of diarrhea. Which is the nurse’s best response to this client?

a. “Diarrhea is expected; that’s how your body gets rid of ammonia.” b. “You may take Kaopectate liquid daily for loose stools.”

c. “Do not take any more of the medication until your stools firm up.”

d. “We will need to send a stool specimen to the laboratory.”

A client is receiving lactulose (Heptalac). Which laboratory value leads the nurse to intervene?

a. Serum potassium, 2.6 mEq/L b. Serum sodium, 132 mEq/L c. Serum glucose, 108 mg/dL

d. Blood urea nitrogen, 16 mg/dL

A client has been diagnosed with hepatitis A. The nurse evaluates that teaching regarding the disease is understood when the client makes

which statement? a. “Some medications have been known to induce hepatitis A.”

b. “I may have been exposed when we ate shrimp last weekend.” c. “I may have been infected through a recent blood transfusion.”

d. “My infection with Epstein-Barr virus can co-infect me with hepatitis A.”

The nurse is assessing health fair participants for risks for hepatitis. The nurse recognizes which client as being at greatest risk for developing

hepatitis B?

a. College student who has had several sexual partners b. Woman who takes acetaminophen daily for headaches

c. Businessman who travels frequently d. Older woman who has eaten raw shellfish

A client with hepatitis C is being treated with ribavirin (Copegus). What nursing action takes priority?

a. Educating the client on ways to remain complaint with the drug regimen

b. Teaching the client that transient muscle aching is a common side effect

c. Ensuring that the client returns to the clinic each week for follow-up care

d. Showing the client how to take and record a radial pulse for 1 minute

A client is admitted with jaundice and suspected hepatitis B. Which intervention does the nurse add to the client’s care plan?

a. Encourage rest during this period. b. Assist the client with ambulation.

c. Place the client on a clear liquid diet. d. Administer PRN prochlorperazine maleate (Compazine).

Which statement made by a client traveling to a nonindustrialized country indicates the need for further teaching regarding the prevention

of viral hepatitis? a. “I will drink bottled water while I’m gone.”

b. “I will not share my drinking glass.” c. “I should eat plenty of fresh fruits and vegetables.”

d. “I will use careful handwashing.”

The nurse monitors for which serologic marker in the client who is a carrier of chronic hepatitis B?

a. Anti-hepatitis C virus (HCV) antibodies b. Anti-hepatitis B (HBs) antibodies

c. Hepatitis B surface antigen (HBsAg) antibodies d. Hepatitis A virus (HAV) antibodies

A client is diagnosed with hepatitis B. Which information does the nurse include in the teaching plan as a priority?

a. “Avoid drinking any alcohol until the doctor says you can.” b. “You will need aggressive control of your serum lipids.”

c. “Once your lab work returns to normal, you can donate blood

again.” d. “Wash your hands well after handling meat and shellfish.”

A client is in the emergency department after a motor vehicle crash. In assessing the client, which clinical sign alerts the nurse to the presence of

possible liver trauma? a. Abdominal pain referred to the right shoulder

b. Left upper quadrant abdominal pain and swelling c. Abdominal pain referred to the spine and legs

d. Abdominal pain with accompanying rebound tenderness

A client is in the emergency department after a motor vehicle crash, and the nurse notices a “steering wheel mark” across the client’s chest.

Which action by the nurse is most appropriate? a. Ask the client where in the car he or she was during the crash.

b. Assess the client by gently palpating the abdomen for tenderness. c. Notify the laboratory to come draw blood for blood type and

crossmatch. d. Place the client on the stretcher in reverse Trendelenburg position.

The nurse recognizes which client as being at greatest risk for the development of carcinoma of the liver?

a. Middle-aged client with a history of diabetes mellitus b. Young adult client with a history of blunt liver trauma

c. Older adult client with a history of cirrhosis d. Older adult client with malnutrition

A client who underwent liver transplantation 2 weeks ago reports a temperature of 101° F (38.3° C) and right flank pain. Which is the

nurse’s best response? a. “The anti-rejection drugs you are taking made you susceptible to

infection.” b. “You should go to the hospital immediately to have your new liver

checked out.” c. “You should take an additional dose of cyclosporine today.”

d. “Take acetaminophen (Tylenol) every 4 hours until you feel better.”

A client who had a liver transplant a month ago is admitted with fever and tachycardia. Which medication does the nurse prepare to administer

to this client? a. Ceftriaxone (Rocephin)

b. Cyclosporine (Sandimmune) c. Azithromycin (Zithromax)

d. Ribavirin (Copegus)

The nurse is meeting a client post-liver transplantation for the first time and notices a tremor as they shake hands. The client states this has not

happened before. Which action by the nurse is most appropriate? a. Conduct a thorough assessment, then notify the surgeon of the

findings. b. Review today’s laboratory work, including liver function studies.

c. Assess the client’s vital signs, and offer acetaminophen if the client is febrile.

d. Perform an assessment of the client’s gross and fine motor skills.

Which statement by a client with alcohol-induced cirrhosis indicates the need for further teaching?

a. “I cannot drink any alcohol at all anymore.” b. “I need to avoid protein in my diet.”

c. “I should not take over-the-counter medications.” d. “I should eat small, frequent, balanced meals.”

A client admitted with hepatopulmonary syndrome is experiencing dyspnea but does not want oxygen increased because the client’s nose

keeps bleeding from it. The client becomes agitated when discussing this with the nurse. The client’s oxygen saturation is 92%. What intervention

by the nurse is best? a. Instruct the client to sit in as upright a position as possible.

b. Tell the client that humidity can be added, but that the oxygen must be worn.

c. Document the client’s refusal in the chart, and call the health care provider.

d. Call the health care provider to request an extra dose of the client’s diuretic.

A client is scheduled for a paracentesis. Which activity does the nurse delegate to the unlicensed assistive personnel?

a. Have the client sign the informed consent form. b. Assist the client to void before the procedure.

c. Help the client lie flat in bed, on the right side. d. Get the client into a chair after the procedure.

A client has liver cancer. Which statement by the client about treatment options demonstrates an accurate understanding?

a. “I guess it’s a good thing that surgery is usually so successful.” b. “I choose hepatic arterial infusion of chemo to limit side effects.”

c. “Because I have only local metastases, I am thinking about transplant.”

d. “This disease is so rare, no wonder no good treatments are available.”

MULTIPLE RESPONSE

The infection control nurse wants to decrease the number of health care professionals who contract viral hepatitis at work. Which actions does

the nurse initiate? (Select all that apply.) a. Strengthen policies related to consistent use of Standard

Precautions. b. Mandate hepatitis vaccination for workers in high-risk areas.

c. Implement a needleless system for IV therapy. d. Reduce the number of “sharps” needed for client care where

possible. e. Provide postexposure prophylaxis in a timely manner.

Chapter 62: Care of Patients with Problems of the Biliary System and Pancreas

Test Bank

MULTIPLE CHOICE

The client with obstructive jaundice asks the nurse why his skin is so itchy. Which is the nurse’s best response?

a. “Bile salts accumulate in the skin and cause the itching.” b. “Toxins released from an inflamed gallbladder lead to itching.”

c. “Itching is caused by the release of calcium into the skin.” d. “Itching is caused by a hypersensitivity reaction.”

The nurse is caring for a client with cholecystitis. Which assessment finding indicates to the nurse that the condition is chronic rather than

acute? a. Abdomen that is hyperresonant to percussion

b. Hyperactive bowel sounds and diarrhea c. Clay-colored stools and dark amber urine

d. Rebound tenderness in the right upper quadrant

A client is admitted for suspected cholecystitis. On reviewing laboratory results, the nurse notes that the client’s amylase is elevated. Which action

by the nurse is best? a. Document the finding in the chart.

b. Ask the client about drinking habits. c. Notify the health care provider. d. Place the client on clear liquids.

The nurse is providing discharge teaching for a client who has just undergone laparoscopic cholecystectomy surgery. Which statement by

the client indicates understanding of the instructions? a. “I will drink at least 2 liters of fluid a day.” b. “I need a diet without a lot of fatty foods.”

c. “I should drink fluids between meals rather than with meals.” d. “I will avoid concentrated sweets and simple carbohydrates.”

The nurse is caring for a client who has just undergone traditional cholecystectomy surgery and has a Jackson-Pratt (JP) drain in place.

The nurse notes serosanguineous drainage present in the drain. Which is the nurse’s priority action?

a. Gently milk the drain tubing. b. Notify the surgeon immediately.

c. Document the finding in the client’s chart. d. Irrigate the drain with sterile normal saline.

The nurse is providing discharge teaching for a client who will be going home with a T-tube following cholecystectomy surgery. Which statement

by the client indicates the need for additional teaching? a. “I will keep the drainage bag lower than the tube itself.”

b. “I will inspect the T-tube drainage site daily for signs of infection.” c. “I will be careful not to pull on the tube or to accidentally pull it

out.” d. “I will slowly pull about an inch of the tube out each day until it’s

out.”

The nurse is caring for a postoperative client who reports pain in the shoulder blades following laparoscopic cholecystectomy surgery. Which direction does the nurse give to the nursing assistant to help relieve the

client’s pain? a. “Ambulate the client in the hallway.”

b. “Apply a cold compress to the client’s back.” c. “Encourage the client to take sips of hot tea or broth.”

d. “Remind the client to cough and deep breathe every hour.”

The nurse is teaching a client with a history of cholelithiasis to select menu items for dinner. Which selections made by the client indicate that

the nurse’s teaching was effective? a. Lasagna, tossed salad with Italian dressing, 2% milk

b. Grilled cheese sandwich, tomato soup, coffee with cream c. Caesar salad with chicken, soft breadstick with butter, diet cola

d. Roasted chicken breast, baked potato with chives, hot tea with sugar

The nurse is caring for a client who had a T-tube placed 3 days ago. Which assessment finding indicates to the nurse that the procedure was

successful? a. Sclera that is slightly icteric

b. Positive Blumberg’s sign c. Soft, brown, formed stool this morning

d. Sips of clear liquid tolerated without nausea

The nurse is caring for a client with acute pancreatitis. During the physical assessment, the nurse notes a grayish-blue discoloration of the

client’s flanks. Which is the nurse’s priority action? a. Prepare the client for emergency surgery. b. Place the client in high Fowler’s position.

c. Insert a nasogastric (NG) tube to low intermittent suction. d. Ensure that the client has a patent large-bore IV site.

The nurse is caring for a client with acute pancreatitis. Which nursing intervention best reduces discomfort for the client?

a. Administering morphine sulfate IV every 4 to 6 hours as needed b. Maintaining NPO status for the client with IV fluids

c. Providing small, frequent feedings, with no concentrated sweets d. Placing the client in semi-Fowler’s position at elevation of 30 degrees

The nurse is caring for a client who has undergone surgery to drain a pancreatic pseudocyst with placement of a pancreatic drainage tube.

Which nursing intervention prevents complications from this procedure? a. Positioning the client in a right side-lying position

b. Applying a skin barrier around the drainage tube site c. Clamping the drainage tube for 2 hours every 12 hours

d. Irrigating the drainage tube daily with 30 mL of sterile normal saline

The nurse is providing discharge teaching for a client who will be receiving pancreatic enzyme replacement at home. Which statement by

the client indicates that additional teaching is needed? a. “The capsules can be opened and the powder sprinkled on

applesauce if needed.” b. “I will wipe my lips carefully after I drink the enzyme preparation.”

c. “The best time to take the enzymes is immediately after I have a meal or a snack.”

d. “I will not mix the enzyme powder with food or liquids that contain protein.”

The nurse is caring for a client with chronic pancreatitis. Which instruction by the nurse is most appropriate?

a. “You will need to limit your protein intake.” b. “We need to call the dietitian to get help in planning your diet.”

c. “You cannot eat concentrated sweets any longer.” d. “Try to eat less red meat and more chicken and fish.”

The postanesthesia care unit nurse is caring for a client who has just undergone an open Whipple procedure. The client has multiple tubes and drains in place after the surgery. Which does the nurse assess first?

a. Endotracheal tube with 40% fraction of inspired oxygen (FiO2)

b. Foley catheter to bedside drainage c. Nasogastric tube to low intermittent suction

d. Triple-lumen IV catheter with lactated Ringer’s solution

The nurse is caring for a client with end-stage pancreatic cancer. The client asks the nurse, “Why is this happening to me?” Which is the

nurse’s best response? a. “I don’t know. I wish I had an answer for you, but I don’t.”

b. “It’s important to keep a positive attitude for your family right now.”

c. “Scientists have not determined why cancer develops in certain people.”

d. “I think that this is a trial so you can become a better person because of it.”

The nurse is caring for a client who has just been diagnosed with end- stage pancreatic cancer. The nurse assesses the client’s emotional

response to the diagnosis. Which is the nurse’s initial action for the assessment?

a. Bring the client to a quiet room for privacy. b. Pull up a chair and sit next to the client’s bed.

c. Determine whether the client feels like talking about his or her feelings.

d. Review the health care provider’s notes about the prognosis for the

client.

The nurse is teaching a community group about pancreatic cancer. Which risk factor does the nurse instruct is known for development of

this type of cancer? a. Hypothyroidism

b. Cholelithiasis c. BRCA2 gene mutation

d. African-American ethnicity

The nurse is caring for a client who had undergone a Whipple procedure 2 days previously. The nurse notes that the client’s hands and feet are

edematous, and urine output has decreased from the previous day. Which intervention does the nurse expect to provide for the client?

a. Increase the client’s IV fluid infusion rate. b. Monitor the client’s blood sugar level every 4 hours.

c. Add colloids to the client’s IV solutions. d. Reinsert the client’s nasogastric (NG) tube.

A client is hospitalized with acute pancreatitis. The nursing assistant reports to the nurse that when a blood pressure cuff was applied, the client’s hand had a spasm. Which additional finding does the nurse

correlate with this condition? a. Serum calcium, 5.8 mg/dL b. Serum sodium, 166 mEq/L

c. Serum creatinine, 0.9 mg/dL d. Serum potassium, 4.2 mEq/dL

The nurse is caring for a client with cholecystitis. The client is a poor historian and is unable to tell the nurse when the symptoms started. Which assessment finding indicates to the nurse that the condition is

chronic rather than acute? a. Temperature of 100.1° F (37.8° C)

b. Positive Murphy’s sign c. Light-colored stools

d. Upper abdominal pain after eating

MULTIPLE RESPONSE

The nurse is caring for a client after a Whipple procedure. Which manifestations might indicate that a complication from the operation has

occurred? (Select all that apply.) a. Urinary retention

b. Substernal chest pain c. Shortness of breath

d. Lack of bowel sounds or flatus e. Urine output of 20 mL/6 hr

The nurse is caring for a female client with cholelithiasis. Which assessment findings from the client’s history and physical examination may have contributed to development of the condition? (Select all that

apply.) a. Body mass index (BMI) of 46

b. Vegetarian diet c. Drinking 4 ounces of red wine nightly

d. Pregnant with twins e. History of metabolic syndrome

f. Glycosylated hemoglobin level of 15%

The nurse is caring for a client who is being discharged from the hospital after an attack of acute pancreatitis. Which discharge instructions does the nurse provide for the client to help prevent a recurrence? (Select all

that apply.)

a. “Take a 20-minute walk at least 5 days each week.” b. “Attend local Alcoholics Anonymous (AA) meetings weekly.” c. “Choose whole grains rather than foods with simple sugars.” d. “Use cooking spray when you cook rather than margarine or

butter.” e. “Stay away from milk and dairy products that contain lactose.”

f. “We can talk to your doctor about a prescription for nicotine patches.”

COMPLETION

The nurse is caring for a client with acute pancreatitis. The client’s health care provider has ordered gentamicin (Garamycin) 3 mg/kg/day in three divided doses. The client weighs 264 lb. The client will receive

milligrams/dose of Garamycin.

Chapter 63: Care of Patients with Malnutrition and Obesity

Test Bank

MULTIPLE CHOICE

The nurse is caring for a female client who is 5 feet, 7 inches tall and weighs 115 pounds. The client asks the nurse if she needs to lose weight.

Which response by the nurse is best? a. “Yes. Your body mass index suggests you are slightly overweight.”

b. “Maybe. Let’s look at your risks for cardiovascular disease.” c. “Your weight is just fine. Don’t worry about it.”

d. “No. In fact, your body mass index suggests that you are already underweight.”

The nurse is caring for a male client who is 6 feet, 1 inch tall and weighs 215 pounds. The client asks the nurse if his weight is appropriate for his

height. Which is the nurse’s best response? a. “Your weight is just about right for someone your height.”

b. “Your weight is a few pounds under the ideal for your height.” c. “Your weight is a few pounds over the ideal for your height.”

d. “Your weight is quite a few pounds over the ideal for your height.”

The nurse is caring for a client who is a vegan and has developed B12

deficiency. Which foods does the nurse encourage the client to include in the diet?

a. Fortified cereals and tofu b. Pumpkin seeds and blackstrap molasses

c. Kale, spinach, and whole grain bread d. Strawberries and sweet red peppers

The nurse is caring for a client who has a new small-bore nasoduodenal tube for feedings. Which intervention most effectively prevents clogging

of the tube? a. Administering medications that have been thoroughly crushed and

dissolved in cold water b. Flushing the feeding tube with 60 mL of cranberry juice or

carbonated beverage four times daily c. Irrigating the tube with water before and after administration of

medications using 20 to 30 mL d. Diluting the tube feeding to half-strength with cold water before

infusion into the feeding tube

The nurse is reviewing recent laboratory values for a client who is being treated for malnutrition. Which laboratory finding indicates that the

client is not receiving adequate iron supplementation? a. Hematocrit, 31%

b. Serum albumin, 3.5 g/dL

c. Creatine phosphokinase (CPK), 55 U/mL d. Erythrocyte sedimentation rate (ESR), 15.8 mm/hr

The nurse is caring for a client on a limited income who has been diagnosed with kwashiorkor. Which foods does the nurse suggest to improve the client’s nutritional status with minimal increase in food

costs? a. Oatmeal and bananas

b. Tomato soup with oyster crackers c. Omelet made with cheddar cheese

d. Whole wheat pasta with tomato sauce

The nurse is preparing to administer tube feedings through a client’s new Salem sump nasogastric tube. The nurse is unable to withdraw any fluid from the tube before starting the feeding. Which is the priority action of

the nurse? a. Start the tube feeding as ordered and check the residual in 30

minutes. b. Inject air into the nasogastric tube while auscultating the client’s

epigastric area. c. Lower the head of the client’s bed and attempt to aspirate fluid

again.

d. Obtain orders for a chest x-ray to confirm placement before starting the feeding.

The nurse is caring for a client who was started on total parenteral nutrition (TPN) 2 days previously. The client reports blurred vision, dry

mouth, and frequent urination. Which is the nurse’s priority action? a. Weigh the client.

b. Assess the client’s vital signs. c. Slow down the TPN infusion.

d. Assess the client’s blood sugar.

The nurse is caring for an anorexic client who is severely malnourished. A nasogastric feeding tube is inserted, and tube feedings are started.

Which laboratory finding is the best indication that the client’s nutritional status is improving?

a. Sodium has risen from 130 to 144 mg/dL. b. Creatinine has dropped from 1.9 to 0.5 mg/dL. c. Prealbumin level has risen from 9 to 13 mg/dL.

d. Blood urea nitrogen (BUN) level has dropped from 15 to 11 mg/dL.

A client who is malnourished has a total lymphocyte count of 1450/mm3. Which instruction does the nurse provide to the unlicensed assistive

personnel helping to care for this client? a. “Wash your hands or use hand foam when you first enter the room.”

b. “Be sure to offer this client a glass of water each time you are with the client.”

c. “You may need to open cartons and packages on the client’s food tray.”

d. “Record all of the client’s food and drink intake for the shift.”

Which client is at highest risk for developing dehydration and hypernatremia as a result of enteral feedings?

a. Client receiving an isotonic enteral feeding solution and an IV of D5W (dextrose 5% in water) at 83 mL/hr

b. Client receiving a hypertonic enteral feeding solution and an IV of normal saline (0.9 NS) at 125 mL/hr

c. Client who can drink liquids and is receiving a supplemental hypertonic enteral feeding solution

d. Client receiving a hypertonic enteral feeding solution and an IV of 0.45% NS (0.45 NS) infusing at 125 mL/hr

Which statement indicates that the client needs additional discharge teaching after gastric bypass surgery?

a. “I hope my type 2 diabetes is cured and I won’t need insulin anymore.”

b. “As soon as I get home, I’m going to enjoy a nice bowl of fruit.” c. “If I get nauseated, I know I’m eating too much at one time.”

d. “I will be sure to report any back, shoulder, or abdominal pain.”

The postanesthesia care nurse is caring for a client who had gastric banding surgery and was extubated an hour ago. The client’s blood gases are as follows: pH, 7.22; HCO3– 21 mEq/L; PCO2, 65 mm Hg; and PO2,

58 mm Hg. Which is the priority action by the nurse? a. Assess the client’s airway.

b. Increase the client’s oxygen flow rate. c. Check the client’s oxygen saturation level.

d. Document findings in the client’s chart.

The nurse is caring for an overweight client who gained 10 pounds during the previous 2 weeks. The client states that she is hungry all the

time and doesn’t understand why. Which assessment finding could explain the client’s weight gain and hunger?

a. The client started taking dexamethasone (Decadron) daily. b. The client started taking naproxen sodium (Naprosyn) daily.

c. The client’s glycosylated hemoglobin level is 6%. d. The client’s thyroxine (T4) level is 8 mcg/dL.

The nurse is caring for an obese client who will be taking orlistat (Xenical) to help her lose weight. Which statement indicates that the

client understands teaching about orlistat? a. “This medication will help speed up my metabolism.”

b. “I may have loose stools after meals if I eat too much fat.” c. “This medication will suppress my appetite so I won’t be hungry.” d. “This medication will make me feel full after I eat small amounts.”

Which dietary adjustments does the nurse recommend to an older adult client asking what changes she should institute to prevent or manage

constipation? a. “Increase your calcium intake.”

b. “Limit your fluid intake.” c. “Include plenty of fiber.”

d. “Take a laxative with every meal.”

The new nursing supervisor at a long-term care facility is concerned

about the number of residents who appear malnourished. Which action by the nurse is best?

a. Institute daily weighing for at-risk or underweight residents. b. Provide a supply of easy to access high-calorie snacks.

c. Ask dining room personnel about residents coughing at meals. d. Assess the residents’ opinions on the quality of food served.

A severely malnourished client was started on enteral feedings. The following day, the client is confused, has a heart rate of 112 beats/min,

and reports feeling weak. Which laboratory value does the nurse correlate with this condition?

a. Serum phosphate, 1.8 mg/dL b. Serum potassium, 3.1 mEq/L c. Serum sodium, 143 mEq/L d. Serum glucose, 110 mg/dL

A client has a small-bore nasoenteric feeding tube. The nurse assesses the following vital signs: temperature, 100.2° F (37.8° C); pulse, 112

beats/min; respiratory rate, 22 breaths/min; and blood pressure, 106/62 mm Hg. Which action by the nurse takes priority?

a. Remove the tube immediately and notify the heath care provider. b. Auscultate lung sounds and obtain oxygen saturation.

c. Add blue dye to the feeding tube formula. d. Auscultate bowel sounds and slow the feeding down.

A facility is beginning to perform bariatric surgery on obese clients. Which action by the nursing manager is most important?

a. Obtain appropriately sized equipment for these clients. b. Select a dedicated group of staff members for these clients. c. Send personnel to sensitivity training as part of orientation.

d. Establish multidisciplinary rounding for clients in this program.

MULTIPLE RESPONSE

A client is malnourished and needs encouragement and assistance to eat. Which activities does the nurse delegate to the unlicensed assistive personnel (UAP) when giving this client a food tray? (Select all that

apply.) a. Open food packages and cut food if needed. b. Remove the urinal from the bedside table.

c. Assess the client’s ability to swallow. d. Report to the nurse pain described by the client. e. Sit with the client and do not rush the feeding.

The nurse is teaching a health promotion class about weight loss and asks students to list health risks that can occur as a result of obesity.

Which student responses indicate that additional teaching is required? (Select all that apply.)

a. Sleep apnea b. Infertility

c. Rheumatoid arthritis d. Cervical cancer

e. Cholecystitis f. Hypothyroidism

When reviewing an older client’s medical record, which findings lead the nurse to perform a nutrition assessment? (Select all that apply.)

a. Widow/widower status b. Chronic constipation c. History of depression

d. Random blood sugar level of 198 mg/dL e. Cholecystectomy 4 years ago

f. Inability to afford a new pair of glasses

Chapter 64: Assessment of the Endocrine System

Test Bank

MULTIPLE CHOICE

A client is taking a drug that blocks a hormone’s receptor site. What is the effect on the client’s hormone response?

a. Greater hormone metabolism b. Decreased hormone activity c. Increased hormone activity

d. Unchanged hormone response

How does a tropic hormone differ from other hormones? a. Tropic hormones are given to clients who have a hormone

deficiency. b. Tropic hormones are exclusively involved in the production of sex

hormones. c. Tropic hormones stimulate other endocrine glands to secrete

hormones. d. Tropic hormones are not under negative feedback control.

A client has a deficiency of aldosterone. Which assessment finding does the nurse correlate with this condition?

a. Increased urine output b. Vasoconstriction

c. Blood glucose, 98 mg/dL d. Serum sodium, 144 mEq/L

A male client reports fluid secretion from his breasts. What does the nurse assess next in this client?

a. Posterior pituitary hormones b. Adrenal medulla functioning c. Anterior pituitary hormones

d. Parathyroid functioning

A client has a condition of excessive catecholamine release. Which assessment finding does the nurse correlate with this condition?

a. Decreased blood pressure b. Increased pulse

c. Decreased respiratory rate

d. No change in vital signs

A client is admitted to the hospital with exacerbation of heart failure, which had been stable for several years. Which finding does the nurse

associate with the client’s current condition? a. Recent prescription for thyroid hormone replacement medication

b. Recent onset of menopause c. Patchy areas of depigmentation on the face

d. Absence of fish in the diet, but inclusion of the iodized form of table salt

A client has abnormal calcium levels. Which hormone does the nurse anticipate testing for? a. Thyroxine (T4)

b. Triiodothyronine (T3) c. Thyrocalcitonin (calcitonin)

d. Propylthiouracil (PTU)

Which is the expected clinical manifestation for a client who has

excessive production of melanocyte-stimulating hormone? a. Hypoglycemia and hyperkalemia

b. Irritability and insomnia c. Increased urine output d. Darkening of the skin

Which pulse rate finding in a client taking a drug that stimulates beta1 receptors requires immediate action by the nurse?

a. 50 beats/min b. 95 beats/min c. 85 beats/min d. 100 beats/min

Which situation or condition is likely to result in increased production of thyroid hormones?

a. Starvation b. Dehydration

c. Adequate sleep d. Cold environmental temperature

A client has bilateral patchy areas of skin depigmentation on the arms and the face. Which action by the nurse is best?

a. Assess the client’s mucous membranes. b. Draw a laboratory specimen for thyroid hormone levels.

c. Schedule the client for fasting blood glucose. d. Question the client about sexual functioning.

A new nurse is palpating a client’s thyroid gland. Which action requires intervention from the nurse’s mentor?

a. The nurse stands behind, instead of in front of, the client. b. The client is asked to swallow while the nurse finds the thyroid

gland. c. The nurse palpates the right lobe with his or her left hand.

d. The client is placed in a sitting position with the chin tucked down.

Which client statement indicates the need for clarification regarding the instructions for collecting a 24-hour urine specimen for assessment of

endocrine function? a. “I will continue to take all my prescribed medicine during the test.” b. “I will add the preservative to the container at the beginning of the

test.” c. “I will start the collection by saving the first urine of the morning.” d. “At the end of 24 hours, I will urinate and save that last specimen.”

A female client with an endocrine problem has hirsutism. Which question or statement by the nurse is most appropriate?

a. “Do you have the money to pay for treatment?” b. “I’m interested in knowing how you feel about yourself.” c. “Many treatment options are available for this problem.”

d. “What can you do to prevent this from happening?”

A client asks why a 24-hour urine collection is necessary to measure excreted hormones instead of a random voided specimen. Which

response by the nurse is most accurate? a. “We are testing for a hormone secreted on a circadian rhythm.”

b. “The hormone is so dilute in urine, we need a large volume.” c. “We want to see when the hormone is secreted in both large and

small amounts.” d. “You’d have to be here at a specific time of the day for a random

urinalysis.”

An older client is being admitted to the hospital for pneumonia. The

client has no other health problems. Which action by the nurse is best? a. Place the client on airborne precautions. b. Offer the client fluids every hour or two.

c. Leave the bathroom light on at night. d. Palpate the client’s thyroid gland on admission.

The nurse is teaching a client about self-care after menopause. Which teaching topic is the priority? a. Weight-bearing exercise

b. Skin care c. Intimacy needs

d. Body image changes

The nursing assistant reports that while pouring urine into a 24-hour urine container, some urine splashed the nursing assistant’s hand. Which

action by the nurse is best? a. Ask the assistant if he or she washed the hands afterward.

b. Call the laboratory to see if the container has preservative in it. c. Have the assistant fill out an incident report.

d. Send the assistant to Employee Health right away.

MULTIPLE RESPONSE

Which are common key features of hormones? (Select all that apply.) a. Hormones may travel long distances to get to their target tissues. b. Continued hormone activity requires continued production and

secretion. c. Control of hormone activity is caused by negative feedback

mechanisms. d. Most hormones are stored in the target tissue for use later.

e. Most hormones cause target tissues to change activities by changing gene activity.

A client has a hypofunctioning anterior pituitary gland. Which hormones does the nurse expect to be affected by this? (Select all that apply.)

a. Thyroid-stimulating hormone b. Vasopressin

c. Follicle-stimulating hormone d. Calcitonin

e. Growth hormone

Chapter 65: Care of Patients with Pituitary and Adrenal Gland Problems

Test Bank

MULTIPLE CHOICE

A client has a hormone deficiency. Which deficiency is the highest priority?

a. Growth hormone b. Luteinizing hormone

c. Thyroid-stimulating hormone d. Follicle-stimulating hormone

An adult client has been diagnosed with a deficiency of gonadotropin and growth hormone. Which fact reported in the client’s history could have

contributed to this problem? a. Mother with adult-onset diabetes mellitus

b. Experienced head trauma 5 years ago c. Severe allergy to shellfish and iodine

d. Has used oral contraceptives for 5 years

Which safety measure does the nurse use for the adult client who has growth hormone deficiency?

a. Avoid intramuscular medications. b. Place the client in protective isolation. c. Use a lift sheet to reposition the client.

d. Assist the client to change positions slowly.

The male client with hypopituitarism asks the nurse how long he will have to take testosterone hormone replacement therapy. Which is the

nurse’s best answer? a. “When your blood levels of testosterone are normal, the therapy is

no longer needed.” b. “When your beard thickens and your voice deepens, the dose is

decreased, but treatment will continue forever.” c. “When your sperm count is high enough to demonstrate fertility,

you will no longer need this therapy.” d. “When you start to have undesirable side effects, the dose is

decreased to the lowest possible level, and treatment is continued until you are 50 years old.”

When performing personal care on a middle-aged woman, the nurse observes that the client has very little pubic and axillary hair. Which is

the nurse’s best action? a. Ask the client if she has less pubic hair now than 5 years ago.

b. Ask the client the date of her last menstrual period. c. Examine the client’s scalp hair for texture and thickness.

d. Draw blood for hormonal immune assays.

A client thought to have a problem with the pituitary gland is given a stimulation test using insulin. A short time later, blood analysis reveals

elevated levels of growth hormone (GH) and adrenocorticotropic hormone (ACTH). Which is the nurse’s interpretation of this finding?

a. Pituitary hypofunction b. Pituitary hyperfunction

c. Pituitary-induced diabetes mellitus d. A normal pituitary response to insulin

A client has documented acromegaly. During a physical assessment before surgery for a knee replacement, the nurse discovers that she has a

moderately enlarged liver. Which is the nurse’s best action? a. Counsel the client on the health risks of alcoholism.

b. Assess for jaundice of the skin and eyes. c. Document the finding and monitor the client.

d. Draw blood for liver function studies.

A client just diagnosed with acromegaly is scheduled for a hypophysectomy. Which statement made by the client indicates a need

for clarification regarding this treatment? a. “I will drink whenever I feel thirsty after surgery.”

b. “I’m glad no visible incision will result from this surgery.” c. “I hope I can go back to wearing size 8 shoes instead of size 12.” d. “I will wear slip-on shoes after surgery so I don’t have to bend

over.”

A client who had a trans-sphenoidal hypophysectomy 2 days ago now has nuchal rigidity. Which is the nurse’s priority action?

a. Have the client do active range-of-motion exercises for the neck. b. Document the finding and monitor the client.

c. Take the client’s temperature and other vital signs. d. Assess using a pain scale and administer pain medication.

A client is going home after an endoscopic transnasal hypophysectomy. Which statement by the client indicates an adequate understanding of

discharge instructions? a. “I will wear dark glasses whenever I am outdoors.”

b. “I will keep food on upper shelves so I do not have to bend over.” c. “I will wash the incision line every day with peroxide and redress it

immediately.” d. “I will remember to cough and deep breathe every 2 hours while I

am awake.”

A client with suspected syndrome of inappropriate antidiuretic hormone (SIADH) has a serum sodium of 114 mEq/L. Which action by the nurse is

best? a. Consult with the registered dietitian about increased dietary sodium.

b. Restrict the client’s fluid intake to 900 mL/24 hr. c. Handle the client gently by using turn sheets for repositioning. d. Instruct the nursing assistants to measure intake and output.

Which safety measure is most important for the nurse to institute for a client who has Cushing’s disease?

a. Pad the siderails of the client’s bed. b. Assist the client to change positions slowly.

c. Use a lift sheet to change the client’s position.

d. Keep suctioning equipment at the client’s bedside.

Which dietary alterations does the nurse make for a client with Cushing’s disease?

a. High carbohydrate, low potassium b. Low carbohydrate, low sodium

c. Low protein, low calcium d. High carbohydrate, low potassium

A client who has been taking high-dose corticosteroid therapy for 1 month to treat a severe inflammatory condition, which has now resolved, asks the nurse why she needs to continue taking corticosteroids. Which is

the nurse’s best response? a. “It is possible for the inflammation to recur if you stop the drugs.” b. “Once you start corticosteroids, you have to be weaned off them.” c. “You must decrease the dose slowly so your hormones will begin to

work again.” d. “The drug suppresses your immune system, which needs to be built

back up.”

A client has received vasopressin (DDAVP) for diabetes insipidus. Which assessment finding indicates a therapeutic response to this therapy?

a. Urine output is increased; specific gravity is increased. b. Urine output is increased; specific gravity is decreased. c. Urine output is decreased; specific gravity is increased. d. Urine output is decreased; specific gravity is decreased.

A client with hypercortisolism has an irregular pulse. Which is the nurse’s priority intervention?

a. Documenting the finding and reassessing in 1 hour b. Assessing blood pressure in both arms

c. Administering atropine sulfate d. Assessing the telemetry reading

The client with adrenal hyperfunction screams at her husband, bursts into tears, and throws her water pitcher against the wall. She then tells

the nurse, “I feel like I am going crazy.” Which is the nurse’s best response?

a. “I will ask your doctor to order a psychiatric consult for you.” b. “You feel this way because of your hormone levels.”

c. “Can I bring you information about support groups?” d. “I will close the door to your room and restrict visitors.”

A client on medication after a bilateral adrenalectomy calls the clinic asking to be seen for “stomach flu” with nausea and vomiting. Which

response by the nurse is best? a. “I will call in a prescription for an antiemetic medication for you.”

b. “Try to drink extra fluids until you can come in for an appointment.”

c. “You need to go to the nearest emergency department today.” d. “Double the dose of your medication today and tomorrow.”

A client with hyperaldosteronism is being treated with spironolactone (Aldactone) before surgery. Which precautions does the nurse teach this

client? a. “Read the label before using salt substitutes.” b. “Do not add salt to your food when you eat.”

c. “Avoid exposure to sunlight.” d. “Take Tylenol instead of aspirin for pain.”

The new nurse is assessing a client with suspected pheochromocytoma. Which action by the nurse requires the precepting nurse to intervene?

a. Auscultating, palpating, and percussing the client’s abdomen b. Taking the client’s blood pressure for reports of chest pain

c. Assessing the client’s diet for red wine and aged cheeses d. Limiting visitors while the client is sleeping

The client has chronic hypercortisolism. Which intervention is the highest priority for the nurse?

a. Wash the hands when entering the room. b. Keep the client in protective isolation.

c. Observe the client for increased white blood cell counts. d. Assess the daily chest x-ray.

A female client is beginning treatment with bromocriptine (Parlodel). The nurse has initiated teaching sessions about potential side effects.

Which is the most important point of instruction? a. “Take and record your temperature daily.”

b. “Be sure to eat 20 to 30 grams of fiber daily.” c. “Plan to take the medication on an empty stomach.” d. “I will need to teach you how to give the injection.”

The nurse is caring for a client who has undergone a hypophysectomy. Which is the nurse’s priority postoperative intervention? a. Keep the head of the bed flat and the client supine.

b. Instruct the client to cough, turn, and deep breathe hourly. c. Report clear or yellow drainage from the nose or incision site.

d. Apply petroleum jelly to the client’s lips to avoid mouth dryness.

A client has cortisol deficiency and is being treated with prednisone (Deltasone). Which instruction by the nurse is most appropriate?

a. “You will need to learn how to rotate the injection sites.” b. “If you work outside when it’s hot, you may need another drug.”

c. “Be sure to stay on your salt restriction even though it’s difficult.” d. “Take one tablet in the morning and two tablets at night to start.”

A client is brought to the emergency department via rescue squad in acute adrenal crisis. Which action by the nurse is the priority?

a. Start an IV line if the client does not already have one. b. Administer hydrocortisone sodium succinate (Solu-Cortef).

c. Instruct the nursing assistant to check the client’s blood glucose. d. Administer 20 units of insulin and 20 mg of dextrose in normal

saline.

A female client has a decrease in all pituitary hormones. Which assessment question by the nurse elicits the best information?

a. “Do you have any biological children?” b. “Do you have a decreased sex drive?”

c. “Have you noticed increased facial hair?” d. “Are you more intolerant of heat?”

MULTIPLE RESPONSE

Which physical characteristics are indicative of anterior pituitary hyperfunction? (Select all that apply.)

a. Protrusion of the lower jaw b. High-pitched voice

c. Enlarged hands and feet d. Kyphosis

e. Barrel-shaped chest f. Excessive sweating

Which conditions may cause hypopituitarism? (Select all that apply.) a. Benign pituitary tumors

b. Diplopia c. Anorexia nervosa

d. Hypotension e. Shock

f. Weight gain

Which serum laboratory values alert the nurse to the possibility of hyperaldosteronism? (Select all that apply.)

a. Sodium, 150 mEq/L b. Sodium, 130 mEq/L

c. Potassium, 2.5 mEq/L d. Potassium, 5.0 mEq/L

e. pH, 7.28 f. pH, 7.50

Chapter 66: Care of Patients with Problems of the Thyroid and Parathyroid Glands

Test Bank

MULTIPLE CHOICE

A client presents with elevations in triiodothyronine (T3) and thyroxine (T4) and with normal thyroid-stimulating hormone (TSH) levels. Which

is the nurse’s priority intervention? a. Administer levothyroxine (Synthroid).

b. Administer propranolol (Inderal). c. Monitor the apical pulse.

d. Assess for Trousseau’s sign.

Which is the best instruction for the nurse to give a client scheduled for a thyroid scan?

a. “You will have external beam radiation.” b. “No radiation is used for this scan.”

c. “No special radiation precautions are needed.” d. “Your thyroid will be radioactive for weeks.”

Which dietary modification does the nurse provide for a client with

hyperthyroidism? a. Decreased calories and proteins and increased carbohydrates b. Elimination of carbohydrates and increased proteins and fats

c. Increased calories, proteins, and carbohydrates d. Supplemental vitamins and reduction of calories

A client with hyperthyroidism is taking lithium carbonate. Which finding indicates that the client is having side effects of this therapy?

a. Blurred vision b. Increased thirst and urination

c. Increased sweating and diarrhea d. Decreased attention span and insomnia

A client scheduled for a partial thyroidectomy asks the nurse why she is being given an iodine preparation before surgery. Which is the nurse’s

best response? a. “Iodine will help make the internal surgical environment sterile.” b. “It is given to stimulate the storage of excess thyroid hormones.”

c. “This will replace the hormones you will lose after your operation.” d. “It will prevent excessive bleeding during surgery.”

Twelve hours after a total thyroidectomy, the client develops stridor. Which is the nurse’s priority intervention?

a. Reassure the client that the voice change is temporary. b. Document the finding and assess the client hourly. c. Hyperextend the client’s neck and apply oxygen.

d. Prepare for emergency tracheostomy and call the health care provider.

On the second postoperative day after a subtotal thyroidectomy, the client tells the nurse that he feels numbness and tingling around his

mouth. Which is the nurse’s priority intervention? a. Offer mouth care.

b. Loosen the dressing. c. Assess Chvostek’s sign. d. Assess the client hourly.

Which client statement alerts the nurse to the possibility of hypothyroidism?

a. “My sister has thyroid problems.” b. “I seem to feel the heat more than other people.”

c. “Food just doesn’t taste good without a lot of salt.” d. “I am always tired, even with 10 or 12 hours of sleep.”

A client has been diagnosed with hypothyroidism. Which medication is the nurse prepared to administer to treat the client’s bradycardia?

a. Atropine sulfate b. Levothyroxine sodium (Synthroid)

c. Propranolol (Inderal) d. Epinephrine (Adrenalin)

A client has hypothyroidism. Which problem does the nurse address as a priority for this client? a. Heat intolerance

b. Body image problems c. Depression and withdrawal

d. Obesity

A client has hypothyroidism and has been started on levothyroxine (Synthroid). Which assessment finding leads the nurse to conclude that

the treatment is effective? a. Thirst is recognized and the client drinks fluids appropriately.

b. Weight has been the same for 3 weeks. c. Total white blood cell count is 6000 cells/mm3.

d. Heart rate is 70 beats/min and regular.

A client with hypothyroidism as a result of Hashimoto’s thyroiditis asks the nurse how long she will have to take thyroid medication. Which is the

nurse’s best response? a. “You will need to take the thyroid medication until the goiter is

completely gone.” b. “Thyroiditis is cured with antibiotics. Then you won’t need thyroid

medication.” c. “You’ll need thyroid pills for life because your thyroid won’t start

working again.” d. “When blood tests indicate normal thyroid function, you can stop

the medication.”

The nurse is reviewing client medical histories. Which client is at greatest risk for hyperparathyroidism?

a. Client with pregnancy-induced hypertension b. Client receiving dialysis for end-stage kidney disease

c. Older adult client with moderate heart failure d. Older adult client on home oxygen therapy

A client has hyperparathyroidism. Which intervention is the priority for the nurse to add to the client’s plan of care?

a. Instruct the client to place both hands behind the neck when moving.

b. Use a lift sheet to assist the client with position changes. c. Instruct the client to use a soft-bristled toothbrush.

d. Strain all urine for at least 24 hours and send stones to the laboratory.

When taking the blood pressure of a client after a parathyroidectomy, the nurse notes that the client’s hand has gone into flexion contractions. Which laboratory result does the nurse correlate with this condition?

a. Serum potassium, 2.9 mEq/L b. Serum potassium, 5.8 mEq/L c. Serum sodium, 122 mEq/L d. Serum calcium, 6.9 mg/dL

The client is receiving methimazole (Tapazole). Which statement by the client indicates good understanding of teaching regarding this

medication?

a. “If I become pregnant, I need to notify my health care provider immediately.”

b. “Liver problems can occur with this drug so I need to report jaundice.”

c. “I will take my pulse daily, and if it is too fast, I will call my provider.”

d. “This medication may cause dyspnea or vertigo. I will be careful with activity.”

A client has diabetes mellitus. Her daughter has recently been diagnosed with Graves’ disease. The client asks the nurse if she is responsible for

the fact that her daughter has Graves’ disease. Which is the best response of the nurse?

a. “No connection is known between Graves’ disease and diabetes, so you can be certain that the fact that you have diabetes did not cause your

daughter to have Graves’ disease.” b. “An association has been noted between Graves’ disease and

diabetes, but the fact that you have diabetes did not cause your daughter to have Graves’ disease.”

c. “Graves’ disease is associated with autoimmune diseases such as rheumatoid arthritis, but not with a disease such as diabetes.”

d. “Unfortunately, Graves’ disease is associated with diabetes, and your diabetes could have led to your daughter having Graves’ disease.”

The nurse is assessing a client with Graves’ disease and finds that the client’s temperature has risen 1° F. Before notifying the health care

provider, which action by the nurse takes priority? a. Turn the lights down in the client’s room and shut the door.

b. Call for an immediate electrocardiogram (ECG). c. Calculate the client’s apical-radial pulse deficit. d. Administer a dose of acetaminophen (Tylenol).

A client has undergone a complete thyroidectomy. Which statement by the client indicates that further instruction is needed? a. “I may need calcium replacement after surgery.”

b. “After surgery, I won’t need to take thyroid medication.” c. “I’ll need to take thyroid hormones for life.”

d. “I can receive pain medication if I feel that I need it.”

A client being treated for hypothyroidism has been admitted for pneumonia. Which activity does the nurse include as a priority in this

client’s care plan? a. Monitor the client’s IV site every shift.

b. Administer acetaminophen (Tylenol) for fever. c. Ensure that working suction equipment is in the room.

d. Assess vital signs every 4 hours.

MULTIPLE RESPONSE

A client has been admitted with hypoparathyroidism. The client’s serum laboratory values are as follows: calcium, 7.2 mg/dL; sodium, 144

mEq/L; magnesium, 1.2 mEq/L; potassium, 5.7 mEq/L. Which medications does the nurse anticipate administering? (Select all that

apply.) a. Potassium chloride orally

b. Calcium chloride IV c. 3% NS IV solution

d. 50% magnesium sulfate e. Calcitriol (Rocaltrol) orally

Chapter 67: Care of Patients with Diabetes Mellitus

Test Bank

MULTIPLE CHOICE

In preparing a staff in-service presentation about diabetes mellitus, the nurse includes which information?

a. Diabetes increases the risk for development of epilepsy. b. The cure for diabetes is the administration of insulin.

c. Diabetes increases the risk for development of cardiovascular disease.

d. Carbohydrate metabolism is altered in diabetes, but protein metabolism is normal.

A client with diabetes asks the nurse why it is necessary to maintain blood glucose levels no lower than about 60 mg/dL. Which is the nurse’s

best response? a. “Glucose is the only fuel used by the body to produce the energy

that it needs.” b. “Your brain needs a constant supply of glucose because it cannot

store it.” c. “Without a minimum level of glucose, your body does not make red

blood cells.” d. “Glucose in the blood prevents the formation of lactic acid and

prevents acidosis.”

The nurse is monitoring a client with hypoglycemia. Glucagon provides

which function? a. It enhances the activity of insulin, restoring blood glucose levels to

normal more quickly after a high-calorie meal. b. It is a storage form of glucose and can be broken down for energy

when blood glucose levels are low. c. It converts excess glucose into glycogen, lowering blood glucose

levels in times of excess. d. It prevents hypoglycemia by promoting release of glucose from liver

storage sites.

A client with untreated diabetes mellitus has polyuria, is lethargic, and has a blood glucose of 560 mg/dL. The nurse correlates the polyuria with

which finding? a. Serum sodium, 163 mEq/L

b. Serum creatinine, 1.6 mg/dL c. Presence of urine ketone bodies

d. Serum osmolarity, 375 mOsm/kg

A client with diabetes has a serum creatinine of 1.9 mg/dL. The nurse correlates which urinalysis finding with this client?

a. Ketone bodies in the urine during acidosis b. Glucose in the urine during hyperglycemia

c. Protein in the urine during a random urinalysis d. White blood cells in the urine during a random urinalysis

A young adult client newly diagnosed with type 1 diabetes mellitus has been taught about self-care. Which statement by the client indicates a

good understanding of needed eye examinations? a. “At my age, I should continue seeing the ophthalmologist as I

usually do.” b. “I will see the eye doctor whenever I have a vision problem and

yearly after age 40.” c. “My vision will change quickly now. I should see the

ophthalmologist twice a year.” d. “Diabetes can cause blindness, so I should see the ophthalmologist

yearly.”

During assessment of a client with a 15-year history of diabetes, the nurse notes that the client has decreased tactile sensation in both feet.

Which action does the nurse take first? a. Document the finding in the client’s chart.

b. Test sensory perception in the client’s hands. c. Examine the client’s feet for signs of injury.

d. Notify the health care provider.

A client’s father has type 1 diabetes mellitus. The client asks if she is in danger of developing the disease as well. Which is the nurse’s best

response? a. “Your risk of diabetes is higher than that of the general population,

but it may not occur.” b. “No genetic risk is associated with the development of type 1

diabetes.” c. “The risk for becoming diabetic is 50% because of how it is

inherited.” d. “Female children do not inherit diabetes, but male children will.”

A client has newly diagnosed diabetes. To delay the onset of microvascular and macrovascular complications in this client, the nurse

stresses that the client take which action? a. Control hyperglycemia. b. Prevent hypoglycemia. c. Restrict fluid intake.

d. Prevent ketosis.

Which client is at greatest risk for undiagnosed diabetes mellitus?

a. Young, muscular white man b. Young African-American man

c. Middle-aged Asian woman d. Middle-aged American Indian woman

The nurse is teaching a client about self-monitoring of blood glucose levels. To prevent bloodborne infection, which statement by the nurse is

best? a. “Wash your hands after completing the test.” b. “Do not share your monitoring equipment.”

c. “Blot excess blood from the strip.” d. “Use gloves during monitoring.”

A client with diabetes has frequent blood glucose readings higher than 300 mg/dL. Which action does the nurse teach the client about self-care?

a. Check urine ketones when blood glucose readings are high. b. Increase the insulin dose after two high glucose readings in a row.

c. Change the diet to include a 10% increase in protein. d. Work out on the treadmill whenever glucose readings are high.

A client who has type 2 diabetes is prescribed glipizide (Glucotrol). Which precautions does the nurse include in the teaching plan related to

this medication? a. “Change positions slowly when you get up.”

b. “Avoid taking nonsteroidal anti-inflammatory drugs.” c. “If you miss a dose of this drug, you can double the next dose.”

d. “Discontinue the medication if you develop an infection.”

The client with type 2 diabetes has recently been changed from the oral antidiabetic agents glyburide (Micronase) and metformin (Glucophage)

to glyburide-metformin (Glucovance). The nurse includes which information in the teaching about this medication?

a. “Glucovance is more effective than glyburide and metformin.” b. “Glucovance contains a combination of glyburide and metformin.”

c. “Glucovance is a new oral insulin and replaces all other oral antidiabetic agents.”

d. “Your diabetes is improving and you now need only one drug.”

Which statement made by a client with type 2 diabetes taking nateglinide (Starlix) indicates understanding of this therapy?

a. “I’ll take this medicine with my meals.” b. “I’ll take this medicine right before I eat.”

c. “I’ll take this medicine just before I go to bed.”

d. “I’ll take this medicine when I wake up in the morning.”

A client who has been taking pioglitazone (Actos) for 6 months reports to the nurse that his urine has become darker since starting the medication.

Which is the nurse’s first action? a. Review results of liver enzyme studies.

b. Document the report in the client’s chart. c. Instruct the client to increase water intake.

d. Test a sample of urine for occult blood.

A client with diabetes asks why more than one injection of insulin is required each day. Which is the nurse’s best response?

a. “You need to start with multiple injections until you become more proficient at self-injection.”

b. “A single dose of insulin each day would not match your blood insulin levels and your food intake patterns closely enough.”

c. “A regimen of a single dose of insulin injected each day would require that you could eat no more than one meal each day.”

d. “A single dose of insulin would be too large to be absorbed predictably, so you would be in danger of unexpected insulin shock.”

A client has been taught to inject insulin. Which statement made by the client indicates a need for further teaching?

a. “The abdominal site is best because it is closest to the pancreas.” b. “I can reach my thigh the best, so I will use different areas of the

same thigh.” c. “By rotating the sites in one area, my chance of having a reaction is

decreased.” d. “Changing injection sites from the thigh to the arm will change

absorption rates.”

A client who has used insulin for diabetes control for 20 years has a spongy swelling at the site used most frequently for insulin injection.

Which is the nurse’s best action? a. Apply ice to this area for 20 minutes.

b. Document the finding in the client’s chart. c. Assess the client for other signs of cellulitis.

d. Instruct the client to use a different site for injection.

A client with diabetes is prescribed insulin glargine once daily and regular insulin four times daily. One dose of regular insulin is scheduled at the same time as the glargine. How does the nurse instruct the client to

administer the two doses of insulin?

a. “Draw up and inject the insulin glargine first, then draw up and inject the regular insulin.”

b. “Draw up and inject the insulin glargine first, wait 20 minutes, then draw up and inject the regular insulin.”

c. “First draw up the dose of regular insulin, then draw up the dose of insulin glargine in the same syringe, mix, and inject the two insulins

together.” d. “First draw up the dose of insulin glargine, then draw up the dose of

regular insulin in the same syringe, mix, and inject the two insulins together.”

A client on an intensified insulin regimen consistently has a fasting blood glucose level between 70 and 80 mg/dL, a postprandial blood glucose

level below 200 mg/dL, and a hemoglobin A1c level of 5.5%. Which is the nurse’s interpretation of these findings?

a. Increased risk for developing ketoacidosis b. Increased risk for developing hyperglycemia

c. Signs of insulin resistance d. Good control of blood glucose

A client with diabetes is visually impaired and wants to know whether syringes can be prefilled and stored for later use. Which is the nurse’s

best response? a. “Yes. Prefilled syringes can be stored for 3 weeks in the refrigerator

in a vertical position with the needle pointing up.” b. “Yes. Prefilled syringes can be stored for up to 3 weeks in the

refrigerator, placed in a horizontal position.” c. “Insulin reacts with plastic, so prefilled syringes are okay, but they

must be made of glass.” d. “No. Insulin cannot be stored for any length of time outside of the

container.”

A client has a new insulin pump. Which is the nurse’s priority instruction in teaching the client?

a. “Test your urine daily for ketones.” b. “Use only buffered insulin.”

c. “Keep the insulin frozen until you need it.” d. “Change the needle every 3 days.”

A client has been newly diagnosed with diabetes mellitus. Which statement made by the client indicates a need for further teaching

regarding nutrition therapy? a. “I should be sure to eat moderate to high amounts of fiber.”

b. “Saturated fats should make up no more than 7% of my total calorie intake.”

c. “I should try to keep my diet free from carbohydrates.” d. “My intake of plain water each day is not restricted.”

A client newly diagnosed with type 2 diabetes tells the nurse that since increasing fiber intake, he is having loose stools, flatulence, and

abdominal cramping. Which is the nurse’s best response? a. “Decrease your intake of water and other fluids until your stools

firm up.” b. “Decrease your intake of fiber now and gradually add it back into

your diet.” c. “You must have allergies to high-fiber foods and will need to avoid

them.” d. “Taking an antacid 1 hour before or 2 hours after meals will help

this problem.”

The nurse has been reviewing options for insulin therapy with several clients. For which client does the nurse choose to recommend the pen-

type injector insulin delivery system? a. Older adult client who lives at home alone but has periods of

confusion b. Client on an intensive regimen with frequent, small insulin doses

c. Client from the low-vision clinic who has trouble seeing the syringe d. “Brittle” client who has frequent episodes of hypoglycemia

A client is learning to inject insulin. Which action is important for the nurse to teach the client?

a. “Do not use needles more than twice before discarding.” b. “Massage the site for 1 full minute after injection.”

c. “Try to make the injection deep enough to enter muscle.” d. “Keep the vial you are using in the pantry or the bedroom drawer.”

To reduce complications of diabetes, the nurse teaches a client with normal kidney function to modify intake of which nutritional group?

a. Fats b. Fiber

c. Proteins d. Carbohydrates

A client with diabetes has proliferative retinopathy, nephropathy, and peripheral neuropathy. Which statement by the client indicates a good

understanding of the disease and exercise? a. “Because I have so many complications, I guess exercise is not a

good idea.” b. “I have so many complications that I better exercise hard to keep

from getting worse.” c. “I love to walk outside, but I probably better avoid doing that now.”

d. “I should look into swimming or water aerobics to get my exercise.”

A client in the emergency department has been diagnosed with ketoacidosis. Which manifestation does the nurse correlate with this

condition? a. Increased rate and depth of respiration

b. Extremity tremors followed by seizure activity c. Oral temperature of 102° F (38.9° C)

d. Severe orthostatic hypotension

The nurse determines that which arterial blood gas values are consistent with ketoacidosis in the client with diabetes?

a. pH 7.38, HCO3– 22 mEq/L, PCO2 38 mm Hg, PO2 98 mm Hg b. pH 7.28, HCO3– 18 mEq/L, PCO2 28 mm Hg, PO2 98 mm Hg c. pH 7.48, HCO3– 28 mEq/L, PCO2 38 mm Hg, PO2 98 mm Hg d. pH 7.28, HCO3– 22 mEq/L, PCO2 58 mm Hg, PO2 88 mm Hg

A client has diabetic ketoacidosis and manifests Kussmaul respirations. What action by the nurse takes priority?

a. Administration of oxygen by mask or nasal cannula b. Intravenous administration of 10% glucose

c. Implementation of seizure precautions d. Administration of intravenous insulin

A client with type 1 diabetes asks whether an occasional glass of wine is allowed in the diet. Which is the nurse’s best response?

a. “Drinking any wine or alcohol will increase your insulin requirements.”

b. “Because of poor kidney function, people diagnosed with diabetes should avoid alcohol at all times.”

c. “You shouldn’t drink alcohol because it will make you hungry and overeat.”

d. “One glass of wine is okay with a meal and is counted as two fat exchanges.”

The home care nurse visits an older client with diabetes. For which nutritional problem does the nurse monitor this client?

a. Obesity b. Malnutrition c. Alcoholism

d. Hyperglycemia

The nurse is teaching a client with diabetes about self-care. Which activity does the nurse teach that can decrease insulin needs?

a. Reducing intake of liquids to 2 L/day b. Eating animal organ meats high in insulin c. Limiting carbohydrate intake to 100 g/day

d. Walking 1 mile each day

The nurse is teaching a client with diabetes about exercise. Which statement by the client indicates a need for further teaching?

a. “I won’t exercise if I find ketones in my urine.” b. “If my blood glucose is over 200, I should not exercise.” c. “Exercise will help me keep my blood glucose down.”

d. “My risks for heart disease can be modified with exercise.”

Two months after a simultaneous pancreas-kidney (SPK) transplantation, a client is diagnosed as being in acute rejection. The

client states, “I was doing so well with my new organs, and the thought of having to go back to living on hemodialysis and taking insulin is so

depressing.” Which is the nurse’s best response? a. “You should have followed your drug regimen better.”

b. “You should be glad that at least dialysis treatment is an option for you.”

c. “One acute rejection episode does not mean that you will lose the new organs.”

d. “Our center is high on the list for obtaining organs from the national registry.”

Which statement made by a client getting ready for discharge after pancreas transplantation indicates a need for further teaching about the

prescribed drug regimen? a. “If I develop an infection, I should stop taking my corticosteroid.”

b. “If I have pain over the transplant, I will call the surgeon immediately.”

c. “I should avoid people who are ill or who have an infection.” d. “I should take my cyclosporine exactly the way I was taught.”

The nurse correlates which laboratory value with inadequate functioning of a transplanted pancreas?

a. Total white blood cell count 5000/mm3 b. 50% decrease in urine amylase level

c. Blood urea nitrogen 30 mg/dL d. Elevated bilirubin level

Three hours after surgery, the nurse notes that the breath of the client with type 1 diabetes has a “fruity” odor. Which is the nurse’s best first

action? a. Document the finding in the client’s chart.

b. Increase the IV fluid flow rate. c. Test the serum for ketone bodies.

d. Perform pulmonary hygiene.

A client with type 1 diabetes has a blood glucose level of 160 mg/dL on arrival at the operating room. Which is the nurse’s best action?

a. Document the finding in the client’s chart. b. Administer a bolus of regular insulin IV.

c. Call the physician to cancel the operation. d. Draw blood gases to assess the metabolic state.

A diabetic client has numbness and reduced sensation. Which intervention does the nurse teach this client to prevent injury?

a. “Examine your feet daily using a mirror.”

b. “Rotate your insulin injection sites.” c. “Wear white socks instead of colored socks.”

d. “Use a bath thermometer to test the water temperature.”

A client with a 20-year history of diabetes mellitus is reviewing his medications with the nurse. The client holds up the bottle of duloxetine

(Cymbalta) and states, “My cousin has depression and is on this drug. Do you think I’m depressed?” What is the nurse’s best response?

a. “Many people with long-term diabetes become depressed after a while.”

b. “It’s for peripheral neuropathy. Do you have burning pain in your feet or hands?”

c. “This antidepressant also has anti-inflammatory properties for diabetic pain.”

d. “That is possible, but most medications are used for several different things.”

A client has long-standing diabetes mellitus. Which finding alerts the nurse to decreased kidney function in this client?

a. Urine specific gravity of 1.033 b. Presence of glucose in the urine

c. Presence of ketone bodies in the urine d. Sustained elevation in blood pressure

A client with a history of diabetes mellitus has new onset of microalbuminuria. Which component of the diet must the client reduce?

a. Percentage of total calories derived from carbohydrates b. Percentage of total calories derived from proteins

c. Percentage of total calories derived from fats d. Total caloric intake

Which statement made by a diabetic client who has a urinary tract infection indicates that teaching was effective regarding antibiotic

therapy? a. “If my temperature is normal for 3 days in a row, I can stop taking

my medicine.” b. “If my temperature goes above 100° F (37.8° C), I should double the

dose.” c. “Even if I feel completely well, I should take the medication until it

is gone.” d. “When my urine no longer burns, I will no longer need to take the

antibiotics.”

The home care nurse finds a client who has diabetes awake and alert, but shaky, diaphoretic, and weak. The nurse gives the client cup of orange juice. The client’s clinical manifestations have not changed 5 minutes

later. Which is the nurse’s best next action? a. Give the client another cup of orange juice.

b. Call the rescue squad for transportation to the hospital. c. Administer 10 units of regular insulin subcutaneously.

d. Administer 1 mg glucagon intramuscularly.

The nurse has given a client an injection of glucagon. Which action does the nurse take next?

a. Apply pressure to the injection site. b. Position the client on his or her side.

c. Have a padded tongue blade available. d. Elevate the head of the bed.

A client is receiving IV insulin for hyperglycemia. Which laboratory value requires immediate intervention by the nurse?

a. Serum chloride level of 98 mmol/L b. Serum calcium level of 8.8 mg/dL

c. Serum sodium level of 132 mmol/L d. Serum potassium level of 2.5 mmol/L

The nurse is teaching a client about sick day management. Which statement by the nurse is most accurate?

a. “Continue your prescribed exercise regimen even if you are sick.” b. “Avoid eating or drinking to reduce vomiting and diarrhea.” c. “Do not use insulin or take your oral antidiabetic agent if you

vomit.” d. “Monitor your blood glucose levels at least every 4 hours.”

The nurse is teaching a client with type 2 diabetes about acute complications. Which teaching point by the nurse is most accurate?

a. Ketosis is less prevalent among obese adults owing to the protective effects of fat.

b. People with type 2 diabetes have normal lipid metabolism, so ketones are not made.

c. Insulin produced in type 2 diabetes prevents fat catabolism but not hyperglycemia.

d. Oral antidiabetic agents do not promote the breakdown of fat for fuel (lipolysis).

A client is being treated for hyperglycemic-hyperosmolar state (HHS). Which clinical manifestation indicates to the nurse that the therapy

needs to be adjusted? a. Serum potassium level has increased from 2.8 to 3.2 mEq/L.

b. Blood osmolarity has decreased from 350 to 330 mOsm. c. Score on the Glasgow Coma Scale is unchanged from 3 hours ago.

d. Urine has remained negative for ketone bodies for the past 3 hours.

The nurse administers 6 units of regular insulin and 10 units NPH insulin at 7 AM. At what time does the nurse assess the client for

problems related to the NPH insulin? a. 8 AM b. 4 PM c. 8 PM d. 11 PM

The nurse has been teaching a client about a new diagnosis of diabetes mellitus. Which statement by the client indicates a good understanding

of self-management? a. “After bathing each day, I will inspect my feet and rub lotion

between my toes and on my heels.”

b. “I can store 3 months’ worth of insulin at room temperature as long as the bottles are not open.”

c. “My medical alert bracelet is important to identify me as having diabetes if I am unconscious.”

d. “If I travel eastward to see my family, I should plan on using more insulin on the day I travel.”

A client was admitted with diabetic ketoacidosis (DKA). Which manifestations does the nurse monitor the client most closely for?

a. Shallow slow respirations and respiratory alkalosis b. Decreased urine output and hyperkalemia c. Tachycardia and orthostatic hypotension

d. Peripheral edema and dependent pulmonary crackles

A client has been taught about lifestyle changes to help manage newly diagnosed diabetes mellitus type 2. Which statement by the client

indicates good understanding? a. “Weight gain may lead to type 1 diabetes and I would need insulin.”

b. “I may not need to take medications if my weight is maintained.” c. “I do not have to check my blood glucose if my weight is in the

proper range.” d. “My vision and foot pain may go away if I lose some weight.”

Which statement by a client with type 2 diabetes indicates a need for further teaching about diabetic management and follow-up care?

a. “I need to have an annual appointment, even if my glucose levels are in good control.”

b. “Because my diabetes is controlled with diet and exercise, I have to be seen only if I am sick.”

c. “I can still develop complications, even though I do not have to take insulin at this time.”

d. “If I have surgery or get very ill, I may have to receive insulin injections for a short time.”

A client recently diagnosed with type 1 diabetes tells the nurse, “I will never be able to stick myself with a needle.” Which is the nurse’s best

response? a. “Try not to worry about it. We will give you your injections here in

the hospital.” b. “Everyone gets used to giving themselves injections. It really does

not hurt.” c. “I am not sure how your disease can be managed if you refuse to

give yourself the shots.” d. “Tell me what it is about the injections that is concerning you.”

The nurse is caring for a critically ill client who has diabetic ketoacidosis (DKA). The nurse finds the following assessment data: blood pressure, 90/62; pulse, 120 beats/min; respirations, 28 breaths/min; urine output,

20 mL/1 hour per catheter; serum potassium, 2.6 mEq/L. The health care provider orders a 40 mEq potassium bolus and an increase in the IV flow

rate. Which action by the nurse is most appropriate? a. Give the potassium after increasing the IV flow rate.

b. Increase the IV rate; consult the provider about the potassium. c. Increase the IV rate; hold the potassium for now.

d. Infuse the potassium first before increasing the IV flow rate.

MULTIPLE RESPONSE

The nurse is performing health screening in a local mall. Which people does the nurse counsel to be tested for diabetes? (Select all that apply.)

a. African-American or American Indian b. Person with history of pancreatic trauma

c. Woman with a 30-pound weight gain during pregnancy d. Male with a body mass index greater than 25 kg/m2

e. Middle-aged woman with physical inactivity most days of the week f. Young woman who gave birth to a baby weighing more than 9

pounds

OTHER

In mixing regular and NPH insulin, the nurse completes the following actions. Place these actions in the correct order. (Separate letters by a

comma and space as follows: a, b, c, d.) Inspect bottles for expiration dates. Gently roll bottle of NPH in hands.

Wash your hands. Inject air into the regular insulin.

Withdraw the NPH insulin. Withdraw the regular insulin. Inject air into the NPH bottle.

Clean rubber stoppers with an alcohol swab.

Chapter 68: Assessment of the Renal/Urinary System

Test Bank

MULTIPLE CHOICE

The nurse is palpating a client’s kidneys. The client’s right kidney is easily palpated, but the nurse cannot palpate the left kidney. What is the

nurse’s interpretation of this finding? a. The problem involves the right kidney. b. The problem involves the left kidney.

c. Both kidneys are in the normal position. d. The client is at increased risk for kidney impairment.

A client’s urinalysis results show a protein level of 1.8 mg/dL. Which action by the nurse is best?

a. Inform the health care provider. b. Ask the client about his or her protein intake.

c. Obtain the client’s weight. d. Document the finding in the chart.

The nurse is reviewing a client’s urinalysis and notes a positive glucose. Which action by the nurse is best?

a. Document the finding and call the health care provider. b. Collect and send another urinalysis sample to the laboratory.

c. Review the client’s recent dietary selections. d. Perform a finger stick blood glucose on the client.

Which condition is associated with oversecretion of renin? a. Alzheimer’s disease

b. Hypertension c. Diabetes mellitus d. Diabetes insipidus

A client’s urinalysis results reveal a urine osmolarity of 1200 mOsm/L. Which action by the nurse is most appropriate?

a. Initiate a fluid restriction. b. Prepare to administer a diuretic.

c. Institute seizure precautions. d. Encourage the client to increase fluid intake.

Which is the result of stimulation of erythropoietin production in the kidney tissue?

a. Increased blood flow to the kidney b. Inhibition of vitamin D and loss of bone density

c. Increased bone marrow production of red blood cells d. Inhibition of active transport of sodium and hyponatremia

The nurse is reviewing a client’s laboratory test results and notes a blood urea nitrogen (BUN) of 35 mg/dL and a creatinine of 1.0 mg/dL. What

new order does the nurse anticipate? a. Increase the client’s IV fluids. b. Prepare the client for dialysis.

c. Place the client on a fluid restriction. d. Obtain urine for culture and sensitivity.

The nursing assistant is using a bladder scanner on a client. Which action by the nursing assistant requires further education on the use of

this device? a. Consistently choosing the female icon for all female clients

b. Consistently choosing the male icon for all male clients c. Applying ultrasound gel to the scanning head and removing it when

finished d. Taking at least two readings by using the aiming icon to place the

scanning head.

A client has an increased BUN/creatinine ratio. Which action by the nurse is most appropriate?

a. Assess the client’s dietary habits. b. Inquire about the use of NSAIDs.

c. Hold the client’s metformin (Glucophage). d. Notify the health care provider immediately.

A client’s urine specific gravity is 1.040. Which action by the nurse is best?

a. Obtain a urine culture and sensitivity. b. Place the client on restricted fluids. c. Review the client’s creatinine level.

d. Increase the client’s fluid intake.

Which assessment maneuver does the nurse perform first when assessing the renal system at the same time as the abdomen?

a. Abdominal percussion b. Abdominal auscultation

c. Abdominal palpation d. Renal palpation

A client was admitted for a myocardial infarction and cardiogenic shock. Two days later, which laboratory test results does the nurse expect to

see? a. Blood urea nitrogen (BUN) of 52 mg/dL

b. Creatinine of 2.3 mg/dL c. BUN of 10 mg/dL

d. BUN-creatinine ratio of 8:1

A client’s urine specific gravity is 1.018. Which is the nurse’s best action? a. Ask the client for a 24-hour recall of liquid intake.

b. Document the finding in the client’s chart. c. Obtain a specimen for culture. d. Notify the health care provider.

Which condition would trigger the release of antidiuretic hormone (ADH)?

a. Overhydration b. Dehydration c. Hemorrhage

d. Edema

The female client’s urinalysis shows all the following results. Which does the nurse document as abnormal?

a. pH 5.6 b. Ketone bodies present

c. Specific gravity of 1.030 d. Two white blood cells per high-power field

A client scheduled for intravenous urography informs the nurse of the following allergies. Which one does the nurse report to the health care

provider immediately? a. Seafood

b. Penicillin c. Bee stings

d. Red food dye

A client scheduled to have intravenous urography has diabetes and is taking the antidiabetic agent metformin (Glucophage). What does the

nurse tell this client? a. “Call your diabetes doctor and tell him or her that you are having

an intravenous urogram performed using dye.”

b. “Do not take your metformin the morning of the test because you are not going to be eating anything and you could become

hypoglycemic.” c. “You must start on an antibiotic before this test because your risk of

infection is greater as a result of your diabetes.” d. “You must take your metformin immediately before the test is

performed because the IV fluid and the dye contain significant amounts of sugar.”

A client is going home after urography. Which instruction or precaution does the nurse teach this client?

a. “Avoid direct contact with the urine for 24 hours until the radioisotope clears.”

b. “You may have some dribbling of urine for several weeks after this procedure.”

c. “Be sure to drink at least 3 liters of fluids today to help eliminate the dye faster.”

d. “Your skin may become slightly yellow from the dye used in this procedure.”

Two hours after a closed percutaneous kidney biopsy, the client reports a dramatic increase in pain. What is the nurse’s best first action?

a. Reposition the client on the operative side. b. Administer the prescribed opioid analgesic.

c. Assess the pulse rate and blood pressure. d. Check the Foley catheter for kinks.

A client is scheduled to have renography (kidney scan). The client voices concern about discomfort during the procedure. Which is the nurse’s

best response? a. “Before the test, you will be given a sedative to reduce any pain.” b. “A local anesthetic agent will be used, so you won’t feel any pain.” c. “No more discomfort is felt with the scan than with an ordinary x-

ray.” d. “The only pain will occur when you have your IV line started.”

To obtain a sterile urine specimen from a client with a Foley catheter, the nurse begins by applying a clamp to the drainage tubing distal to the

injection port. What does the nurse do next? a. Clamp another section of the tube to create a fixed sample section

for retrieval. b. Insert a syringe into the injection port and aspirate the quantity of

urine required. c. Clean the injection port cap of the drainage tubing with povidone-

iodine solution. d. Withdraw 10 mL of urine and discard it; then withdraw 10 mL

more for the sample.

MULTIPLE RESPONSE

Which results are normal in a urinalysis? (Select all that apply.) a. pH, 6

b. Specific gravity, 1.015 c. Protein, 1.2 mg/dL d. Glucose, negative

e. Nitrate, small f. Leukocyte esterase, positive

Chapter 69: Care of Patients with Urinary Problems

Test Bank

MULTIPLE CHOICE

Which client is at greatest risk for development of a bacterial cystitis? a. Older woman not taking estrogen replacement b. Older man with mild congestive heart failure

c. Middle-aged woman who has never been pregnant d. Middle-aged man taking cyclophosphamide for cancer therapy

A client has a fungal urinary tract infection. Which assessment by the nurse is most helpful?

a. Palpating and percussing the kidneys and bladder b. Assessing medical history and current medical problems c. Performing a bladder scan to assess post-void residual

d. Inquiring about recent travel to foreign countries

The nurse is assessing the laboratory findings of a client with a urinary tract infection. The laboratory report notes a “shift to the left” in a client’s white blood cell count. Which action by the nurse is most

appropriate? a. Request that the laboratory perform a differential analysis on the

white blood cells. b. Notify the health care provider and start an IV line for parenteral

antibiotics. c. Instruct the client to begin straining all urine for renal calculi.

d. Document the finding in the client’s chart and continue to monitor.

Which client statement indicates a good understanding regarding antibiotic therapy for recurrent urinary tract infections?

a. “If my urine becomes lighter and clearer, I can stop taking my medicine.”

b. “Even if I feel completely well, I should take the medication until it is gone.”

c. “When my urine no longer burns, I will no longer need to take the antibiotics.”

d. “If I have a fever higher than 100° F (37.8° C), I should take twice as much medicine.”

A postmenopausal female client has had two episodes of bacterial urethritis in the last 6 months. She asks her nurse why this is happening

to her now. Which is the nurse’s best response? a. “Your immune system becomes less effective as you age.”

b. “Low estrogen levels can make the tissue more susceptible to infection.”

c. “You should be more careful with your personal hygiene in this area.”

d. “It is likely that you have an untreated sexually transmitted infection.”

A client is hospitalized with urinary retention, has an indwelling catheter, and is getting IV fluids. Which intervention does the nurse add to the

care plan to address the priority problem for this client? a. Perform catheter care per policy every shift.

b. Encourage fluid intake to 1 liter/day. c. Apply a moisture barrier cream daily.

d. Document accurate intake and output (I&O) each shift.

A client with severe bacterial cystitis is prescribed cefadroxil (Duricef) and phenazopyridine (Pyridium). What statement by the client indicates

an accurate understanding of these medications? a. “I will not take these drugs with food or milk.”

b. “I will stop these drugs if I think I am pregnant.” c. “An orange color in my urine won’t alarm me.”

d. “I will try to drink a liter of cranberry juice daily.”

Which statement made by a client with stress incontinence indicates a need for clarification of nutrition therapy? a. “I will limit my total intake of fluids.”

b. “I will avoid drinking alcoholic beverages.” c. “I will avoid drinking caffeinated beverages.”

d. “I will try to lose about 10% of my body weight.”

The nurse is working in an incontinence clinic and sees older clients. The nurse plans a habit training program for the client with which condition?

a. Confusion b. Diabetes

c. Early kidney failure d. Arthritis

The nurse is working in a long-term care facility where many clients use habit training to manage incontinence. Which action by unlicensed

assistive personnel (UAP) requires intervention by the nurse? a. Toileting clients after meals

b. Changing incontinence briefs when wet c. Encouraging clients to drink fluids d. Recording incontinence episodes

A client has overflow incontinence. Which intervention does the nurse add to this client’s care plan to assist with elimination?

a. Stroking the medial aspect of the thigh b. Using intermittent catheterization c. Providing digital anal stimulation

d. Using the Valsalva maneuver

The caretaker of a confused client with functional incontinence asks about having an in-dwelling catheter placed. Which is the nurse’s best

response? a. “You must be very aggravated about this situation. I will call the

provider with this request.” b. “I will teach you how to insert the catheter, which should be used

just at night.” c. “We can teach you how to perform intermittent catheterization to

drain the bladder.” d. “Because the client is confused, we need to place priority on keeping

the skin clean and dry.”

A confused client is hospitalized for possible pneumonia and is admitted from the emergency department with an indwelling catheter in place.

During interdisciplinary rounds the following day, what question by the nurse takes priority?

a. “Do you want daily weights on this client?” b. “Will the client be able to return home?”

c. “Can we discontinue the in-dwelling catheter?”

d. “Should we get another chest x-ray today?”

A client has a history of renal calculi. Which statement by the client indicates a good understanding of preventive measures?

a. “I know I should drink at least 3 to 4 liters of fluid every day.” b. “I can’t eat much dairy or other sources of calcium.”

c. “Aspirin and aspirin-containing products can lead to stones.” d. “The doctor will give me antibiotics at the first sign of a stone.”

A client has kidney stones from secondary hyperoxaluria. Which medication does the nurse anticipate administering?

a. Phenazopyridine (Pyridium) b. Propantheline (Pro-Banthine)

c. Tolterodine (Detrol-LA) d. Allopurinol (Zyloprim)

A client with a renal calculus has just returned from an extracorporeal shock wave lithotripsy procedure, and the nurse finds an ecchymotic

area on the client’s right lower back. Which is the nurse’s priority

intervention? a. Notify the health care provider.

b. Apply ice to the site. c. Place the client in the prone position.

d. Document the observation in the chart.

A client has been admitted from a nursing home for a workup to determine the cause of several recent falls. What intervention by the

nurse takes priority? a. Obtain a clean catch or catheterized urine specimen.

b. Document the number of and causative factors for falls. c. Review the results of recent laboratory work for kidney function.

d. Facilitate neurologic and social work consultations.

A client who has undergone a nephrolithotomy procedure 24 hours ago now has a fever of 101° F (38.3° C). What is the nurse’s priority

intervention? a. Apply a cooling blanket. b. Strain the client’s urine.

c. Notify the health care provider. d. Document the finding in the client’s chart.

Which personal factor places a client at risk for bladder cancer? a. Working in a lumber yard for 10 years b. 50-pack-year cigarette smoking history c. Numerous episodes of bacterial cystitis d. History of sexually transmitted diseases

A client with bladder cancer is scheduled to have intravesical chemotherapy. Which statement made by the client indicates correct

understanding of this therapy? a. “My hair will start growing back in 3 to 6 weeks after chemotherapy

is over.” b. “My white blood cell count will drop and I will be at increased risk

for infection.” c. “This type of chemotherapy is used when no distant metastases are

present.” d. “Chemotherapy only controls cancer, so I will also need radiation.”

A client with bladder cancer has undergone a complete cystectomy with ileal conduit. Four hours after the surgery, the nurse observes the stoma

to be cyanotic. Which is the nurse’s priority action? a. Reassess in 2 hours.

b. Loosen the dressing. c. Notify the surgeon.

d. Apply oxygen.

Which is an initial priority intervention for a client with stress incontinence?

a. Beginning medication and dietary teaching b. Referring the client to an incontinence clinic c. Assisting the client in finding absorbent pads

d. Instructing the client to maintain an incontinence diary

A client presents with senile dementia, Alzheimer’s type (SDAT), and incontinence. Which therapy will best help this client?

a. Bladder training b. Habit training

c. Exercise therapy d. Electrical stimulation

A client is being admitted with a suspected diagnosis of bladder cancer.

Which question assists the nurse in determining risk factors? a. “Do you smoke cigarettes?”

b. “Do you use alcohol?” c. “Do you use recreational drugs?”

d. “Do you take any prescription drugs?”

A client is scheduled to undergo the surgical creation of an ileal conduit. He expresses his anxiety and fear regarding the procedure. Which is an

appropriate response from the nurse? a. “Do you think something for your nerves would be helpful?”

b. “Would you like to discuss the procedure with your doctor once more?”

c. “Are you ready for your sleeping medication now?” d. “Would you like to speak with someone who has an ileal conduit?”

A client is receiving treatment with levofloxacin (Levaquin). Which teaching topics does the nurse include in this client’s care plan?

a. How to assess blood pressure b. How to assess a radial pulse c. How to assess a carotid pulse

d. How to assess respirations

A young woman is being treated with amoxicillin (Amoxil) for a urinary tract infection. Which is the highest priority instruction for the nurse to

give this client? a. “Use a second form of birth control while on the drug.”

b. “You will experience increased menstrual bleeding while on this drug.”

c. “You may experience an irregular heartbeat while on the drug.” d. “Watch for blood in your urine while taking this drug.”

A client is receiving treatment with liquid nitrofurantoin (Furadantin). Which is the highest priority instruction that the nurse can provide to

this client regarding accurate administration of the medication? a. “The medication should be mixed with cold water before drinking

it.” b. “Urine will turn orange immediately after you swallow the drug.”

c. “You should ask the pharmacist for a syringe to measure the dose.” d. “The drug is available in granules that must be dissolved.”

A client is receiving acetohydroxamic acid (Lithostat). Which statement

by the client indicates a good understanding of this therapy? a. “I should finish this antibiotic even if I am feeling better.”

b. “I need to drink a full glass of water when I take this drug.” c. “My blood will be drawn occasionally for kidney function tests.” d. “This medication may turn my urine bright orange and stain my

clothes.”

Which type of incontinence is most common after a difficult vaginal delivery?

a. Stress b. Urge

c. Reflex d. Overflow

A client has functional urinary incontinence. Which instruction by the nurse to the client and family helps meet an expected outcome for this

condition? a. “You must clean around your catheter daily with soap and water.” b. “Wash the vaginal weights with a 10% bleach solution after each

use.” c. “Operations to repair your bladder are available, and you can

consider these.” d. “Buy slacks with elastic waistbands that are easy to pull down.”

A client in the emergency department reports extreme dry mouth, constipation, and an inability to void. The client’s history includes

incontinence. Which question by the nurse is most important? a. “Are you drinking plenty of water?”

b. “Do you take anticholinergic medication?” c. “Have you tried laxatives or enemas?”

d. “Has this type of thing ever happened before?”

A client is beginning to undergo urinary bladder training. Which is an effective instruction to give this client?

a. “Use the toilet at the first urge, rather than at specific intervals.” b. “Try to consciously hold your urine until the scheduled toileting

time.” c. “Initially try to use the toilet at least every half-hour for 24 hours.”

d. “The toileting interval can be increased once you have been continent for 1 week.”

MULTIPLE RESPONSE

A client has had two episodes of bacterial cystitis in the last 6 months. Which questions should the nurse ask? (Select all that apply.)

a. “How much water do you drink every day?” b. “Do you take estrogen replacement?”

c. “Does anyone in your family have a history of cystitis?” d. “Do you have any condition that affects your immune system?”

e. “Are you on steroids or other immune suppressant drugs?” f. “Do you drink grapefruit juice every day?”

The nurse is teaching a client about self-catheterization in the home setting. Which instructions are applicable? (Select all that apply.)

a. “Wash your hands before and after self-catheterization.” b. “Use a large-lumen catheter for each catheterization.”

c. “Use lubricant on the tip of the catheter before insertion.” d. “Self-catheterize every 12 hours.”

e. “Use sterile gloves for the procedure.” f. “Maintain a specific schedule for catheterization.”

Which statements about urge incontinence and stress incontinence are true? (Select all that apply.)

a. Urge incontinence involves a post-voiding residual volume less than 50 mL.

b. Stress incontinence occurs because of weak pelvic floor muscles. c. Stress incontinence usually occurs in people with dementia.

d. Urge incontinence can be managed by increasing fluid intake. e. Urge incontinence occurs because of abnormal bladder contractions.

Chapter 70: Care of Patients with Renal Disorders

Test Bank

MULTIPLE CHOICE

On assessment of a client with polycystic kidney disease (PKD), which finding is of greatest concern to the nurse?

a. Flank pain b. Periorbital edema

c. Bloody and cloudy urine d. Enlarged abdomen

A client with autosomal dominant polycystic kidney disease (ADPKD) asks whether his children could develop this disease. Which is the nurse’s

best response? a. “No genetic link is known, so your children are not at increased

risk.” b. “The disease is sex linked, so only your sons could be affected.”

c. “Both you and your wife must have the disease for your children to develop it.”

d. “Each of your children has a 50% risk of having ADPKD.”

A client with polycystic kidney disease and hypertension is prescribed a diuretic for blood pressure control. Which statement by the client

indicates the need for further teaching regarding these orders? a. “I will weigh myself every day at the same time.”

b. “I will drink only 1 liter of fluid each day.” c. “I will avoid aspirin and aspirin-containing drugs.”

d. “I will avoid nonsteroidal anti-inflammatory drugs.”

A client with polycystic kidney disease (PKD) has received extensive teaching in the clinic. Which statement by the client indicates that an

important goal related to nutrition is being met? a. “I take a laxative every night before going to bed.”

b. “I have a soft bowel movement every morning.” c. “Food tastes so much better since I can use salt again.”

d. “The white bread I am eating does not cause gas.”

A client has a large renal calculus. Which assessment finding may indicate the development of a complication?

a. Blood pressure of 178/94 mmHg b. Urine output of 5600 mL/24 hr

c. Client reports of pain on urination d. Asymmetric, tender flank area

A client is hospitalized with a urinary tract infection (UTI). Which clinical manifestation alerts the nurse to the possibility of a complication

from the UTI? a. Burning on urination b. Cloudy, dark urine

c. Fever and chills d. Hematuria

A middle-aged client with diabetes mellitus is being treated for the third episode of acute pyelonephritis in the past year and asks what can be

done to help prevent these infections. Which is the nurse’s best response? a. “Test your urine daily for the presence of ketone bodies and

proteins.” b. “Use tampons rather than sanitary napkins during your menstrual

period.” c. “Drink more water and empty your bladder every 2 to 3 hours

during the day.” d. “Keep your hemoglobin A1c under 9% by keeping your blood sugar

controlled.”

In assessing a client recently diagnosed with acute glomerulonephritis, the nurse asks which question to determine potential contributing

factors? a. “Are you sexually active?”

b. “Do you have pain or burning on urination?” c. “Has anyone in your family had chronic kidney problems?”

d. “Have you had a cold or sore throat within the last 2 weeks?”

The nurse completes which assessment in a client with acute glomerulonephritis and periorbital edema?

a. Auscultating breath sounds b. Checking blood glucose levels

c. Measuring deep tendon reflexes d. Testing urine for protein

A client with glomerulonephritis has a glomerular filtration rate (GFR) of 40 mL/min, as measured by a 24-hour creatinine clearance. Which is

the nurse’s interpretation of this finding? a. Excessive GFR, client at risk for dehydration

b. Excessive GFR, client at risk for fluid overload c. Reduced GFR, client at risk for dehydration

d. Reduced GFR, client at risk for fluid overload

An older client is hospitalized with suspected heart failure. After 2 days of treatment, the client is not improving. Which laboratory value does

the nurse report to the provider? a. Potassium, 3.7 mEq/L b. Sodium, 144 mEq/L

c. Glomerular filtration rate, 55 mL/min d. Creatinine, 0.9 mg/dL

Which clinical manifestation indicates to the nurse that a client with glomerulonephritis (GN) is responding as expected to the prescribed

treatment? a. The client has lost 11 pounds in the past 10 days.

b. The client’s urine specific gravity is 1.048. c. No blood is observed in the client’s urine.

d. The client’s blood pressure is 152/88 mm Hg.

A client has nephrotic syndrome and a normal glomerular filtration. Which dietary selection shows that the client understands nutritional

therapy for this condition? a. Decreased intake of protein b. Increased intake of protein

c. Decreased intake of carbohydrates d. Increased intake of carbohydrates

A client has nephrotic syndrome. Which finding shows that therapy is effective?

a. Serum albumin level, 2.8 g/dL b. Serum albumin level, 4 g/dL

c. Urine protein level, 3.7 g/24 hr d. Potassium, 4.2 mEq/L

In planning care for a client with renal cell carcinoma, the nurse monitors for which electrolyte imbalance?

a. Hyponatremia b. Hypernatremia c. Hypocalcemia d. Hypercalcemia

In assessing a client 6 hours after a radical nephrectomy for renal cell carcinoma, the nurse notes that the client’s blood pressure has decreased

from 134/90 to 100/56 mm Hg and urine output is 20 mL for this past hour. Which is the nurse’s best action?

a. Position the client so that the remaining kidney is not dependent. b. Measure the specific gravity of the client’s urine.

c. Document the findings in the client’s record. d. Assess the pulse rate and quality, and then notify the provider.

A client is in the emergency department after experiencing kidney trauma. The abdomen is tender and distended, and blood is visible at the

urinary meatus. Which action by the nurse is most appropriate? a. Assess vital signs and abdominal pain every 5 to 15 minutes.

b. Consult with the provider before inserting a catheter. c. Monitor the client’s IV rate and prepare to give blood.

d. Assist with obtaining informed consent for surgery if needed.

A client with diabetes is hospitalized with recurrent pyelonephritis. The provider orders IV gentamicin (Garamycin) before culture results come

back. Which action by the nurse is best? a. Monitor the client’s blood sugar before and after each dose. b. Consult with the pharmacist about the antibiotic selection.

c. Monitor the client’s daily blood urea nitrogen and creatinine levels. d. Check the client’s most recent hemoglobin A1c result.

Which statement by a client with hypertension secondary to renal disease indicates the need for further teaching?

a. “I can prevent more damage to my kidneys by managing my blood pressure.”

b. “If I have increased urination at night, I need to drink less fluid during the day.”

c. “I need to see the dietitian to discuss limiting my protein intake.” d. “It is important that I take my antihypertensive medications as

directed.”

A client returned to the nursing unit after having a nephrostomy performed. Over the next 6 hours, drainage in the tube has gone from 40

mL/hr to 12 mL over the last hour. Which intervention by the nurse is most appropriate?

a. Document the finding in the client’s record. b. Evaluate the tube as working in the hand-off report.

c. Clamp the tube in preparation for removing it. d. Assess the client’s abdomen and vital signs.

A client who had kidney trauma required a nephrectomy. What does the nurse teach the client about this condition?

a. “You need to avoid participating in contact sports like football.” b. “You probably will end up on dialysis a few years from now.”

c. “You need medication to control your high blood pressure from the injury.”

d. “You will always be required to restrict your salt and fluid intake.”

The visiting nurse has many clients who are African American. Which intervention is most important for the nurse to accomplish when seeing

these clients? a. Weigh the clients and compare their weights.

b. Assess the clients’ blood pressure. c. Observe the clients for any signs of abuse.

d. Ask the clients about their medications.

MULTIPLE RESPONSE

In interviewing a client with a family history of polycystic kidney disease (PKD), the nurse assesses for which clinical manifestations most

carefully? (Select all that apply.) a. Nocturia

b. Flank pain c. Diarrhea d. Dysuria

e. Bloody urine f. Increased abdominal girth

The nurse monitors for which clinical manifestations in a client with nephrotic syndrome? (Select all that apply.)

a. Proteinuria, >3.5 g/24 hr b. Hypoalbuminemia

c. Dehydration d. Lipiduria e. Dysuria

f. Costovertebral angle (CVA) tenderness

COMPLETION

A client presents to the emergency department with severe dehydration and is ordered to receive 3 L of fluid over 6 hours. The nurse sets the

intravenous pump at a rate of mL/hr.

Chapter 71: Care of Patients with Acute Kidney Injury and Chronic Kidney Disease

Test Bank

MULTIPLE CHOICE

Which client is most at risk for developing postrenal kidney failure? a. Client diagnosed with renal calculi b. Client with congestive heart failure

c. Client taking NSAIDs for arthritis pain d. Client recovering from glomerulonephritis

A client is admitted to the hospital with a serum creatinine level of 2 mg/dL. When taking the client’s history, which question does the nurse

ask first? a. “Do you take any nonprescription medications?”

b. “Does anyone in your family have kidney disease?” c. “Do you have yearly blood work done?”

d. “Is your diet low in protein?”

A client with a decreased glomerular filtration rate asks how to prevent further damage to the kidneys. Which is the nurse’s best response? a. “The diuretics you are taking will prevent further damage.”

b. “Kidney damage is inevitable as you age.” c. “Avoid taking NSAIDs.”

d. “You will need to follow a high-protein diet.”

A client who has chronic kidney disease is being discharged from the hospital after receiving treatment for a hip fracture. Which information is most important for the nurse to provide to the client before discharge?

a. “Increase your intake of foods with protein.” b. “Monitor your daily intake and output.”

c. “Maintain bedrest until the fracture is healed.” d. “Take your aluminum hydroxide (Nephrox) with meals.”

Which intervention is most important for the nurse to implement in a client after kidney transplant surgery?

a. Promote acceptance of new body image. b. Monitor magnesium levels daily.

c. Place the client on protective isolation. d. Remove the indwelling (Foley) catheter as soon as possible.

During a hot summer day, an older adult client tells the clinic nurse, “I am not drinking or voiding that much these days.” The nurse notes a

heart rate of 100 beats/min and a blood pressure of 90/60 mm Hg. Which action does the nurse take first?

a. Give the client something to drink. b. Insert an intravenous catheter.

c. Teach the client to drink 2 to 3 liters a day. d. Perform a bladder scan to assess urine volume.

A client is taking furosemide (Lasix). To detect a common adverse effect, the nurse obtains which assessment as a priority?

a. Breath sounds b. Heart sounds

c. Intake and output d. Nutritional patterns

A client with acute kidney failure and on dialysis asks how much fluid will be permitted each day. Which is the nurse’s best response? a. “This is based on the amount of damage to your kidneys.”

b. “You can drink an amount equal to your urine output, plus 700 mL.”

c. “It is based on your body weight and changes daily.” d. “You can drink approximately 2 liters of fluid each day.”

Which statement by a client who has undergone kidney transplantation indicates a need for more teaching?

a. “I will need to continue to take insulin for my diabetes.” b. “I will have to take my cyclosporine for the rest of my life.”

c. “I will take the antibiotics three times daily until the medication is finished.”

d. “My new kidney is working fine. I do not need to take medications any longer.”

Which staff member does the charge nurse assign to care for a client newly diagnosed with chronic kidney disease?

a. Licensed practical nurse who usually works on the unit b. Registered nurse floated from the hemodialysis unit

c. Registered nurse who has taken care of this client before d. Registered nurse with the most years of experience

A client has been missing some scheduled hemodialysis sessions. Which intervention is most important for the nurse to implement?

a. Discussing with the client his or her acceptance of the disease b. Discussing with the client the option of peritoneal dialysis c. Rescheduling the sessions to another day or another time

d. Stressing to the client the importance of going to the sessions

Assessment findings reveal that a client with chronic kidney disease is refusing to take prescribed medications because of the “cost.” The client also is having difficulty performing activities of daily living and prefers to sleep most of the day. To which health care team member does the

nurse refer the client? a. Home health aide b. Physical therapist

c. Psychiatric nurse practitioner d. Physician

A client has a serum creatinine level of 2 mg/dL and a urine output of 1000 mL/day. How does the nurse categorize the client’s kidney injury?

a. Intrarenal b. Nonoliguric

c. Prerenal d. Postrenal

A client has a serum creatinine level of 2.5 mg/dL, a serum potassium level of 6 mmol/L, an arterial pH of 7.32, and a urine output of 250

mL/day. Which phase of acute kidney failure is the client experiencing?

a. Intrarenal b. Nonoliguric

c. Oliguric d. Postrenal

A client has been diagnosed with acute postrenal kidney injury. Which assessment finding does the nurse assess most carefully for?

a. Blood urea nitrogen (BUN), 35 mg/dL b. Creatinine, 2.5 mg/dL

c. Feeling of urgency d. Weight gain and edema

A client with chronic hypertension is seen in the clinic. Which assessment indicates that the client’s hypertension is not under control?

a. Heart rate of 55 beats/min b. Serum creatinine level of 1.9 mg/dL

c. Blood glucose level of 128 mg/dL d. Irregular heart sounds

A client is scheduled to have dialysis in 30 minutes and is due for the following medications: vitamin C, B-complex vitamin, and cimetidine

(Tagamet). Which action by the nurse is best? a. Give medications with a small sip of water.

b. Hold all medications until after dialysis. c. Give the supplements, but hold the Tagamet. d. Give the Tagamet, but hold the supplements.

When evaluating the effects of a low-protein diet in a client with chronic kidney disease, the nurse is most concerned with which result?

a. Albumin level of 2 g/dL b. Calcium level of 8.0 mg/dL

c. Potassium level of 5.2 mmol/L d. Magnesium level of 3 mEq/L

A client with acute kidney injury is placed on a fluid restriction. To determine whether outcomes related to fluid balance are being met, the

nurse assesses for which finding? a. Absence of lung crackles

b. Decreased serum creatinine level c. Decreased serum potassium level

d. Increased muscle strength

)

A client with chronic kidney disease is scheduled to be given the following medications: digoxin (Lanoxin) and epoetin alfa (Epogen). The

client reports nausea and vomiting and wishes to wait to take the medications. Which action by the nurse is most appropriate?

a. Administer both medications with soda crackers. b. Allow the client to wait an hour before taking the medications.

c. Review today’s potassium level and notify the health care provider. d. Call the health care provider to get an order for anti-nausea

medication.

A client is receiving continuous arteriovenous hemofiltration (CAVH). Which laboratory value does the nurse monitor most closely?

a. Hemoglobin b. Glomerular filtration rate

c. Sodium d. White blood cells

A client who is 2 days post–femoral vein cannulation begins to have

difficulty with outflow of blood during dialysis. For which complication does the nurse assess?

a. Hematoma at cannula insertion site b. Infection c. Oliguria

d. Skin necrosis at cannula insertion site

A client is admitted with a 3-day history of vomiting and diarrhea. The client’s vital signs are blood pressure, 85/60 mm Hg; and heart rate, 105

beats/min. Which intervention by the nurse takes priority? a. Obtain blood and urine cultures.

b. Start an IV of normal saline as ordered. c. Administer antiemetic medications.

d. Assess the client’s recent travel history.

Which response by a client indicates an understanding of measures to facilitate the flow of peritoneal dialysate fluid? a. “I will take my stool softeners every day.”

b. “I will keep the drainage bag at the level of my abdomen.” c. “Flushing the catheter is needed with each exchange.”

d. “Warmed dialysate infusion increases the speed of flow.”

When providing care for a client receiving peritoneal dialysis, the nurse notices that the effluent is cloudy. Which intervention is most important

for the nurse to carry out? a. Irrigate the peritoneal catheter with saline. b. Send a specimen for culture and sensitivity. c. Document the finding in the client’s chart.

d. Change the dialysate solution and catheter tubing.

During hemodialysis, a client with chronic kidney disease develops headache, nausea, vomiting, and restlessness. After notifying the health

care provider, which action by the nurse is most appropriate? a. Administer a bolus of dextrose solution. b. Draw blood for sodium and potassium. c. Order a blood urea nitrogen level stat.

d. Prepare to administer phenytoin (Dilantin),

A client’s temperature after dialysis is 99° F (37.2° C) and was normal before dialysis. Which is the nurse’s best action?

a. Continue to monitor the temperature. b. Encourage the client to drink fluids.

c. Obtain a white blood cell count.

d. Prepare to culture the fistula site.

The RN has assigned a client with a newly placed arteriovenous (AV) fistula in the right arm to an LPN. Which information about the care of

this client is most important for the RN to provide to the LPN? a. “Avoid movement of the right extremity.”

b. “Place gentle pressure over the fistula site after blood draws.” c. “Start any IV lines below the site of the fistula.”

d. “Take blood pressure in the left arm.”

The nurse is providing dietary teaching to a client who was just started on peritoneal dialysis (PD). Which instruction does the nurse provide to

this client regarding protein intake? a. “Your protein needs will not change, but you may take more fluids.”

b. “You will need more protein now because some protein is lost by dialysis.”

c. “Your protein intake will be adjusted according to your predialysis weight.”

d. “You no longer need to be on protein restriction.”

A client was just admitted to the emergency department for new-onset confusion. As the nurse starts the IV line, the client says he just finished a hemodialysis session. The IV site is bleeding briskly. What action by the

nurse takes priority? a. Assess for a bruit and thrill over the vascular access site.

b. Draw blood for coagulation studies and white blood cell count. c. Prepare to administer protamine sulfate.

d. Hold constant firm pressure with a gauze pad for 5 minutes.

A client with acute kidney injury had normal assessments 1 hour ago. Now the nurse finds that the client’s respiration rate is 44 breaths/min and the client is restless. Which assessment does the nurse perform?

a. Obtain an oxygen saturation level. b. Send blood for a creatinine level. c. Assess the client for dehydration. d. Perform a bedside blood glucose.

A client has end-stage kidney disease (ESKD). Which food selection by the client demonstrates understanding of a low-sodium, low-potassium

diet? a. Bananas

b. Ham c. Herbs and spices d. Salt substitutes

A client who is admitted to the hospital with a history of kidney disease begins to have difficulty breathing. Vital signs are as follows: blood

pressure, 90/70 mm Hg; heart rate, difficult to feel peripheral pulses. His heart sounds are difficult to hear. Which intervention does the nurse

prepare for? a. Administration of digoxin (Lanoxin)

b. Draining of pericardial fluid with a needle c. Emergency hemodialysis

d. Placement of a pacemaker

A client with chronic kidney disease states that he will be going to the dentist for a planned tooth extraction. Which is the nurse’s best

response? a. “Rinse your mouth with an antiseptic solution after the procedure.”

b. “Kidney disease is probably what caused your dental decay.” c. “You should receive prophylactic antibiotics before any dental

procedure.” d. “You may take any medication for pain that the dentist prescribes.”

A client hospitalized for worsening kidney injury suddenly becomes restless and agitated. Assessment reveals tachycardia and crackles

bilaterally at the bases of the lungs. Which is the nurse’s first intervention?

a. Begin ultrafiltration. b. Administer an antianxiety agent.

c. Place the client on mechanical ventilation. d. Place the client in high Fowler’s position.

The nurse is caring for a client with chronic kidney disease who has developed uremia. Which assessment finding does the nurse correlate

with this problem? a. Decreased breath sounds

b. Foul-smelling urine c. Heart rate of 50 beats/min

d. Respiratory rate of 40 breaths/min

A client has end-stage kidney disease (ESKD). The nurse observes tall,

peaked T waves on the client’s cardiac monitor. Which action by the nurse is best?

a. Check the serum potassium level. b. Document the finding in the client’s chart.

c. Prepare to give sodium bicarbonate. d. Call the health care provider to request an electrocardiogram

(ECG).

Which assessment parameter does the nurse monitor in a client with chronic kidney disease to determine fluid and sodium retention status?

a. Capillary refill b. Intake and output c. Muscle strength

d. Weight and blood pressure

The nurse is caring for a client who is receiving peritoneal dialysis (PD). Which nursing intervention has the greatest priority when a dialysis

exchange is performed? a. Adding potassium and antibiotic to the dialysate bags

b. Positioning the client on either side c. Using sterile technique when hooking up dialysate bags

d. Warming the dialysate fluid in a microwave oven

The nurse is providing a client with a peritoneal dialysis exchange. The nurse notes the presence of cloudy peritoneal effluent. Which action by

the nurse is most appropriate? a. Document the finding in the client’s chart. b. Collect a sample to send to the laboratory.

c. Reposition the client on the left side. d. Increase the free water content in the next bag.

A client is 12 hours post–kidney transplantation. The nurse notes that the client has put out 2000 mL of urine in 10 hours. Which assessment does

the nurse carry out first? a. Skin turgor

b. Blood pressure c. Serum blood urea nitrogen (BUN) level

d. Weight of the client

A client who underwent kidney transplantation 7 days ago has developed the following signs: urine output, 50 mL/12 hr; temperature, 102.2° F (39° C); lethargy; serum creatinine, 2.1 mg/dL; blood urea nitrogen

(BUN), 54 mg/dL; and potassium, 5.6 mEq/L. Which initial intervention

does the nurse anticipate for this client? a. Immediate hemodialysis

b. Increased dose of immune suppressive drugs c. Initiation of IV antibiotics after cultures are obtained

d. Placement of a catheter for peritoneal dialysis

The nurse is assessing a client with acute kidney injury and hears the following sound when auscultating the lungs. For what complication does

the nurse plan care? (Click the media button to hear the audio clip.) a. ac tamponade b. Pericarditis

c. Pulmonary edema d. Myocardial Infarction

MULTIPLE RESPONSE

A client asks the nurse, “What are the advantages of peritoneal dialysis over hemodialysis?” Which response by the nurse is accurate? (Select all

that apply.) a. “It will give you greater freedom in your scheduling.”

b. “You have less chance of getting an infection.” c. “You need to do it only three times a week.”

d. “You do not need a machine to do it.” e. “You will have fewer dietary restrictions.”

Chapter 72: Assessment of the Reproductive System

Test Bank

MULTIPLE CHOICE

The nurse is counseling a postmenopausal woman about her new stress incontinence. Which statement by the nurse is most important?

a. “You can try a variety of briefs and undergarments.” b. “It will be important to keep that area clean and dry.”

c. “I can refer you to a good incontinence clinic.” d. “Unfortunately, incontinence is common in women your age.”

An older woman is asking the nurse about her husband’s sexual functioning. Which statement by the nurse is most accurate?

a. “Men his age tend to have a rapid decline in sexual abilities.” b. “His testosterone levels will decrease only slightly until he is quite

old.” c. “Changes in testosterone levels do not affect sexual performance.”

d. “You are lucky your husband is healthy enough for sexual activity.”

The nurse is preparing a teaching plan for a client who is scheduled to undergo mammography for the first time. What instruction by the nurse

is accurate? a. “The test should be carried out even if you are pregnant.”

b. “Do not use deodorant on breasts or underarms before the test.” c. “You will not experience any discomfort because this is just an x-

ray.” d. “The entire test should not take longer than 1 hour.”

The nurse is conducting a reproductive assessment of a postmenopausal woman. Which assessment finding reported by the client requires

immediate intervention by the nurse?

a. Urinary incontinence b. Vaginal dryness

c. Painful intercourse d. Returning periods

The nurse is working with a client who is recovering after a cervical biopsy. Which statement by the client indicates a need for further

instruction? a. “I can resume vaginal intercourse after 6 weeks.”

b. “I should report heavy bleeding to the health care provider.” c. “I must not lift heavy objects for about 2 weeks.” d. “I will use the antiseptic rinse on a regular basis.”

The nurse is working with a client who is recovering after a laparoscopy. Which assessment finding is considered a priority by the nurse?

a. Slight drainage from the incision site b. Grogginess after the anesthesia c. Discomfort from the catheter

d. Reports of shoulder pain

A postmenopausal client says that she is experiencing difficulty with vaginal dryness during intercourse and wonders what might be causing

this. Which is the nurse’s best response? a. “The less frequently you have intercourse, the drier the vaginal

tissues become.” b. “Estrogen deficiency causes the vaginal tissues to become drier and

thinner.” c. “Drinking at least 3 liters of water each day will make all your

tissues less dry.” d. “Try using a water-soluble lubricant during intercourse.”

The nurse is assessing a client with a history of irregular periods. Which condition does the nurse possibly correlate with this problem?

a. Childhood mumps b. Past valve replacement surgery

c. Diabetes mellitus d. Mild intermittent asthma

A client tells the nurse she is happy that she never had children because she has less risk of developing cancer. Which response by the nurse is

best?

a. “Actually, your risk of breast cancer is slightly higher.” b. “You’re right; your risk of all reproductive cancer is quite low.”

c. “In reality, smoking is the leading risk factor for all types of cancer.” d. “Your risk of uterine cancer is higher because you had no children.”

A client who has had numerous children is having her annual examination. The nurse wishes to discuss contraception, but the client is

not interested. Which action by the nurse is most appropriate? a. Provide education on the value of spacing children.

b. Explain the many alternatives from which to choose. c. Ask the client how her husband feels about so many children.

d. Assess the client’s religious and cultural background.

The nurse is teaching a postmenopausal woman about nutrition. Which statement by the nurse is most appropriate?

a. “Be sure to eat cereal fortified with folic acid and B vitamins.” b. “Make sure you take a calcium supplement every day.”

c. “Vitamin C is important for the postmenopausal woman.” d. “You can get all the iron you need in two daily meat servings.”

When performing an assessment of the external genitalia of an older man, the nurse observes the scrotum to have smooth skin and to be very

pendulous. Which action by the nurse is most appropriate? a. Suggest to the client that he should wear an athletic supporter while

awake. b. Ask the client if he has been treated for a sexually transmitted

disease. c. Document the observation and continue the assessment.

d. Notify the health care provider and facilitate a scrotal ultrasound.

The nurse is counseling a mother who wants her teenage daughter to have a Pap smear and pelvic examination. Which statement by the nurse

is most accurate? a. “If your daughter is over 18, she needs a pelvic examination and Pap

smear.” b. “A teenager does not need this examination unless she is sexually

active.” c. “Teach her to have her first examination by the age of 21 at the

latest.” d. “It is not needed unless you are worried about sexually transmitted

diseases.”

A young woman is not pregnant but has not had a menstrual period for 5 months. Which factors does the nurse explore as a possible cause of the

amenorrhea? a. The client’s mother having type 2 diabetes mellitus

b. Running 10 to 15 miles/day c. Taking aspirin daily

d. Having a diet high in protein

When scheduling an annual pelvic examination and Pap test, the client asks if she should abstain from intercourse before the test. Which is the

nurse’s best response? a. “Yes. Avoid having intercourse for 24 hours before the test.” b. “Yes. Avoid having intercourse for 2 hours before the test.”

c. “No. Intercourse does not interfere with this test.” d. “No. Intercourse can actually enhance the test results.”

During examination of the male client’s external genitalia, the nurse observes a discharge from the urethra when compressing the glans.

Which is the nurse’s next action? a. Document the observation.

b. Ask the client to turn his head and cough.

c. Obtain a specimen for culture. d. Test the cremasteric reflex.

A client is in the clinic for an annual examination and questions the need for a pelvic examination and Pap smear because she had a hysterectomy

many years ago. Which response by the nurse is most appropriate? a. “Do you still have your cervix?”

b. “Are you sexually active?” c. “We can skip it if you like.”

d. “Let’s see what the doctor says.”

A client is scheduled for an ultrasound to evaluate for possible uterine fibroids. Which instruction by the nurse is most appropriate?

a. “Do not eat or drink anything after midnight.” b. “Take these laxatives the morning of the test.”

c. “Do not urinate an hour before the test; a full bladder will give best results.”

d. “Have a designated driver because you will be sleepy from the anesthesia.”

An African-American client has a prostate-specific antigen (PSA) of 12 ng/mL. Which action by the nurse is best?

a. Remind the client to repeat the test in 1 year. b. Prepare the client for further diagnostic testing.

c. Ask if the client ejaculated within 48 hours of the test. d. Assess the client for alcohol and tobacco use.

MULTIPLE RESPONSE

The nurse is teaching high school girls about the female reproductive tract. Which statements by the nurse are accurate? (Select all that

apply.) a. The vagina has an acidic environment.

b. The cervix is where the Pap smear is taken from. c. The ovum is fertilized in the uterus.

d. Ovaries produce sex steroid hormones. e. The breasts contain fat tissue.

A young adult client is in the clinic for evaluation of amenorrhea lasting

3 months. She takes birth control pills but is on no other medications. Which actions by the nurse are most appropriate? (Select all that apply.)

a. Instruct the client on collecting a urinalysis for a pregnancy test. b. Assess the client’s urinary and bowel habits.

c. Perform a physical assessment on the client’s abdomen. d. Weigh the client and calculate the body mass index.

e. Reassure the client that amenorrhea can occur with oral contraception.

Chapter 73: Care of Patients with Breast Disorders

Test Bank

MULTIPLE CHOICE

A client recently had a mammogram. Which statement by the client indicates a need for clarification regarding the importance or purpose of

this procedure? a. “Now that I have had a mammogram, my risk for getting breast

cancer is reduced.” b. “I will still do a breast self-examination monthly even after the

mammogram.” c. “Yearly mammograms can reduce my risk of dying from breast

cancer.” d. “The amount of radiation exposure from a mammogram is very

low.”

When performing a clinical breast examination on a client, the nurse palpates a thickened area where the skin folds under the breast. Which is

the nurse’s best action? a. Proceed with the examination.

b. Determine whether the thickness is bilateral. c. Ask how long the thickness has been present.

d. Change the client’s position and re-assess.

A client who has discovered a lump in her breast becomes tearful when scheduling a mammogram. Which is the nurse’s best response?

a. “All lumps are considered cancerous until proven otherwise.” b. “Unless you have a relative with breast cancer, this lump is probably

benign.” c. “Diagnosing cancer at this early stage is most likely to result in a

cure.” d. “Many women have breast lumps, and most of the lumps are

benign.”

A client has just been diagnosed with fibrocystic breast disease. She asks what this means in terms of her health. Which is the nurse’s best

response? a. “This increases your risk for breast cancer, so schedule yearly

mammograms.” b. “This will increase as you age, especially if you have never been

pregnant.” c. “This will diminish with menopause if you don’t take replacement

hormones.” d. “This is genetic and you should teach your daughters about it.”

Which client does the nurse encourage to seek genetic counseling regarding her risk for BRCA1 or BCRA2 gene mutation–related breast

cancer? a. Woman whose father had lung cancer and mother had leukemia b. Woman whose sister has breast cancer and mother has ovarian

cancer c. Woman whose fraternal twin sister has breast cancer d. Older woman who has bilateral benign breast disease

Which statement made by a client about breast cancer indicates correct

understanding of the disease? a. “Breast cancer is the leading cause of cancer deaths among women

in the United States.” b. “Breast cancer is the leading type of cancer among women in North

America.” c. “Late onset of menses and early menopause increase the risk for

breast cancer.” d. “Breast cancer decreases with age, and very old women have

virtually no risk.”

Which comment made by a client with breast cancer indicates correct understanding regarding cancer causes and prevention?

a. “I will prevent recurrence of my cancer by eating a low-fat diet from now on.”

b. “If I had breast-fed my children, this would not have happened to me.”

c. “I hope this doesn’t increase my risk for bone cancer or lung cancer.”

d. “I will have regular mammograms on my other breast to detect cancer early.”

A client has advanced breast cancer and bone metastasis. Which problem does the nurse consider the priority?

a. Pain

b. Mobility problems c. Risk for infection

d. Malnutrition

A client had a mastectomy nearly a year ago and is distressed over continued tingling and burning in the ipsilateral arm. What orders does

the nurse prepare to implement? a. Teach the client about gabapentin (Neurontin).

b. Demonstrate the use of heat therapy to the axilla. c. Discuss ways to prevent constipation with pain meds.

d. Reassure the client that this will disappear shortly.

A client had a mastectomy and axillary node dissection. The nurse empties sanguineous drainage from the client’s incisional Jackson-Pratt drain on the first postoperative day. Which other action regarding the

drain is of high priority for the nurse? a. Flushing the tubing with urokinase to ensure patency b. Compressing and closing the drain to ensure suction

c. Advancing the tubing inch from the insertion site d. Clamping the drain for 2 hours and releasing it for 2 hours

A client is postoperative from a left-sided mastectomy. She says that the incision and the inner side of her arm from the armpit to the elbow are

numb. Which is the nurse’s best action? a. Teach the client to avoid lifting heavy objects.

b. Measure the circumference of the client’s left arm. c. Reassure the client that this is an expected finding.

d. Notify the surgeon as soon as possible.

A client receiving tamoxifen (Tamofen) asks how this therapy helps fight breast cancer. Which is the nurse’s best response?

a. “This agent decreases estrogen levels. so the cancer stops growing.” b. “The drug causes you to secrete testosterone, which limits cancer

growth.” c. “Tamoxifen kills estrogen-secreting cells and growth of blood vessels

to cancer cells.” d. “It blocks estrogen receptors, and this limits cancer cell growth.”

A client asks how soon after a mastectomy she can engage in sexual activity. Which is the nurse’s best response?

a. “When do you want to resume sexual activity?” b. “Most surgeons say to wait several weeks after the operation.”

c. “As soon as the incision has healed completely.”

d. “You shouldn’t worry about sexuality right now.”

Which exercise plan or activity does the nurse teach the client for the first postoperative day after a modified radical mastectomy?

a. “Perform no movement or exercise today. Keep the arm supported and the elbow flexed, and as close to your body as possible.”

b. “Without moving your shoulder, straighten your elbow three times hourly and squeeze a rubber ball with your fingers.”

c. “Face the wall and extend your arm straight out to the wall. Walk your fingers as far above your head as your arm will reach, and then

walk them back down.” d. “Hold your operative arm straight out from the shoulder to the side. Use your nonoperative arm to pull the operative arm completely straight

above your head.”

A client is experiencing lymphedema in the arm on the operative side after a modified radical mastectomy. Which statement indicates correct

understanding of managing this problem? a. “I will reduce my intake of salt and water.” b. “I will elevate my arm on a pillow at night.”

c. “I will try to drink at least 3 liters of water each day.” d. “I will wear long sleeves to prevent sun exposure.”

A woman is asking about monthly breast self-examination (BSE). What information does the nurse provide to the client?

a. “It is a valuable tool for finding breast lumps early.” b. “After menopause, it is no longer useful.”

c. “BSE should be combined with other assessments.” d. “Women in their 30s should begin monthly BSE.”

A client with a family history of breast cancer tells the nurse that she has made several recent lifestyle changes. Which question by the nurse about

these practices is most important? a. “Are you a vegetarian?”

b. “Do you drink green tea?” c. “What supplements do you use?”

d. “Do you smoke cigarettes?”

A client has large breasts. Which health problem is she most likely to develop?

a. Breast tenderness b. Breast cancer

c. Chest pain

d. Back pain

A client is undergoing treatment for breast cancer and asks the nurse about “natural” treatments for her chemotherapy-induced nausea.

Which is the most appropriate response by the nurse? a. “Anything you can take will interfere with your chemotherapy.”

b. “I don’t know of any recommended complementary treatments for nausea.”

c. “Black cohosh and flaxseed are good for combating nausea.” d. “Ginger has been used for nausea; would you consider taking it?”

A client had a mastectomy with reconstruction, and several axillary nodes were dissected. Which statement by the client indicates good

understanding of discharge instructions? a. “I must be careful not to injure the arm or hand on the side of my

surgery.” b. “I’m glad that lymphedema is no longer a problem, as it was in my

mother’s day.” c. “I will have a hard time waiting for a whole year to see how my

breast will look.” d. “I need to pull my drains out by inch each day until they are totally

out.”

A client is being treated with anastrozole (Arimidex) for breast cancer. The nurse is developing a plan of care for the client. Which intervention

is the highest priority? a. Teach the client to weigh herself each day at the same time.

b. Instruct the client to keep a symptom journal for menopausal symptoms.

c. Monitor the client closely for evidence of osteoporosis. d. Review the client’s dietary habits to prevent weight gain.

A client with a history of breast cancer is admitted through the emergency department with shortness of breath, weakness, fatigue, and new lower extremity edema. The client’s oxygen saturation is 88%. After

stabilizing the client, which action by the nurse is most important? a. Obtain a list of the client’s medications.

b. Orient her to her room and surroundings. c. Place the client on intake and output.

d. Assess the client’s family cardiac history.

The clinic nurse is preparing a client for a physical and breast examination. The nurse notes the client’s breast appears as shown in the

photograph below. Which action by the nurse takes priority?

a. Continue preparations and note the finding in the client’s chart. b. Ask the client how long this problem has been present.

c. Alert the health care provider and prepare to order a mammogram. d. Question the client about routine drug and alcohol intake.

MULTIPLE RESPONSE

The nurse is assessing a client with a history of ductal ectasia. Which signs and symptoms supporting this diagnosis does the nurse correlate

with this condition? (Select all that apply.) a. A soft mass on palpation

b. Greenish-brown nipple discharge c. Enlarged axillary nodes

d. A mass with regular borders e. Redness and edema over the site of the mass

f. Mass tenderness on palpation

Which factors are considered to be indicative of a moderately increased risk of a client’s developing breast cancer? (Select all that apply.)

a. High postmenopausal bone density b. Ionizing radiation

c. Family history of one first-degree relative d. Genetic factors

e. First child born after age 30 f. Biopsy-confirmed atypical hyperplasia

Chapter 74: Care of Patients with Gynecologic Problems

Test Bank

MULTIPLE CHOICE

A client is in the clinic reporting stress incontinence. Which other assessment is the priority for the nurse to perform?

a. Ask the client about vaginal discharge or bleeding. b. Have the client perform a 24-hour fluid recall.

c. Inquire about fever, chills, and burning on urination. d. Obtain the client’s reproductive history.

A woman reports cyclical abdominal pain, and her pelvic examination reveals tender nodules in the posterior vagina. The nurse plans to

educate the woman about which treatment? a. Medroxyprogesterone (Depo-Provera)

b. Radiation therapy c. Doxycycline (Vibramycin)

d. Endometrial ablation

A woman has endometriosis and is visibly upset. She tells the nurse that she just got married and wants to have children but is distressed because

now she will be infertile. Which response by the nurse is most appropriate?

a. “Treatment for endometriosis often causes infertility; I can refer you to a support group.”

b. “Endometriosis is more common in infertile women, but not all treatments cause infertility.”

c. “You shouldn’t worry about fertility until after we know that this didn’t cause cancer.”

d. “Unfortunately, you will have to take birth control pills for the rest of your life.”

A 48-year-old woman reports to the nurse about new “flooding” with her periods. Which other complaint is the nurse prepared to investigate more

thoroughly? a. Hot flashes and sweating episodes

b. Fatigue during typical activity c. More frequent periods than usual d. Abdominal cramping with periods

For which problem are Kegel exercises recommended? a. Cyst

b. Fistula c. Cystocele d. Rectocele

When the history of a female client is taken, which client statement does the nurse refer to the health care provider?

a. “I had a fibroadenoma of the breast when I was 22 years old.” b. “I had my first child when I was 26 years old and my second child

when I was 32.” c. “I stopped using oral contraceptives when I was no longer sexually

active.”

d. “I had my menopause 2 years ago and have started to have vaginal bleeding again.”

Which symptom experienced by a woman in her 20s alerts the nurse to the possibility of endometriosis? a. Bleeding between periods b. Cessation of menstruation

c. Premenstrual tension headache d. Pain before the onset of menstrual flow

Which action does the nurse teach the client to prevent toxic shock syndrome?

a. “Use a barrier method of contraception.” b. “Wash your hands before inserting a tampon.”

c. “Avoid intercourse with more than one partner.” d. “Empty your bladder immediately after intercourse.”

A young woman calls the clinic to report a fever and a funny rash with peeling skin on the palms of her hands and the soles of her feet. Which

action by the nurse is most appropriate? a. Make an appointment for her to be seen the next day at the clinic. b. Instruct her to take warm baths with oatmeal added to the water.

c. Tell her to go to the emergency department immediately. d. Have her take acetaminophen (Tylenol) every 4 hours and drink

fluids.

A client with pelvic organ prolapse has chosen treatment with a vaginal mesh. Which action by the nurse before the procedure is most

important? a. Administering the preoperative sedative medication

b. Giving the woman the manufacturer’s labeling information c. Ensuring that the woman has a ride home after she recovers

d. Witnessing the client signature on the informed consent

Which statement made by a woman who is being discharged after a hysterectomy indicates understanding and acceptance?

a. “I wish I had delayed this surgery so that I could have had one more child.”

b. “I will diet to prevent the weight gain most women have after hysterectomy.”

c. “Now that my uterus will be gone, I’ll probably develop stress incontinence.”

d. “My husband and I hope to have more sex because I won’t have so

much bleeding.”

A client had a posterior colporrhaphy. Which statement by the client indicates an adequate understanding of discharge instructions? a. “I’ll eat a high-fiber diet so I won’t get constipated again.”

b. “I’ll expect my periods to decrease within the next 6 months.” c. “I’ll need to eat a low-residue diet.”

d. “I’ll call the surgeon if I saturate more than one pad in 4 hours.”

A woman is receiving radiation via brachytherapy for endometrial cancer. Which statement by the woman indicates a need for further

education about the procedure? a. “I can go about my usual activities between sessions.”

b. “I might experience more fatigue than usual during therapy.” c. “I should report any fever over 100 degrees to my doctor.”

d. “I must stay away from my young grandchildren for 6 weeks.”

Which clinical manifestation in a client with invasive cervical cancer alerts the nurse to the possibility of metastasis?

a. Amenorrhea b. Weight gain

c. Breast tenderness d. Swelling of one leg

A client has undergone cryosurgery for stage I cervical cancer. Which precaution or action does the nurse teach this client?

a. “Use sanitary napkins to manage discharge for the next several weeks.”

b. “Avoid sexual intercourse or becoming pregnant for the next 12 months.”

c. “If you should become pregnant, you will be at increased risk for preterm labor.”

d. “Your next menstrual cycle will be delayed because of this procedure.”

A client had a uterine artery embolization and has just returned to the nursing unit. She is asking when she can get up to go to the bathroom.

Which question does the nurse ask during hand-off report? a. “Was a vascular closure device used?”

b. “What was her estimated blood loss?” c. “Is there an order for a catheter?”

d. “When was the client’s last sedation?”

A client has returned to the nursing unit after a total abdominal hysterectomy. The nurse auscultates the client’s abdomen and does not

hear bowel sounds. Which is the nurse’s priority intervention? a. Document the finding in the chart. b. Position the client on the right side.

c. Irrigate the nasogastric tube. d. Measure abdominal girth.

Why are the death rates from ovarian cancer so high? a. The causative oncovirus is resistant to chemotherapy and to

radiation. b. No symptoms are obvious during the early stages of this disorder.

c. Radiation therapy is ineffective because the ovaries are located deep in the pelvis.

d. Ovarian cancer occurs mostly in women over the age of 70 who have other health problems.

A client had a total abdominal hysterectomy with bilateral salpingo- oophorectomy and pelvic lymph node dissection 2 days ago. The nurse

finds the client short of breath, tachycardic, and anxious. What intervention takes priority?

a. Assess oxygen saturation and apply oxygen if needed. b. Have the client cough and deep breathe or use the spirometer.

c. Call respiratory therapy to provide a nebulizer treatment. d. Prepare to administer furosemide (Lasix) IV push.

Which intervention is essential for the nurse to perform for a client who had a total abdominal hysterectomy?

a. Instruct the client on a low-fat diet. b. Monitor for the onset of menopause. c. Assess for problems with intercourse.

d. Teach exercises to prevent incontinence.

A woman has had recurrent Bartholin cysts. Which intervention is most appropriate for the nurse to add to the client’s care plan?

a. Assess the woman for sexually transmitted diseases (STDs). b. Prepare a family diagram to investigate a familial pattern.

c. Teach the woman about surgical marsupialization. d. Instruct the woman to wear only cotton underwear.

The client has been diagnosed with possible vulvovaginitis pending the outcome of laboratory tests. What information does the nurse teach the

client? a. “Use sanitary pads, not tampons, when you have your period.”

b. “Limit douching to once a month or so, after your period.” c. “Scrub your vulvar area with antibacterial soap when you bathe.”

d. “Wear only cotton underwear and wear looser jeans or slacks.”

A client has recently undergone an anterior colporrhaphy. Which is the most important discharge instruction that the nurse can provide?

a. “Avoid sexual intercourse for 2 weeks.” b. “Call us for fever and pain that does not improve.”

c. “Sutures will need to be removed in 2 weeks.” d. “An ice pack on your incision will help the pain.”

A client is scheduled to have a hysteroscopic myomectomy. Which statement by the client indicates that she understands the procedure?

a. “I will need to deliver future children by cesarean section.” b. “I need to schedule this during the last part of my cycle.”

c. “My uterus will be removed through tiny incisions.” d. “This operation will make me infertile.”

A client is recovering from a hysteroscopic myomectomy. The nurse assesses the client and finds the following: 2+ bilateral pedal edema;

pulse, 108 beats/min; and respiratory rate, 28 breaths/min. Which action by the nurse takes priority?

a. Assess lung sounds and oxygen saturation. b. Call for an immediate electrocardiogram (ECG).

c. Notify the health care provider as soon as possible. d. Review the client’s intake and output pattern.

A woman has been told she has cervical polyps. Which statement by the client indicates a good understanding of the teaching the nurse provided?

a. “I hope my polyp doesn’t turn cancerous like colon polyps can.”

b. “These can be removed easily in the doctor’s office with little pain.” c. “I will need to have more frequent screening for cervical cancer.” d. “I will need to finish all my medication before having sex again.”

A client is crying because her endometrial cancer is scheduled to be treated with chemotherapy agents that will cause hair loss. Which is the

most appropriate response from the nurse? a. “You should prepare yourself for total hair loss all over your body.”

b. “You can start shopping for wigs and scarves now so you’ll have them available.”

c. “Why not shave your hair off now so that you can have it made into a wig?”

d. “Would you like me to put you in touch with a former client so that you can talk?”

A woman had returned to the nursing unit after a total abdominal hysterectomy. After settling the client and performing a thorough assessment including vital signs, which action by the nurse is most

important? a. Consult with physical therapy about ambulating the client.

b. Obtain and apply sequential compression devices. c. Order the client’s next-day chest x-ray and laboratory work.

d. Assist the client to order light food items for dinner.

MULTIPLE RESPONSE

The nurse is teaching a woman’s group about gynecologic cancers. Which does the nurse teach are risk factors? (Select all that apply.)

a. Nulliparity b. Multiple sex partners

c. Obesity d. Smoking

e. Delayed first intercourse

Chapter 75: Care of Male Patients with Reproductive Problems

Test Bank

MULTIPLE CHOICE

The nurse is assisting a client with limited mobility into position for examination of his prostate gland. How does the nurse best assist the

client? a. Assist the client to bend over the examination table.

b. Hold the client up as he bends over the bedside table. c. Help the client lie down in a side-lying fetal position.

d. Assist the client to lie in a prone position.

A client’s laboratory findings reveal an elevated serum acid phosphatase level and a high-normal prostate-specific antigen level. How does the

nurse interpret this information? a. The client shows evidence of renal disease and should be evaluated

further. b. These results may indicate prostate cancer. He should be further

evaluated. c. These results are not abnormal. He does not need to be evaluated

further. d. These results may indicate an infection. He should be evaluated

further.

Which client statement indicates understanding about a transrectal ultrasound?

a. “This will determine if the outlet of my bladder is obstructed.” b. “This will determine the amount of residual urine present.”

c. “This is performed to view the interior of the bladder and urethra.” d. “This is performed to view the prostate and do a tissue biopsy.”

Which statement indicates that a client understands the most appropriate time of day to take an alpha blocker drug for treatment of

benign prostatic hyperplasia (BPH)? a. “I’ll take my medication at bedtime.”

b. “As soon as I get up, I will take my medication.” c. “I will take my medication with food or milk.”

d. “I’ll take my medication on an empty stomach.”

The nurse is assessing a client who has undergone a transurethral resection of the prostate (TURP). Which assessment finding requires

immediate action by the nurse? a. Passing small blood clots after catheter removal

b. Experiencing urinary frequency after catheter removal c. Having bright red drainage with multiple blood clots

d. Having the urge to void continuously while the catheter is inserted

A client’s prostate-specific antigen (PSA) level is 2.0 ng/mL. Which action by the nurse is most appropriate?

a. Inform the client that the results are normal and no cancer has been detected.

b. Inform the client that results are normal and schedule a digital rectal examination.

c. Let the client know that the results are elevated and he is at risk for prostate cancer.

d. Tell the client that cancer is indicated and that the health care provider recommends watchful waiting.

Which client diagnosed with prostate cancer is not a candidate for watchful waiting?

a. Client with very early cancer of the prostate b. Client who is asymptomatic

c. Client who wants to avoid urinary incontinence as a result of treatment

d. Client who refuses frequent digital rectal examinations (DREs)

A client is scheduled for a prostatectomy for benign prostatic hyperplasia (BPH). On the morning of surgery, the laboratory report on the client’s

urine indicates the presence of red blood cells, white blood cells, and

bacteria. Which is the nurse’s highest priority action? a. Document the report in the client’s chart.

b. Insert a new Foley catheter before surgery. c. Strain the client’s urine.

d. Assess the client’s vital signs and notify the health care provider.

A client has been taking finasteride (Proscar) for 4 weeks and reports that he has not yet seen a reduction in symptoms. Which response by the

nurse is most appropriate? a. “Have you been taking the medication as ordered?”

b. “It may take several months to see results.” c. “It may not be the right drug for you.”

d. “We can try dutasteride (Avodart) next.”

A client had a transurethral prostatectomy and has incontinence. Which statement by the client indicates a need for clarification about managing

this condition? a. “I will practice stopping the urine stream to strengthen my

sphincter control.” b. “I will limit my fluid intake every day to prevent incontinence.”

c. “I will avoid vigorous activity for the first 3 weeks after surgery.” d. “I will avoid caffeinated beverages and spicy foods.”

A client with prostate cancer reports pain in his lower back and legs. Which action by the nurse is most appropriate?

a. Discuss medications for arthritis. b. Perform a bladder scan.

c. Facilitate imaging studies. d. Encourage weight-bearing exercises.

A client diagnosed with early prostate cancer is confused that surgery has not been planned. Which is the nurse’s best response?

a. “The disease is slow-growing. The risks of surgery at your age are not justified by the outcome.”

b. “Your disease is so advanced that surgery at this point would not increase your chances of cure.”

c. “Your disease is in a very early stage and is slow-growing. Your doctor will monitor you.”

d. “This stage indicates that you do not really have cancer, so surgery is not necessary.”

A client with a catheter in place after a suprapubic prostatectomy has decreased urinary output. Which action by the nurse is most

appropriate? a. Assess the client for bladder spasms.

b. Encourage the client to drink more water. c. Administer pain medication.

d. Have the client try to void around the catheter.

A client had a seminoma removed via an open procedure. The following day, the client’s abdomen is soft and nontender, but no bowel sounds are

present. Which action by the nurse is most appropriate? a. Encourage the client to ambulate several times a day.

b. Reassess the client’s abdomen in 4 hours. c. Document the finding and call the surgeon immediately. d. Give the client a laxative and encourage high-fiber food.

Which client is most likely to have organic erectile dysfunction? a. Middle-aged man who first had sexual intercourse at age 15 b. Middle-aged man who has had diabetes mellitus for 25 years

c. Young man who had a myocardial infarction 2 years ago d. Young man who has a job that causes him high stress levels

A client is going home after outpatient surgery for a hydrocele. Which information does the nurse emphasize in teaching this client?

a. “Report to the doctor immediately any drainage from your drain.” b. “Use a condom during intercourse to prevent incisional infection.” c. “Sit when you urinate until all swelling is gone and drainage has

stopped.” d. “Wear the scrotal support device for at least 3 weeks after surgery.”

A client had a spermatocele removed in an outpatient surgical center. Which statement by the client indicates good understanding of discharge

instructions? a. “The heavy drainage will go away within a few days.”

b. “I need to buy dressing supplies at the drugstore.” c. “I should report any redness or drainage from the incision.”

d. “Because of all the narcotics I’ll be taking, I will need laxatives.”

A client who was treated 1 year ago for testicular cancer now has an elevated serum alpha-fetoprotein level. Which topic is most important

for the nurse to teach this client about?

a. Sperm banking options b. Effects of chemotherapy

c. Hospice philosophy d. Importance of yearly monitoring

A client is scheduled for a penectomy for penile cancer. Which action by the nurse is most important?

a. Teaching the client to sit when he urinates b. Demonstrating dressing changes and wound cleaning c. Assessing the client’s psychosocial status and support

d. Explaining the purpose of the in-dwelling catheter

Which client statement indicates understanding about post-orchiectomy care for testicular cancer?

a. “I will avoid contact sports to prevent injury and development of cancer in my remaining testis.”

b. “I will always use a condom because I am at increased risk for acquiring a sexually transmitted disease.”

c. “I will wear an athletic supporter and cup to prevent testicular cancer in my remaining testicle.”

d. “I will continue to perform testicular self-examination (TSE) monthly on my remaining testicle.”

The client with sickle cell anemia has had an erection for longer than 4 hours. How does the nurse intervene?

a. Administer a diuretic to increase urine output. b. Attempt to relieve pressure by catheterizing the client.

c. Document the finding and reassess in 4 hours. d. Notify the health care provider and prepare to give meperidine

(Demerol).

Which intervention helps the client with chronic prostatitis prevent spread of infection to other areas of the urinary tract?

a. “Wear a condom during intercourse.” b. “Avoid alcohol and caffeinated beverages.”

c. “Be sure to empty your bladder completely at each voiding.” d. “Sexual intercourse or masturbation can help drain the prostate.”

A client with BPH asks why his enlarged prostate is causing difficulty with urination. Which is the nurse’s most accurate response? a. “It compresses the urethra, blocking the flow of urine.”

b. “It presses on the kidneys, decreasing urine formation.” c. “It secretes acids that weaken the bladder, causing dribbling.” d. “It destroys nerves, decreasing awareness of a full bladder.”

A client has decided to treat his enlarged prostate with saw palmetto. Which is the nurse’s best response?

a. “You’ll need to get permission from your health care provider to make that decision.”

b. “Saw palmetto is a well-respected alternative therapy for benign prostatic hyperplasia.”

c. “Have you discussed this decision with your family?” d. “What has your health care provider told you about this choice of

therapy?”

A client is receiving brachytherapy for prostate cancer. Which intervention is most important for the nurse to include in the client’s care

plan? a. Reassure the client that erectile dysfunction does not occur with

brachytherapy. b. Help the client plan activities interspersed with rest periods during

the day. c. Remind the client that while hospitalized, he cannot have any

visitors. d. Discuss hospice philosophy and availability with the client and

family.

A client is taking goserelin (Zoladex). What periodic assessment does the nurse plan for this client?

a. Weight and abdominal girth b. Pulmonary function tests

c. Bone density testing d. Abdominal ultrasound

The nurse is performing a psychosocial assessment of a young man diagnosed with testicular cancer. Which does the nurse include as a

priority in the assessment? a. Encouraging the client to verbalize his thoughts and feelings to his

health care provider b. Assisting the client in locating a support group for men with

testicular cancer

c. Asking the client to rate his fears of sexual deficiency on a scale of 1 to 10

d. Identifying all components of his support system, including his partner

The nurse is caring for a young adult who just got married and has been diagnosed with testicular cancer. To which community resource does the

nurse refer him? a. American Cancer Society

b. Red Cross c. Sperm bank

d. Public Health Department

An older client with benign prostatic hyperplasia (BPH) and hypertension is being treated with doxazosin (Cardura) while staying in

the hospital. Which activity does the nurse delegate to the unlicensed assistive personnel (UAP) as a priority?

a. Helping the client choose low-sodium meal items b. Assisting the client whenever he gets out of bed

c. Encouraging the client to use the spirometer hourly d. Frequently re-orienting the client to his surroundings

MULTIPLE RESPONSE

Which symptoms are expected in orchitis? (Select all that apply.) a. Scrotal pain

b. Dysuria c. Scrotal edema

d. Priapism e. Penile discharge

f. Inability to ejaculate

A client has returned to the nursing unit after a prostatectomy. Which activities does the nurse delegate to the unlicensed assistive personnel

(UAP)? (Select all that apply.) a. Demonstrating how to use the incentive spirometer

b. Measuring and recording output from the in-dwelling catheter c. Encouraging the client to get out of bed and into the chair d. Irrigating the catheter with normal saline for blood clots

e. Re-taping the catheter tape if the client reports pain

Chapter 76: Care of Patients with Sexually Transmitted Disease

Test Bank

MULTIPLE CHOICE

The nurse is teaching a young woman about her risk of contracting a sexually transmitted disease (STD). Which statement by the client

indicates that further instruction is needed? “I am at decreased risk for an STD if I don’t rely on contraceptive

sponges or foams to protect me.” “I am at decreased risk for an STD because I am using an intrauterine

device for contraception.” “I am at increased risk for an STD because of the way that my body is

designed as a woman.” “I will be at increased risk for an STD if I rely on oral contraceptives to

protect me from contracting a disease.”

Which statement by a middle-aged woman indicates that further instruction is needed for her and her partner regarding prevention of

sexually transmitted diseases (STDs)?

“I’m glad we don’t have to use condoms anymore because I can’t get pregnant.”

“Changes in my vagina may make me more likely to be at risk for an STD.”

“I told my partner that we need to switch to condoms instead of the pill now.”

“I should report any evidence of infection, even if symptoms are minor.”

The nurse is conducting an assessment on a client and identifies a lesion that appears as a smooth indurated area. Which is the highest priority

action on the part of the nurse? Question the client further regarding sexual practices.

Ask the client about any associated symptoms. Document the findings and obtain a specimen of fluid from the lesion.

Don gloves before continuing to assess the lesion any further.

The nurse is counseling a client who has recently been diagnosed with syphilis. Which is the highest priority instruction that the nurse provides

to the client regarding sexual partners? “As long as both of you are being treated, abstinence is not necessary.”

“If you both have the same disease, you can continue to have sex.” “Your partner must be treated with antibiotics within the next 90 days.”

“Once the health department gets your partner’s name, confidentiality is not considered to be important.”

A client has been diagnosed with genital herpes. Which statement by the client indicates an accurate understanding of the disease and treatment?

“Antiviral drugs can cure genital herpes and prevent a recurrence.” “I can prevent outbreaks with suppressive antiviral therapy.”

“Suppressive therapy will prevent shedding of the virus.” “Medication should be taken only when symptoms are present.”

A client has recently been diagnosed with gonorrhea. The client comes from a deeply religious family. When the nurse finds the client weeping, the client tells the nurse, “I’m being punished for having an affair.” How

does the nurse respond? “Surely you don’t really believe that.” “Why don’t we get you a sedative?” “Tell me more about how you feel.” “Which religion do you practice?”

A client has just been diagnosed with a recurrence of genital herpes simplex. She asks how this is possible because she has not had sex since

she was diagnosed and treated 1 year ago. Which is the nurse’s best response?

“Sometimes one course of therapy is not enough to eradicate the disease.”

“The disease can be controlled but is never cured, and outbreaks are common.”

“Did you take the medication exactly the way it was prescribed for you?” “If you have more than one sex partner, you may have more than one

strain.”

A client has secondary syphilis. What precautions are necessary for the nurse to take when caring for this client?

No precautions in addition to Standard Precautions are necessary. Gloves should be worn whenever direct contact with the client’s skin is

required. Handwashing is required before and after contact with the client.

A mask should be worn by anyone entering the client’s room.

A client with primary syphilis was treated with an intramuscular injection of benzathine penicillin G. Later, the client reports a hard

painful lump at the injection site and aching joints. Which is the nurse’s highest priority initial action?

Assess the client’s vital signs. Give the client acetaminophen (Tylenol).

Document the finding in the chart. Apply a warm compress to the site.

Which statement made by a client about condom use indicates a need for clarification?

“I will use a new condom each time I have intercourse.” “I will use an oil-based lubricant whenever I have intercourse.”

“I will always use a latex condom rather than a natural membrane condom.”

“I will keep the condom on until I have withdrawn from the vagina.”

A client has been diagnosed with Trichomonas vaginalis. Which statement by the client indicates an accurate understanding of this

disease? “I need to have a throat culture for Trichomonas.”

“This will affect only my vagina and can cause itching.” “My partner does not need to be treated.”

“My lymph nodes may stay swollen after treatment.”

A client with pelvic inflammatory disease (PID) from gonorrhea asks how this can cause sterility. Which is the nurse’s response?

“The infection damages the ovary so that less estrogen is secreted and ovulation is not possible.”

“The infection remains in your body and can infect your baby, so it is best if you don’t become pregnant.”

“If the infection is present in the fallopian tubes, it can cause enough scarring to block the tubes permanently.”

“The infection causes such damage to the cervix that it cannot contain a pregnancy inside the uterus for longer than 3 months.”

A client is brought to the emergency department by the family, who state that the client was diagnosed last week with gonorrhea but has not taken the medications yet. Today the family notes that the client is “not acting

right” and seems confused. Which action by the nurse is most appropriate?

Start an IV and notify the health care provider about the client’s diagnosis.

Perform a thorough neurologic assessment and document the findings. Administer acetaminophen (Tylenol) if the client has a fever.

Ask the client why he or she has not started the medication regimen yet.

A client was diagnosed with chancroid. Which manifestation does the nurse associate with this condition?

Vaginal discharge High fever

History of ectopic pregnancies Genital ulcers

Which disease process places the client at greatest risk for development of an ectopic pregnancy?

Chlamydia infection Genital herpes

Human papilloma virus infection Pelvic inflammatory disease (PID)

Which should be the nursing focus for a female client during the initial outbreak of genital herpes simplex?

Instruction in condom use Promotion of comfort

Prevention of pregnancy Institution of isolation

A female client is diagnosed with human papilloma virus (HPV) infection. Which intervention by the nurse takes priority? Instruct the client on using podofilox (Condylox) cream. Prepare the client for a Pap test and HPV DNA testing.

Teach the client to take all medications until they are gone. Encourage the client to drink 8 to 10 glasses of water daily.

A 24-year-old woman has just been diagnosed with human papilloma virus (HPV) infection. She is very angry at her ex-boyfriend, who has

been her only sexual contact. She is crying and says that she isn’t going to tell him that he is infected. Which is the nurse’s best response?

“You do not have to tell him because this is not a reportable disease in this state.”

“Because there is no cure for this disease, telling him would be of no benefit.”

“He should be told so he can take precautions to prevent the spread of infection.”

“You should tell him because he may not know that this can cause cancer.”

A client has been diagnosed with anal cancer. Which test does the nurse prepare the client for?

Darkfield microscopy Culture of discharge

Blood draw for the Venereal Disease Research Laboratory (VDRL) test Human papilloma virus (HPV) DNA

A female client admitted for cardiac problems also has condyloma acuminatum. Which type of precautions does the nursing staff

implement with this client? Standard Airborne Contact Droplet

Why are women more likely than men to have silent sexually transmitted disease (STD) infection?

Women are less susceptible to STDs and are not assessed often for them. Lesions may not be visible, or the woman can be asymptomatic.

A man’s longer urethra provides increased opportunity for bacteria to multiply.

Symptoms of infection in women are likely to be systemic and vague, not local.

The nurse manages a clinic in an area with a high rate of sexually transmitted diseases (STDs). Which strategy best helps decrease the rate

of infection? Start an expedited partner treatment program.

Use a single-dose drug given in the clinic. Provide referrals to a low-cost pharmacy.

Plan occasional community educational programs.

The nurse assesses a client and finds the manifestation shown in the photograph. Which drug does the nurse prepare to administer to the

client?

Doxycycline (Vibramycin) Ceftriaxone (Rocephin)

Acyclovir (Zovirax) Podophyllin (Pododerm)

COMPLETION

A client weighing 110 lb is admitted with acute pelvic inflammatory disease. The client is ordered to receive an initial dose of gentamicin

(Garamycin), 2 mg/kg. The client will receive an initial dose of gentamicin of milligrams.